You are on page 1of 339

Mc lc

Li ni u . . . . . . . . . . . . . . . . . . . . . . . . . . . . . . . . . . . . . . .

Cc thnh vin tham gia chuyn . . . . . . . . . . . . . . . . . . . . . . . .

I CNG V PHNG TRNH HU T

Phng trnh bc ba . . . . . . . . . . . . . . . . . . . . . . . . . . . . . . . . . . .

Phng trnh bc bn . . . . . . . . . . . . . . . . . . . . . . . . . . . . . . . . . .

13

Phng trnh dng phn thc . . . . . . . . . . . . . . . . . . . . . . . . . . . . . .

19

Xy dng phng trnh hu t . . . . . . . . . . . . . . . . . . . . . . . . . . . . . .

22

Mt s phng trnh bc cao

25

. . . . . . . . . . . . . . . . . . . . . . . . . . . . . .

PHNG TRNH, H PHNG TRNH C THAM S

28

Phng php s dng o hm . . . . . . . . . . . . . . . . . . . . . . . . . . . . .

28

Phng php dng nh l Lagrange - Rolle . . . . . . . . . . . . . . . . . . . . . .

38

Phng php dng iu kin cn v . . . . . . . . . . . . . . . . . . . . . . . . .

42

Phng php ng dng hnh hc gii tch v hnh hc phng . . . . . . . . . . . . .

51

Hnh hc khng gian v vic kho st h phng trnh ba n . . . . . . . . . . . . .

74

Mt s bi phng trnh, h phng trnh c tham s trong cc k thi Olympic . . .

79

CC PHNG PHP GII PHNG TRNH

92

Phng php dng lng lin hp . . . . . . . . . . . . . . . . . . . . . . . . . . . .

92

Phng php dng n iu hm s . . . . . . . . . . . . . . . . . . . . . . . . . .

106

Phng php dng bt ng thc . . . . . . . . . . . . . . . . . . . . . . . . . . . .

114

PHNG TRNH M-LOGARIT

123

L thuyt . . . . . . . . . . . . . . . . . . . . . . . . . . . . . . . . . . . . . . . . .

123

Phng php t n ph . . . . . . . . . . . . . . . . . . . . . . . . . . . . . . . . .

123

Phng php dng n iu hm s . . . . . . . . . . . . . . . . . . . . . . . . . .

131

Phng php bin i ng thc . . . . . . . . . . . . . . . . . . . . . . . . . . . .

135

Bi tp tng hp . . . . . . . . . . . . . . . . . . . . . . . . . . . . . . . . . . . . .

139

H PHNG TRNH

143

Cc loi h c bn . . . . . . . . . . . . . . . . . . . . . . . . . . . . . . . . . . . .

143

H phng trnh hon v . . . . . . . . . . . . . . . . . . . . . . . . . . . . . . . . .

150

Phng php bin i ng thc . . . . . . . . . . . . . . . . . . . . . . . . . . . .

171

Phng php dng n iu hm s . . . . . . . . . . . . . . . . . . . . . . . . . .

180

Phng php h s bt nh . . . . . . . . . . . . . . . . . . . . . . . . . . . . . . .

188

Phng php t n ph tng - hiu . . . . . . . . . . . . . . . . . . . . . . . . . .

197

Phng php dng bt ng thc . . . . . . . . . . . . . . . . . . . . . . . . . . . .

203

Tng hp cc bi h phng trnh . . . . . . . . . . . . . . . . . . . . . . . . . . . .


H phng trnh hu t . . . . . . . . . . . . . . . . . . . . . . . . . . . . . . .
H phng trnh v t . . . . . . . . . . . . . . . . . . . . . . . . . . . . . . . .

216
216
236

SNG TO PHNG TRNH - H PHNG TRNH

256

Xy dng mt s phng trnh c gii bng cch a v h phng trnh

. . . .

256

S dng cng thc lng gic sng tc cc phng trnh a thc bc cao . . . .

266

S dng cc hm lng gic hyperbolic . . . . . . . . . . . . . . . . . . . . . . . . .

269

Sng tc mt s phng trnh ng cp i vi hai biu thc . . . . . . . . . . . . .

271

Xy dng phng trnh t cc ng thc. . . . . . . . . . . . . . . . . . . . . . . .

277

Xy dng phng trnh t cc h i xng loi II. . . . . . . . . . . . . . . .


Xy dng phng trnh v t da vo tnh n iu ca hm s. . . . .
Xy dng phng trnh v t da vo cc phng trnh lng gic. . . .
S dng cn bc n ca s phc sng to v gii h phng trnh. . .
S dng bt ng thc lng gic trong tam gic . . . . . . . . . . . .
S dng hm ngc sng tc mt s phng trnh, h phng trnh.

.
.
.
.
.
.

280
283
287
290
297
303

Sng tc h phng trnh . . . . . . . . . . . . . . . . . . . . . . . . . . . . . . . .

308

Kinh nghim gii mt s bi h phng trnh . . . . . . . . . . . . . . . . . . . . .

312

Ph lc 1: GII TON BNG PHNG TRNH - H PHNG TRNH

321

Ph lc 2: PHNG TRNH V CC NH TON HC NI TING

326

.
.
.
.
.
.

.
.
.
.
.
.

.
.
.
.
.
.

Lch s pht trin ca phng trnh . . . . . . .


C my cch gii phng trnh bc hai? .
Cuc thch chn ng th gii ton hc
Nhng vinh quang sau khi qua i . . .

.
.
.
.

.
.
.
.

.
.
.
.

.
.
.
.

.
.
.
.

.
.
.
.

.
.
.
.

.
.
.
.

.
.
.
.

.
.
.
.

.
.
.
.

.
.
.
.

.
.
.
.

.
.
.
.

.
.
.
.

.
.
.
.

.
.
.
.

.
.
.
.

.
.
.
.

.
.
.
.

326
326
328
332

Tu s mt s nh ton hc ni ting
Mt cuc i trn bia m . . . .
Ch v l sch qu hp! . . . . . .
Sng hay cht . . . . . . . . . . .

.
.
.
.

.
.
.
.

.
.
.
.

.
.
.
.

.
.
.
.

.
.
.
.

.
.
.
.

.
.
.
.

.
.
.
.

.
.
.
.

.
.
.
.

.
.
.
.

.
.
.
.

.
.
.
.

.
.
.
.

.
.
.
.

.
.
.
.

.
.
.
.

.
.
.
.

.
.
.
.

335
335
336
337

Ti liu tham kho

.
.
.
.

.
.
.
.

.
.
.
.

.
.
.
.

.
.
.
.

339

Li ni u
Phng trnh l mt trong nhng phn mn quan trng nht ca i s v c nhng ng
dng rt ln trong cc ngnh khoa hc. Sm c bit n t thi xa xa do nhu cu tnh
ton ca con ngi v ngy cng pht trin theo thi gian, n nay, ch xt ring trong Ton
hc, lnh vc phng trnh c nhng ci tin ng k, c v hnh thc (phng trnh hu t,
phng trnh v t, phng trnh m - logarit) v i tng (phng trnh hm, phng trnh
sai phn, phng trnh o hm ring, . . . )
Cn Vit Nam, phng trnh, t nm lp 8, l mt dng ton quen thuc v c
yu thch bi nhiu bn hc sinh. Ln n bc THPT, vi s h tr ca cc cng c gii tch
v hnh hc, nhng bi ton phng trnh - h phng trnh ngy cng c trau chut, tr
thnh nt p ca Ton hc v mt phn khng th thiu trong cc k thi Hc sinh gii, thi
i hc.
c rt nhiu bi vit v phng trnh - h phng trnh, nhng cha th cp mt
cch ton din v nhng phng php gii v sng to phng trnh. Nhn thy nhu cu c
mt ti liu y v hnh thc v ni dung cho c h chuyn v khng chuyn, Din n
MathScope tin hnh bin son quyn sch Chuyn phng trnh - h phng trnh m
chng ti hn hnh gii thiu n cc thy c gio v cc bn hc sinh.
Quyn sch ny gm 6 chng, vi cc ni dung nh sau:
> Chng I: i cng v phng hu t cung cp mt s cch gii tng qut phng
trnh bc ba v bn, ngoi ra cn cp n phng trnh phn thc v nhng cch xy dng
phng trnh hu t.
> Chng II: Phng trnh, h phng trnh c tham s cp n cc phng php
gii v bin lun bi ton c tham s ,cng nh mt s bi ton thng gp trong cc k thi
Hc sinh gii.
> Chng III: Cc phng php gii phng trnh ch yu tng hp nhng phng
php quen thuc nh bt ng thc, lng lin hp, hm s n iu, . . . vi nhiu bi ton
m rng nhm gip bn c c cch nhn tng quan v phng trnh.
Chng ny khng cp n Phng trnh lng gic, v vn ny c trong chuyn
Lng gic ca Din n.
> Chng IV: Phng trnh m logarit a ra mt s dng bi tp ng dng ca hm
s logarit, vi nhiu phng php bin i a dng nh t n ph, dng ng thc, hm n
iu, ...
> Chng V: H phng trnh l phn trng tm ca chuyn . Ni dung ca chng

4
bao gm mt s phng php gii h phng trnh v tng hp cc bi h phng trnh hay
trong nhng k thi hc sinh gii trong nc cng nh quc t.
> Chng VI: Sng to phng trnh - h phng trnh a ra nhng cch xy dng mt bi
hay v kh t nhng phng trnh n gin bng cc cng c mi nh s phc, hm hyperbolic,
hm n iu, . . .
Ngoi ra cn c hai phn Ph lc cung cp thng tin ng dng phng trnh, h phng
trnh trong gii ton v v lch s pht trin ca phng trnh.
Chng ti xin ng li cm n nhng thnh vin ca Din n chung tay xy dng
chuyn . c bit xin chn thnh cm n thy Chu Ngc Hng, thy Nguyn Trng Sn,
anh Hong Minh Qun, anh L Phc L v anh Phan c Minh v h tr v ng gp
nhng kin qu gi cho chuyn .
Nim hi vng duy nht ca nhng ngi lm chuyn l bn c s tm thy nhiu iu
b ch v tnh yu ton hc thng qua quyn sch ny. Chng ti xin n nhn v hoan nghnh
mi kin xy dng ca bn c chuyn c hon thin hn. Mi gp xin vui lng
chuyn n anhhuy0706@gmail.com

Thnh ph H Ch Minh, ngy 11 thng 7 nm 2012


Thay mt nhm bin son
Nguyn Anh Huy

Cc thnh vin tham gia chuyn


hon thnh c cc ni dung trn, chnh l nh s c gng n lc ca cc thnh vin ca
din n tham gia xy dng chuyn :

Ch bin: Nguyn Anh Huy (10CT THPT chuyn L Hng Phong - TP HCM)
Ph trch chuyn : Nguyn Anh Huy (10CT THPT chuyn L Hng Phong - TP HCM),
Nguyn An Vnh Phc (TN Ph thng Nng khiu- TP HCM)
i cng v phng trnh hu t: Hunh Phc Trng (THPT Nguyn Thng Hin
TP HCM), Phm Tin Kha (10CT THPT chuyn L Hng Phong - TP HCM)
Phng trnh, h phng trnh c tham s: thy Nguyn Trng Sn (THPT Yn M A
Ninh Bnh), V Trng Hi (12A6 THPT Thi Phin - Hi Phng), nh V Bo Chu
(THPT chuyn L Qu n - Vng Tu), Hong B Minh ( 12A6 THPT chuyn Trn
i Ngha - TP HCM), Nguyn Hong Nam (THPT Phc Thin - ng Nai), Ong Th
Phng (11 Ton THPT chuyn Lng Th Vinh - ng Nai)
Phng php t n ph: thy Mai Ngc Thi (THPT Hng Vng - Bnh Phc), thy
Nguyn Anh Tun (THPT L Qung Ch -H Tnh), Trn Tr Quc (11TL8 THPT
Nguyn Hu - Ph Yn), H c Khnh (10CT THPT chuyn Qung Bnh), on Th
Ho (10A7 THPT Long Khnh - ng Nai)
Phng php dng lng lin hp: Ninh Vn T (THPT chuyn Trn i Ngha TPHCM) , inh V Bo Chu (THPT - chuyn L Qu n, Vng Tu), on Th
Ha (THPT Long Khnh - ng Nai)
Phng php dng bt ng thc: Nguyn An Vnh Phc (TN Ph thng Nng khiuTP HCM), Phan Minh Nht, L Hong c (10CT THPT chuyn L Hng Phong - TP
HCM), ng Hong Phi Long (10A10 THPT Kim Lin H Ni), Nguyn Vn Bnh
(11A5 THPT Trn Quc Tun - Qung Ngi),
Phng php dng n iu: Nguyn Anh Huy (10CT THPT chuyn L Hng Phong
- TP HCM), Hong Kim Qun (THPT Hng Thi H Ni), ng Hong Phi Long
(10A10 THPT Kim Lin H Ni)
Phng trnh m logarit: V Anh Khoa, Nguyn Thanh Hoi (i hc KHTN- TP
HCM), Nguyn Ngc Duy (11 Ton THPT chuyn Lng Th Vinh - ng Nai)
Cc loi h c bn: Nguyn Anh Huy (10CT THPT chuyn L Hng Phong - TP HCM)

6
H phng trnh hon v: thy Nguyn Trng Sn (THPT Yn M A Ninh Bnh),
Nguyn Anh Huy (10CT THPT chuyn L Hng Phong TP HCM), Nguyn nh Hong
(10A10 THPT Kim Lin - H Ni)
Phng php bin i ng thc: Nguyn nh Hong (10A10 THPT Kim Lin - H
Ni), Trn Vn Lm (THPT L Hng Phong - Thi Nguyn), Nguyn c Hunh (11
Ton THPT Nguyn Th Minh Khai - TP HCM)
Phng php h s bt nh: L Phc L (i hc FPT TP HCM), Nguyn Anh Huy,
Phan Minh Nht (10CT THPT chuyn L Hng Phong TP HCM)
Phng php t n ph tng - hiu: Nguyn Anh Huy (10CT THPT chuyn L Hng
Phong TP HCM)
Tng hp cc bi h phng trnh: Nguyn Anh Huy (10CT THPT chuyn L Hng Phong
TP HCM), Nguyn Thnh Thi (THPT chuyn Nguyn Quang Diu ng Thp), Trn
Minh c (T1K21 THPT chuyn H Tnh H Tnh), V Hu Thng (11 Ton THPT
Nguyn Th Minh Khai TP HCM)
Sng to phng trnh: thy Nguyn Ti Chung (THPT chuyn Hng Vng Gia Lai),
thy Nguyn Tt Thu (THPT L Hng Phong - ng Nai), Nguyn L Thu Linh (10CT
THPT chuyn L Hng Phong TP HCM)
Gii ton bng cch lp phng trnh: Nguyn An Vnh Phc (TN Ph thng Nng khiuTP HCM) Lch s pht trin ca phng trnh: Nguyn An Vnh Phc (TN Ph thng
Nng khiu- TP HCM), Nguyn Hong Nam (THPT Phc Thin - ng Nai)

Chng I: I CNG V PHNG TRNH


HU T

PHNG TRNH BC BA
Mt s phng php gii phng trnh bc ba
F Phng php phn tch nhn t:
Nu phng trnh bc ba ax3 + bx2 + cx + d = 0 c nghim x = r th c nhn t (x r) do
c th phn tch
ax3 + bx2 + cx + d = (x r)[ax2 + (b + ar)x + c + br + ar2 ]
T ta a v gii mt phng trnh bc hai, c nghim l

b ra b2 4ac 2abr 3a2 r2


2a
F Phng php Cardano:
Xt phng trnh bc ba x3 + ax2 + bx + c = 0 (1).
a
Bng cch t x = y , phng trnh (1) lun bin i c v dng chnh tc:
3
y 3 + py + q = 0(2)
a2
2a3 9ab
,q = c +
3
27
Ta ch xt p, q 6= 0 v p = 0 hay q = 0 th a v trng hp n gin. t y = u + v. Thay
vo (2), ta c:
(u + v)3 + p(u + v) + q = 0 u3 + v 3 + (3uv + p)(u + v) + q = 0 (3)
Chn u, v sao cho 3uv + p = 0 (4).
Nh vy, tm u v v, t (3) v (4) ta c h phng trnh:

u3 + v 3 = q
3
u3 v 3 = p
27
Trong : p = b

Theo nh l Viete, u3 v v 3 l hai nghim ca phng trnh:


X 3 + qX

t =

q 2 p3
+
4
27
7

p3
= 0(5)
27

8
> Khi > 0, (5) c nghim:
q
q
u3 = + , v 3 =
2
2
Nh vy, phng trnh (2) s c nghim thc duy nht:
r
r
q
q
3
y = + + 3
2
2
r
q
> Khi = 0, (5) c nghim kp: u = v = 3
2
Khi , phng trnh (2) c hai nghim thc, trong mt nghim kp.
r
y1 = 2

q
, y2 = y3 =
2

r
3

q
2

> Khi < 0, (5) c nghim phc.


p
Gi u30 l mt nghim phc ca (5), v03 l gi tr tng ng sao cho u0 v0 = .
3
Khi , phng trnh (2) c ba nghim phn bit.
y1 = u0 + v0

3
1
(u0 v0 )
y2 = (u0 + v0 ) + i
2
2
1
3
y3 = (u0 + v0 ) i
(u0 v0 )
2
2
F Phng php lng gic ho - hm hyperbolic:
Mt phng trnh bc 3, nu c 3 nghim thc, khi biu din di dng cn thc s lin quan
n s phc.V vy ta thng dng phng php lng gic ho tm mt cch biu din
khc n gin hn, da trn hai hm s cos v arccos
C th, t phng trnh t2 + pt + q = 0 () ta t t = u cos v tm u c th a () v
dng
4 cos3 3 cos cos 3 = 0
r
p
u3
Mun vy, ta chn u = 2
v chia 2 v ca () cho
c
3
4
r
r
3q
3
3q
3
3
4 cos 3 cos .
= 0 cos 3 = .
2p
p
2p
p
Vy 3 nghim thc l
r
r


p
1
3q
3
2i
ti = 2
. cos
arccos( .
)
vi i = 0, 1, 2.
3
3
2p
p
3
Lu rng nu phng trnh c 3 nghim thc th p < 0 (iu ngc li khng ng) nn cng
thc trn khng c s phc.
Khi phng trnh ch c 1 nghim thc v p 6= 0 ta cng c th biu din nghim bng cng
thc hm arcosh v arsinh:
2|q|
>t =
.
q

r


p
1
3|q| 3
cosh .arcosh(
.
) nu p < 0 v 4p3 + 27q 2 > 0.
3
3
2p
p

9
r
r


p
1
3q
3
>t = 2
. sinh .arsinh( .
) nu p > 0
3
3
2p
p
Mi phng php trn u c th gii quyt phng trnh bc 3 tng qut. Nhng mc ch
ca chng ta trong mi bi ton lun l tm li gii ngn nht, p nht. Hy cng xem qua
mt s bi tp v d:

Bi tp v d
Bi 1: Gii phng trnh x3 + x2 + x =

1
3
Gii

Phng trnh khng c nghim hu t nn khng th phn tch nhn t. Trc khi ngh ti
cng thc Cardano, ta th quy ng phng trnh:
3x3 + 3x2 + 3x + 1 = 0
i lng 3x2 +3x+1 gi ta n mt hng ng thc rt quen thuc x3 +3x2 +3x+1 = (x+1)3 .
Do phng trnh tng ng:
(x + 1)3 = 2x3
hay

x + 1 = 3 2x
1
.
T suy ra nghim duy nht x =
1+ 32
~ Nhn xt: V d trn l mt phng trnh bc ba c nghim v t, v c gii nh kho lo
bin i ng thc. Nhng nhng bi n gin nh th ny khng c nhiu. Sau y ta s i
su vo cng thc Cardano:
Bi 2: Gii phung trnh x3 3x2 + 4x + 11 = 0

Gii
u tin phi kh bc 2. t x = y + 1 . Th vo phng trnh u bi, ta c phng trnh:
y 3 + 1.y + 13 = 0
4567
4
>0
Tnh = 132 + .13 =
27
27
p dng cng thc Cardano suy
v ra:
v
r
r
u
u
u
u
4567
4567
u
u
3
3 13
t 13 +
t
27
27
y=
+
2
2
v
v
r
r
u
u
u
4567 u
4567
u
u
3 13 +
3 13
t
t
27
27
Suy ra x =
+
+ 1.
2
2
~ Nhn xt: V d trn l mt ng dng c bn ca cng thc Cardano. Tuy nhin cng
thc ny khng h d nh v ch c dng trong cc k thi Hc sinh gii. V th, c l chng
ta s c gng tm mt con ng "hp thc ha" cc li gii trn. l phng php lng
gic ho.

10
Bi 3: Gii phng trnh x3 + 3x2 + 2x 1 = 0

Gii
V mt hnh thc, bi 3 khng khc g bi 2. t x = y 1. Phng trnh tng ng:
y 3 y 1 = 0 (1)
T y s dng cng thc Cardano, ta s c ngay kt qu. Nhng xin nhc li rng, ta ang i
tm mt con 
ng 
khc, sng to hn v cng d chp nhn hn. Hy th cch t sau y:
1
1
(). Th vo phng trnh (1), c phng trnh tng ng:
t y = t +
t
3
t3
1
+
1=0
3 3 3 3t3
Sau khi quy ng, ta s c mt phng trnh bc hai n t3 ! Vic gii phng trnh ny khng
c g kh khn, xin dnh li cho bn c. Ta tm c nghim:


1
1 3 1
1 2
r
3
3

23
+
x=


3 2
3 1
3 3 23
2
~ Nhn xt: Cu hi t ra l: "S dng phng php trn nh th no?". Mun tr li, ta
cn lm sng t 2 vn :
1) C lun tn ti t tho mn cch t trn?"
2
p n l khng. Coi () l phng trnh bc hai theo t ta s tm c iu kin |y| >
3
1
1
1
1
2
(tht ra c th tm nhanh bng cch dng AM-GM:|y| = | (t + )| = (|t| + ) > ).
t
|t|
3
3
3
2
Vy trc ht ta phi chng minh (1) khng c nghim |y| < .
3



3
2
3
y < 1. Do tn ti [0, ] sao cho
y = cos .
Nu |y| < suy ra
2
2
3
r

Phng trnh tng ng:


hay
2
Do |y| > .
3

2
8
cos3 cos 1 = 0
3 3
3

3 3
cos 3 =
(v nghim!)
2

2
2) V sao c s ?
3
tng ca ta l t phng trnh x3 + ax + b = 0 a v mt phng trnh
rtrng phng theo
1
p
t3 qua cch t x = k(t + ). Khai trin v ng nht h s ta c k =
t
3
Nh vy cch t trn c th gii quyt nhng phng trnh bc 3 c nghim duy nht. Ta
cng tip tc khai thc phng php lng gic ho i vi phng trnh c 3 nghim thc:
Bi 4: Gii phng trnh:x3 x2 2x + 1 = 0

Gii

11
1
t y = x . Phng trnh tng ng:
3
7
7
y3 y +
= 0()
3
27

Tuy nhin, phng trnh ban u ca chng ta c ti ba nghim phn bit, trong khi cc
cch t nu v d trc ch tm c mt nghim duy nht. Do , ta s "lng gic ha"
phng trnh
().


3y
7 cos
2 7
3y
2
Vi |y| <
th < 1. Do tn ti [0, ] sao cho cos = hay y =
.
3
3
2 7
2 7
Th vo (), ta c:

7
cos 3 =
14
y l phng trnh lng gic c bn. D dng tm c ba nghim ca phng trnh ban
u:

!
7

arccos 14

1
2 7
+
cos
x1 =

3
3
3

!
7

arccos 14

1
2
2 7
+
cos
+
=

3
3
3

x2,3

Do phng
trnh bc ba c ti a ba nghim phn bit nn ta khng cn quan tm trng hp
2 7
|y| >
. Bi ton c gii quyt.
3

2 7
bng cch t
~ Nhn xt: Ta cng c th chng minh phng trnh v nghim khi |y| >
3

7
1
y=
(t + ) ging nh bi 3, t dn ti mt phng trnh trng phng v nghim.
3
t
r
p
1
Tng kt li, ta dng php t n ph y =
(t + ) () nh sau:
3
t
r
p
> Nu phng trnh c 1 nghim thc, chng minh phng trnh v nghim khi |y| < 2
,
3
trng hp cn li dng () a v phng trnh trng phng.
r
p
> Nu phng trnh c 3 nghim thc, chng minh phng trnh v nghim khi |y| > 2
3
r
p
bng php t () (a v phng trnh trng phng v nghim theo t). Khi |y| 6 2
th
3
|y|
t r
= cos , t tm c 3 nghim.
p
2
3
Cch t trn dng khi p < 0, cn khi p > 0 khng kh chng minh phng trnh c nghim
duy nht.
Bi 5: Gii phng trnh x3 + 6x + 4 = 0

Gii

12
1
~ tng: Ta s dng php t x = k(t ) a v phng trnh trng phng.
t
php t ny khng cn iu kin ca x, v n tng ng k(t2 1) xt = 0. Phng trnh
ny lun c nghim theo t.
Nh vy t phng trnh u ta c
1
1
1
) 3k 3 (t ) + 5k(t ) + 4 = 0
3
t
t
t

Cn chn k tho 3k 3 = 5k k = 2
Vy ta c li gii bi ton nh sau:
~ Li gii:

1
t x = 2(t ) ta c phng trnh
t
s

3 1

3
3 1
3
6

2 2(t 3 ) + 4 = 0 t 1 + 2t = 0 t1,2 =
t
2
k 3 (t3

Lu rng t1s
.t2 = 1 theo nh
nn ta ch nhn c mt gi tr ca x l
s l Viete

1 + 3
1 3

+ 3
)2
t1 + t2 = 2( 3
2
2
Bi 6: Gii phng trnh 4x3 3x = m (1) vi |m| > 1

Gii
Nhn 
xt rng
 khi |x| 6 1 th |V T | 6 1 < |m| (sai) nn |x| > 1. V vy ta c th t
1
1
x=
t+
.
2
t
Ta c phng trnh tng ng:


1 3 1
t + 3 =m
2
t
T :

p
p

1 p
3
3
3
t = m m2 1 x =
m + m2 1 + m m2 1 .
2
Ta chng minh y l nghim duy nht.
Gi s phng trnh c nghim x0 th x0 6 [1, 1] v |x0 | > 1. Khi :
4x3 3x = 4x30 3x0
hay
(x x0 )(4x2 + 4xx0 + 4x20 3) = 0
Xt phng trnh:
4x2 + 4xx0 + 4x20 3 = 0
c 0 = 12 12x20 < 0 nn phng trnh bc hai ny v nghim.
Vy phng trnh u bi c nghim duy nht l

p

1 p
3
3
2
2
m + m 1 + m m 1 .
x=
2

13

Bi tp t luyn
Bi 1:Gii cc phng trnh sau:
a) x3 + 2x2 + 3x + 1 = 0
b) 2x3 + 5x2 + 4x + 2 = 0
c) x3 5x2 + 4x + 1 = 0

d) 8x3 + 24x2 + 6x 10 3 6 = 0
Bi 2: Gii v bin lun phng trnh:
4x3 + 3x = m vi m R
Bi 3: Gii v bin lun phng trnh:
x3 + ax2 + bx + c = 0

PHNG TRNH BC BN
Phng trnh dng ax4 + bx3 + cx2 + bkx + ak 2 = 0 (1)
Ta c:
(1) a(x4 + 2x2 .k + k 2 ) + bx(x2 + k) + (c 2ak)x2 = 0
a(x2 + k)2 + bx(x2 + k) + (c 2ak)x2 = 0
n y c hai hng gii quyt:
Cch 1: a phng trnh v dng A2 = B 2 :
Thm bt, bin i v tri thnh dng hng ng thc dng bnh phng ca mt tng, chuyn
cc hng t cha x2 sang bn phi.
Cch 2: t y = x2 + k y > k
Phng trnh (1) tr thnh ay 2 + bxy + (c 2ak)x2 = 0
Tnh x theo y hoc y theo x a v phng trnh bc hai theo n x.
V d: Gii phng trnh: x4 8x3 + 21x2 24x + 9 = 0 (1.1)
Cch 1:
(1.1) (x4 + 9 + 6x2 ) 8(x2 + 3) + 16x2 = 16x2 21x2 + 6x2 (x2 4x + 3)2 = x2

"
"
5 13
x2 4x + 3 = x
x2 5x + 3 = 0
x=
2

5 + 13
x2 4x + 3 = x
x2 3x + 3 = 0
x=
2
Cch 2:
(1.1) (x4 + 6x2 + 9) 8x(x2 + 3) + 15x2 = 0 (x2 + 3)2 8x(x2 + 3) + 15x2 "
=0
y = 3x
t y = x2 + 3. (1.1) tr thnh: y 2 8xy + 15x2 = 0 (y 3x)(y 5x) = 0
y = 5x
2
Vi y = 3x: Ta c x + 3 = 3x: Phng trnh v nghim

5 13
x=
2
Vi y = 5x: Ta c x2 + 3 = 5x x2 5x + 3 = 0
5 + 13
x=
(

2)
5 + 13 5 13
Vy phng trnh (1.1) c tp nghim: S =
;
2
2

14

Bi tp t luyn
Gii cc phng trnh sau:
1) x4 13x3 + 46x2 39x + 9 = 0
2) 2x4 + 3x3 27x2 + 6x + 8 = 0
3) x4 3x3 6x2 + 3x + 1 = 0
4) 6x4 + 7x3 36x2 7x + 6 = 0
5) x4 3x3 9x2 27x + 81 = 0
Nhn xt: Mi phng php gii c li th ring. Vi cch gii 1, ta s tnh c trc tip m
khng phi thng qua n ph. Vi cch gii 2, ta s c nhng tnh ton n gin hn v t b
nhm ln.
Phng trnh dng (x + a)(x + b)(x + c)(x + d) = ex2 (2) vi ad = bc = m:
Cch 1: a v dng A2 = B 2
(2) (x + px + m)(x2 + nx + m) = ex2 (ad = bc = m, p = a + d, n = b + c)
!
!
p
+
n
n

p
n

p
p
+
n
x2 +
x+m
x
x+m+
x = ex2
x2 +
2
2
2
2
!2 "
#
 n p 2
p
+
n
x2 +
x+m =
+ e x2
2
2
Cch 2: Xt xem x = 0 c phi l nghim ca phng trnh khng.
Trng hp x 6= 0:

(2)

m
x+
+p
x

!
m
x+
+n =e
x

p
m
. iu kin: |u| > 2 |m|
x
(2) tr thnh (u + p)(u + n) = e Gii phng trnh bc hai theo u tm x.
t u = x +

V d: Gii phng trnh: (x + 4)(x + 6)(x 2)(x 12) = 25x2 (2.1)


Cch 1:
(2.1) (x2 + 10x + 24)(x2 14x + 24) = 25x2
(x2 2x + 24 + 12x2 )(x2 2x + 24 12x2 ) = 25x2
"
x2 2x + 24 = 13x
(x2 2x + 24)2 = 169x2
x2 2x + 24 = 13x

x = 8
"
x = 3
x2 15x + 24 = 0

x2 + 11x + 24 = 0
15 129
x=
2
Cch 2:
(2.1) (x2 + 10x + 24)(x2 14x + 24) = 25x2
Nhn thy x = 0 khng phi 
l 
nghim ca phng
trnh.

24
24
x 6= 0 : (2.1) x +
+ 10
x+
14 = 25
x
x

15
t y = x +

24
|y| > 4 6. (2.1) tr thnh:
x

(y + 10)(y 14) = 25 (y + 11)(y 15) = 0

"

y = 11
y = 15

Vi y = 11: Ta c phng trnh:


"
x = 3
24
= 11 x2 + 11x + 24 = 0
x+
x
x = 8
Vi y = 15: Ta c phng trnh:

15

129
24
= 15 x2 15x + 24 = 0 x =
x+
x
2
(
)

15 129 15 + 129
Phng trnh (2.1) c tp nghim S = 3; 8;
;
2
2

Bi tp t luyn
Gii cc phng trnh sau:
1) 4(x + 5)(x + 6)(x + 10)(x + 12) = 3x2
2) (x + 1)(x + 2)(x + 3)(x + 6) = 168x2
3) (x + 3)(x + 2)(x + 4)(x + 6) = 14x2
4) (x + 6)(x + 8)(x + 9)(x + 12) = 2x2
19
5) 18(x + 1)(x + 2)(x + 5)(2x + 5) = x2
4
Nhn xt: Trong cch gii 2, c th ta khng cn xt x 6= 0 ri chia xung m c th t n
ph y = x2 + m thu c phng trnh bc hai n x, tham s y hoc ngc li.
Phng trnh dng (x + a)(x + b)(x + c)(x + d) = m (3) vi a + b = c + d = p:
Ta c (3) (x2 + px + ab)(x2 + px + cd) = m
Cch 1:
!
!
ab
+
cd
ab

cd
ab
+
cd
ab

cd
(3) x2 + px +
+
x2 + px +

=m
2
2
2
2
!2
!2
ab
+
cd
ab

cd
=m+
x2 + px +
2
2
Cch 2:
p2
t y = x2 + px iu kin: y > . (3) tr thnh:(y + ab)(y + cd) = m
4
Gii phng trnh bc 2 n y tm x.
V d: Gii phng trnh: x(x + 1)(x + 2)(x + 3) = 8 (3.1)
Cch 1:
Ta c
(3.1) (x2 + 3x)(x2 + 3x + 2) = 8
(x2 + 3x + 1 1)(x2 + 3x + 1 + 1) = 8
"
x2 + 3x + 1 = 3
(x2 + 3x + 1)2 = 9
x2 + 3x + 1 = 3
"

x2 + 3x 2 = 0
3 17

x=
2
x2 + 3x + 4 = 0
Cch 2:

16
(3.1) (x2 + 3x)(x2 + 3x + 2) = 8
9
t y = x2 + 3x y >
4
(3.1) tr thnh:
"
y(y + 2) = 8 y 2 + 2y 8 = 0

y=2
y=2
y = 4(loi)

Vi y = 2: Ta c phng trnh:

3 17
x + 3x 2 = 0 x =
2
(
)

3 + 17 3 17
;
Phng trnh (3.1) c tp nghim: S =
2
2
2

Bi tp t luyn
Gii cc phng trnh sau:
1. (x + 2)(x + 3)(x 7)(x 8) = 144
2. (x + 5)(x + 6)(x + 8)(x + 9) = 40

1 
3 
1 
4  39879
3. x +
x+
x+
x+
=
4
5
20
5
40000
2
4. (6x + 5) (3x + 2)(x + 1) = 35
5. (4x + 3)2 (x + 1)(2x + 1) = 810
Nhn xt: Nh dng (2), ngoi cch t n ph trn, ta c th t mt trong cc dng
n ph sau:
> t y = x2 + px + ab
2
> t y = x
+ cd
 + px
p 2
> t y = x +
2
ab + cd
2
> t y = x + px +
2
Phng trnh dng (x + a)4 + (x + b)4 = c(c > 0) (4):
a+b
t x = y
. (4) tr thnh:
2

4 
4
ab
ab
y+
+ y
=c
2
2
S dng khai trin nh thc bc 4, ta thu c phng trnh:
4

2 2

2y + 3(a b) y + 2

ab
2

4
=c

Gii phng trnh trng phng n y tm x.


V d: Gii phng trnh: (x + 2)4 + (x + 4)4 = 82 (4.1)
t y = x + 3. Phng trnh (4.1) tr thnh:
(y + 1)4 + (y 1)4 = 82
(y 4 + 4y 3 + 6y 2 + 4y + 1) + (y 4 4y 3 + 6y 2 4y + 1) = 82

17
2y 4 + 12y 2 80 = 0 (y 2 4)(y 2 + 10) = 0
y 2 = 4 y = 2
Vi y = 2, ta c x = 1
Vi y = 2, ta c x = 5
Vy phng trnh c tp nghim: S = {1; 5}

Bi tp t luyn
Gii cc phng trnh sau:
1. (x + 2)4 + (x + 8)4 = 272

2. (x + 2)4 + (x + 1)4 = 33 + 12 2
3. (x + 10)4 + (x 4)4 = 28562
4. (x + 1)4 + (x 3)4 = 90
Phng trnh dng: x4 = ax2 + bx + c (5):
a (5) v dng A2 = B 2 :
(5) (x2 + m)2 = (2m + a)x2 + bx + c + m2
Trong , m l mt s cn tm.
Tm m f (x) = (2m + a)x2 + bx + c + m2 c = 0. Khi , f (x) c dng bnh phng ca
mt biu thc.

x 2 + m = 0
Nu 2m + a < 0 : (5) (x2 + m)2 + g 2 (x) = 0[f (x) = g 2 (x)]
g(x) = 0
"
x2 + m = g(x)
Nu2m + a > 0 : (5) (x2 + m)2 = g 2 (x)[f (x) = g(x)2 ]
x2 + m = g(x)
V d: Gii phng trnh: x4 + x2 6x + 1 = 0 (5.1)
Ta c:
(5.1) x4 + 4x2 + 4 = 3x2 + 6x + 3 (x2 + 2)2 = 3(x + 1)2
"

x2 + 2 = 3(x + 1)

x2 + 2 = 3(x + 1)
"

x2 3x + 2 3 = 0

x2 + 3 + 2 + 3 = 0

3 4 35
x=
p2

3+ 4 35
x=
2
(
)
p
p

3 4 35 3+ 4 35
Phng trnh (5.1) c tp nghim: S =
;
2
2

Bi tp t luyn
Gii cc phng trnh sau:
1. x4 19x2 10x + 8 = 0
2. x4 = 4x + 1

18
3. x4 = 8x + 7
4. 2x4 + 3x2 10x + 3 = 0
5. (x2 16)2 = 16x + 1
6. 3x4 2x2 16x 5 = 0
Nhn xt: Phng trnh dng x4 = ax + b c gii theo cch tng t.
Phng trnh = 0 l phng trnh bc ba vi cch gii c trnh by trc. Phng
trnh ny c th cho 3 nghim m, cn la chn m sao cho vic tnh ton l thun li nht. Tuy
nhin, d dng nghim m no th cng cho cng mt kt qu.
Phng trnh dng af 2 (x) + bf (x)g(x) + cg 2 (x) = 0 (6):
Cch 1: Xt g(x) = 0, gii tm nghim v th li vo (6).
2

f (x)
f (x)
+ b.
+c=0
Trng hp g(x) 6= 0: a
g(x)
g(x)
f (x)
t y =
, gii phng trnh bc hai ay 2 + by + c = 0 ri tm x.
g(x)
Cch 2: t u = f (x), v = g(x), phng trnh tr thnh:
au2 + buv + cv 2 = 0(6)
Xem (6) l phng trnh bc hai theo n u, tham s v. T tnh u theo v.
V d: Gii phng trnh: 20(x 2)2 5(x + 1)2 + 48(x 2)(x + 1) = 0 (6.1)
t u = x 2, v = x + 1. Phng trnh (6.1) tr thnh:
20u2 + 48uv 5v 2 = 0 (10u v)(2u + 5v) = 0
10u = v

2u = 5v
7
Vi 10u = v, ta c: 10(x 2) = x + 1 x =
3
1
Vi 2u = 5v, ta c: 2(x 2) = 5(x + 1) x =

7
7 1
Vy phng trnh (6.1) c tp nghim: S =
;
3 7
f (x)
Nhn xt: Nu chn y =
Vi f (x) v g(x) l hai hm s bt k (g(x) 6= 0), ta s to
g(x)
c mt phng trnh. Khng ch l phng trnh hu t, m cn l phng trnh v t.

Bi tp t luyn
Gii cc phng trnh sau:
1. (x 5)4 12(x 2)4 + 4(x2 7x + 10)2 = 0
2. (x 2)4 + 3(x + 3)4 4(x2 + x 6)2 = 0
3. 4(x3 1) + 2(x2 + x + 1)2 4(x 1)2 = 0
4. 2(x2 x + 1)2 + 5(x + 1)2 + 14(x3 + 1) = 0
5. (x 10)4 15(x + 5)4 + 4(x2 5x 50)2 = 0

Phng trnh bc 4 tng qut ax4 + bx3 + cx2 + dx + e = 0 (7)


Phn tch cc hng t bc 4, 3, 2 thnh bnh phng ng, cc hng t cn li chuyn sang v
phi.

19
(7) 4a2 x4 + 4bax3 + 4cax2 + 4dax + 4ae = 0
(2ax2 + bx)2 = (b2 4ac)x2 4adx 4ae
Thm vo hai v mt biu thc 2(2ax2 + bx)y + y 2 (y l hng s) v tri thnh bnh phng
ng, cn v phi l tam thc bc hai theo x:
f (x) = (b2 4ac 4ay)x2 + 2(by 2ad)x 4ae + y 2
Tnh y sao cho v phi l mt bnh phng ng. Nh vy, ca v phi bng 0. Nh vy ta
phi gii phng trnh = 0. T ta c dng phng trnh A2 = B 2 quen thuc.
V d: Gii phng trnh: x4 16x3 + 66x2 16x 55 = 0 (7.1)
(7.1) x4 16x3 + 64x2 = 2x2 + 16x + 55
(x2 8x)2 + 2y(x2 8x) + y 2 = (2y 2)x2 + (16 16y)x + 55 + y 2
Gii phng trnh = 0 (8 8y)2 (55 + y 2 )(2y 2) = 0 tm c y = 1, y = 3, y = 29.
Trong cc gi tr ny, ta thy gi tr y = 3 l thun li nht cho vic tnh ton.
Nh vy, chn y = 3, ta c phng
trnh:
"
2
x 8x + 3 = 2(x 4)
(x2 8x + 3)2 = 4(x 4)2
x2 8x + 3 = 2(x 4)
"
"

x2 10x + 11 = 0
x = 3 14

x = 5 14
x2 6x 5 = 0



Phng trnh (7.1) c tp nghim S = 3 + 14; 3 14; 5 + 14; 5 14
Nhn xt: V d trn cho ta thy phng trnh = 0 c nhiu nghim. C th chn y = 1
nhng t ta c phng trnh (x2 8x + 1)2 = 56 th khng thun li lm cho vic tnh ton,
tuy nhin, kt qu vn nh nhau.
b
Mt cch gii khc l t phng trnh x4 + ax3 + bx2 + cx + d = 0 t x = t + , ta s thu c
4a
phng trnh khuyt bc ba theo t, ngha l bi ton quy v gii phng trnh t4 = at2 + bt + c.

Bi tp t luyn
1.
2.
3.
4.
5.

x4 14x3 + 54x2 38x 11 = 0


x4 16x3 + 57x2 52x 35 = 0
x4 6x3 + 9x2 + 2x 7 = 0
x4 10x3 + 29x2 20x 8 = 0
2x4 32x3 + 127x2 + 38x 243 = 0

PHNG TRNH DNG PHN THC


Phng trnh cha n mu c bn:
t iu kin xc nh cho biu thc mu. Quy ng ri gii phng trnh.
1
x
V d: Gii phng trnh:
+
= 2 (1.1)
2 x 2x 1
1
iu kin: x 6= 2; x 6= .
2

20
(1.1)

2x 1 + x(2 x)
= 2 2x 1 + 2x x2 = 2(4x 2 2x2 + x)
(2 x)(2x 1)

3x2 6x + 3 = 0 x = 1(tha iu kin)


Vy phng trnh (1.1) c tp nghim S = {1}
a2 x 2
Phng trnh dng x2 +
= b (2):
(x + a)2

2
ax
ax
(2) x
+ 2x.
=b
(x + a)
x+a
 2 2
x
x2

+ 2a.
+ a2 = b + a2
x+a
x+a
x2
t y =
. Gii phng trnh bc hai theo y tm x.
x+a
9x2
V d: Gii phng trnh: x2 +
= 7 (2.1)
(x + 3)2
iu kin:
 x 6= 3. 2
3x
x2
(2.1) x
+ 6.
=7
x+3
x+3
 2 2
x
x2

=7
+ 6.
x+3
x+3
x2
. Ta c phng trnh:
t y =
x+3
"
y=1
y 2 + 6y 7 = 0
y = 7
Vi y = 1: Ta c phng
trnh:
1

13
x2 = x + 3 x =
2
Vi y = 7: Ta c phng trnh:
x2 + 7x + 21 = 0 (v nghim)
(

)
1 + 13 1 13
Vy phng trnh (2.1) c tp nghim: S =
;
2
2
Nhn xt: Da vo cch gii trn, ta c th khng cn phi t n ph m thm bt hng s
to dng phng trnh quen thuc A2 = B 2

Bi tp t luyn
Gii cc phng trnh sau:
4x2
1. x2 +
= 12
(x + 2)2
25x2
2. x2 +
= 11
(x + 5)2
9x2
3. x2 +
= 14
(x 3)2
25
49
4. 2
=1
x
(x 7)2
9
8
5.
+1=
2
4(x + 4)
(2x + 5)2

21

x2 + nx + a
x2 + qx + a
Phng trnh dng 2
+
= b (3):
x + mx + a x2 + px + a

x2 + mx + a 6= 0
iu kin:
x2 + px + a 6= 0
Xt xem x = 0 c phi l nghim phng trnh khng.
Trng hp x 6= 0:
a
a
x+ +n
x+ +q
x
x
(2)
+
=b
a
a
x+ +m x+ +p
x
x
p
a
t y = x + . iu kin: |y| > 2 |a|
x
y+n
y+q
(3) tr thnh
+
=b
y+m y+p
Gii phng trnh n y sau tm x.
1
x2 3x + 5 x2 5x + 5
2
= (3.1)
V d: Gii phng trnh: 2
x 4x + 5 x 6x + 5
4
iu kin: x 6= 1, x 6= 5.
x = 0 khng phi l nghim ca phng trnh.
Xt x 6= 0
5
5
x+ 3 x+ 5
1
x
x
(3.1)

=
5
5
4
x+ 4 x+ 6
x
x

5
t y = x + |y| > 2 5, y 6= 6.
x
y3 y5
1
Phng trnh (3.1) tr thnh:

=
y4 y6 " 4
y=2
2
1
2
= y 2 10y + 16 = 0
y 10y + 24
4
y=8
Vi y = 8: Ta c phng trnh:

5
x + = 8 x2 8x + 5 = 0 x = 4 11
x
Vi y = 2: Loi do khng tha iu kin.



Vy phng trnh (3.1) c tp nghim: S = 4 + 11; 4 11
Nhn xt: Cc dng phng trnh sau c gii mt cch tng t:
nx
mx
~ Dng 1: 2
+ 2
=p
ax + bx + d ax + cx + d
ax2 + mx + c
px
~ Dng 2:
+ 2
=b
2
ax + nx + c
ax + qx + c

Bi tp t luyn
Gii cc phng trnh sau:
4x
3x
1) 2
+ 2
=1
4x 8x + 7 4x 10x + 7
2x
13x
2) 2
+ 2
=6
2x 5x + 3 2x + x + 3
3x
7x
3) 2
+ 2
= 4
x 3x + 1 x + x + 1
x2 10x + 15
4x
4) 2
= 2
x 6x + 15
x 12x + 5

22
x2 + 5x + 3 x2 + 4x + 3
+
=7
x2 7x + 3 x2 + 5x + 3
Tng kt
Qua cc dng phng trnh trn, ta thy phng trnh hu t thng c gii bng mt trong
cc phng php:
[1.] a v phng trnh tch
[2.] t n ph hon ton
[3.] t n ph a v h phng trnh
[4.] a v ly tha ng bc (thng l dng A2 = B 2 )
[5.] Chia t v mu cho cng mt s
[6.] Thm bt to thnh bnh phng ng
Tuy nhin, c mt s dng phng trnh c nhng phng php gii c trng. Nhng phng
trnh ny s c trnh by c th hn nhng phn khc.

5)

Bi tp tng hp phn phng trnh hu t


Phn 1:
1) x3 3x2 + 18x 36 = 0
1
2) 8x2 6x =
2
3) x3 4x2 4x + 8 = 0
4) x3 21x2 + 35x 7 = 0
5) x3 6x2 + 8 = 0
Phn 2:
1) 6x5 11x4 11x + 6 = 0
2) (x2 6x)2 2(x 3)2 = 81
3) x4 + (x 1)(3x2 + 2x 2) = 0
4) x4 + (x + 1)(5x2 6x 6) = 0
5) x5 + x2 + 2x + 2 = 0
6) (x2 16)2 = 16x + 1
7) (x + 2)2 + (x 
+ 3)3 +
(x + 4)4 = 2
1
1
8) x3 + 3 = 13 x +
x
x

2 
2
x1
x1
40
9)
+
=
x
 x2  9
x(3 x)
3x
10)
x+
= 15
x1
x1
1
1
1
1
1
1
1
1
11) +
+
+
=
+
+
+
x x+2 x+5 x+7
x+1 x+3 x+4 x+6

XY DNG PHNG TRNH HU T


Bn cnh vic xy dng phng trnh t h phng trnh, vic xy dng phng trnh t nhng
ng thc i s c iu kin l mt trong nhng phng php gip ta to ra nhng dng

23
phng trnh hay v l. Di y l mt s ng thc n gin.
4.1 T ng thc "(a + b + c)3 = a3 + b3 + c3 + 3(a + b)(b + c)(c + a) (1) ":
V d: Gii phng trnh: (x 2)3 + (2x 4)3 + (7 3x)3 = 1 (1.1)
Nhn xt: Nu t a = x 2, b = 2x 4, c = 7 3x. Khi ta c phng trnh: a3 + b3 + c3 =
(a + b + c)3 . T ng thc (1), d dng suy ra (a + b)(b + c)(c + a) = 0. T , ta c li gii:
3
3
(1) (x 2)3 + (2x 4)3 + (7 3x)
= [(x 2) + (2x 4) + (7 3x)]
x=2

x=3
(3x 6)(3 x)(5 2x) = 0

5
x=
2 

5
Vy phng trnh (1.1) c tp nghim: S = 2; 3;
2
Vi bi ton trn, cch t nhin nht c l l khai trin ri thu v phng trnh bc ba. Tuy
nhin, vic khai trin c th khng cn hiu qu vi bi ton sau:
V d: Gii phng trnh: (x2 4x + 1)3 + (8x x2 + 4)3 + (x 5)3 = 125x3 (1.2)
1
1 1 1
(a + b)(b + c)(c + a) = 0 (2) ":
4.2 T bi ton " + + =
a b c
a+b+c
1
1
1
1
V d: Gii phng trnh:
+
+
=
(2.1)
x 8 2x + 7 5x + 8
8x + 7
7
8
7
iu kin: x 6= 8, x 6= , x 6= , x 6=
2
5
8
T bi ton (2), ta c:
1
x=

(2.1) (x 8 + 2x + 7)(x 8 + 5x + 8)(2x + 7 + 5x + 8) = 0 x = 0

15
x=
7


15
1
; 0;
Phng trnh c tp nghim: S =
3
7
3
3
3
4.3 T ng thc "a + b + c 3abc = (a + b + c)(a2 + b2 + c2 ab bc ca) (3) ":

V d: Gii phng trnh 54x3 9x + 2 = 0 (3.1)


Ta tm cch vit v tri ca phng trnh di dng x3 + a3 + b3 3abx. Nh vy th a, b l
nghim ca h phng trnh

3
3
a + b =
54

1
3
3

a .b = 3
18

a3 , b3 l nghim ca phng trnh

t2

2
1
1
1
t+ 2 =0t= a=b=
54
18
54 2
3 2

24
Khi phng trnh cho tng ng vi
(x + a + b) x2 + a2 + b2 a x bx ab = 0

2




x
=

2
2
1
3 2
x+
x2 x +
=0

1
18
3 2
3 2
x=
3 2
2
1
Vy (3.1) c tp nghim S = { ; }
3 2 3 2
1 1 1
4.3 T bi ton "Nu xyz = 1 v x + y + z = + + th (x 1)(y 1)(z 1) = 0 (4) ":
x y z
1
1
1
V d: Gii phng trnh:
=
+
+ 10x2 18x + 7 (4.1)
2
10x 11x + 3
2x 1 5x 3
1
3
iu kin: x 6= , x 6=
5
2
1
Nhn xt: Nu t a = 2x 1, b = 5x 3, c =
2
10x 11x + 3
1 1 1
Th ta c: abc = 1 v a + b + c = + +
a b c

1
T (3.1) (2x 1 1)(5x 3 1)
1 =0
10x2 11x + 3

x=1
x=1

4
x= 4

x
=
5

5
11 41
2
10x 11x + 2 = 0
x=
20
(
)
4 11 41
Phng trnh (3.1) c tp nghim: S = 1; ;
5
20
Qua cc v d trn, ta c th hnh dung c bn vic s dng ng thc xy dng phng
trnh. Hi vng da vo vn hiu bit v kh nng sng to ca mnh, bn c c th to ra
nhng phng trnh p mt v c o hn na. Sau y l mt s bi tp t luyn t cc
ng thc khc.

Bi tp t luyn
1. (x 2)6 + (x2 5x + 4)3 = (2x2 + 3)3 + (5 9x)3
2. (x2 2x + 3)5 + (8x x2 + 7)5 = (9x + 5)5 + (5 3x)5
3. (x3 5x+4)2 (127x5x2 )+(x3 +2x2 +7)(7x2 +12x9) = (x3 +7x2 +7x5)2 (2x2 +5x+3)
4. (x 5)4 (4x x2 ) + (2x + 7)4 (x2 3x + 12) = (x2 2x + 7)4 (12 + x)
(x2 5x + 3)3
(x2 + 4x 1)3
(7x 4)3
5.
+
+
=7
(4 9x)(x2 12x + 7) (9x 4)(x2 3x + 3) (12x 7 x2 )(x2 + 3x 3)
1
1
1
1
6. 2
+ 2
+ 2
+ 2
=5
x + 4x + 3 x + 8x + 15 x + 12x + 35 x + 16x + 63
7. (x2 8x + 5)7 + (7x 8)7 = (x2 x 3)7

25

MT S PHNG TRNH BC CAO


Nh ton hc Abel chng minh rng khng c cng thc nghim tng qut cho phng
trnh bc cao (> 4). y cng khng phi l dng ton quen thuc ph thng. V th bi
vit ny ch cp n mt s phng trnh bc cao c bit, c th gii bng bin i s cp.

Bi 1: Gii phng trnh x5 x4 x3 11x2 + 25x 14 = 0


Gii
Phng trnh cho tng ng
(x5 2x4 ) + (x4 2x3 ) + (x3 2x2 ) + (9x2 + 18x) + (7x 14) = 0
(x 2)(x4 + x3 + x2 9x + 7) = 0
x = 2 x4 + x3 + x2 9x + 7 = 0 ()
Xt (*) ta c
() (x4 + x3 + x2 9x + 6) + 1 = 0
(x4 x3 + 2x3 2x2 + 3x2 3x 6x + 6) + 1 = 0
(x 1)2 (x3 + 3x + 6) + 1 = 0 (v nghim)
Vy phng trnh c tp nghim S = {2} 2
Bi 2: Gii phng trnh x6 7x2 +

6 = 0 ()

Gii
R rng ta khng th on nghim ca phng trnh ny v bc cao v h s xu. Mt cch t

nhin ta t 6 = a. Lu rng ta hi vng c th a (*) v phng trnh bc hai theo a, do


ta phn tch 7 = a2 + 1. Cng vic cn li l gii phng trnh ny:

t 6 = a, khi
() x6 x2 (a2 + 1) + a = 0 a2 x2 a + x2 x6 = 0 ()

() c: 4 = 1 + 4x2 (x6 x2 ) = (2x4 1)2 nn c nghim:

a1 = x 2
2
a = 2 x
2
x2

r
2
Vy () x2 = 6 hoc 2 x2 = 6x2 x = 6 hoc x =
2
6+1

Bi 3: Gii phng trnh x6 15x2 + 68 = 0 ()


Gii

26
Do x = 0 khng tho (*) nn x 6= 0. Vit li (*) di dng

68
2 17
15
17 2
3
3
x + 3 =
x +
=
3
x
x
x
x

t a = 17 > 0 ta c phng trnh


x3 +

2a
a2 2
x2 a2 2a x6 2x2 = 0 ()
=
x3
x

Coi (**) l phng trnh n a ta tm c nghim

a = x2 (loi do a > 0)
a=

2 + x4
x2

2 + x4
x4 17x2 + 2 = 0 x2 =
Vy ta c 17 =
2
x
r
17 3
lun: (*) c tp nghim S = {
}2
2

r
17 3
17 3
x=
Kt
2
2

Bi 4: Chng minh phng trnh x5 5x4 + 30x3 50x2 + 55x 21 = 0 c nghim duy
nht

5
5
5
5
x = 1 + 2 4 + 8 16

Gii
t f (x) = x5 5x4 + 30x3 50x2 + 55x 21.
Ta c f 0 (x) = 5x4 20x3 + 90x2 100x + 55 = 5(x2 2x + 3)2 + 10(2x 1)2 > 0 x, do
phng trnh f (x) = 0 c khng qu 1 nghim.

Ta s chng minh nghim l x = 1 + 5 2 5 4 + 5 8 5 16:


t

5
5
5
5
x = 1 + 2 4 + 8 16

5
5
5
5
5
2x = 2 + 4 8 + 16 2

5
5
5
x + 2x = 2 2 1 x + 1 = 2(2 x)
(x + 1)5 = 2(2 x)5
Khai trin biu thc trn, sau rt gn, ta c phng trnh:
x5 5x4 + 30x3 50x2 + 55x 21 = 0
Vy ta c iu phi chng minh. 2
Bi 5: Chng minh phng trnh sau c 7 nghim thc:
g(x) = x9 9x7 + 3x6 + 27x5 18x4 27x3 + 27x2 1 = 0 ()

Gii

27
t f (x) = x3 3x + 1 th g(x) = f (f (x)). Ta s tm nghim ca g 0 (x) = f 0 (x).f 0 (f (x)):
> Nghim ca f 0 (x) l: f 0 (x) = 0 3x2 3x = 0 x = 1.
> tm nghim ca f 0 (f (x)) ta tm nghim ca f (x) = 1 v f (x) = 1:
f (x) = 1 x3 3x + 2 = 0 x {2; 1}

f (x) = 1 x3 3x = 0 x {0; 3}

Nh vy tp nghim ca g 0 (x) = 0 l {2; 3; 1; 0; 1; 3}.


Suy ra g(x) c ti a 7 nghim. Li c:

g(x) khi x

g(2) = 3 > 0 g(x) c 1 nghim trong (; 2)

g(
3)
=
1
<
0

g(x)
c
1
nghim
trong
(2;

3)

g(1) = 19 > 0 g(x) c 1 nghim trong ( 3; 1)

g(0) = 1 < 0 g(x) c 1 nghim trong (1; 0)

g(1) = 3 > 0 g(x) c 1 nghim trong (0; 1)

g( 3) = 1 < 0 g(x) c 1 nghim trong (1; 3)

g(x) + khi x + g(x) c 1 nghim trong (3; +)


Nh vy g(x) = f (f (x)) c 7 nghim thc. 2

Chng II: PHNG TRNH, H PHNG


TRNH C THAM S

PHNG PHP S DNG O HM


L thuyt
i vi bi ton tm iu kin ca tham s phng trnh f (x) = g(m) c nghim min D ta
da vo tnh cht: phng trnh c nghim khi v ch khi hai th ca hai hm s y = f (x)
v y = g(m) ct nhau. Do bi ton ny ta tin hnh theo cc bc sau:
Bc 1: Lp bng bin thin ca hm s y = f (x) .
Bc 2: Da vo bng bin thin ta xc nh m ng thng y = g(m) ct th hm s
y = f (x).
Ch : Nu hm s y = f (x) lin tc trn D v m = M in f (x) , M = M ax f (x) th phng
D

trnh: k = f (x) c nghim m 6 k 6 M


Sau y l mt s bi tp v d:

Bi tp v d
Bi 1:Tm m phng trnh

x2 + x + 1

x2 x + 1 = m c nghim.

Gii
Xt hm s f (x) =

x2 + x + 1

x2 x + 1 vi x R c f 0 (x) =

2x 1

2 x2 x + 1
Ta s tm nghim ca f 0 (x):

2x + 1

2 x2 + x + 1

2x 1
2x + 1
f 0 (x) = 0

=0
2
2 x + x + 1 2 x2 x + 1

(2x + 1) x2 x + 1 = (2x 1) x2 + x + 1
1 2
1 2 3
1 2
1 2 3
(x + ) [(x ) + ]=(x ) [(x + ) + ]
2
2
4
2
2
4
2
2
1
1
(x + ) = (x ) x = 0
2
2
Th li ta thy x = 0 khng l nghim ca f 0 (x). Suy ra f 0 (x) khng i du trn R, m
f 0 (0) = 1 > 0 f 0 (x) > 0x R . Vy hm s f (x) ng bin trn R.
2x

Mt khc: lim f (x) = lim


= 1 v lim f (x) = 1.
x+
x+
x
x2 + x + 1 + x2 x + 1
Da vo bng bin thin ta thy phng trnh cho c nghim khi v ch khi m (1; 1) 2

28

29
Bi 2: Tm m phng trnh

x2 + 1

x = m c nghim
Gii

KX: x > 0

Xt hm s f (x) = 4 x2 + 1 x vi x > 0 ta c f (x) lin tc trn [0; +)


1
x
1
x
<
= 0 x > 0.
Li c: f 0 (x) = q
4
2 x
2 x6 2 x
2 4 (x2 + 1)3
Suy ra hm s f (x) nghch bin trn [0; +)
Mt khc: lim f (x) = 0
x+

Do phng trnh cho c nghim khi v ch khi m (0; 1] 2 .


~ Nhn xt: i khi ta phi tm cch c lp m a phng trnh v dng trn.
Bi 3: Tm m phng trnh sau c nghim:

x4 13x + m + x 1 = 0 (*)

Gii
Ta c:
()

(
x4 13x + m + x 1 = 0

1x>0
4

x 13x + m = (1 x)

Xt hm s f (x) = 4x3 6x2 9x vi x 6 1

1>x
4x3 6x2 9x = 1 m

2 .
Ta c: f 0 (x) = 12x2 12x 9, f 0 (x) = 0
1
x=
2
3
5
Da vo bng bin thin suy ra phng trnh c nghim khi v ch khi 1 m 6 m > 2
2
2
x=

Bi 4: Tm m phng trnh sau c nghim: x x + x + 12 = m( 5 x + 4 x)


Gii
KX: x [0; 4].
Khi phng trnh tng ng vi

(x x + x + 12)( 5 x 4 x) = m

Xt hm s f (x) = (x x + x + 12)( 5 x 4 x) lin tc trn on [0; 4].


3
1
1
1
Ta c: f 0 (x) = (
x+
)(

) > 0x [0; 4]
2
2 x + 12 2 4 x 2 5 x
Vy f (x) l hm ng bin trn [0; 4].

Suy ra phng trnh c nghim khi v ch khi 2 3( 5 2) 6 m 6 12 2 .
~ Nhn xt : Khi gp h phng trnh trong mt phng trnh ca h khng cha tham s
th ta s i gii quyt phng trnh ny trc. T phng trnh ny ta s tm c tp nghim
(i vi h mt n) hoc s rt c n ny qua n kia. Khi nghim ca h ph thuc vo
nghim ca phng trnh th hai vi kt qu ta tm c trn.

45x
2x2 6 ( 1 )
2
Bi 5: Tm m h sau c nghim: ()

2
3x mx x + 16 = 0

30
Gii
Ta c:
(
()

x2 6 4 + 5x

3x2 mx x + 16 = 0

x [1; 4]

m=

3x2 mx x + 16 = 0

3x2 + 16

x x

3x2 + 16

Xt f (x) =
vi x [1; 4]. Ta c:
x x

6x2 x
x(3x2 + 16)
3 x(x2 16)
0
2
f (x) =
=
6 0x [1; 4]
x3
2x3
Nh vy m = f (x) nghch bin trn [1; 4], do f (4) 6 m 6 f (1) 8 6 m 6 19
Vy h c nghim khi v ch khi m [8; 19] 2
(
Bi 6: Tm m h sau c nghim:

72x+

x+1

72+

x+1

+ 2007x 6 2007

x2 (m + 2)x + 2m + 3 = 0
Gii

Ta c:

72x+

x+1

72+

x+1

+ 2007x 6 2007 72+

x+1

(72x+2 1) 6 2007(1 x) ()

> Nu x > 1 72+ x+1 (72x2 1) > 0 > 2007(1 x).Suy ra (*) v nghim.

> Nu x 6 1 72+ x+1 (72x2 1) 6 0 6 2007(1 x). Suy ra (*) ng .


Suy ra h c nghim khi v ch khi phng trnh x2 (m + 2)x + 2m + 3 = 0 c nghim vi
x2 2x + 3
c nghim vi x [1; 1].
x [1; 1]. Hay phng trnh m =
x2
x2 2x + 3
x2 4x + 1 0
Xt hm s f (x) =
vi x [1; 1], c: f 0 (x) =
2 ; f (x) = 0 x =
x2
(x

2)

2 3.

Da vo bng bin thin suy ra h c nghim khi v ch khi m 6 2 3.


(
Bi 7: Tm m h phng trnh sau c nghim: ()

xy+m=0

y + xy = 2

Gii
KX: xy > 0. T h ta cng
c y 6= 0.

xy+m=0

xy+m=0
y 2 4y + 4
4y 4
y 2 4y + 4
()
x=
m=y
=
(y 6 2)

y
y
y
xy = 2 y

y 6 2
4y 4
4
(y 6 2). Ta c: f 0 (y) = 2 > 0(y 6= 0), suy ra hm s f (y) ng
y
y
bin trn cc khong (; 0) v (0; 2].
Mt khc, lim f (y) = 4, lim+ f (y) = ; lim f (y) = +
Xt hm s f (y) =

y0

y0

31
Suy ra h c nghim khi v ch khi m (; 2] (4; +) 2
~ Nhn xt: Khi bi ton yu cu xc nh s nghim ca phng trnh th ta phi lu :
S nghim ca phng trnh f (x) = g(m) chnh l s giao im ca th hai hm s y = f (x)
v y = g(m) . Do phng trnh c k nghim hai th trn ct nhau ti k giao im.
Bi 8: Tm m
phn bit

x4 4x3 + 16x + m+

p
4
x4 4x3 + 16x + m() = 6 c ng hai nghim

Gii
Ta c:
()

x4 4x3 + 16x + m = 2 m = x4 + 4x3 16x + 16

Xt hm s f (x) = x4 + 4x3 16x + 16 vi x R. Ta c:


"
0

f (x) = 4x + 12x 16; f (x) = 0

x = 1
x=2

Da vo bng bin thin suy ra phng trnh c hai nghim phn bit khi v ch khi m < 27.

Bi 9: Tm m phng trnh m x2 + 2 = x + m () c ba nghim phn bit.


Gii
x
T (*) ta c: () m =
2
x +21
x
vi x R.
Xt hm s: f (x) =
x2
+21

2 x2 + 2
0
Ta c: f 0 (x) =
;
f
(x)
=
0

x
=

2.

2
x2 + 2( x2 + 2 1)

Da vo bng bin thin phng trnh c nghim khi v ch khi 2 < m < 2 2
 
Bi 10: Tm m phng trnh: mx2 + 1 = cos x () c ng mt nghim x 0;
2
Gii
Ta thy (*) c nghim th m 6 0. Khi
2 x
sin
( )
cos x 1
2
mx + 1 = cos x m =
2m = x 22
2
x
( )
2

sin2 t

(t

(0;
))
t2
4
2t2 sin t cos t 2tsin2 t
2 sin t(t cos t sin t)
sin 2t(t tan t)

Ta c: f 0 (t) =
=
=
< 0 t (0; )
4
3
3
t
t
t
4

Suy ra hm sf (t) nghch bin trn (0; ).


4

Da vo bng bin thin suy ra phng trnh c ng mt nghim trn (0; ) khi v ch khi
4
8
1
4
< 2m < 1 < m < 2 2 .
2
2

Xt hm s f (t) =

32
(
Bi 11: Tm m h phng trnh ()

3(x + 1)2 + y m = 0
c ba cp nghim phn

x + xy = 1

bit
Gii
iu kin xy > 0
Ta c
(
()

3(x + 1)2 + y = m

3(x + 1)2 + y = m
x2 2x + 1
x2 2x + 1
2

m
=
3(x
+
1)
+
y=

x
x
xy = 1 x

y61

x2 2x + 1
m 3 = 3x + 6x +
x
2

Xt hm s: f (x) = 3x2 + 6x +

x2 2x + 1
(x 6 1) ta c
x

x = 1

6x3 + 7x2 1
x =
=
0

f 0 (x) =

2
x2

1
x=
3
Da vo bng bin thin ta thy h phng trnh c ba nghim phn bit khi
15
20
m [4;
] [ ; 12] 2
4
3
~ Nhn xt: Khi t n ph ta phi tm min xc nh ca n ph v gii quyt bi ton n
ph trn min xc nh va tm. C th:
* Khi t t = u(x)(x D), ta tm c t D1 v phng trnh f (x, m) = 0 (1) tr thnh
g(t, m) = 0 (2). Khi (1) c nghim x D (2) c nghim t D1 .
* tm min xc nh ca t ta c th s dng cc phng trnh tm min gi tr (v min xc
nh ca t chnh l min gi tr ca hm u(x)).
* Nu bi ton yu cu xc nh s nghim th ta phi tm s tng ng gia x v t, tc l mi
gi tr t D1 th phng trnh t = u(x) c bao nhiu nghim x D.

Bi 12: Tm m phng trnh m( x 2 + 2 4 x2 4) x + 2 = 2 4 x2 4 c nghim


Gii
KX; x > 2

Ta thy x = 2 khng l nghim ca phng trnh nn ta chia hai v phng trnh cho 4 x2 4:
r
r
x+2
4 x 2
+ 2) 4
= 2 ()
m(
x+2
x2
r
x+2
1
t2 + 2t
t t = 4
(t > 1). Khi (*) tr thnh: m( + 2) t = 2 m =
.
x2
t
2t + 1
t2 + 2t
2t2 + 2t + 2
Xt hm s f (t) =
(t > 1) ta c f 0 (t) =
> 0 t > 1.
2t + 1
(2t + 1)2

33
Vy hm s f (t) ng bin trn (1; +).
Vy phng trnh c nghim khi v ch khi m > 1 2
~ Nhn xt: Trong cc bi ton trn sau khi t n ph ta thng gp kh khn khi xc nh
min xc nh ca t . tm c iu kin ca n ph t, chng ta c th dng cng c hm
s, bt ng thc, lng gic ha. . .
Bi 13: Tm m phng trnh log23 x +

h i
p 2
log3 x + 1 2m 1 = 0 c nghim trn 1; 3 3
Gii

t t =

log23 x + 1. Do x 1; 3

16t62

Phng trnh cho tr thnh: t2 + t = 2m + 2


Xt hm s f (t) = t2 + t vi 1 6 t 6 2, ta thy f (t) l hm ng bin trn [1; 2]
Vy phng trnh c nghim khi v ch khi 2 6 2m + 2 6 6 0 6 m 6 2 2
Bi
14:
Xc
nh
m

log 3 (x + 1) log 3 (x 1) > log3 4

sau

nghim

phn

bit

log2 (x2 2x + 5) mlog(x2 2x+5) 2 = 5


Gii
x+1
> log3 2 x (1; 3).
x1
t t = log2 (x2 2x + 5)(t (2; 3))v phng trnh th hai ca h tr thnh

iu kin : x > 1. T bt phng trnh th nht ca h ta c: log3

t+

m
= 5 t2 5t = m
t

T cch t t ta c: Vi mi gi tr t (2; 3) th cho ta ng mt gi tr x (1; 3). Suy ra h c


2 nghim phn bit khi v ch khi phng trnh t2 5t = m c 2 nghim phn bit t (2; 3).
5
Xt hm s f (t) = t2 5t vi t (2; 3). Ta c f 0 (t) = 2t 5; f 0 (t) = 0 t = . Da vo bng
2
25
bin thin ta c, h phng trnh c 2 nghim phn bit khi v ch khi m (6; ) 2
4
Bi 14:( thi H khi B - 2004) Tm m phng trnh c nghim:

m( 1 + x2 1 x2 + 2) = 2 1 x4 + 1 + x2 1 x2 ()
Gii
iu kin: 1 6 x 6 1
Trc tin, ta nhn thy rng: (1 x2 ) (1 + x2 ) = 1 x4 v 1 x2 + 1 + x2 = 2 nn ta c php
t n ph nh sau:

t t = 1 + x2 1 x2
Phng trnh cho tr thnh:
m (t + 2) = t2 + t + 2

Do 1 + x2 > 1 x2 t > 0

Mt khc: t2 = 2 2 1 x4 6 2 t 6 2

t2 + t + 2
= m (1)
t+2

34
 
t2 + t + 2
t2 4t
, t 0; 2 , ta c:f 0 (t) =
60
t+2
(t + 2)2 
 
Vy hm f (t) nghch bin trn on 0; 2 . M hm s lin tc trn 0; 2 nn phng trnh
 
cho c nghim x khi phng trnh (1) c nghim t 0; 2 .
iu ny tng ng vi:
h i
 
min f (t) 6 m 6 max f (t) t 0; 2 f
2 6 m 6 f (0)
Ta xt hm s: f (t) =

Vy cc gi tr m cn tm l

2 1 6 m 6 1 2.

Bi 15: Tm m phng trnh sau c nghim:

x x + x + 12 = m 5 x + 4 x
Gii
Cng ging nh nhng bi ton trc, bi ny ta ngh ngay l phi a bi ton v dng
f (x) = m ri s dng tng giao gia 2 th v suy ra iu kin m.
Ta gii bi ton nh sau:
iu kin: 0 6 x 6 4

x x + x + 12

=m
Phng trnh cho tng ng vi:
5

x
+
4

x x + x + 12

t f (x) =
x [0; 4]
5x+ 4x
Tuy nhin, nu n y ta kho st hm s ny th c v kh phc tp v di dng. V vy ta
s gii quyt theo mt hng khc. rng, ta c tnh cht sau:
f (x)
f (x)
Vi hm s y =
. Nu y = f (x) ng bin v y = g (x) nghch bin th y =
ng
g (x)
g (x)
bin.
Ta vn dng tnh cht trn nh sau:

Xt hm s g (x) = x x + x + 12 ta c:
1
3
x+
g 0 (x) =
> 0 nn hm g(x) l ng bin.
2
2 x+ 12

Xt hm s h(x) = 5 x + 4 x ta c:
1
1
h0 (x) =

< 0 nn hm h(x)l nghch bin
2 5x 2 4x
g (x)
Vy, theo tnh cht trn ta c hm y =
ng bin x [0; 4].
h (x)
Do phng trnh f (x) = m c nghim khi v ch khi


f (0) 6 m 6 f (4) 2 15 12 6 m 6 12 2.
Bi 16: Gii v bin lun phng trnh sau theo m:

x2 2x + m2 = |x 1| m (1)
Gii

Xt x > 1 ta c:
(1)

(
x2 2x + m2 = x 1 m

x1m>0
x2 2x + m2 = (x 1 m)2

x>1+m
2mx = 2m + 1

35
> Nu m = 0: h trn v nghim
2m + 1
> Nu m 6= 0 x =
2m

2m + 1
2
2
2m2 + 1
Ta c x > 1 + m
>1+m
>0m6
0<m<
2m
2m
2
2

m>0
2
2m + 1
>1
Kt hp iu kin trn ta c 0 < m <
.
Li c x > 1
1
2m
2
m6
2
(

1xm>0
- Xt x < 1, (1) x2 2x + m2 = 1 x m

x2 2x + m2 = (1 x m)2
(
2mx = 2m 1
x61m
Kt lun:
> Nu m = 0: h v nghim

1
1 + m 3m2 + 10m 3
> Nu 0 < m < m > 3: phng trnh c 2 nghim:x =
3
1m
> Nu m = 3: phng trnh c nghim duy nht x = 1
1
> Nu m = : phng trnh c nghim duy nht x = 1.2
3
Bi 17: Tm m phng trnh sau c nghim
x2 + x2 + x + 1

2

= x2 + 1

2

+ m x2 x + 1

2

Gii
2

rng: (x2 + 1) x2 = (x2 + x + 1) (x2 x + 1)


Do , phng trnh cho tng ng vi:
x2 + x + 1

2

= x2 + x + 1


2
x 2 x + 1 + m x2 x + 1

Do x2 x + 1 > 0, x nn chia hai v ca phng trnh cho x2 x + 1 ta c:


 2
2
x +x+1
x2 + x + 1

=m
x2 x + 1
x2 x + 1
x2 + x + 1 1
, 6t63
x2 x + 1 3
Phng trnh trn tr thnh: t2 t = m
y l mt phng trnh bc hai n gin nn vic kho st xin dnh cho bn c.
iu lu y l iu kin ca t. Thc cht y ta tm gi tr ln nht v nh nht
x2 + x + 1
(c th dng phng php min gi tr).
ca biu thc 2
x x+1
t t =

Bi 18: Tm m phng trnh sau c nghim:





12 4 + x 3x2 = 3x 24 + m 3 x + 1 + 2 4 3x
Gii
4
iu kin: 1 6 x 6

3
t t = 3 x + 1 + 2 4 3x

36



4
Xt hm s f (x) = 3 x + 1 + 2 4 3x, x 1;
3
 

4
Ta c: f (1) = 2 7, f
= 21
3



3
4
3(
4

3x

2
x + 1)
3
p
ta c: f 0 (x) =
Vi x 1;
=

3
4 3x
2 x+1
2 (x + 1)(4 3x)

0
f (x) = 0 4 3x 2 x + 1 = 0 x = 0



Ta c: f (0) = 7 21 6 f (x) 6 7 t
21; 7 .

Li c t2 = 25 3x + 12 4 + x 3x2 12 4 + x 3x2 = t2 25 + 3x

Bi ton tr thnh tm m t2 1 = mt c nghim t [ 21; 2 7]

1
1
Xt hm s f (t) = t , t [ 21; 2 7] ta c: f 0 (t) = 1 + 2 > 0 vi t [ 21; 2 7] nn
t
t

20
27 7
f ( 21) 6 f (t) 6 f (2 7) 6 f (t) 6
14
21
20
27 7
Vy phng trnh c nghim khi 6 f (t) 6
2
14
21
Bi 19: Tm m h sau c nghim thc:

1
1

x + + y + = 5
x
y
1
1

x3 + 3 + y 3 + 3 = 15m 10
x
y
Gii
Cch 1:
1
Nhn vo h ny, ta thy ngay c hng i l phi t n ph v gia cc i lng x +
x
1
v x3 + 3 dng nh c mi lin h vi nhau. Vi tng , ta c php t nh sau:
x
1

a = x +
x
t
, (|a| > 2, |b| > 2).
1

b = y +
y 

3


1
1
1

3 x+
= a3 3a
x3 + 3 = x +
x
x
x

3


Ta c

1
1
1

3
y +
= y+
3 y+
= b3 3b
y3
y
y
Khi , h cho tr thnh:
(
(
a+b=5
a+b=5

a3 + b3 3 (a + b) = 15m 10
ab = 8 m
D thy a, b l nghim ca phng trnh X 2 5X + 8 m = 0 X 2 5x + 8 = m (1) Xt
hm s f (X) = X 2 5X + 8, |X| > 2, ta c:
5
f 0 (X) = 2X 5 f 0 (X) = 0 X =
2
T , k bng bin thin v ch rng h cho c nghim thc khi v ch khi phng trnh
7
(1) c nghim |X| > 2. Ta tm c: 6 m 6 2 m > 22.
2
Cch 2:
Ta nhn thy rng phng trnh th nht khng c cha tham s nn ta s xut pht t

37
phng trnh ny. Khai trin phng trnh ny ra, ta c:
x3 y 3 + 4 (x y) = 9y + 8 3x2 + 6y 2 x3 + 3x2 + 4x = y 3 + 6y 2 + 13y + 8
(x + 1)3 + (x + 1) = (y + 2)3 + (y + 2)
Xt hm s f (t) = t3 + t, d thy l hm s ny ng bin nn
f (x + 1) = f (y + 2) x + 1 = y + 2 x = y + 1
T y, thay x = y + 1 vo phng trnh th hai ta c:
p

15 + 2y y 2 = 2m +

4 y2

p
p
(5 y) (y + 3) 4 y 2 = 2m

n y tng r, ta ch cn chuyn v tng giao gia hai th.


(
Bi 20: Tm m h sau c nghim thc:

x3 + (y + 2) x2 + 2xy = 2m 3
x2 + 3x + y = m

Gii
~ tng: h ny ta quan st thy bi ton cn cha r ng li no v c hai phng
trnh trong h u cha n tham s m. V vy i n hng gii quyt tt ta nn bt u
vo cng ph hai v tri trong hai phng trnh trong h. C th ta c:
x3 + (y + 2) x2 + 2xy = x3 + yx2 + 3x2 + 2xy = x2 (x + y) + 2x (x + y) = (x + y) x2 + 2x

Mt khc:
x2 + 3x + y = x2 + 2x + x + y
R rng bc phn tch ny ta tm ra li gii cho bi ton ny chnh l t n ph.
~
Gii
(
t:

a = x2 + 2x
b=x+y

, a > 1. Khi h cho tr thnh:


(

a+b=m
ab = 2m 3

a2 3 = (a + 2) m (1)
b=ma

a2 3
= m (2)
a+2
H cho c nghim khi v ch khi phng trnh (2) c nghim a > 1.
x2 3
Xt hm s: f (x) =
vi x > 1
x+2
n y ta ch cn lp bng bn thin. Cng vic tip theo xin dnh cho bn c.
T phng trnh (1) trong h ta c:

Bi tp t luyn
Bi 1: Tm m phng trnh tan2 x + cot2 x + m(cot x + t anx) = 3 c nghim

38

Bi 2: Tm m phng trnh x + x + 9 = 9x x2 + m c nghim

Bi 3: Tm m phng trnh 3 + x + x + 6 18 + 3x x2 = m c nghim


Bi 4: Tm m phng trnh x3 4mx2 + 8 = 0 c 3 nghim phn bit.
Bi 5: Tm m phng trnh x3 + 3x2 + (3 2m) x + m + 1 = 0 c ng mt nghim ln
hn 1.

Bi 6: Tm m phng trnh sau c ng (


2 nghim thc phn bit 4x2 2mx+1 = 3 8x3 + 2x
x3 + (y + 2) x2 + 2xy = 2m 3
Bi 7: Tm m h sau c nghim thc:
x2 + 3x + y = m

PHNG PHP DNG NH L LAGRANGE-ROLLE


L thuyt
F nh l Rolle: : Nu f (x) l hm lin tc trn on [a; b], c o hm trn khong (a; b)
v f (a) = f (b) th tn ti c (a; b) sao cho f 0 (c) = 0.
T ta c 3 h qu:
> H qu 1: Nu hm s f (x) c o hm trn (a; b) v f (x) c n nghim (n l s nguyn
dng ln hn 1) trn (a; b) th f 0 (x) c t nht n - 1 nghim trn (a; b).
> H qu 2: Nu hm s f (x) c o hm trn (a; b) v f 0 (x) v nghim trn (a; b) th f (x)c
nhiu nht 1 nghim trn (a; b).
> H qu 3: Nu f (x)c o hm trn (a; b) v f 0 (x) c nhiu nht n nghim (n l s nguyn
dng) trn (a; b) th f (x) c nhiu nht n + 1 nghim trn (a; b).
Cc h qu trn vn ng nu cc nghim l nghim bi (khi f (x) l a thc) v cho ta
tng v vic chng minh tn ti nghim cng nh xc nh s nghim ca phng trnh, v
nu nh bng mt cch no ta tm c tt c cc nghim ca phng trnh th ngha l
khi phng trnh c gii.
F nh l Lagrange: : Nu f (x)l hm lin tc trn on [a; b], c o hm trn khong
f (b) f (a)
(a; b)th tn ti c (a; b) sao cho f 0 (c) =
.
ba
Sau y l mt s ng dng ca hai nh l trn:

Bi tp v d
F Dng nh l Lagrange - Rolle bin lun phng trnh:
Bi 1: Chng minh rng phng trnh a cosx + b cos 2x + c cos 3x lun c nghim vi mi
a, b, c.
Gii
b sin 2x c sin 3x
+
ta c f 0 (x) = a cosx + b cos 2x + c cos 3x M f (0) =
2
3
f () = 0 x0 (0; ) : f 0 (x0 ) = 0, suy ra iu phi chng minh 2

Xt f (x) = a sin x +

39
~ Nhn xt: Bi ton trn c dng tng qut:
Bi 1*: Cho hm s f (x) lin tc trn [a; b], chng minh rng phng trnh f (x) = 0 c t
nht mt nghim trn (a; b)
Hng dn: Xt hm F (x) tha mn F (x) lin tc trn [a; b], F 0 (x) = f (x).g(x) x (a; b),
g(x) v nghim trn (a; b) v F (a) = F (b). Theo nh l Rolle suy ra iu phi chng minh 2
Bi 2: Cho a + bc = 0. Chng minh rng phng trnh
a sin x + 9b sin3x + 25c sin 5x = 0
c t nht 4 nghim thuc [0; ]
Gii
chng minh f (x) c t nht n nghim ta chng minh F (x) c t nht n + 1 nghim vi F (x)
l mt nguyn hm ca f (x) trn (a; b) (c th phi p dng nhiu ln)
Xt hm s:f (x) = a sin x bsin3x csin5x, ta c:
f 0 (x) = a cos x 3b cos 3x 5c cos 5x; f 00 (x) = a sin x + 9b sin 3x + 25c sin 5x
3

3
3

Ta c f (0) = f ( ) = f ( ) = f () = 0 x1 (0; ), x2 ( ; ), x3 ( ; ) sao cho


4
4
4
4 4
4
f (0) = f 0 (x1 ) =0 f (x2 ) = f 0 (x3 ) = 0 x4 (x1 ; x2 ), x5 (x2 ; x3 )|f 00 (x4 ) = f 00 (x5 ) = 0
m f 00 (0) = f 00 () = 0, suy ra iu phi chng minh 2
Bi 3:Chng minh phng trnh x5 5x + 1 = 0 c ng ba nghim thc.
Gii
t f (x) = x5 5x + 1 th f (2) = 21 < 0, f (0) = 1 > 0, f (1) = 3 < 0, f (2) = 23 > 0 nn
t y suy ra phng trnh cho c ba nghim thc
Gi s c nghim th t ca phng trnh. p dng nh l Rolle ta c:
f 0 (x) = 0 5x4 5 = 0. Phng trnh ny c hai nghim thc nn suy ra mu thun. Vy ta
c pcm 2.
Bi 4: Cho a thc P (x) v Q(x) = aP (x) + bP 0 (x) trong a, b l cc s thc, a 6=0.
Chng minh rng nu Q(x) v nghim th P (x) v nghim.
Gii
Ta c deg P (x) = deg Q(x)
V Q(x) v nghim nn deg Q(x) chn. Gi s P (x) c nghim, v deg P (x) chn nn P (x) c
t nht 2 nghim.
> Khi P(x) c nghim kp x = x0 ta c x0 cng l mt nghim ca P 0 (x) suy ra Q(x) c
nghim.
> Khi P(x) c hai nghim phn bit x1 < x2 :

40
* Nu b = 0 th hin nhin Q(x) c nghim.
a
x
* Nu b 6=0 : Xt f (x) = e b P (x) th f (x)c hai nghim phn bit x1 < x2 v
a
a
a
a
x
a x
1 x
1 x
0
0
f (x) = e b P (x) + e b P (x) = e b (aP (x) + bP (x)) = e b Q(x)
b
b
b
0

V f (x)c hai nghim suy ra f 0 (x)c t nht 1 nghim hay Q(x) c nghim.
Tt c trng hp u mu thun vi gi thit Q(x) v nghim. Vy khi Q(x) v nghim th
P (x) v nghim 2
Bi 5: Cho phng trnh
a0 xn + a1 xn1 + ... + an1 x + an = 0, (a0 6= 0)
c n nghim phn bit. Chng minh (n 1)a21 > 2na0 a2
Gii
t a0 xn + a1 xn1 + ... + an1 x + an = f (x)
Nhn xt f kh vi v hn trn R nn suy ra f 0 (x) c n-1 nghim phn bit.
00
f (x) c n-2 nghim phn bit.
n!
. . . .. f [n2] (x) = a0 x2 + (n 1)!a1 x + (n 2)!a2 c 2 nghim phn bit.
2
Nhn thy > 0 nn ((n 1)!a1 )2 2n!a0 (n 2)!a2 > 0 t suy ra iu phi chng minh
2.
Bi 6:(VMO 2007) Cho s thc a > 2 v fn (x) = a10 x10+n + xn + xn1 + ... + x + 1.
Chng minh rng vi mi s nguyn dng n, phng trnh fn (x) = a lun c ng mt
nghim dng duy nht xn
Gii
t Fn (x) = fn (x) a, ta c Fn (x) lin tc, ng bin trn [0; +) v Fn (0) = 1 a <
0, Fn (1) = a10 + n + 1 a > 0. Suy ra phng trnh fn (x) = a lun c ng mt nghim xn
dng duy nht.
a1
t b =
fn (b) = bn (a 1)[(a 1)9 1] + a fn (b) > a b > xn , n N
a
Theo nh l Lagrange, lun tn ti cn (xn ; b) tha mn:
fn (b) fn (xn ) = f 0 (cn )(b xn )
T ta c iu phi chng minh 2
n
P

1
1
= , vi n l s nguyn dng. Chng
1
2
i=1
minh rng vi mi s nguyn dng n, phng trnh nu trn c mt nghim duy nht ln
hn 1; k hiu nghim l xn
Bi 7: (VMO 2002) Xt phng trnh

i2 x

Gii

41
n
P

1
1

, ta c: fn (x) lin tc v nghch bin trn (1; +)


2
2
i=1 i x 1
1
M lim+ fn (x) = +, lim fn (x) = fn (x) = 0 c mt nghim duy nht ln hn 1 2
x+
x1
2
Xt fn (x) =

F Dng nh l Lagrange -Rolle gii phng trnh:


Bi 8: Gii phng trnh: 3x + 5x = 2.4x ()

Gii
Nhn xt: x = 0; x = 1 l nghim ca phng trnh (*).
Gi x0 l nghim ca phng trnh cho. Ta c:
3x0 + 5x0 = 2.4x0 5x0 4x0 = 4x0 3x0 (1a)
Xt hm s f (t) = (t + 1)x0 tx0 , ta c (1a) f (4) = f (3)
V f (t) lin tc trn [3; 4] v c o hm trong khong (3; 4), do theo nh l Rolle tn ti
c (3; 4) sao cho
"
x0 = 0
x0 1
0
x0 1
f (c) = 0 x0 [(c + 1)
c
]=0
x0 = 1
Vy phng trnh (*) c tp nghim S = {0; 1} 2
Bi 9: Gii phng trnh 5x 3x = 2x ()

Gii
Nhn xt: x = 0; x = 1 l nghim ca phng trnh (2). Gi x0 l nghim ca phng trnh
cho, ta c:
5x0 5x0 = 3x0 3x0 (2a)
Xt hm s: f (t) = tx0 tx0 , khi : (2a) f (5) = f (3)
V f (t) lin tc trn [3; 5] v c o hm trn (3; 5), do theo nh l Lagrange lun tn ti
c (3; 5) sao cho
"
x0 = 0
f 0 (c) = 0 x0 (cx0 1 1)=0
x0 = 1
Vy phng trnh (*) c tp nghim S = {0; 1} 2
Bi 10: Gii phng trnh: 3x + 2.4x = 19x + 3 ()

Gii
Ta c () 3x + 2.4x 19x 3 = 0
Xt hm s: y = f (x) = 3x + 2.4x 19x 3 ta c: f 0 (x) = 3x ln 3 + 2.4x ln 4 19 ta c
f 00 (x) = 3x (ln 3)2 + 2.4x (ln 4)2 > 0, x R hay f 00 (x) v nghim, suy ra f 0 (x) c nhiu nht 1

42
nghim, suy ra f (x) c nhiu nht 2 nghim.
M f (0) = f (2) = 0 do (*) c ng hai nghim x = 0, x = 2 2
Bi 11: Gii phng trnh: (1 + cos x)(2 + 4cos x ) = 3.4 cos x ()

Gii
t t = cos x, (t [1; 1])
Ta c: () (1 + t)(2 + 4t ) = 3.4t (1 + t)(2 + 4t ) 3.4t = 0
Xt hm s: f (t) = (1 + t)(2 + 4t ) 3.4t ta c:
f 0 (t) = 2 + 4t + (t 2)4t ln 4, f 00 (t) = 2.4t ln 4 + (t 2)4t ln2 4
2
f 00 (t) c mt nghim duy nht
Li c: f 00 (t) = 0 t = 2 +
ln 4
f 0 (t) c nhiu nht hai nghim f (t) c nhiu nht ba nghim.
1
1
Mt khc d thyf (0) = f ( ) = f (1) = 0, do f (t) c ba nghim t = 0, , 1.
2
2

Kt lun: Nghim ca phng trnh (*) l: x = + k2, x = + k2, x = k2, k Z 2


2
3
Bi 12: Gii phng trnh 3cos x 2cos x = 2cos x 2 cos x ()
Gii
Xt hm f (t) = tcos t cos , f 0 (t) = cos (tcos 1 1)
Ta nhn thy f (3) = f (2) v f (x) kh vi trn [2;3] nn p dng nh l Lagrange ta c:
f (3) f (2)
cos (ccos 1 1) = 0
1

T ta suy ra nghim ca phng trnh (*) l x = + k, x = k2(k Z) 2


2
c [2; 3] : f 0 (c) =

PHNG PHP DNG IU KIN CN V


L thuyt
F Bi ton:
Cho h phng trnh (hoc h bt phng trnh) cha tham s c dng:

f (x, m) = 0

f (x, m) > 0
(I) x Dx
hoc (II) x Dx

m Dm
m Dm
Trong x l bin s, m l tham s, Dx , Dm l min xc nh ca x v m.
Yu cu t ra: ta phi tm gi tr ca tham s m h (I) hc (II) tha mn mt tnh cht
no .
F Phng php gii:

43
I Bc 1 (iu kin cn): Gi s h tha mn tnh cht P no m u bi i hi. Khi ,
da vo c th ca tnh cht P v dng ca phng trnh ta s tm c mt rng buc no
i vi tham s m v rng buc y chnh l iu kin cn c tnh cht P. iu c ngha
l: nu vi m0 khng tha mn rng buc trn th chc chn ng vi m0 , h khng c tnh cht P.
I Bc 2 (iu kin ): Ta tm xem trong cc gi tr ca m va tm c, gi tr no lm cho
h tha mn tnh cht P. bc ny ni chung ta cng ch cn gii nhng h c th khng
cn tham s. Sau khi kim tra, ta s loi i nhng gi tr khng ph hp v nhng gi tr cn
li chnh l p s ca bi ton.
Nh vy, tng ca phng php ny kh r rng v n gin. Trong rt nhiu bi ton v
bin lun th phng php ny li th hin u th r rt. Tuy nhin, thnh cng ca phng
php cn nm ch ta phi lm th no pht hin iiu kin cn mt cch hp l v chn
iu kin mt cch ng n.

Bi tp v d
a)S dng tnh i xng ca cc biu thc c mt trong bi ton:
Bi 1: Tm m phng trnh sau c nghim duy nht

x+

1x+

x+

1 x = m (1)

Gii
~ iu kin cn:
Gi s (1) c nghim duy nht x =
D thy nu (1) c nghim x = th (1) cng c nghim x = 1 . V nghim l duy nht
1
nn = 1 =
2

1
Thay = vo (1) ta tm c m = 2 + 4 8.
2
~ iu kin :

Gi s m = 2 + 4 8, khi (1) c dng sau:

4
4
x + 4 1 x + x + 1 x = 2 + 8 (2)
Theo bt ng thc AM-GM ta c:

4
x + 1 x 6 2 v 4 x + 4 1 x 6 8
1
Do (2) x = 1 x x = .
2

Vy (1) c nghim duy nht th iu kin cn v l m = 2 + 4 8 2


Bi 2: Tm a v b phng trnh sau c nghim duy nht
q
q

3
3
3
3
2
(ax + b) + (ax b)2 + a2 x2 b2 = b (1)

44
Gii
~ iu kin cn:
Gi s (1) c nghim duy nht x = x0 , khi d thy x = x0 cng l nghim ca (1). Do
t gi thit ta suy ra x0 = 0. Thay x0 = 0 vo (1) ta c :
"

b=0
3
3
b2 = b
b=1
~ iu kin :
> Khi b = 0, (1) c dng:

a2 x 2 +

a2 x 2 +

a2 x 2 = 0 a2 x 2 = 0

Do (1) c nghim duy nht khi v ch khi a 6= 0


> Khi b = 1, (1) c dng:
q
q

3
3
3
2
(ax + 1) + (ax 1)2 + a2 x2 1 = 1 ()

t u = 3 ax + 1; v = 3 ax 1, ta thy:
( 3
(
(
(
uv =2
u=1
ax + 1 = 1
u v3 = 2
()

ax = 0
u2 + uv + v 2 = 1
u2 + uv + v 2 = 1
v = 1
ax 1 = 1
Vy (*) c nghim duy nht khi v ch khi a 6= 0
"
Tm li, phng trnh (1) c nghim duy nht th iu kin cn v l

a 6= 0; b = 0
b=1

Bi 3: Tm m h sau c nghim duy nht:

7 + x + 11 x 4 = m 4 3 10 3m
q

p7 + y + p11 y 4 = m 4 310 3m
Gii
74
10
6m6
27
3
Tr theo v hai phng trnh ta c:
p
p

x + 7 11 x = y + 7 11 y

Xt hm s: f (t) = t + 7 11 t; 7 6 t 6 11 ta c:
1
1
f 0 (t) =
+
> 0 Vy hm s ng bin, suy ra: f (x) = f (y) x = y.
2 t + 7 2 11 t
Thay vo mt trong hai phng trnh ca h ta c:
q

7 + x + 11 x 4 = m 4 3 10 3m ()
iu kin: 7 6 x, y 6 11;

~ iu kin cn:
Ta thy l nu x0 l mt nghim ca phng trnh th 4 x0 cng l nghim ca phng trnh.
Nn h cho c nghim duy nht khi v ch khi
x0 = 4 x0 x0 = 2

45
Thay vo phng trnh (*) ta c:
q

4 3 10 m = m 2 ()
Gii phng trnh (**) ta tm c m = 3.
~ iu kin :

7 + x + 11 x = 6
Vi m = 3, ta thu c h phng trnh:
7 + y + 11 y = 6

V x = y nn ta ch vic gii phng trnh 7 + x + 11 x = 6 x = 2


Vy m = 3 l gi tr cn tm h cho c nghim duy nht. 2
Bi 4: Tm a,b h sau c nghim duy nht:

xyz + z = a

xyz 2 + z = b

x 2 + y 2 + z 2 = 4
Gii

~ iu kin cn:
Gi s (x0 ; y0 ; z0 ) l nghim ca h phng trnh cho th (x0 ; y0 ; z0 ) cng l nghim.
Do

z0 = a
tnh duy nht nn x0 = x0 ; y0 = y0 x0 = y0 = 0 Thay tr li vo h , ta c: z0 = b

z 2 = 4
0

T y ta suy ra a = b = 2 hoc a = b = 2
~ iu kin :
Xt hai trng hp sau:
> Nu a = b = 2: Khi h c dng:

xyz + z = 2 (1)

xyz 2 + z = 2 (2)

x2 + y 2 + z 2 = 4 (3)

Ly (1) (2) ta c xyz(1 z) = 0, t (1) li c z 6= 0 do xy(1 z) = 0


* Nu x = 0 z = 2 y = 0
* Nu y = 0
z=2x=0
x 2 + y 2 = 3
* Nu z = 1
.
xy = 1
H trn c nghim (x1 ; y1 ) 6= (0; 0).V vy ngoi nghim (0, 0, 2), h cn c nghim khc
(x1 ; y1 ; 1) do h khng c nghim duy nht. Trng hp ny khng tha mn.
> Nu a = b = 2:
Khi h c dng:

xyz = 2
xyz 2 2 + z = 2

x 2 + y 2 + z 2 = 4

46

Tin hnh lm nh trng hp trn ta i n:


* Nu x = 0 z = 2 y = 0
* Nu y = 0
z = 2 x = 0
x 2 + y 2 = 3
* Nu z = 1
xy = 3
Ta thy t h phng trnh trn, ta suy ra x2 + y 2 < 2|xy| nn h v nghim.
Vy trong trng hp ny h c duy nht nghim (x; y; z) = (0, 0, 2)
Vy iu kin cn v h phng trnh cho c nghim duy nht l a = b = 2 2

b) S dng im thun li:


Bi 5: Tm a phng trnh sau nghim ng vi mi x:
log2 (a2 x2 5ax2 +

6 a = log2+x2 (3

a 1) ()

Gii
~ iu kin cn:

Gi s (*)
ng vi mi x. Vi x = 0 ta c log2 6 a = log2 (3 a 1)

1 6 a 6 6

Li c:
a {2; 5}
a1<3

a1+ 6a=3
~ iu kin :
> Nu a = 2 th () log2 (2 12x2 ) = log2+x2 2 ()
R rng (**) khng ng vi mi x, v log2 (2 12x2 ) c ngha th phi c 12x2 < 2
> Nu a = 5 th () log2 1 = log2+x2 1 (lun ng)
Vy a = 5 l iu kin cn v (*) ng vi mi x 2
Bi 6: Tm a h phng trnh n (x; y) c nghim vi mi b:

2bx + (a + 1)by 2 = a2
(a 1)x3 + y 2 = 1
Gii
~ iu kin cn:
Gi s h c nghim vi mi b, thay b = 0 ta c

a2 = 1
(a 1)x3 + y 2 = 1
Do iu kin cn l a = 1
~ iu kin :

47

2bx + 2by 2 = 1
> Nu a = 1: ta c h
y 2 = 1
1
h v nghim. Vy trng hp ny loi.
2

2bx = 1
> Nu a = 1: ta c h
2x3 + y 2 = 1
Khi b >

H trn lun c nghim (x; y) = (0; 1)


Vy a = 1 l iu kin cn v h phng trnh c nghim vi mi b 2
Bi 7: Tm a h phng trnh n (x; y) c nghim vi mi b:

(x2 + 1)a + (b2 + 1)y = 2


a + bxy + x2 y = 1
Gii
~ iu kin cn:
Gi s h c nghim vi mi b, thay b = 0 ta c

"
(x2 + 1)a = 1
a = 0; x2 y = 1
()

a {0; 1}
a + x2 y = 1
x2 + 1 = a + x2 y = 1
~ iu kin :

(b2 + 1)y = 1 (1)


> Nu a = 0 : ta c
bxy + x2 y = 1 (2)
Nu b 6= 0 b2 + 1
6= 1 nn t (1) c y = 0, nhng khng tho (2). Vy trng hp ny loi.
x2 + (b2 + 1)1
> Nu a = 1: ta c
bxy + x2 y = 0
H trn lun c nghim x = y = 0.
Vy a = 1 l iu kin cn v h cho c nghim vi mi b 2
Bi 8: Tm iu kin ca a, b, c, d, e, f hai phng trnh n (x; y) sau l tng ng:

ax2 + bxy + cy 2 + dx + ey + f = 0 (1)


x2 + y 2 = 1 (2)
Gii
~ iu kin cn:

1
1
1 1
Ta thy (x; y) = (0; 1), (1; 0), ( ; ), ( ; ) l nghim ca (2). Do (1) cng
2 2
2
2
phi c cc
nghim
trn.

c + e + f = c e + f = a + d + f = a d + f = 0

Nh vy a + b + c + 2d + 2e + 2f
a + b + c 2d 2e + 2f

=
=0
2
2

48

Gii h trn ta tm c iu kin cn ca bi ton l ()

b = d = e = 0
a = c = f 6= 0

~ iu kin :
D thy vi (*) th (2) trng vi (1).
Vy (*) l iu kin cn v (1) (2) 2
Bi 9: Cho phng trnh
x3 + ax + b = 0 (1)
Tm a, b phng trnh trn c ba nghim phn bit x1 < x2 < x3 cch u nhau.
Gii
~ iu kin cn:
Gi s phng trnh (1) c 3 nghim khc nhau x1 , x2 , x3 tha gi thit x1 + x3 = 2x2
Theo nh l Viete vi phng trnh bc 3 ta c: x1 + x2 + x3 = 0 3x2 = 0 x2 = 0
Thay x2 = 0 vo (1) ta c b = 0
~ iu kin :
Gi s b = 0 , khi (1) tr thnh:
x3 + ax = 0 x(x2 + a) = 0 (2)
Ta thy (2) c 3 nghim phn bit nu a < 0. Khi cc nghim ca (2) l

x
1 = a

x2 = 0

x3 = a

Cc nghim trn cch u nhau nn iu kin cn v (1) c nghim tha mn bi l


b = 0, a < 0 2
Bi 10: Cho phng trnh
x3 3x2 + (2m 2) x + m 3 = 0
Tm m phng trnh c ba nghim x1 , x2 , x3 sao cho x1 < 1 < x2 < x3 .
Gii
~ iu kin cn:
t f (x) = x3 3x2 + (2m 2) x + m 3
Gi s phng trnh f (x) = 0 c 3 nghim x1 , x2 , x3 tha mn x1 < 1 < x2 < x3
Ta c: f (x) = (x x1 ) (x x2 ) (x x3 )
Suy ra: f (x) > 0 khi x1 < x < x2
f (1) > 0 hay m 5 > 0 m < 5.
~ iu kin :
Gi s m < 5

49
Do lim f (x) = nn ta phi c < 1 m f () < 0
x

Li c: f (1) = m 5 > 0 v f (x) lin tc nn suy ra tn ti mt < x1 < 1 sao cho


f (x1 ) = 0
Ta c: f (0) = m 3 < 0 ( do m < 5)
Vy tn ti 1 < x2 < 0 sao cho f (x2 ) = 0
Mt khc, do lim f (x) = + nn phi c > 0 sao cho f () > 0.
x+

T , tn ti x3 m 0 < x3 < sao cho f (x3 ) = 0.


Nh vy, phng trnh f (x) = 0 khi m < 5 c 3 nghim x1 , x2 , x3 tha mn x1 < 1 < x2 <
0 < x3 . Vy m < 5 chnh l iu kin cn v tha mn bi. 2
Bi 11: Tm m phng trnh sau c nghim:
20x2 + 10x + 3
= x2 + 2 (2m 3) x + 5m2 16m + 20
2
3x + 2x + 1
Gii
~ iu kin cn:
Gi s phng trnh cho c mt nghim l x0 .
20x2 + 10x + 3
t f (x) =
v g (x) = x2 + 2 (2m 3) x + 5m2 16m + 20
3x2 + 2x + 1
Khi d thy rng:
max f (x) > f (x0 ) = g (x0 ) > min g (x)
Do min g (x) = g ( (2m 3)) = m2 4m + 11
Gi y0 l gi tr ty ca f (x), khi ta c:
y0 =

20x2 + 10x + 3
(20 3y0 ) x2 + 2 (5 y0 ) x + 3 y0 = 0 (1)
2
3x + 2x + 1

Suy ra (1) c nghim khi: 0 = 2y0 2 + 19y0 35 > 0

5
6 y0 6 7 Vy max f (x) = 7
2

Nh vy, t iu kin max f (x) > min g (x) ta c:


7 > m2 4m + 11 (m 2)2 6 0 m = 2
~ iu kin :
Gi s m = 2, khi phng trnh cho tr thnh
20x2 + 10x + 3
= x2 + 2x + 8 (2)
3x2 + 2x + 1
(
= 7 x = 1
20x2 + 10x + 3
Ta nhn thy rng g (x) = x2 + 2x + 8
Mt khc: f (x) =
67
3x2 + 2x + 1
> 7 x 6= 1
vi x v f (1) = 6, 5. T y suy ra (2) v nghim.
Vy khng tn ti gi tr no ca m tha mn gi thit. 2
Bi 12: Tm m phng trnh x2 3x + m = 0 (1) c mt nghim gp i nghim ca
phng trnh x2 x + m = 0 (2)

50
Gii
~ iu kin cn: Gi s tn ti m tho mn iu kin u bi, tc l phng trnh (2) c
nghim x0 , cn (1) c nghim 2x0 . Vy ta c

4x2 6x0 + m = 0
0
x2 x0 + m = 0
0

5
Tr v vi v ca hai phng trnh trn cho nhau ta s tm c x0 = 0 hoc x0 =
3
10
Thay hai gi tr x0 vo mt trong hai phng trnh trn ta c m = 0 v m =
9
y cng chnh l iu kin cn ca bi ton. ~ iu kin :

x2 3x = 0

x 2 x = 0
10
ta s ln lt gii cc phng trnh
Xt khi m = 0 v m =
10

9
x2 3x +
=0

x2 x + 10 = 0
9
D thy nghim ca 2 cp phng trnh ny tho gi thit.
10
Vy m {0; } l iu kin cn v tha mn bi. 2
9
Bi 13: Tm a,b sao cho vi mi c phng trnh sau c khng qu hai nghim dng:
x3 + ax2 + bx + c = 0
Gii
gii bi ny, ta s dng phn b. Ngha l ta tm a, b sao cho tn ti c phng trnh
x3 + ax2 + bx + c = 0 c 3 nghim dng.
~ iu kin cn:
Gi s tn ti c phng trnh f (x) = x3 + ax2 + bx + c = 0 c 3 nghim dng x1 < x2 < x3 .
Khi f (x) = (x x1 )(x x2 )(x x3 )
Vy hm s f (x) c cc tr ti , > 0 (vi x1 < < x2 < < x3 )
Phng trnh f 0 (x) = 3x2 + 2ax + b = 0 (1) c hai nghim dng.
Nh vy

0
2

=
a

3b
>
0

a2 > 3b

b > 0
b
P = >0
b>0
()

a < 3b

2a
S =
a < 0
>0
3
(*) l iu kin cn ca bi ton.
~ iu kin :
Gi s a, b tho mn (*) th r rng phng trnh 3x2 + 2ax + b = 0 c 2 nghim dng
0<<
Suy ra hm s f (x) = x3 + ax2 + bx + c c cc i ti x = v cc tiu ti x = .
Do > 0 nn tm c x1 (c; ) sao cho f () < f (x1 ) < f () phng trnh f (x) = f (x1 )
c ba nghim dng. t c = f (x1 ) th phng trnh x3 + ax2 + bx + c = 0 c 3 nghim
dng.

51
Vy (*) l iu kin cn v tn ti c sao cho phng trnh x3 + ax2 + bx + c = 0 c 3
nghim dng.

T , ta suy ra: b 6 0 hoc a > 3b l iu kn cn v sao cho phng trnh


x3 + ax2 + bx + c = 0 c khng qu 2 nghim dng. 2

Bi tp t luyn
Bi 1: Tm m hai phng trnh sau l tng ng x2 + (m2 5m + 6) x = 0 v x2 +
2 (m 3) x + m2 7m + 12 = 0
Bi 2: Tm m phng trnh sau c nghim duy nht:
|x + m|2 + |x + 1| = |m + 1|
Bi 3: Tm m phng trnh sau c nghim duy nht:
p

x + 3 + 6 x (3 + x) (6 x) = m
(p


x2 + y 2 1 a x + y 1 = 1
Bi 4: Tm a h sau c ng mt nghim:
x + y = xy + 1
(
x2 + 3 + |y| = a
Bi 5: Tm a h sau c ng mt nghim: p

y 2 + 5 + |x| = x2 + 5 + 3 a
(
(x + y)4 + 13 = 6x2 y 2 + m
Bi 6: Cho h phng trnh

xy x2 + y 2 = m
Tm m h c nghim duy nht.
x3 my 3 = 1 (m + 1)2
2
Bi 7: Tm s thc m sao cho h:
c nghim (x; y) tho x + y = 0
3
2
x + mx y + xy 2 = 1

PHNG PHP NG DNG HNH HC GII TCH


V HNH HC PHNG
L thuyt

Trong mt phng ta Oxy, cho cc vector:


u = (x1 , y1 ),
v = (x1 , y2 ) khi ta c:
p

|
u +
v | 6 |
u | + |
v | (x1 + x2 )2 + (y1 + y2 )2 6

x21 + y12 +

q
x22 + y22

x1
y1

Du bng xy ra khi v ch khi hai vector


u v
v cng hng
=
= k > 0.
x2
y2

Vi hai vector
u,
v bt k trong khng gian th:
u .
v = |
u | . |
v | . cos (
u ,
v) 6

| u | . | v |.
(nh l cosin trong tam gic) Cho a, b, c l ba cnh ca tam gic ABC v A l gc
nh A ca tam gic khi :
a2 = b2 + c2 2bc cos A

52
Cho tam gic ABC c ba gc nhn v im M ty trong mt phng khi gi T l
im nhn cc cnh BC, CA, AB di cng mt gc 1200 th vi mi im M trn mt
phng ta c:
MA + MB + MC > T A + T B + T C
T c gi l im Torricelli ca tam gic ABC.

Sau y l mt s dng ton thng gp:


F Kho st h phng trnh cha dng tuyn tnh v phn tuyn tnh

x 2 + y 2 = r
Bi 1: Bin lun s nghim ca h
ax + by = c

(r>0)

~ Phn tch: H cho gm phng trnh ng trn (C) v phng trnh ng thng
d. Nh vy ta cn kho st s giao im ca (C) v d.
> Cch 1: Lp cng thc tnh khong cch t gc to n d. Bin lun s giao im ca
ng thng v ng trn bng cch so snh khong cch vi bn knh ca (C).
b nh nht cha (C). Bin lun v tr ca
> Cch 2: Tm di mt phng P hoc min gc Q
ng thng d i vi cc min phng . Nu pht hin ng thng i qua mt im trong
ng trn th h phng trnh lun c nghim phn bit.

(x p)2 + (y q)2 = r
Bi 2: Bin lun s nghim ca h phng trnh
ax + by = c

(r>0)

~ Phn tch: t u = x p; v = y p a v bi ton 1.

px2 + qy 2 = r
Bi 3: Bin lun s nghim ca h phng trnh
ax + by = c

(p, q, r > 0)

~ Phn tch: H cho gm phng trnh ng elip (E) v phng trnh ng thng d.
Nh vy ta cn kho st s giao im ca d v (E). S dng php co - dn bin (E) thnh
ng trn (C) v bin d thnh d0 , ta a v kho st s im chung ca d0 v (C). Ta bit
rng s im chung khng ln hn
r 2.
q
~ Phng php: t u = x; v =
y h cho tr thnh
p

r
2
2

u + v =
r p
()
p

au + (b
v) = c
q
T y lm tip nh bi 1. Nu pht hin ng thng i qua mt im nm trong (E) th h
lun c 2 nghim phn bit.

53

p(x k)2 + q(y h)2 = r


Bi 4: Bin lun s nghim ca h phng trnh
ax + by = c

(p, q, r > 0)

~ Phng php: t u = x k; v = y h a v bi ton 3.

px2 + kxy + qy 2 = r
Bi 5: Bin lun s nghim ca h phng trnh
ax + by = c

(p, q, r > 0; k =

1)
~ Phng php: Bin i phng trnh u v dng mu2 + nv 2 = r a v bi ton 4.

x 2 + y 2 = r
Bi 6: Kho st tnh cht nghim ca h
ax + by = c

(r > 0)

~ Phng php: Coi mi nghim ca h l mt im vi to l cp s . Bin i h


phng trnh v iu kin ca h thnh ngha hnh hc. iu kin bi ton thng lin quan
n mt s tnh cht nh tp hp im thuc phn chung ca cc na mt phng hoc min
trn, min elip hoc khong cch gia 2 im, tch v hng ca 2 vect c lp ra t cc
im , s o gc to bi 2 vect hoc 2 ng thng.

px2 + qy 2 = r
Bi 7: Kho st tnh cht nghim ca h phng trnh
ax + by = c
~ Phng php: t u =

(p, q, r > 0)

px; v = qy h cho tng ng

u2 + v 2 = r
a0 u + b 0 v = c 0

b
a
Trong a0 = ; b0 = . Bi ton a v bi 6.
p
q

F a phng trnh v t mt n sang h phng trnh hai n

Bi 1: Kho st nghim ca phng trnh a bx2 = kx + m vi a, b > 0.


r
a
~ Phng php: t y =
x2 khi ta c h
b

x2 + y 2 =

b
() kx by = m

y > 0
Bi ton a v kho st nghim ca h ().

54
Bi 2: Kho st nghim ca phng trnh
~ Phng php: t z = x + c; y =

p
a b(x + c)2 = kx + m vi a, b > 0

a2 + bz 2 v a v bi 1.

Bi 3: Kho st nghim ca phng trnh a sin t + b cos t = c vi t [; ]


~ Phng php: t x = sin t; y = cos t ta c h

ax + by = c
()
x 2 + y 2 = 1
Bi ton a v kho st nghim ca h () vi x, y tho iu kin xc nh.

Bi 4: Kho st s nghim ca phng trnh p a + bx + q c + dx = m vi bd < 0; q = 1


r
r
a
c
+ x; z =
x ta c h
~ Phng php: Coi b > 0 d < 0. t y =
b
d

a c

y2 + z2 =

b d

pby + q dz = m
()

y>0

z > 0
[ nm trong min gc P xc
Bi ton a v kho st s nghim ca h () gm cung trn AnB
nh bi cc min x > 0, y > 0 v ng thng d.

Bi tp v d
F Cc bi tp v h tuyn tnh v phn tuyn tnh
Bi 1: Bin lun s nghim ca h

(x 1)2 + (y 2)2 = 5
x + my = m + 1
Gii
t u = x 1; v = y 2 ta c h cho tng ng:

u2 + v 2 = 5
u + mv = m

55

(C)

A(0,-1)
O

H trn gm phng trnh ng trn (C) : x2 + y 2 = 5 tm l gc ta v bn knh l 5


v phng trnh ng thng d : x + my + m = 0 ph thuc m. Hn na, ng thng d lun
i qua im c nh A(0; 1) nm trong (C) nn d lun ct (C) ti 2 im phn bit vi mi
m.2

Bi 2: Tm m h c nghim x > 0; y > 1:

x 2 + y 2 = 4
()
2x y = m

Gii

iu kin ca bi ton xc nh min gc P vi x > 0; y > 1. () gm phng trnh ng trn


(C) : x2 + y 2 = 4 v phng trnh ng thng d : 2x y = m ph thuc m. Phn chung ca

[ c cc u mt A( 3; 1) v B(0; 2). Yu cu bi ton l tm m d v


(C) v P l cung AnB
[ c t nht 1 im chung.
cung AnB

56

y
d3
B(0; 2)

y=1

d2

d1

A( 3, 1)

M2

M1

M(

m
2

; 0)

Phng trnh ng thng d1 , d2 i qua A, B v cng phng vi d l 2x y = 2 3 1 v

1
2x y = 2. Cc ng thng ct Ox ti M1 ( 3 ; 0) v M2 (1; 0). Cc im nm
2
trong (C) nn giao im th hai ca d1 , d2 vi (C) thuc na mt phng P1 = {(x; y) : y < 0}.
[ l A v B.
V vy d1 , d2 ch c mt im chung vi AnB
[ ng thng d ct on thng
Gi Q l di mt phng bin d1 , d2 , khi Q cha AnB.
m
M1 M2 ti M ( ; 0) khc M1 , M2 .
2

T suy ra 2 < m < 2 3 1 l tp hp cc gi tr m tho gi thit. 2


Bi 3: Tm m h c 2 nghim phn bit tho x1 x2 + y1 y2 > 0:

x 2 + y 2 = 4
()
x + y = m

Gii

57
y

B
H
A
O

Gi s h c hai nghim phn bit l (x1 , y1 ) v (x2 , y2 ), khi ta c: Gi A(x1 ; y1 ), B(x2 ; y2 ),


th do
 

OA.OB = OA.OB. cos OA, OB = x1 x2 + y1 y2
Nn suy ra:
 
[ <
cos OA, OB > 0 AOB
2
Hn na do (x1 , y1 ) v (x2 , y2 ) u l nghim ca h nn cc im A v B u nm trn ng
trn (C) : x2 + y 2 = 4 v ng thng d : x + y = m, nh vy AB l mt dy ca ng trn
(C), ng thi AB cng chnh l ng thng d.
Nh vy h c nghim tha iu kin x1 x2 + y1 y2 > 0 th AB phi ct ng trn (C) ti
[ <
hai im phn bit, hn na phi tha mn AOB
2
|m|
AB ct (C) ti hai im phn bit dO/d < 2 < 2
2
[ < AOH
\<
Gi H l trung im ca dy AB khi OH AB. Ta c: AOB
4
2
[ < OH > 2.
AH < OH. V OH 2 + AH 2 = 4 nn suy ra AOB
2

|m|
Nh vy h c hai nghim phn bit tha x1 x2 +y1 y2 > 0 2 > > 2 2 2 > |m| > 2 2
2
Bi 4: Tm m h c 2 nghim phn bit tho (x1 x2 )2 + 4(y1 y2 )2 = 3:

x2 + 4y 2 = 16
()
x my = m
Gii
H () gm phng trnh elip (E) :

x2 y 2
+
= 1 v ng thng d : x my = m ph thuc m.
16 4

58
y

O
x

A -1

Ta c ng thng d lun i qua im c nh A(0; -1) hn na A nm trong (E). Vy d lun


ct (E) ti 2 im phn bit, do h () c 2 nghim phn bit vi mi m.
Gi B(x1 ; y1 ), D(x2 ; y2 ) v t u = x; v = 2y (1). Cng thc (1) xc nh mt php dn vi h
s 2 bng trc dn Ox. Php dn bin cc im B, D thnh B 0 (u1 ; v1 ), D0 (u2 ; v2 ) v h ()
tr thnh:

u2 + v 2 = 16
()
2u mv = 2m
To cc im B 0 , D0 l nghim ca (). H gm phng trnh ng trn (C) v phng
trnh ng thng d0 l nh ca d qua php dn .
y

P'
O
x
-2
Q'

Mt khc
B 0 D02 = (u1 u2 )2 + (v1 v2 )2 = (x1 x2 )2 + 4(y1 y2 )2 = 3

0 0
Ngha l d0 ct (C) ti 2 im B 0 , D0 sao cho
B
D
=
3 hn na (C) c tm l gc ta
r nn

0 02
B
D
61
122
|2m|
d2O/d0 +
= R2 = 16 Suy ra dO/d0 =
, do d =
nn tm c m =
2
4
2
3
4 + m2
Bi 5: Tm m h c 2 nghim phn bit v (x1 x2 )2 + (y1 y2 )2 t gi tr nh nht:

x2 + y 2 = 4
()
2x + my = m + 2

59
Gii
y

d
A
(C)

H () gm phng trnh ng trn (C) : x2 + y 2 = 4 v ng thng d ph thuc vo


m. Gi A, B l giao im ca d v (C) th ta cn tm m AB 2 t gi tr nh nht. Do
M (1; 1) d m v M nm trong (C) nn d lun ct (C) ti 2 im phn bit A, B.
Ta c: AB Min d OM ().

Gi
u = (m; 2) l vect ch phng ca d th

()
u .OM = 0 2 m = 0 m = 2
Do vi m = 2 th h c hai nghim phn bit tha (x1 x2 )2 + (y1 y2 )2 t gi tr nh nht.
Bi 6. Bin lun s nghim ca h phng trnh sau:

|x| + 2 |y| = 4
(x 2a) (y a) = 0
Gii
Xt h phng trnh:

|x| + 2 |y| = 4

(1)

(x 2a) (ya) = 0 (2)


Ta thy rng cc im tha mn (1) l bn cnh ca hnh thoi ABCD trong :
A((4; 0) ; B (0; 2) ; C (4; 0) ; D (0; 2)
ng thi, cc im tha mn (2) nm trn hai ng thng d1 : x = 2a v d2 : y = a. S
nghim ca h hai phng trnh (1) v (2) chnh l s giao im ca bn cnh hnh thoi vi
hai ng thng ni trn.
Trc tin ta tm xem khi no 3 ng thng x = 2a; y = a v |x| + 2 |y| = 4 ng quy. Gi
(x0 ; y0 ) l im chung ca ba ng thng ny, khi ta c h sau:

x0 = 2a
x0 = 2a
y0 = a

|x0 | + 2 |y| = 4

y0 = a

4 |a| = 4

60

|a| = 1
x0 = 2; y0 = 1

x0 = 2; y0 = 1
y

y
x = 2a

x=2

2 B

2 B
y=1
4

-4

y=a

-4

-2 D

-2 D

Da vo th trn ta c kt lun sau:

Nu |a| > 2: H (1), (2) v nghim.


Nu |a| = 2: H (1), (2) c hai nghim.
Nu |a| < 2 v |a| =
6 1: H (1), (2) c 4 nghim.
Nu |a| = 1: H (1), (2) C 3 nghim.
F a phng trnh v t v h phng trnh

Bi 7: Bin lun s nghim ca phng trnh

m 9 x2 x + 5m = 0 (1)
Gii
t y =

5 x2 ta c h

2
2

x + y = 9
() x my = 5m

y > 0

H () gm phng trnh ng thng d ph thuc m v na ng trn (C) xc nh bi h


bt phng trnh

x2 + y 2 = 9
y > 0
Ta c A(0; 5) d m do A nm ngoi (C)

61

y
d
(C)

-3

M2

5m

M1

Cc im M1 (3; 0), M2 (3; 0) l u mt ca ng knh ca (C). Cc tia AM1 , AM2 l bin


3
ca min gc P. ng thng d ct M1 M2 ti M (5m; 0) nn vi m tho 3 6 5m 6 3 6
5
3
m 6 th d v na trn ca (C) c duy nht 1 giao im. Trng hp ny phng trnh c 1
5
nghim.
Vi cc gi tr cn li ca m th d v na trn ca (C) khng c giao im nn phng trnh
v nghim. 2

Bi 8: Bin lun theo m nghim ca phng trnh sau:

4 x2 = mx + 2 m ()

Gii

Ta bit rng s nghim ca phng trnh () l s giao im ca hai ng y = mx + 2 m

v y = 4 x2 .

Hn na v y = 4 x2 x2 + y 2 = 4, y > 0 nn th ca y = 4 x2 l na ng trn
tm l gc ta bn knh bng hai v nm pha trn trc honh.
Cn y = mx + 2 m l mt h ng thng lun i qua im c nh A(1; 2). Ta nhn thy

c hai tip tuyn vi ng (C) : y = 4 x2 , mt l ng thng y = 3 song song vi trc


honh, v tip tuyn AD.

62

D
B

-2

Gi B(2, 0) v C(2, 0) l hai u mt ca ng knh BOC, gi s m1 , m2 , m3 , m4 tng


ng l h s gc ca cc ng thng AC, AD, AB, AE th ta c cc iu sau:

[ = 2.
m1 = tan ACO


\ = tan EAD
\ = tan 2OAE
[ = 4.
m2 = tan DCO
3
[ = 2.
m3 = tan ABO
3
m4 = 0.
T suy ra:

2
0<m6 3
1. Phng trnh () c hai nghim
4
2 6 m 6
3
2

3
m < 2

2. Phng trnh () c mt nghim


m=0

4
m=
3

m>

4
3. Phng trnh () v nghim < m < 0 2
3
Bi 9: Bin lun s nghim ca phng trnh sau theo a:



2
a 9x +x xa 3 =0

Gii

63

t y = 9 x2 x2 + y 2 = 9
Phng trnh (4) tng ng:
(

x2 + y 2 = 9, y > 0

(ay + x) x a 3 = 0

D thy phng trnh th nht biu din phn pha trn trc honh ca ng trn tm l gc

1
ta bn knh l 3 cn phng trnh th hai biu din hai ng thng x = a 3 v y = .x
a
(nu a 6= 0).
S nghim ca h chnh l s giao im ca hai ng thng vi na ng trn. Ta ch cn
xt khi a > 0 (v khi a < 0 ta c kt qu tng t v khi a = 0 th (4) x = 0 v lc h
c 1 nghim).
1
Thy rng ng thng y = x nn n i qua O v v vy lun ct na ng trn ti mt
a

im. Ta ch cn quan tm n v tr tng i ca x = a 3 vi ng trn v ng thng


1
y = x.
a

Do y > 0 nn nu ng trn v hai ng thng ng quy th phi c x = a 3 > 0 v


1
y = x < 0. iu ny l v l. Do s khng c trng hp ba ng ny ng quy. Nh
a
th nhn vo th ta s thu c kt lun:

y
y=-

1
a

y=-

x=a 3
a> 3

1
a

x=a 3
0<a< 3

Do ta c kt lun sau:
"
Phng trnh (4) c mt nghim

a=0

|a| > 3

Phng trnh (4) c hai nghim 0 < |a| <

32

Bi 10: Bin lun s nghim ca phng trnh:

9 2x x2 = x + m (()
Gii

64
Quan st mt cht ta thy bi ny c cu trc hi khc cc bi trn nhng tht ra chng nh
nhau:
p
() 10 (x + 1)2 = x + m
t z = x + 1; y =

10 z 2 ta c h:

2
2

y + z = 10
yz =m1

y > 0

Bi ton a v bin lun s im chung ca ng thng y = z + m 1 v na ng trn

y 2 + z 2 = 10
y > 0
n y tng t nh Bi 1, xin dnh cho bn c gii quyt tip phn cn li ca bi ton.
Bi 11: Bin lun s nghim ca phng trnh


2
;
2 sin t (m + 3) cos t = m 1, t
3 3

Gii

t x = sin t; y = cos t v a iu kin ca bi ton thnh:

x2 + y 2 = 1

2x (m + 3)y = m 1

6x61

1 6 y 6 1
2
2

(i)
(ii)
(iii)
(iv)

Do hai iu kin (iii) v (iv) nn ta ch cn kho st nghim trong hnh ch nht ABCD trong
:
!
!





3 1
1
1
3 1
A
;
, C 1;
,D
;
, B 1;
2 2
2
2
2 2
Do bi ton a v kho st s giao im ca ng thng d : 2x (m + 3)y = m 1 v
ng trn (C) : x2 + y 2 = 1 nm trong hnh ch nht ABCD, h c nghim th ch c nghim
duy nht.

65

A B
S
x

O
DC
I

Cc nh A v D thuc ng trn n v. Cc nh cn li nm ngoi ng trn. Hn na


ng thng d lun i qua im c nh I(2, 1), do h c nghim th ng thng d phi
!

3 3+1m
nm trong phn gii hn ca hai tia IA, ID. Li c d ct AD ti im S
,
2
m+3
Do h c nghim duy nht
1

6
2

3+1m
3+4
1
1
1
1
sqrt3 + 4
3
2 31
6
6
6 6
6 5+2 3 > 5 >
m+3
2
2
m+3
2
2
m+3
2
3

Min gc cha tp hp im tho mn h iu kin trn c bin l cc ng thng ng vi

1
2
cc gi tr ca m = ( 3 ) v m = 2 3 + 5 2
3
2

F Mt vi bi ton khc

Hai phn trn chng ta tip cn vi cch gii ton bng hnh hc, th, trong phn ny,
chng ta s thc s thy c v p ca vic gii ton bng hnh hc, cc bi ton trong phn
ny thng khng mu mc v c cch gii quyt khc nhau ty dng ton, ta bt u vi v
d sau:

Bi 12: Tm cc b s (x, y) dng tha mn phng trnh:


q
q
q

2
2
2
2
x + b bx 3 + y + a ay 3 + x2 + y 2 xy 3 = a2 + b2
Trong a, b l cc s thc dng cho trc.
Gii

66
~ tng: Cc bi ton hai phn trc ch cp n bin lun nghim ca phng trnh,
h phng trnh, nhng y l mt bi v gii phng trnh, cch tip cn hon ton khc.
Ta ch n nhn xt sau: v tri l ba cn thc c dng gn ging nh l Cosin trong tam gic,
v phi l biu thc gn ging cng thc tnh ng cho ca cnh huyn (nh l Pytagore).
Hn na li c mt kt qu quen thuc trong hnh hc phng l AM + M N + N B > AB. Do
ta i n li gii nh sau:
~ Li gii:

iu kin: x2 + b2 bx 3, y 2 + a2 ay 3, x2 + y 2 xy 3 > 0 Dng tam gic vung OAB c


OA = a, OB = b. Ox v Oy l hai ng chia ba cc gc ca tam gic, trn Ox ly M v trn
Oy ly N sao cho OM = y v ON = x (nh hnh v)
x
A

M
N

y
x

p
Khi ta c c AM = OA2 + OM 2 2OA.OM. cos = a2 + y 2 ay. 3.
6
p
p

2
2
2
2
Tng t: M N = x + y xy. 3, BN = x + b bx. 3, AB = a2 + b2 .
Hn na AM + M N + BN > AB nn:
q
q
q

2
2
2
2
x + b bx 3 + y + a ay 3 + x2 + y 2 xy 3 > a2 + b2
r

Do iu kin ca bi ton nn du bng xy ra, ni cch khc M v N ln lt l giao im


ca Ox v Oy vi AB.
n y ta ch cn tm OM , ON na l c!

x
A
y

M
a

O
b

Ta c:
SOAB = SOAM + SOBM

1
3
2ab

ab = ay +
by y =
2
2
a + 3.b

2ab
Tng t ta tm c x =
3.a + b
Do nghim ca bi ton l x =

2ab
2ab

,y =
2
3.a + b
a + 3.b

67
~ Nhn xt: Bi ton trn cho thy c vic s dng hnh hc gii ton s a bi ton
v mt bi ton kh nh nhng. Ta cng n vi v d tip theo:

Bi 13: Gii phng trnh:

r




2x2 +
3 + 1 x + 1 + 2x2
31 x+1=3

r
2x2 2x + 1 +

Gii
~ tng: Cng ging nh trn, v tri gi ta dng nh l Cosin cho tam gic nhng khng
th c. Ta s phi s dng cch gii khc.
Phn tch t biu thc n gin nht ta c c: 2x2 2x + 1 = x2 + (x 1)2 . Do tng
ca chng ta cng l phn tch cc biu thc cn li thnh tng hai bnh phng, do ta s
c li gii sau:
~ Li gii:
!
!

3 1
3 1
, , C
, .
Trong h ta Oxy ly cc im M (x, x), A(0, 1), B
2
2
2
2
y
M

x
1

O
B

Khi ta c:

M
A
=
2x2 2x + 1


M B = 2x2 +
3+1 x+1

M C = 2x2 3 1 x + 1

Nh vy V T = M A + M B + M C > T A + T B + T C vi T l im Torricelli ca tam gic


ABC.
D c c tam gic ABC l tam gic u v T chnh l gc ta , do
M A + M B + M C > 3T A = 3
Theo iu kin bi ton th du ng thc xy ra hay M T , do x = 0 2
~ Nhn xt: Bng cch phn tch cc biu thc nh trn ta hon ton c th gii bng cch

68
s dng BT Cauchy-Schwarz. Cng vic ca ta l nh gi c BT sau:
v
v
u
!2 
!2 
2 u
2
u
u
p
3
3
1
1
t
t
2
2
+
>3
x + (x 1) +
x+
+ x+
x
+ x+
2
2
2
2
C th thit lp bt ng thc trn bng cch chn tham s.
Mi cch lm u c u nhc im. Dng im Toricelli s cho li gii nh nhng, nhng nu
4ABC khng u vic tnh ton s gp kh khn. Dng tham s c th tng qut bi ton
nhng i khi nghim nhn c rt phc tp.
Di y s l mt v d vi cch gii khc hon ton vi hai bi ton trn.

Bi 14. Gii phng trnh sau:

x x + 1 + 3 x = 2 x2 + 1
Gii
~ tng: Bi ny r rng vic a cc biu thc cha cn thnh di mt on thng
gp kh khn v cc biu thc khng c dng bc hai. Tuy th trong bi ny phng php hnh
hc vn pht huy tc dng. Bin i phng trnh mt cht ta c:

x. x + 1 + 1. 3 x =

r

x+1

2

2
3 x . x2 + 1

V phi khin ta ngh n tch di cn v tri gip ta ngh n tch v hng ca hai vector,
t ta hnh thnh li gii nh sau:
~ Li gii:
iu kin: 1 6 x 6 3.

a
.
b
=
x
x
+
1
+
3x


q


t
a = (x, 1); b =
x + 1, x + 3 ta c

2
2

|
a | . b = x2 + 1.
x+1 +
3x

Hn na ta li c a . b 6 | a | . b
Do ta c:

x. x + 1 + 1. 3 x 6

r
2

2
x+1 +
3 x . x2 + 1

Nh vy theo iu kin ta phi tm x trong trng hp du bng xy ra tc l ta c:

a k b x. 3 x = x + 1 x3 3x2 + x + 1 = 0

x=1

2
(x 1)(x 2x 1) = 0 x = 1 + 2
2

x = 1 2 (loi)
Ba v d trn chng ta tip cn cch gii phng trnh bng hnh hc, sau y s l mt v
d v gii h phng trnh.

69

Bi 15: Gii h phng trnh:

2
2
2
2

x = y a + z a
y = z 2 b2 + x 2 b2
p

z = x 2 c2 + y 2 c2
Trong a, b, c l cc s thc dng cho trc tha mn

1 1 1
, , l ba cnh ca tam gic
a b c

khng t.
Gii
~ tng: R rng ngay khi nhn vo bi th gi thit "a, b, c l cc s thc dng cho
1 1 1
trc tha mn , , l ba cnh ca tam gic khng t" khin ta cm thy kh ri. Tuy
a b c
nhin nu ch n phn tch sau th mi vic hon ton d dng:
Vi mt tam gic khng t c ba cnh l a, b, c cc ng cao tng ng l ha , hb , hc v din
tch l S. ta c:

1
a

ha
2S
b
aha = bhb = chc = 2S
= hb

2S

c = hc
2S
a d c
T ch rng
,
,
l ba cnh ca tam gic khng t ng dng vi tam gic c ba
2S 2S 2S
1
1 1 1
cnh l a, b, c theo t l
do
, ,
l ba cnh ca tam gic khng t. Vy ta c li
2S
ha hb hc
gii nh sau:
~ Li gii:
iu kin a2 , b2 6 z 2 ; b2 , c2 6 x2 ; a2 , c2 6 y 2 .

z
b

c
a
x

h phng trnh c nghim th x, y, z phi dng nn suy ra a, b 6 z; b, c 6 x; a, c 6 y. Do


ta lun c th dng cc tam gic sau:

4A1 B1 C1 c A1 B1 = y, A1 C1 = z v ng cao ti nh A1 bng a ng thi hai tia


A1 B1 v A1 C1 nm v hai pha i vi ng cao ti A1 . Khi theo phng trnh th
nht d dng c c x = B1 C1 .

70
4A2 B2 C2 c B2 A2 = z, B2 C2 = x ng cao ti nh B2 l b ng thi hai tia B2 A2 ,
B2 C2 nm v hai pha i vi ng cao nh B2 th y = A2 C2 .

4A3 B3 C3 c C3 A3 = y, C3 B3 = x ng cao ti nh C3 bng c ng thi hai tia C3 A3 ,


C3 B3 nm v hai pha i vi ng cao ti nh C3 , khi z = A3 B3 .
Do ba tam gic A1 B1 C1 , A2 B2 C2 , A3 B3 C3 c ba cnh bng nhau nn chng bng nhau do
din tch ca chng bng nhau suy ra ax = by = cz = 2S.
p
x+y+z
p dng cng thc Heron ta c: S = p(p a)(p b)(p c), trong p =
2


1 1 1
2S
+ +
2S
2S
2S
x+y+z
a b c
Ta c: x =
,y=
,z=
,p=
=
. Do suy ra:
a
b
c
2
2
s




1 1 1
1 1 1
1 1 1
1 1 1
2
+ +
+ +
+
+
S=S
a b c
a b c
a b c
a b c
Do :
S = s

T ta c c: x =

1
1 1 1
+
a b c



1 1 1
+
a b c



1 1 1
+
a b c

2S
2S
2S
,y=
,z=
2.
a
b
c

Bi 16: Gii h phng trnh n (a; b; c; d) sau:

a2 + b 2 = 1
c+d=3

ac + bd + cd = 9 + 6 2
4
Gii
~ tng: R rng bi ton trn l mt bi ton khng h mu mc, ch c ba phng trnh
nhng li c ti 4 n, hn na phng trnh th 3 kh phc tp. Kh c th bin i i s,
nn chng ta ngh ti phng php hnh hc.
Vi nhng g chng ta phn tch trn, tng hnh hc cn bn ca ta s l xt ng
trn (C) : x2 + y 2 = 1 khi im M (a, b) s thuc ng trn ny. Cn phng trnh th 2
ta xt ng thng (d) : x + y = 3 th im N (c, d) s thuc ng thng ny.
Qu tch nhng im M (a, b) tha phng trnh th nht l ng trn (O; 1).
Qu tch nhng im N (c, d) tha mn phng trnh th hai l ng thng (d) : x + y = 3.
Hn na ta li c:
2(ac + bd + cd) = 2ac + 2bd + (c + d)2 c2 d2
= 2ac + 2db + 10(a2 + b2 + c2 + d2 )
= 10 (a c)2 (b d)2 = 10 M N 2

71

9+6 2
11 6 2
3 2
2
Suy ra 10 M N =
MN =
MN =
2
2
2
Do ta s tm hai im M , N nmtrn ng trn (O; 1) v ng thng (d) : x + y = 3 sao
3 2
cho khong cch gia chng l .
2
~ Li gii:
2

N
N0
M

M0
1

Gi N0 l hnh chiu ca O ln M N , v M0 v M00 l giao im ca ON0 vi ng trn n


v (O; 1).
Nhn xt rng nu h phng trnh c nghim th ta phi c N M0 6 M N 6 N M00 , do ta
s tnh N0 M0 v N0 M00 .

3
3 2
Ta c: M0 N0 = ON0 1 = 1 =
= MN
2
2
T suy ra M M0 v N N!
.
0


2 2
3 3
D dng tnh c M0
,
,
v N0 =
2 2
2 2
T suy ra nghim ca h 2
~ Nhn xt: Bi ton trn
rt c th v may mn l M N t gi tr nh nht. Trong mt
9+6 2
v trng hp s
ca ta thay bng s khc m vn tha mn h c nghim v M N
4
khng t cc tr th vic lm bi ton s rt kh khn v h s c v s nghim, do thng
thng cc bi ton dng ny s l v nghim hay c nghim duy nht.

Bi tp t luyn

x2 xy + 4y 2 = m
r
Bi 1: Bin lun s nghim theo m ca h
3

x +
y=1
7

x 2 + y 2 = 8
Bi 2: Tm m h
c nghim x, y > 0
x + y = m

x 2 + y 2 = 4
Bi 3: Tm m h
c nghim x 6 0 6 y
2x my = m + 2

72
Bi
Bi
Bi
Bi

4:
5:
6:
7:

p
Bin lun theo m s nghim ca phng trnh 5 2(x + m)2 = x 2
Bin lun theo m s nghim ca phng trnh 4 sin t 3 cos t = 2m 1 vi t [0; ]

Bin lun theo m s nghim ca phng trnh 5 4x + m + 9x = 1


Cho h bt phng trnh:

(x + 1)2 + y 2 6 a
9 + (y + 1)2 6 a

Tm a h c nghim duy nht.


Bi 8: Cho h phng trnh:

x2 + y 2 = 2 (1 + a)
(x + y)2 = 4
Tm a h c hai nghim.
Bi 9: Cho h:

x2 + (5a + 2) x + 4a2 + 2a < 0


x 2 + a2 = 4
Tm a h c nghim.
Bi 10: Cho h phng trnh:

x + mx m = 0
x 2 + y 2 x = 0
1. Bin lun s nghim ca h theo m.
2. Khi h c hai nghim (x1 , y1 ), (x2 , y2 ). Xt i lng:
d = (x2 x1 )2 + (y2 y1 )2 .
Tm m d t gi tr ln nht.
Bi 11: Cho h:

2ax + y = 1
x + 2y > a
Tm m h c t nht mt nghim m (tc l c t nht mt nghim (x, y) trong x < 0 v
y < 0)
Bi 12: Bin lun theo m s nghim ca cc phng trnh sau:

a. 16 4x2 = x m

b. 1 9x2 = m 3x

c. 2x x2 = 2m x

d. 4 x2 = 2mx x2

e. 1 x2 = 1 m2 + 2mx x2

f. 4x2 16 = m 2x
Bi 13: Tm m bt phng trnh sau c nghim:

73

a. 1 x2 > x m

b. x2 9 > x m

Bi 14: Tm m bt phng trnh sau c nghim duy nht: 12 3x2 6 x m

Bi 15: Phng trnh sau c bao nhiu nghim: 4x3 3x = 1 x2


Bi
16: Tm m cc h sau c nghim:

1 x2 = y
a.
3mx 3y = 5m

x + 1 + y + 2 = m
b.
x + y = 3m

4 x2 = y
c.
mx 3y = 2m
Bi
17: Tm m cc h sau c nghim:

x2 + x 2 + x < 1
a.
x2 2(m + 2)x + 1 > 0

x + y > 1
b.
x + y 6 2xy + m

x + y 6 2
c.
p
x + y + 2x(y 1) + a = 2
Bi
18: Tm m cc h sau c nghim duy nht:
x + y + 2xy + m > 1
a.
x + y 6 1

x + y = 1
b.
x + y 6 m
Bi 19: Bin lun theo a s nghim ca cc phng trnh:
2
a. |x|
 (3 4x ) = a.
1
b.
|x| 1 (|x| 1)2 = a.
3
a
c. x2 2x 2 =
|x 1|
x2 + 2 |x| 1
d.
= a.
|x| + 1
x2 + 2
e.
=a
|x| 1
f. (x + 1)2 a |x + 2| = 0
Bi 20: Cho cc s thc dng x, y, z tha mn h phng trnh

y2

x
+
xy
+
= 25

2
3
y
+ z2 = 9

z 2 + zx + x2 = 16
Tm gi tr biu thc P = xy + 2yz + 3xz

74
Bi 21: Cho a, b, c l cc s thc dng tha ng thc:
q
q

a2 2ab + b2 + b2 2bc + c2 = a2 + c2
a+c
b
Bi 22: Cho p, q l cc s thc sao cho phng trnh x2 + px + q = 0 c hai nghim thc c
gi tr tuyt i khng vt qu 1. Tm gi tr ln nht v nh nht ca biu thc:
Tm gi tr nh nht ca

P = p2 (p2 + 2q 2 3) + q 2 (q 2 3)
Bi 23: Kho st s nghim ca h phng trnh theo a:

x 2 y 2
(x a)2 + y 2 = 1
.

HNH HC KHNG GIAN V VIC KHO ST H


PHNG TRNH 3 N
Trong nhng bi ton sau ta quy c (x; y; z) l n cn a, b, c l tham s.

Bi tp v d:
Bi 1: Bin lun s nghim ca h phng trnh vi r > 0:

2
2
2

x + y + z = r

() ax + by + cz = m

a0 x + b0 y + c0 z = m0
tng:
H (*) gm mt phng trnh ca mt mt cu (W) vi tm l gc ta v mt ng thng
d. Ta giao im ca d v (W) l nghim ca (*). V th, bi ton c a v dng bin
lun s giao im ca d v (W).
~ Phng php:
> Vit
( phng trnh mt phng (P ) i qua O(0; 0; 0) v vung gc vi ng thng
ax + by + cz = m
d:
()
a0 x + b0 y + c0 z = m0
> Tm giao im K ca (P) v d. Xt v tr tng i ca K so vi (W).
* Nu K nm trong (W) th d ct (W) ti hai im phn bit v h (*) c hai nghim phn
bit.
* Nu K nm trn (W) th d v (W) c mt im chung v h (*) c nghim duy nht
* Nu K nm ngoi (W) th h (*) v nghim.

75
V d 1: Bin lun s nghim ca h phng trnh sau theo tham s m:

x2 + y 2 + z 2 = 22

x + y + 2z = m

x z = 2.
Gii
Phng trnh mt phng (P) i qua O v vung gc vi d l: x 3y + z =
0.

x + y + 2z = m

Gi K l giao im ca (P) v d. Khi , ta ca K l nghim ca h x z = 2

z 3y + z = 0


3m 2m 3m
Gii h ny ta c ta im K
;
;
.
11 11 11
2m2
* Nu K nm trong (W) khi v ch khi
< 22 11 6 m 6 11. Khi , h cho c hai
11
nghim phn bit.
* Nu K nm trn (W) khi v ch khi m = 11 m = 11. Khi , h cho c nghim duy
nht.
* Nu K nm ngoi (W) khi v ch khi m > 11 m < 11. Khi , h cho v nghim. 2
V d 2: Bin lun s nghim ca h phng trnh sau:

x2 + y 2 + z 2 = 16

() x + my + z = 2m 4

x z = 2
Gii
Trong
h (*) th phng trnh x + my + z = 2m 4 thuc chm ng thng c trc l
(
x + z = 4
.
y2=0
(
x + my + z = 2m 4
Do ng thng d c phng trnh
xz =2
V d i qua im c nh K (1; 2; 3).
Ta thy rng K nm trong mt cu (W): x2 + y 2 + z 2 = 16 do d ct (W) ti hai im phn
bit vi mi m. Vy h cho lun c hai nghim phn bit vi mi m.
~ M rng:
R rng l h phng trnh bi ton 1. cho vi mt cu nhn gc ta lm tm. Tuy nhin,
ta cng c th tng qut phng php trn ln vi h c phng trnh mt cu c tm khng
phi l gc ta :

(x a)2 + (y b)2 + (z c)2 = r

(r > 0)
a1 x + b1 y + c1 z = m1

a x + b y + c z = m
2

Khi , nu ta t: u = x a, v = y b, w = z c th bi ton trn s a v bi ton 1.

76
V d 3: Bin lun s nghim ca h phng trnh sau:

(x m)2 + (y 2)2 + (z + 1)2 = 16

x+y+z =1

x z = 2m
Gii
t u = x m, v = y 2, w = z + 1 h phng trnh cho tng ng vi h sau:

u2 + v 2 + w2 = 16

() u + v + w = m

u w = m 1
n y ta ch cn bin lun s nghim ca h (*) nh phng php bi ton 1.
Bi 2: Bin lun nghim ca h phng trnh:
(p
r x2 y 2 = px + qy + k
ax + by = c
Phng php:
p
Bng cch t z = r x2 y 2 th h cho tng ng vi h:

x2 + y 2 + z 2 = r

z > 0

ax + by = c

px + qy z + k = 0
H ny gm phng trnh ca na mt cu (W) v ng thng d. Bi ton cng chuyn v
dng bi 1.
V d 1: Bin lun s nghim ca h phng trnh sau
(p
6 x2 y 2 = 2x + m
xy =2
Gii
t z =

6 x2 y 2 . H phng trnh cho tr thnh:

x2 + y 2 + z 2 = 6

z > 0
()

2x z = m

x y = 2

H (*) gm na mt cu (W) v ng thng d cha tham s m.


Gi (P) l mt phng i qua tm mt cu (W) v vung gc vi d. Phng trnh mt phng

77
(P ) : x + y + 2z = 0
Gi H l giao im ca d v (P), ta H l nghim ca h phng trnh

x + y + 2z = 0

2x z = m

x y = 2



1 m m 5 m + 2
T ta tm c H
;
;
3
3
3
Vy ng thng d ct na mt cu (W) ti hai im phn bit khi v ch khi

m+2

>0

m2  m




2
2
31 >m>2
21
+
+2 >6

4
2

(m 1) + (m + 5)2 + (m + 2)2

<6
9
ng thng d v na mt cu (W) c mt im chung duy nht khi v ch khi:

m+2

>0



3

m
=
2
3

(m 1)2 + (m + 5)2 + (m + 2)2

=6
9
Kt lun:


* Khi m = 2 3 1 th h cho c nghim duy nht



* Khi 2 3 1 > m > 2 2 1 th h c hai nghim phn bit
* Mi gi tr cn li ca m lm h v nghim. 2
~ Nhn xt: Cng bng cch lm nh m rng ca bi ton 1, ta c th m rng cho bi
ton 2. vi h c phng trnh mt cu khng nhn gc ta lm tm:
q
r (x m)2 (y n)2 = px + qy + k
ax + by = c
By gi ta s m rng thm vi h gm hai phng trnh mt cu.

Bi 3: Bin lun nghim ca h phng trnh:

(x a1 )2 + (y b1 )2 + (z c1 )2 = r1

() (x a2 )2 + (y b2 )2 + (z c2 )2 = r2

ax + by + cz = m
tng:
Ta nhn thy h (*) c hai phng trnh mt cu v mt phng trnh mt phng. Nu hai
mt cu khng c im chung th h v nghim. Nu hai mt cu c im chung th h hoc
v nghim hoc c nghim ph thuc vo im chung thuc hoc khng thuc mt phng
ax + by + cz = m.

78
~ Phng php:
Tr hai v ca hai phng trnh u ta thu c mt phng trnh mt phng c dng:
(a1 a2 ) x + (b1 b2 ) y + (c1 c2 ) z = m0 . Ta gi mt phng ny l mt ng phng ca hai
mt cu. D dng nhn thy mt ng phng ny cha cc im chung ca hai mt cu v
vung gc vi ng thng i qua tm ca hai mt cu.
* Nu hai mt cu tip xc nhau ti K th mt ng phng l tip din chung ca hai mt
cu ti K.
* Nu hai mt cu ct nhau th giao tuyn ca chng l ng trn nm trong mt ng
phng.
Nh vy, vic kho st h phng trnh trn chuyn v bi ton kho st h tng ng l:

(x a1 )2 + (x b1 )2 + (x c1 )2 = r1

(a1 a2 ) x + (b1 b2 ) y + (c1 c2 ) z = m0

ax + by + cz = m

Vic kho st phng trnh ny chnh l ni dung ca bi ton 1.

Bi tp t luyn:
Bi1: Bin lun s nghim ca cc h phng trnh sau:

x2 + y 2 + z 2 = 2m2 2

a) x + y + z = 0

x z = m + 4
(p
45 x2 y 2 = m + 2x
b)
xy =1

x2 + y 2 + z 2 = 29

c) mx + y + z = 3m

x z = 2
q
6 (x 2)2 (y + 2)2 = 2x + m
d)
x y = 2
Bi 2: Tm tt c cc gi tr ca m h phng trnh sau c hai nghim phn bit (x1 ; y1 ; z1 )
v (x2 ; y2 ; z2 ) biu thc (x1 x2 )2 + (y1 y2 )2 + (z1 z2 )2 t gi tr nh nht:

2
2
2

x + y + z = 12
mx + y + z = m 2

x + y = 2

79

MT S BI PHNG TRNH, H PHNG TRNH


C THAM S TRONG CC K THI OLYMPIC
Bi 1: Gii v bin lun h phng trnh sau theo m,n,p,k:

p
2+

m
y m2 = k

(I)
y n2 + z n2 = k

pz p2 + px p2 = k

k 3
Trong m, n, p, k > 0 v tha m + n + p =
2
Gii
Trc ht ta s chng minh (I) nu c nghim th l nghim duy nht. Tht vy: Gi s
(x1 ; y1 ; z1 ) v (x2 ; y2 ; z2 ) l 2 nghim phn bit ca (I). Do vai tr ca x,y,z nh nhau nn ta
gi s x1 > x2
Khi t phng trnh th nht ca (I) ta c:
p
p
p
p
x1 m2 + y1 m2 = x2 m2 + y2 m2
Suy ra y2 > y1
Hon ton tng t t phng trnh th hai ca h ta chng minh c z1 > z2
T phng trnh th 3 ta suy ra c x2 > x1 , iu ny mu thun vi gi thit.
Vy (I) c nghim duy nht.
Li c

m, n, p, k > 0

m + n + p = k
2
Nn tn ti mt 4ABC u cnh k v mt im I nm trong tam gic sao cho khong cch
t I n cc cnh l m, n, p.

Xt 4F IA c IA2 IF 2 = AF 2 IA2 m2 = AF

Tng t ta c IB 2 m2 = BF

Do IA2 m2 + IB 2 m2 = AF + BF = k.
2
2
2
T phng trnh 1 ca
h ta suy ra x = IA , y = IB x = IC

x = IA2

Vy (I) c nghim: y = IB 2

z = IC 2

x = IA2

Vy vi mi m, n, p, k tho bi th (I) y = IB 2 2

z = IC 2
Bi 2: Xc nh gi tr ca a sao cho phng trnh sau c 4 nghim lp thnh cp s nhn:
16a4 a3 + (2a + 7) x2 ax + 16 = 0 (1)

80
Gii
Gi s a R l gi tr m phng trnh cho c 4 nghim phn bit lp thnh cp s nhn
vi cng bi l q.
> D thy x=0 khng l nghim ca phng trnh nn q 6= 0
> Nu (
q=1 th (1) c 4(nghim bng nhau v bng 1 (hoc -1)
a=4
a = 4
Ta c:

2a + 1 = 6
2a + 1 = 6
c hai h phng trnh trn u khng th xy ra. Vy q 6= 1.
> Nu q = 1: Cc nghim ca (1) ln lt l x; x; x; x a = 0
Khi (1) q 6= 1; q 6= 0: khng mt tnh tng qut, ta gi 4 nghim l q, q 2 , q 3 ( 6=
0 v chn |q| > 1) || < |q| < |q 2 | < |q 3 |

1
cng
l
nghim
ca
(1)
i
=
0;
3
Mt khc
q i
1
1
1
1
V
>
>
>
2
||
|q|
|q |
|q 3 |
2
1
3
q = q = 3

1 1
Suy ra 4 nghin ca phng trnh l , 3 , 3 , 1 Theo nh l Viete ta c:

1
1

+ 3 + 3 + 1 =
16
1
2
4
4

3 + 3 + 1 + 1 + 3 + 3 = 2a + 37
16
1
1
t z = 3 + 3 , z > 2, ta c:

z 3 2z =
 15
15
16
4
2
3

3z
=
2
z

2z

z 4 2z 3 3z 2 + 4z +
=0
2a

15

6
6
z 4 3z 2 =
16
5
Gii phng trnh trn kt hp vi |z| > 2, ta tm c z = . Vy a = 170.
2

1 1
Ngc li vi a = 170 phng trnh cho c 4 nghim l
; ; 2; 8 to thnh mt cp s
8 2
nhn vi q = 4. Vy a = 170 l gi tr cn tm. 2
Bi 3: Tm tt c gi tr nguyn dng ca n sao cho h phng trnh sau c nghim
dng:

x1 + x2 + ... + xn = 9
1 + 1 + ... + 1 = 1
x1 x2
xn
Gii
Gi s h cho c nghim x1 ; x2 ; ...; xn . Theo bt ng thc Cauchy ta c:


1
1
1
(x1 + x2 + ... + xn )
+
+ ... +
> n2 n2 6 9 n 6 3
x1 x2
xn

81

x1 + x2 + x3 = 9
> Vi n = 3 th h
c nghim dng x1 = x2 = x3 = 3.
1 + 1 + 1 =1
x1 x2 x3
(
x1 + x 2 = 9
x1 + x2 = 9
> Vi n = 2, xt h:

1
1

+
=1
x1 x2 = 9
x1 x2
Ta thy phng trnh t2 9t + 9 = 0 c 2 nghim phn bit dng. Vy m = 2 tha mn yu
cu bi.

x1 = 9
> Vi n = 1, ta c h:
1 =1
x1
D thy h trn v nghim.
Nh vy n {2; 3} l gi tr cn tm. 2
Bi 4: Tm tt c gi tr ca tham s m h sau c 5 nghim thc:
( 3
x mx y = 0
(I)
y 3 my + x = 0
Gii
Nhn thy h cho lun c nghim
x = y = 0 m
( 4
x mx2 xy = 0
Xt x 6= 0, y 6= 0 ta c (I)
y 4 my 2 + xy = 0
T ta c x4 + y 4 = m(x2 + y 2 ) m > 0.
Bi ton cho tr thnh: tm m > 0 h c 4 nghim (x; y) 6= (0; 0)
Th y = x3 mx vo phng trnh th hai ca h, ta c:
3

x2 (x2 m) m(x2 m) + 1 = 0 (1)


( 4
t + mt3 mt + 1 = 0
2
t t = x m, (1) tng ng vi: ()
t > m
1
t u = t , (*) tr thnh: u2 mu + 2 = 0 (2)
t

Ta phi c > 0 m > 2 2. Khi (2) c 2 nghim

m
+
m2 8

u1 =
2

m2 8
u =
2
2
Xt hai phng trnh sau:

t 1 = u1 f (t) = 2t2 ( m2 8 m)t 2 = 0 (3)


t

t = u2 g(t) = 2t2 + ( m2 8 + m)t 2 = 0 (4)


t
Ta c f (m).f (0) < 0 v g(m).g(0) < 0
Nn (3) c 2 nghim phn bit m < t1 < 0 < t2
V (4) c 2 nghim phn bit m < t3 < 0 < t4

82

ng vi mi t s c hai gi tr ca x = t + m.

Vy h cho c 4 nghim ta phi c t1 = t3 , t2 = t4 u1 = u2 m = 2 2

Th li thy ng. Vy m = 2 2 l gi tr cn tm. 2


Bi 5: Tm iu kin ca cc s a,b h phng trnh sau y c t nht mt nghim vi
x1 > 0, x2 > 0, x3 > 0, x4 > 0 :

x x2 = a

1
x3 x4 = b

x + x + x + x = 1
1

Gii
Gi s h cho c t nht mt nghim x1 > 0, x2 > 0, x3 > 0, x4 > 0.
Ta c |a| = |x1 x2 | < |x1 | + |x2 | = x1 + x2
|b| = |x3 x4 | < |x3 | + |x4 | = x3 + x4
T suy ra |a| + |b| < x1 + x2 + x3 + x4 = 1
Ngc li nu c |a| + |b| < 1 th c = 1 (|a| + |b|) > 0
c
c
Khi ty theo a; b m hay dng m h cho c nghm: x1 = |a| + , x2 = , x3 =
4
4
c
c
|b| + , x4 =
4
4
c
c
c
c
x1 = , x2 = |a| + , x3 = |b| + , x4 =
4
4
4
4
c
c
c
c
x1 = |a| + , x2 = , x3 = , x4 = |b| +
4
4
4
4
Cc nghim ny u tho yu cu bi ton. 2
Bi 6: Cho a, b, c 6= 0. Gii v bin lun h sau theo a, b, c:

a x y + z. x + y z = x yz

(I) b x + y z. y + z x = y xz

cy + z x.x + y z = z xy

Gii
Xt hai trng hp sau y:
~ Nu c t nht mt trong ba s a, b, c < 0. Ta gi s a < 0. Khi t phng trnh u ca
h ta suy ra c hai v u phi bng 0, tc l ta c:
(
(
x yz = 0
xyz = 0

x y + z. x + y z = 0
(x y + z)(x + y z) = 0
T n iu kin cho biu thc trong cn c ngha th tho mn phng trnh th
nht ca h ta phi c: x = 0, y = z > 0 hoc y = 0, x = z > 0 hoc z = 0, x = y > 0
Thay li vo hai phng trnh sau thy ng. chnh l cc nghim ca h phung trnh
trong trng hp ny.

83
~ Nu a, b, c > 0: Khi h cho tng ng vi h

a2 (x y + z)(x + y z) = x2 yz

(II) b2 (x + y z)(y + z x) = y 2 xz

c2 (y + z x)(x + z y) = z 2 xy
R rng (II) lun c nghim (0; 0; 0). Ta th tm nghim khc ca h:
Nhn v vi v ca ba phng trnh trong h trn ri ly cn bc hai, ta c:
abc(x + y z)(x + z y)(y + z x) = x2 y 2 z 2
T ta i n h phng trnh sau:

x= 2

2
2
2

b + c 2 a2
a y + b x = 2abc
abc(y + z x) = a yz
bc(y + z x) = ayz

b
ac(x + y z) = bxz abc(x + y z) = b2 xz b2 z + c2 y = 2abc y = 2

a + c2 b 2

c2 x + a2 z = 2abc
abc(x + z y) = c2 yx
ab(x + z y) = cyx

z =
2
a + b2 c 2
x, y, z > 0, ta phi c: a2 + b2 > c2 , b2 + c2 > a2 , c2 + a2 > b2
Tm li ta i n kt lun sau:
> Nu c t nht mt trong ba s a, b, c < 0 th nghim ca h l: (0; ; ), (; ; 0), (; 0; )
vi > 0 tu .
>Nu c ba s a, b, c u dng th:
* Nu a2 + b2 > c2 , b2 + c2 > a2 , c2 + a2 > b2 th nghim ca h l (0; 0; 0) v
a
b
c
( 2
;
;
)
b + c 2 a2 a2 + c 2 b 2 a2 + b 2 c 2
* Nu c t nht mt trong cc bt ng thc sau: a2 + b2 6 c2 , b2 + c2 6 a2 , c2 + a2 6 b2 th
nghim ca h ch l (0; 0; 0) 2
a
b
c
+
+
= 0. Chng minh rng phng trnh ax2 + bx + c = 0
m+2 m+1 m
c nghim x (0; 1)

Bi 7: Gi s

Gii
a
b
c
Xt hm s f (x) =
xm+2 +
xm+1 + xm , x [0, 1]
m+2
m+1
m
f 0 (x) = axm+1 + bxm + cxm1 = xm1 (ax2 + bx + c)
a
b
c
Ta li c: f (0) = 0 v f (1) =
+
+
=0
m+2 m+1 m
Do theo nh l Lagrange th tn ti x0 (0, 1)sao cho f (1) f (0) = (1 0)f 0 (x0 )
Nn f 0 (x0 ) = 0 x0 m1 (ax0 2 + bx0 + c) = 0
ax0 2 + bx0 + c = 0
Vy ta c iu phi chng minh. 2
Bi 8: Cho a, b, c, d l 4 s i mt khc nhau. Chng minh phng trnh sau c 3 nghim
phn bit:
(x a)(x b)(x c) + (x b)(x c)(x d) + (x c)(x d)(x a) + (x d)(x a)(x b) = 0

84
Gii
Xt phng trnh:
g(x) = (x a)(x b)(x c)(x d) = 0 (1)
R rng ta c: g(a) = g(b) = g(c) = g(d) = 0
Khng gim tnh tng qut gi s: a < b < c < d.
t v tri phng trnh cho l f (x). Ta thy: f (x) = f 0 (x).
Theo nh l Lagrange ta thy tn ti x1 , x2 , x3 sao cho a < x1 < b < x2 < c < x3 < d. m
f (b) f (a) = (b a)f 0 (x1 ); f (c) f (b) = (c b)f 0 (x2 ); f (d) f (c) = (d c)f 0 (x3 ) .
T suy ra f 0 (x1 ) = f 0 (x2 ) = f 0 (x3 ) = 0 vy f (x) = 0 c 3 nghim phn bit x1 , x2 , x3
Mt khc v deg f (x) = 3 nn cng l 3 nghim ti a ca phng trnh cho. 2
Bi 9: Cho phng trnh: x3 2002x2 + 2001bx 2000a = 0 Tm gi tr ln nht ca a
sao cho tn ti b sao cho phng trnh trn c 3 nghim trn [ 2002, 2002]
Gii
Gi s x1 , x2 , x3 l 3 nghim ca phng trnh. Khi theo nh l Viete cho phng trnh bc
ba ta c:

x + x2 + x3 = 2002

1
(I) x1 x2 + x2 x3 + x3 x1 = 2001b

x x x = 2000a
1 2 3

Xy ra cc trng hp sau:
> Nu x2 < 0 hoc x3 < 0 khi x1 + x2 + x3 < x3 6 2002. iu ny mu thun vi phng
trnh th nht ca (I)
> Nu x1 < 0 6 x2 6 x3 , t phng trnh ba ca (I) suy ra a 6 0.
> Nu 0 6 x1 6 x2 6 x3 , theo bt ng thc Cauchy ta c:

20023
3
2002 = x1 + x2 + x3 > 3 3 x1 x2 x3 = 2000a a 6
54000
2002
20022
v khi y b =
3
6003
20023
20022
v b =
By gi ta xt bi ton trong trng hp a =
54000
6003
Khi phng trnh cho tr thnh:
Du = xy ra khi x1 = x2 = x3 =

x3 2002x2 +

20022
20022
2002 3
x
=
0

(x

) =0
3
33
3

Phng trnh trn c 3 nghim x1 = x2 = x3 =


Vy amax =

20023
l gi tr cn tm 2
54000

2002
[-2002,2002]
3

Bi 10: Tm tt c gi tr ca tham s a sao cho phng trnh sau c ng hai nghim


phn bit:
(
x + 4 |y| = |x|
|y| + |x a| = 1

85
Gii
Ta xt hai trng hp sau:
> Nu x > 0 th y = 0 Khi :

Vi a > 1 th h c hai nghim phn bit vi x > 0 l (a 1; 0), (a + 1; 0).


Vi 1 6 a 6 1 h ch c mt nghim vi x > 0 l (a + 1; 0).
Vi a < 1 th h cho v nghim.
x
x
> Nu x < 0: Ta c |y| =
v |x a| = 1 +
2
2

1+ >0
x > 2

2
x
x = 2(1 + a)
xa=1+

Hay
2

x = 2(a 1)
x a = 1 x
3
2
2
Mt khc 2 6 2(a + 1) < 0 2 6 a < 1 v 2 6 (a 1) < 0 2 6 a < 1.
3
R rng nu (x; y) l mt nghim ca h vi x < 0 th y 6= 0 v (x; y) cng l mt nghim
ca h. Do :
Khi a < 2 th h v nghim.
2
Khi a = 2 ta c x = 2(a + 1) = 2 v x = (a 1) = 2 nn h c ng hai nghim
3
phn bit (2; 1).
2
Khi 2 < a < 1 th 2(a + 1) 6= (a 1) v hai gi tr ny cho h c 4 nghim.
3
Khi 1 < a < 1 th h c 3 nghim.
"
Tm li h cho c ng hai nghim khi

a>1
a = 2

Bi 11: Tm a, b, c, d sao cho phng trnh sau c nghim x R:


20

(2x 1)40 (ax + b)40 = (x2 + cx + d)

(1)

Gii
1
2
a = 2b
=0 c
(2)
+d= 1
2
4

V (1) ng vi mi x nn n cng ng khi x =


20
40
1
a
1 c
Vi x = , (1) ( + b) + ( + + d)
2
2
4 2

Thay a = 2b vo (1) ta c:
20

(2x 1)40 = (2bx + b)40 + (x2 + cx + d) x

86
H s x40 ca hai v l 240 v 240 b40 + 1 do suy ra 240 = 240 b40 + 1 b =

1 40
240 1
2

(1) tr thnh
20

(2x 1)40 = b40 (2x + 1)40 + (x2 + cx + d)


20

(2x 1)40 (1 b40 ) = (x2 + cx + d)


1
20
(2x 1)40 40 = (x2 + cx + d) x
2
40

1
20
= (x2 + cx + d) x
x
2

2
1
x
= (x2 + cx + d) x
2

2
1
1
= x2 + cx + d x c = 1 c = 1 v d = (tho (2))
2
4
2
1
= (x2 + cx + d) x: khng c c, d tha mn yu cu bi ton.
2
1
1
Vy a = 2b, b = 40 240 1, c = 1 v d = l cc gi tr cn tm 2.
2
4

> Xt khi x

> Xt khi x

Bi 12: Cho h phng trnh sau (n x, y):


(
x = y2 + a
y = x2 + b
vi a, b l cc tham s thc. Bit rng h phng trnh ny c nghim duy nht (x0 , y0 ).
Tnh gi tr ca tch P = x0 y0 .
Gii
u tin ta cng xem qua li gii sau:
Gi s vi cc gi tr a, b no , h cho c nghim duy nht l (x0 , y0 ). t x0 y0 = k R.
> Nu x0 = 0 th y0 = b v b2 + a = 0, suy ra a 6 0.
Trong h cho, thay y t phng trnh th hai ln phng trnh th nht, ta c
2

(x2 + b) + a = x x4 + 2bx2 + b2 + a = x x(x3 + 2bx 1) = 0 ()


Ta xt phng trnh x3 + 2bx 1 = 0v t f (x) = x3 + 2bx 1, x R.
Ta thy hm ny lin tc trn R v f (0) = 1 < 0, lim f (x) = + nn f (x) = 0 hay phng
x+

trnh x + 2bx 1 = 0 c t nht mt nghim dng x = x1 > 0.


D thy rng iu kin x tha mn bi l x = y 2 + a > a nn c hai nghim nu
x = x0 = 0, x = x1 ca phng trnh () u tha mn. Mi gi tr x cho ta ng mt gi tr
y nn trong trng hp ny h c t nht hai nghim phn bit, khng ng vi bi.
> Nu y0 = 0 th cng xt tng t nh trn.
k
> Nu x0 , y0 6= 0. Thay y0 =
vo hai phng trnh ca h cho, ta c:
x0

( 3
x0 = k + a

x0 = k 2 + ax20
x20

k = k 2 + ax20 + bx0 k 2 k + (ax20 + bx0 ) = 0


3

k
k = x0 + bx0

= x20 + b
x0

87
H c nghim duy nht nn phng trnh bc hai theo k cng c nghim duy nht, tc l
1
1
k = . Vy nu h c nghim duy nht th tch P = x0 y0 = .
2
2
~ Nhn xt: Li gii trn tuy nhn hp l nhng tht ra c mt sai lm rt tinh vi, l
khi cho rng h cho v phng trnh thu c sau php th, cn gi l phng trnh h qu,
l tng ng vi nhau. y, r rng nu h phng trnh ban u c nghim duy nht th
cha hn phng trnh cui theo bin k k 2 k + (ax20 + bx0 ) = 0 c nghim duy nht, trong
vn c th c mt nghim khng tha mn bi. Cch bin i i s dng cho bi ny kh
c th tm ra c kt qu chnh xc.
Li gii ng l nh sau:

Trong h phng trnh cho, thay y bi x2 + b vo phng trnh trn, ta c:


2

x = (x2 + b) + a
D thy cc nghim ca phng trnh ny u tha mn bi v x > a, m mi nghim nh
th cho ta mt gi tr tng ng ca y nn iu kin h cho c nghim duy nht l
phng trnh n x trn c nghim duy nht.
Phng trnh bc 4 c nghim duy nht khi n c nghim kp, iu c ngha l nghim
cng l nghim ca phng trnh o hm bng 0.
Gi x0 l nghim (tng ng vi gi tr y0 ) th ta c h sau
(

x0 = (x20 + b) + a
4x0 (x20 + b) = 1

1
Hn na, do x0 l nghim ca h ban u nn x20 + b = y0 , suy ra x0 y0 = . Vy nu h cho
4
1
c nghim duy nht th tch P = x0 y0 = .
4
M RNG:
gii quyt trn vn bi ton, chng ta s lm r hn cc vn sau:
Vn 1: Nu phng trnh bc 4 c nghim duy nht khi n c nghim kp, iu c ngha
l nghim cng l nghim ca phng trnh o hm bng 0.
~ Li gii:
Xt a thc P (x) = x4 + ax3 + bx2 + cx + d, gi s a thc ny c nghim duy nht l x = x0 .
Khi , theo nh l Bezut, ta c th phn tch thc cho thnh P (x) = (x x0 )Q(x) vi
Q(x) l a thc bc 3 a thc bc ba Q(x) lun c t nht mt nghim nn nghim cng
phi l x = x0 . Ta li phn tch a thc ban u thnh P (x) = (x x0 )2 R(x) vi R(x) l a
thc bc 2
Suy ra P 0 (x) = 2(x x0 )R(x) + (x x0 )2 R0 (x) = (x x0 ) [2R(x) + (x x0 )R0 (x)].
a thc ny cng c nghim l x = x0 nn ta c pcm. 2
Vn 2: Tm iu kin cn ca hai tham s a, b h cho c nghim duy nht.
~ Li gii:

88
T cch gii phn tch trn, ta thy rng a, b phi tha mn iu kin:

(
2
2
x = (x2 + b) + a
x = (x2 + b) + a
x2 + b

+ a = x 3x2 4ax b = 0(**)

x2 + b = 1
4x
4x(x2 + b) = 1
4x
4ax + b
Suy ra x2 =
. Thay vo phng trnh th hai ca h trn, ta c
3




4ax + 4b
4ax + 4b
3 16bx
4x
= 1 4x
= 1 16ax2 + 16bx 3 = 0 x2 =
3
3
16a
Do

3 16bx
4ax + b
9 16ab
=
x=
16a
3
64a2 + 48b
Thay vo phng trnh (), ta c
2



9 16ab
9 16ab
b=0
3
4a
64a2 + 48b
64a2 + 48b
2

3(9 16ab)2 4a(64a2 + 48b)(9 16ab) b(64a2 + 48b) = 0


256(a3 + b3 + a2 b2 ) + 288ab = 27 256(a + b2 )(b + a2 ) = 27 32ab
Biu thc ny i xng gia a v b nn nu lp li qu trnh bin i trn vi y th ta cng c
kt qu tng t. Do , iu kin cn ca hai tham s a, b h cho c nghim duy nht
l a, b phi tha mn ng thc sau
256(a + b2 )(b + a2 ) = 27 32ab


1 1
Mt cp gi tr p tha mn ng thc trn l (a, b) =
,
v tng ng vi nghim duy
4 4


1 1
,
2.
nht ca h l (x0 , y0 ) =
2 2
Vn 3: Bi ton m rng.
Bi ton trn c th m rng ra cho bc cao hn ca x v y nh sau:
Bi 12*: Cho h phng trnh sau (n x, y) vi a, b l cc tham s thc:
(
x = y 2n + a
, n N
2n
y =x +b
Bit rng h phng trnh ny c nghim duy nht (x0 , y0 ). Hy tnh tch Pn = x0 y0 .
~ Li gii:
2n
Thay y = x2n + b vo x = y 2n + a, ta c x = (x2n + b) + a.
a thc ny bc chn nn chng minh tng t nhn xt (1) nu trn, ta thy a, b phi tha
(
2n
x0 = (x2n
+a
0 + b)
(2n)2 x2n1
(x2n
0
0 + b)

2n1

=1

Li thay y0 = x2n
0 + b vo ng thc trn, ta c
(2n)2 x2n1
y02n1 = 1 x0 y0 = (4n2 )
0

12n

89
12n

Vy nu h cho c nghim duy nht th Pn = (4n2 )


2.
Bi ton ban u l trng hp c bit khi n = 1.
Ch rng ta cho bc y chn x0 y0 nhn mt gi tr, bi ton hon ton c th m rng
vi s m ty nhng trong trng hp bc l th c th c 2 gi tr ca x0 y0 .
Ngoi ra kt qu th v t bu thc iu kin cn trn cng cho ta mt bi Bt ng thc
hay:
1
Ta li thy rng nu xt gi thit a + b > 0 th c th chng minh rng ab 6 . Tht vy, nu
16
c mt s m, mt s dng th iu ny ng. Ngc li th ch cn dng BT AM-GM kt
hp vi n ph nh gi.
1
R rng iu kin a + b > 0 c th c thay th bng a + b > c vi c 6 . y c th chn
8
1
c=
.
2012
Chuyn h cho v
( 2
x y =b
y2 x = a
R rng phng trnh trn chuyn c thnh phng trnh di thng qua mt php hon
i v tr gia hai bin.
Nh vy ta c bi ton sau:
Bi 12**: Cho a thc P (x, y) = x y 2 vi x, y R\{0}.
Xt a, b R sao cho tn ti ng mt cp s thc x0 , y0 R\{0} tha mn P (x0 , y0 ) = b v
P (y0 , x0 ) = a.
1
Chng minh rng nu a + b >
th 4ab 6 x0 y0 .
2012

Sau y l mt bi ton v h bt phng trnh:


Bi 13: Tm a h bt phng trnh c nghim:

x2 + 2xy 7y 2 > 1 a ()
1+a
2
2
3x + 10xy 5y 6 2
Gii
~ iu kin cn: Gi s h bt phng trnh c nghim (x0 ; y0 ) v a0 l mt trong cc gi tr
cn tm. Vy th:

x0 2 + 2x0 y0 7y0 2 > 1 a0


2x0 2 4x0 y0 + 14y0 2 6 2 4 (1)
1 + a0
1 + a0

2
2
2
2
3x0 + 10x0 y0 5y0 6 2
3x0 + 10x0 y0 5y0 6 2 (2)
T (1) v (2) suy ra:
x0 2 + 6x0 y0 + 10y0 2 6

4
4
(x0 + 3y0 )2 6
1 + a0 < 0 a0 < 1
1 + a0
1 + a0

90
Vy iu kin cn h c nghim l a < 1.
~ iu kin : Vi a < 1 th 1a
< 1.
(1+a2
x + 2xy 7y 2 = 1
Do nu h phng trnh (I)
c nghim th h bt phng trnh
3x2 + 10xy 5y 2 = 2
cho c nghim.
Ta xt h (I):

1
3
( 2
;y =
x=
x + 2xy 7y 2 = 1

2
2
(I)

2
1
3
(x + 3y) = 0
x = ;y =
2
2
H (I) c nghim nn h bt phng trnh cho c nghim.
Tm li, h bt phng trnh ban u c nghim khi v ch khi a < 1 2
~ Nhn xt: y l mt bi ton tng i kh, tng nm trong k thi i hc. iu c bit
ca li gii trn nm iu kin khi chuyn vic chng minh h bt phng trnh c nghim
v vic gii h phng trnh ng cp bc hai. Tuy nhin, mu cht ca bi ton li nm
iu kin cn. Cc bn bao gi ngh, lm cch no a c h bt phng trnh u
bi v dng f 2 (x0 , y0 ) 6 g(a) cha? Hay ni cch khc, ti sao ngh ti vic nhn s (2) vo
bt phng trnh (*)? Chng ta hy cng xem qua tng sau y:
Trc ht, ta thy rng:
(
a=b=0

c>0

> ax2 + bx + c > 0 x R (


a>0

60
(
a=b=0

c60

> ax2 + bx + c 6 0 x R (
a<0

60
(
a=b=0

c>0

> ax2 + bx + c = f 2 (x)x R (


(vi f (x) = x + )
a>0

=0
(
a=b=0

c>0

>ax2 + bxy + cy 2 = f 2 (x, y) x, y R (


(vi f (x, y) = x + y)
a, c > 0

b2 4ac = 0
Mc tiu ca chng ta l a h bt phng trnh v dng 0 6 f 2 (x0 , y0 ) 6 g(a), chnh v l
do t bt phng trnh (*) ta cn nhn vi mt s < 0. Ta c:

x2 + 2xy 7y 2 6 1 a
x2 + 2xy 7y 2 > 1 a
1+a
1+a
2
2
2
2
3x + 10xy 5y 6 2
3x + 10xy 5y 6 2
1a
( + 3)x2 + 2( + 5)xy (7 + 5)y 2 6
2 (3)
1+a

91
By gi, n v tri ca (3), ta cn bin i v tri ny v bnh phng ca mt biu thc
vi mi x, y. iu ny ch tng ng vi iu kin sau:
(
(
+3>0
+3>0

2 2
2
( + 5) y + ( + 3)(7 + 5)y = 0
( + 5)2 + ( + 3)(7 + 5) = 0

(
= 2
+3>0

5
82 + 36 + 40 = 0
=
2
Tuy nhin, bi ton ny, ta li thy c hai gi tr ca . Ta cn chn la s no s thch
hp.
Ta c:
5(1 a)
2(1 a)
2(1 a) 5(1 a)

2 [
2] =

2(1 + a)
1+a
1+a
2(1 + a)
1a
1a
5(1 a)
2(1 a)
=
> 0 ( do
< 0)
2>
2
2(1 + a)
1+a
2(1 + a)
1+a
Nh vy, t bt phng trnh (3) ta suy ra cn nhn vo hai v ca bt phng trnh (*) vi
s = 2.
T ta cng c th sng to nhiu bi ton mi, v d nh bi sau:
Bi
Tm a cc h bt phng trnh sau c nghim:
13*:
2
5x 4xy + 2y 2 > 3
a)
7x2 + 4xy + 2y 2 6 2a 1
2a + 5

5x2 + 7xy + 2y 2 > 3a + 1


a+2
b)
2
2
3x + xy + y 6 1
2
3x 8xy 8y 2 > 2
c)
x2 4xy + 2y 2 6 a + 1
2a + 1

Chng III: CC PHNG PHP GII PHNG


TRNH

PHNG PHP DNG LNG LIN HP


L thuyt
Phng php lng lin hp l mt cch gii hiu qu i vi phng trnh, h phng trnh
v t. tng chnh ca phng php l trc cn thc v to nhn t chung. Mun vy, ta
cn phi bit c nghim v bin i kho lo.
1) Dng my tnh b ti gii nghim ca phng trnh:

V d tm nghim ca phng trnh x2 x + 3 + x2 + x + 4 = 7:


u tin ta cn nhp vo my tnh b ti:

X2 X + 3 +

X2 + X + 4 7

Sau , bm Shift CALC


Sau khi my tnh gii ra nghim, ta nhn thy phng trnh trn c nghim X = 3. kim
tra phng trnh trn cn st nghim hay khng ta tip tc nhp:
h
i

X 2 X + 3 + X 2 + X + 4 7 : (X 3)
Ri li tip tc bm Shift Solve. Ta s tm c nghim th hai l X =

143
.
48

2) Dng tnh cht s hc on nghim ca phng trnh:


Vn dng phng trnh trn lm v d, ta nhn xt v phi ca phng trnh l 1 s nguyn.
Chnh v vy m ta a ra d on cc s trong cn thc phi l s chnh phng v b hn 7.
V vy ta c th cho mi cn thc bng cc s t 1 7 v gii tng v cn thc.
V d nh:

Cho x2 x + 3 = 1 v x2 + x + 4 = 6 ri gii ln lt tng phng trnh. Nhng nu c


hai v u l cn thc th phng php ny s khng hiu qu.
3) Th th mt vi nghim vo phng trnh cho on nghim:
Da vo iu kin ca bi ton, ta s ln lt th cc nghim gn gi nh 3, 2, 1, 0, 1, 2, 3
vo phng trnh tm xem s no l nghim ca phng trnh. i khi vi mt cht nhy
bn, ta c th d dng nhm ngay nghim ca phng trnh nu l nghim p.
Sau y l mt s k thut dng phng php lng lin hp:
Mt s hng ng thc hay s dng:
1) x2 y 2 = (x y) (x + y)
92

93
2) x3 y 3 = (x y) (x2 + xy + y 2 )
3) x4 y 4 = (x y) (x + y) (x2 + y 2 )
4) xn y n = (x y) (xn1 + xn2 y + ... + xy n2 + y n1 )
........
S dng nhng hng ng thc ny, ta c th quy phng trnh v t ban u v dng phng
trnh tch bng vic lm xut hin cc nhn t chung. T ta c th d dng gii quyt tip.
Ta cng c mt s hng ng thc trc cn thc:

xy

1) x y =

x y

xy

p
2) 3 x 3 y =

3
x2 3 xy + 3 y 2

xy

3) 4 x 4 y =

4
4
( x y)( x + y)

Bi tp v d
k thut thm bt v tch hng t
phng php ny, chng ta s thm bt hng t hoc tch hng t sn c dng phng
php lng lin hp gii phng trnh.
Bi 1: Gii phng trnh: (x + 1)

x2 2x + 3 = x2 + 1 (1.1)
Gii

V x = 1 khng phi l nghim ca phng trnh trn, ta vit phng trnh di dng:

x2 2x + 3 =

x2 + 1
x2 2x 1
x2 2x + 3 2 =
x+1
x+1

x2 2x + 3 + 2 > 0 nn:



x2 2x + 3 2
x2 2x + 3 + 2
x2 2x 1

(1.1)
=
x+1
x2 2x + 3 + 2
2
2
x 2x 1
x 2x 1

=
2
x+1
x 2x + 3 + 2

x
=
1
+
2

x2 2x 1 = 0
x=1 2
1
1

=
1
1


x+1
=
x2 2x + 3 + 2
x+1
x2 2x + 3 + 2

2
D thy phng trnh x 2x + 3 + 2 = x + 1 v nghim.



Vy tp nghim ca phng trnh cho l: S = 1 2; 1 + 2 2
~ Nhn xt: mu cht ca li gii trn l nhn ra lng lin hp tm ra nhn t chung l

x2 2x + 3 + 2. Vy lm cch no nhn ra c iu ny? Ta lm nh sau:

x2 + 1
Xt phng trnh: x2 2x + 3 =
()
x+1

94

x2 + 1
m (m > 0)
x+1

x2 mx m + 1
x2 2x + 3 m =
x+1



x2 2x + 3 m
x2 2x + 3 + m
x2 mx m + 1

x+1
x2 2x + 3 + m
2
2
2
x 2x + 3 m
x mx m + 1

=
2
x+1
x 2x + 3 + m
By gi ta ch cn xc nh m sao cho: x2 2x + 3 m2 = x2 mx m + 1 , m. Suy ra
2 = m 3 m2 = m + 1 m = 2. T ta suy ra li gii nh trnh by.
()

x2 2x + 3 m =

3
Bi 2: Gii phng trnh: 3 x2 + x2 + 8 2 = x2 + 15 (1.2)

Gii

D on c
phng trnh nh
 nghim
 x=1, ta vitli 
 sau:

3
2
2
2
(1.2) 3
x 1 +
x +83 =
x + 15 4
x2 1
x2 1
3 (x2 1)

+
=

3
3
x2 + 8 + 3
x2 + 15 + 4
x4 + x2 + 1

x2 = 1
1
1
1

=
()
+
3
3
x2 + 8 + 3
x2 + 15 + 4
x4 + x2 + 1
Mt khc, ta c:

1
1
x2 + 15 > x2 + 8 x2 + 15 + 4 > x2 + 8 + 3
<
2
2
x + 15 + 4
x +8+3
Nn phng trnh () v nghim. Vy phng trnh c tp nghim S = {1; 1}2.

Bi 3: Gii phng trnh: 2 (x2 + 2) = 5 x3 + 1 (1.3)

Gii

iu kin: x > 1
Do x = 1 khng phi l nghim phng trnh nn ta xt x > 1.

95
Khi vit li (1.3) di dng
p

(1.3) 2 x2 + 2 = 5 (x + 1) (x2 x + 1)
s

(x + 1)2 (x2 x + 1)
2
2 x +2 =5
x+1
s
2 (x2 + 2)
(x + 1)2 (x2 x + 1)

=
5 (x + 1)
x+1
s
x2 x + 1
2 (x2 + 2)

2=
2
5 (x + 1)
x+1
! r
!
r
x2 x + 1
x2 x + 1
2
+2
x+1
x+1
2x2 10x 6
r
=

5 (x + 1)
x2 x + 1
+2
x+1
2x2 10x 6
x2 5x 3
!

=
r
5 (x + 1)
x2 x + 1
+2
(x + 1)
x+1

2
5
+
37
x 5x 3 = 0
x=
r

2
5

x2 x + 1
5 37
=
+2
x=
2
x+1
2
r
5
x2 x + 1
Phng trnh =
+ 2 v nghim nn tp nghim ca (1.3) l
+1
(
2
x)
5 37 5 + 37
S=
;
2
2
2

Bi 4: Gii phng trnh: x3 + 3x2 3 3 3x + 5 = 1 3x (1.4)


Gii
~ tng: nh gi phng trnh, ta thy phng trnh c dng hng ng thc (x + 1)3 nn
ta s bin i phng trnh theo hng ng thc biu thc gn hn. Tht vy t phng
trnh u ta c biu thc gn hn nh sau:

(x + 1)3 = 3 3 3x + 5 + 2
n y, ta c th d dng nhm nghim hn. Tht vy, nhm nghim ca phng trnh ta c
phng trnh c nghim l x = 1. i vi cn bc ba, ta cng lm tng t nh cn bc hai.
~ Li gii:
Vit li (1.4) di dng

(1.4) (x + 1)3 8 = 3 3 3x + 5 6
9(x 1)
(x 1)[(x + 1)2 + 2(x + 1) + 4] = q

3
(3x + 5)2 + 2 3 3x + 5 + 4
"
x=1
q

2
[(x + 1) + 2(x + 1) + 4]( 3 (3x + 5)2 + 2 3 3x + 5 + 4) = 9 ()

96
Do c cn bc ba nn vic gii (*) s c phn phc tp hn. Ta c th dng bt ng thc nh
sau:


() (x + 2) + 3


 
3


2
3x + 5 + 1 + 3 = 9

Li c (x + 2)2 + 3 > 3 v ( 3 3x(+ 5 + 1)2 + 3 > 3 V T () > 9


x = 2

Du bng xy ra khi v ch khi


x = 2
3
3x + 5 + 1 = 0
Vy tp nghim ca phng trnh l: S = {2; 1} 2

Bi 5: Gii phng trnh:

162x3 + 2

27x2 9x + 1 = 1 (1.5)

Gii
Vit li (1.5) di dng
(1.5)

162x3 + 2 2

27x2 9x + 1 + 1 = 0

162x3 6
3x (3x 1)

=0

2
27x2 9x + 1 + 1
162x3 + 2 + 2 3 162x3 + 2 + 4
"
#
2
2 (9x + 3x + 1)
3x
(3x 1)

=0

2
3
3
2
27x 9x + 1 + 1
162x3 + 2 + 2 162x3 + 2 + 4

3

Xt phng trnh
3x
2 (9x2 + 3x + 1)

=0

2
3
2
27x 9x + 1 + 1
162x3 + 2 + 2 162x3 + 2 + 4
3x
2 (9x2 + 3x + 1)


2
3
3
162x3 + 2
162x3 + 2 + 2 3 162x3 + 2 + 4

t a =

162x3 + 2, suy ra


1
4
1
a 2
2 3x +
+ 1 = a + + 2 3x +
+1= + +1
3x
a
3x
2 a
a
2
1
3x = 3x = x =
2
a
3

Vy phng trnh cho c nghim duy nht x =

Bi 6: Gii phng trnh:

x2 + 12 + 5 = 3x +

1
2.
3

x2 + 5 (1.6)

Gii
5
3
~ tng: Ta nhn thy x = 2 l mt nghim ca phng trnh. Nh vy phng trnh
cho c th phn tch c v dng (x 2) Q (x) = 0.
iu kin: x >

97
~ Li gii:
Vit li (1.6) di dng
(1.6)

x2 + 12 4 = 3x 6 +

x2 + 5 3

x2 4
x2 4
= 3 (x 2) +
x2 + 12 + 4
x2 + 5 + 3


x+2
x+2

3 =0
(x 2)
x2 + 12 + 4
x2 + 5 + 3

x=2
x+2
x+2

3 = 0 ()
x2 + 12 + 4
x2 + 5 + 3

1
1
x+2
x+2
<

< 0 nn () v nghim.
x2 + 12 + 4
x2 + 5 + 3
x2 + 12 + 4
x2 + 5 + 3
Vy phng trnh cho c nghim duy nht x = 2 2.
trn ta lm quen vi k thut thm bt v tch s, nhng nu phng trnh c nhiu
nghim th khng th dng cch . Sau y l k thut thm bt v tch n. tng ca
phng php ny l dng n nh vai tr ca s sau khi nhm nghim.
Do

Bi 7: Gii phng trnh:

2x 1 + x2 3x + 1 = 0 (1.7)
Gii

~ tng: Nhm nghim ta thy x = 1 l nghim ca phng trnh.


Ta th k thut tch s vi phng trnh ny:
(1.7)

2x 1 1 + x2 3x + 2 = 0

2(x 1)

+ (x 1)(x 2) = 0
2x 1 + 1

x=1
2

= 2 x ()
2x 1 + 1
Nhn thy () vn cn nghim. Vy vic thm bt s khng th tm c ht nghim. Nu

quy ng phng trnh (), ta li phi gii phng trnh kh phc tp l x = (2 x) 2x 1.


Vy ta cn suy ngh n vic dng n thay th s.
~ Li gii:
1
iu kin: x >
2
Tht vy, t phng trnh u ta c:
(1.7)

2x 1 x + x2 2x + 1 = 0

x=1
(x 1)
1
+ (x 1)2 = 0

= 1()
2x 1 + x
2x 1 + x
2

98
Nh vy ch cn gii (*):

1 2x + x2 = 2x 1

() 1 x = 2x 1
1 > x > 1
2

x2 4x + 2 = 0

x
=
2

1 > x > 1
2


Kt lun: phng trnh c tp nghim S = 1; 2 2

Bi 8: Gii phng trnh:

12x2 + 46x 15

x3 5x + 1 = 2x + 2 (1.8)

Gii
Vit li (1.8) di dng

3
3
(1.8) 12x2 + 46x 15 (2x + 1) x3 5x + 1 1 = 0
8x3 + 40x 16
x3 5x + 2

=0
2

2
2

2x
+
1
3(2x
+
1)
1
3
3
3
12x2 + 46x 15 +
+
x2 5x + 1
+
2
4
2
4
3
3
x 5x + 2
8x 40x + 16
=0

2 +
2
2

3
1
3(2x
+
1)
2x
+
1
3
3
2
( x 5x + 1 ) +
) +
( 12x2 + 46x 15 +
2
4
2
4
3
3
8(x 5x + 2)
x 5x + 2

+
=0
2
2
2

2x
+
1
3(2x
+
1)
1
3
3
3
12x2 + 46x 15 +
+
x2 5x + 1
+
2
4
2
4

x3 5x + 2 = 0

1
8

+
= 0 ()

2
2
2


2x
+
1
3(2x
+
1)
1
3
3
3
12x2 + 46x 15 +
+
x2 5x + 1
+
2
4
2
4


x=2

Do () v nghim nn(1.8) x 5x + 2 = 0 x = 2 1

x= 21



Vy tp nghim ca phng trnh trn l S = 2 1; 2 1; 2 2
~ Nhn xt: Cu hi t ra l: Ti sao xut hin i lng x3 5x + 2 = 0? Bng cch thm
bt ax + b vo phng trnh v ng nht h s, ta s lm xut hin cc i lng nh vy.
in hnh ta xt cc v d sau:
3

Bi 9: Gii phng trnh:

3x2

6x 5 =

(2 x)5 +

p
(2 x)(2x2 x 10) (1.9)

Gii

~ tng: u tin ta c (1.9) 3x2 6x 5 = 2 x(3x2 5x 6)


Dng my tnh bm nghim ta thy phng trnh ra nghim xu. Chnh v th, ta s thm bt

99
n cho ph hp c th nhn lng lin hp:

Do gi thit phng trnh ny c 2 x nn ta s thm bt 2 x vo phng trnh:

3x2 6x 5 2 x = 2 x(3x2 5x 6 )

3x2 x(6 2 ) 5 22

= 2 x(3x2 5x 6 )

2
3x 6x 5 + 2 x
c nhn t chung, ta s ng nht h s ca 2 v:
(
6 2 = 5
a=1
5 + 22 = 6 +
~ Li gii:

x62

3+2 6
x62
3

2
6
x>

x
6
iu kin:

3
3x2 6x 5 > 0

2 6

x6
3
Vit li (1.9) di dng

(1.9) 3x2 6x 5 2 x = 2 x(3x2 5x 7)

3x2 5x 7
= 2 x(3x2 5x 7)

3x2 6x 5 + 2 x

3x2 5x 7 = 0

= 2 x ()

3x2 6x 5 + 2 x
Xt (*) ta c:
p
p
() 1 = 2 x + (2 x)(3x2 6x 5) x 1 = (2 x)(3x2 6x 5)

32 6
2 6
Vi iu kin x 6
th V T 6
<06VP
3
3

5 + 109
(khng tho x < 2)
x=
6

Vy phng trnh (1.9) tng ng: 3x2 5x7 = 0


5 109
x=
(chn)
6

5 109
Vy phng trnh c tp nghim S = {
}2
6

Bi 10: Gii phng trnh: x3 3x + 1 =

8 3x2

(1.10)

Gii

2 6
2 6
iu kin:
6x6
3
3
~ tng: bi ny, tuy bit phng trnh c 2 nghim nhng li l nghim v t nn ta
khng th bit chnh xc nghim. Tuy nhin ta ch quan tm ti biu thc cn thm bt to t
2 nghim trn. Do ta s ln lt dng chc nng Shift Solve tm ra 2 nghim ca phng
trnh l: x1 = 0, 6180339887...; x2 = 1, 618033989... sau gn hai nghim ny vo hai bin

100
A v B.
By gi ta s th tm tam thc bc 2 to t 2 nghim trn. Ngha l ta cn tnh A + B v A.B.
Thu c A + B = 1, AB = 1.
iu chng t A, B l hai nghim ca phng trnh: X 2 X 1 = 0
V t y, ta c th d on c x2 x 1 l mt nhn t ca phng trnh.
Ta vit phng trnh cho li thnh:

(1.10) x3 3x + 1 (px + q) 8 3x2 + px + q = 0


(px + q)2 (8 3x2 )
x3 3x + 1 (px + q) +
= 0 (2)
8 3x2 + px + q
(p2 + 3) x2 + 2pqx + q 2 8

=0
x3 (p + 3) x + 1 q +
8 3x2 + px + q
n y, xut hin nhn t x2 x 1 th (p2 + 3) x2 + 2pqx + q 2 8 = (x2 x 1) vi
l mt h s. Chn = 4 th ta c mt cp (p, q) tha mn l (p, q) = (1; 2).
~ Li gii:
Vit li (2) di dng
x2 x 1
x3 2x 1 + 4
=0
8 3x2 + 2 x



4
2
x x1 x+1+
=0
8 3x2 + 2 x

3x
8 3x2 + 2 x, ta c: f 0 (x) =
1
8 3x2
r
2
3x
=1x=
f 0 (x) = 0
2
3
8 3x
Ta c bng bin thin:
Xt f (x) =

2 6

f 0 (x)

6+2 6
f (x)
%
3

2 6

2
3

6+4 6
3

6+2 6
&
3

6+4 6
0 < f (x) 6
3

4
4
2 6
12
>0
x+1+
=x+1+
>
+1+
f (x)
3
8 3x2 + 2 x
6 +
4 6
1 5
Vy phng trnh cho tng ng x2 x 1 = 0 x =
2
2
By gi chng ta s phi hp cc k thut trn gii bi ton sau:
Bi 11: Gii phng trnh

(x 1) 2x2 5x 15 +

2x3 7x2 + 19
= 2x3 7x2 12x + 17 + 7x (1.11)
2
Gii

101
~ tng: Ta nhn xt thy cc h s c cng s m trong cn thc ging nhau. iu ny
gi cho ta tng thm bt n v s sao cho ph hp nghim.
khng qu ph thuc vo vic on nghim nn ta s thm bt x + tm h s tch

hp l. (x 1) 2x2 5x 15 c dng bc ba ging nh trong cn thc v ngoi cn thc


(v h s ca s m ln nht trong cn v ngoir
cn ging nhau) ta s thm bt (x 1). Vic
2x3 7x2 + 19
nh th no.
ng nht cc h s s cho ta bit nn tch
2
r

2x3 7x2 + 19
(1.11) (x1)( 2x2 5x 15)+
= 2x3 7x2 (12+)x+17++ 7x
2
r
3
2
2
2

(x 1)(2x 5x 15 )
2x 7x + 19

+
= 2x3 7x2 (12 + )x + 17 + + 7x
2
2x2 5x 15 +
r

2x3 7x2 (10 + 2 )x + 15 + 2


2x3 7x2 + 19

= 2x3 7x2 (12+)x+17++ 7x

+
2
2x2 5x 15 +
ng
nht h s, ta c:
(
"
10 + 2 = 12 +
= 1
2

2
=
0

15 + 2 = 17 +
=2

Do nhn lng lin hp vi ax + b c nn ta nhn nghim dng. Vy ta chn = 2.


~ Li gii:

x > 5 + 145
4
iu kin:

x3 7 x2 + 19 > 0
2
2
Vit li (1.11) di dng
r

2x3 7x2 14x + 19


2x3 7x2 + 19
+
= 2x3 7x2 14x + 19 + 7x
(1.11)
2
2x2 5x 15 + 2
r
2x3 7x2 14x + 19
2x3 7x2 + 19
7x = 2x3 7x2 14x + 19

+
2
2
2x 5x 15 + 2
3
2
3
2x 7x2 14x + 19
2x 7x 14x + 19
+r
= 2x3 7x2 14x + 19

3
2

2x2 5x 15 + 2
2x 7x + 19
+ 7x
2
3
2x 7x2 14x + 19 = 0

1
1

2x2 5x 15 + 2 + r 2x3 7x2 + 19 = 1()


+ 7x
2

x=1

x = 5 + 177

177
x=

1
1

r
+
= 1()
2x2 5x 15 + 2
2x3 7x2 + 19
+ 7x
2
D thy
VT ca phng trnh () l hm nghch bin. Mt khc theo iu kin xc nh
5 + 145
x>
> 1. Vy phng trnh (*) v nghim.
4

102

177
Xt vi iu kin xc nh. Vy phng trnh trn c nghim duy nht l x =
2
4
~ Nhn xt: C th nhm c nghim x = 1. T vic tch n s d dng hn. Nhng mt
cch tng qut ta s lm nh trn trong mi trng hp c kh nng dng lng lin hp. Tuy
nhin khuyt im ca phng php ny l nu gp phng trnh khng c bt k 1 nghim
hi t no v sau khi thm bt x + ta khng tm c ; hoc gp 1 h phng trnh phc
tp trong khi ng nht h s th vic tch n dng lng lin hp kh c th t hiu qu.
Nhng lng lin hp li c th mnh trong 1 s bi ton xt iu kin phc tp. in hnh l
nhng phng trnh v t cha tr tuyt i:
5+

Bi 12: Gii phng trnh:

3 x + 2 + x = x3 + x2 4x 4 + |x| + |x 1|

(1.12)

Gii
iu kin: 2 6 x 6 3


 
(1.12)
3 x |x 1| +
2 + x |x| = x3 + x2 4x 4
x2 + x + 2
x2 + x + 2
= (x + 2) (x + 1) (x 2)
+

3 x + |x 1|
2 + x + |x|


1
1
(2 x) (x + 1)
+
+ (x + 2) = 0
3 x + |x 1|
2 + x + |x|
"
x = 1

x=2
Vy phng trnh c tp nghim S = {1; 2} 2
~ Nhn xt: Vi bi ny, vic xut hin thm cc a thc cha tr tuyt i tng chng nh
s gy cho ta thm kh khn trong vic gii quyt. Nhng nh s dng phng php nhn
lng lin hp, bi ton c gii quyt nhanh chng! Khi y, ta ch cn chuyn cc lng
trn v ng v tr v s dng phng php nhn lin hp l .

k thut ghp hng t


Bi 1: Gii phng trnh:

10x + 1 +

3x 5 =

9x + 4 +

2x 2 (2.1)

Gii
5
3
~ tng: i vi k thut p dng vo mt s bi tp, vic on nghim c th khng cn
thit v ta s nhm tng hng t ca 2 v vi nhau to ra nhn t chung. Tht vy, ta c
th thy:
10x + 1 9x 4 = x 3 v 3x 5 2x + 2 = x 3
Vy ta c th d dng thy c nhn t chung cn nhm l x 3 v khng cn nhm nghim.
Do ta c th gii bi phng trnh ny nh sau:
~ Li gii:
iu kin: x >

103
Vit li (2.1) di dng

(2.1) 10x + 1 9x + 4 + 3x 5 2x 2 = 0
x3
x3

+
=0
3x 5 + 2x 2
10x + 1 + 9x + 4

x=3
1
1

= 0 (v nghim)
+
3x 5 + 2x 2
10x + 1 + 9x + 4

x = 3 (chn)

Vy phng trnh c tp nghim S = {3} 2


M RNG:
Sau y ta s xt h s ca cc cn thc. Nu thay i h s ca cc cn thc trn, ta s c
mt v d khc kh hn:
Bi 1*: Gii phng trnh
r

26
28
5
26 10x + 1 +
3x 5 =
9x + 4
2x 2 (2.1)
31
26
806

Gii
5
3
i vi bi ny, tng ca ta l ng nht cc h s to ra nhn t chung: Quan st
phng trnh ny, ta thy c kh nng ghp i xng cc phng trnh ca 2 v to ra nhn
t chung l x 3. Nhng h s y kh phc tp, v th ta cn phi ng nht h s ghp
cho ph hp.

Ta thm bt 9x + 4 v 2x 2 tm h s ; ph hp:
r





28
5
26
26 10x + 1 9x + 4+
3x 5 2x 2 =
9x + 4
+
2x 2
31
26
806
r

26
Ta d on = 26, =
th VT s xut hin nhn t chung l x 3. kim chng
31
li, ta s lm nh sau:

xut hin nhn t chung l x3 th lng lin hp ca 9x


2 phi bng x3
" +4 vi 2x

a 9x + 4 b 2x 2 = x 3

hoc 3 x. Vy tm ra s ; nhanh gn, ta c th tm


.
a 9x + 4 b 2x 2 = 3 x
(tm h s a, b ta a v gii h phng
trnh: r

2
2
2

2
9a 2b = 1

a=

a2 =

13
2
2
13

( 4a + 2b = 3

9a2 2b2 = 1

31

b =
b = 31
4a2 + 2b2 = 3
26
r 26

28
2

=
= r26
26
r13
Vy ta d dng tm h s , :

= 26
5
31


+ =
31
26
806
iu ny ng nh ta d on, vy lm nh tng t nh bi 1 ta s c nghim duy nht
l x = 3. Vy ta t hi c th tm c h s bt k cho phng trnh trn hay khng? Sau y

iu kin: x >

104
ta s xt iu :
Tm mi quan h gia cc h s phng trnh sau c nghim duy nht:

a 10x + 1 + b 3x 5 = c 9x + 4 + d 2x 2
Gii
Do ta thy y l phng trnh c dng lng lin hp i xng.
Ta s tm h s (a, b, c, d) 6= (1; 1; 1; 1).

Vy ta s khng th tch hon ton m s cn vng li a 9x + 4 v b 2x 2.

a( 10x + 1 9x + 4) + b( 3x 5 2x 2) = (c a) 9x + 4 + (d b) 2x 2
Theo nh V d 1 (*)ta s c hrphng trnh
r
sau:

2
2

2
ca=
a=c
(c a)2 =

13
13
13

r
r

31

31

(d b)2 =
db=
b = d + 31
26
26
26

Ta chn
h
s
nh
trn
l

c
th
nhn
lng
lin
hp
cho
a 9x + 4 v b 2x 2.

r
2

a=c
r 13 l h s chun phng trnh c nghim duy nht.
Vy

31

b=d+
26
p

Bi 2: Gii phng trnh:


3x2 5x + 1
x2 2 =
3 (x2 x 1)

x2 3x + 4 (2.2)
Gii
~ tng:
Trc ht, kim tra ta thy c rng phng trnh cho c mt nghim x = 2 nn ta s c
gng a phng trnh trn v phng trnh tch xut hin nhn t (x 2). Ta c nhn xt
rng:
(3x2 5x + 1) (3x2 3x 3) = 2 (x 2) v (x2 2) (x2 3x + 4) = 3 (x 2)
Ta i n li gii nh sau:
~ Li gii:
Vit li (2.2) di dng

3x2 5x + 1 3 (x2 x 1) = x2 2 x2 3x + 4
2x + 4
3x 6
p

x2 2 + x2 3x + 4
3x2 5x + 1 + 3 (x2 x + 1)
"
#
2
3
p

(x 2)
+
=0
x2 2 + x2 3x + 4
3x2 5x + 1 + 3 (x2 x 1)

(2.2)

x=2
Vy phng trnh (2) c nghim duy nht x = 2.

105

Bi tp t luyn
1. Bi tp c bn

1) x2 3x + 3 + x2 3x + 6 = 3

2) 3 x + x2 2 x + x2 = 1

3) rx2 3x + 1 + 2x2 6x + 1 = 2
1x
2x + x2
4)
=
x
1 + x2

3
3
5) x 2 + 2x 3 = 1

6) 3 x2 1 + x 7 = 4

7) 4 x + 2 3 x2 + 7 + x3 + 1 x4 = 1

8) x + 3 + 3 x = 3

9) 5 x + x 1 = x2 + 2x + 1

10) x2 + x + 1 2x2 + 3x 1 = 3

11) x2 + 2x + 3 + 4x2 + 5x + 6 = 7x2 + 8x + 9 + 10x2 + 11x + 12

12) 2x 1 + x2 3x + 1 = 0


13)
4x + 1 3x 2 = x 5

14) x + 1 + x + 4 + x + 9 + x + 16 = x + 100

x+1
15) 4x + 1 3x 2 =
3

16) 3x2 7x + 3 x2 2 = 3x2 5x 1 x2 3x + 4

17) x2 + 12 + 5 = 3x + x2 + 5

18) 3 x2 1 + x = x3 1

19) x2 3x 4 = x 1 (x2 4x 2)

2. Bi tp nng cao

x+3
1) 3 x2 1 + x 3 + x + 1 + x = 2
+5
x 6

2) x2 + x 1 = (x + 2) x2 2x + 2

3) 5x 1 + 3 9 x = 2x2 + 3x 1

4) 3x2 7x + 3 x2 2 = 3x2 5x 1 x2 3x + 4

5) 2x2 5 + 2x2 5 + 3 4x4 29x2 + 25 = 3x + 12x3 9x2 30x

3
6) 2x2 + x + 6 + x2 + x + 3 = 2(x + )
x

7) 2x + 1 + x = 2x2 + 4x 23

8) x2 x 3 + 2x + 5 = 0

9) 2 x2 7x + 10 = x + x2 12x + 20
1
2
1
7
+ 2+
=
10)
2x
4
x 1 x

11) r2 (x2 + 8) = 5 x3 + 8
x2 + x + 1 x2
1
12)
+
=
+2
x+4
2
x2 + 1

x2 1
2
13) 2x 3x + 1 =
2x 3

x3
1

14)
=
2x 1 1
x+3 x3

106

PHNG PHP DNG N IU HM S


Trong nhng bi ton gii phng trnh v t th dng n iu ca hm s l mt phng
php mnh v thng cho ta li gii p. Bi vit ny s gii thiu mt s ng dng ca phng
php trn. Mi bi ton iu c trnh by theo th t tng - Li gii Nhn xt, vi
mong mun cho bn c c mt ci nhn su hn v cch t duy v kinh nghim gii ton.

L thuyt
I nh l 1: Nu hm s y = f (x) lun ng bin (hoc nghch bin) v lin tc trn D th s
nghim trn D ca phng trnh f (x) = k khng nhiu hn mt v x, y D : f (x) = f (y)
x = y.
I nh l 2: Nu hm s f(x) v g(x) n iu ngc chiu v lin tc trn D th s nghim
trn D ca phng trnh f (x) = g(x) khng nhiu hn mt.
I nh l 3: Nu hm s f(x) lun ng bin (hoc lun nghch bin) trn D th
f (x) > f (y) x > y ( hoc x < y)
Vn dng linh hot cc nh l trn, t mt phng trnh n x, ta s a hai v v dng
f (g(x)) = f (h(x)) vi f (t) l mt hm n iu trn min D ang xt. Thng thng ta c th
d on c h(x) v bc ca g(x), t ng nht h s tm g(x) (u tin h s nguyn).
Cc bi ton sau s lm r tng trn:

Bi tp v d
Bi 1: Gii phng trnh:

3x + 1 +

x + 7x + 2 = 4 (1.1)

Gii
~ tng: VT ton du cng nn ta hi vng y l mt hm ng bin theo x. Khi theo
nh l 1, (1.1) c nghim duy nht (d thy l x = 1).
~ Li gii:
x > 1
3
KX: ()
x + 7x + 2 > 0
t VT (1.1) l f(x). Ta c
3
7
1
f 0 (x) =
+ (1 +
). p
> 0 x tho (*)

2 3x + 1
2 7x + 2 2 x + 7x + 2
Vy f (x) = 4 = f (1) x = 1
Th li ta thy x = 1 tho phng trnh.Vy (1.1) c tp nghim S = {1} 2
~ Nhn xt: y l mt ng dng c bn ca phng php n iu. Ta ch vic a cc bin
v cng mt v v xt o hm. Chng ta s khng i su vo dng ny, m tp trung nhiu
hn v cch xy dng hm s.

Bi 2: Gii phng trnh vi a > 0 (x l n): x3 b = a 3 ax + b (2.1)

107
Gii
~ tng: Ta ngh ti vic a hai v v dng f (g(x)) = f (h(x)) trong f (t) = mt3 + nt.

C th xc nh h(x) VP chnh l 3 ax + b, cn g(x) VT c bc nht nn g(x) = px + u.


tip, ta thy VT(2.1) sau khi bin i s tr thnh: m(px + u)3 + n(px + u). Nh vy th
hng t bc ba s l mp3 x3 , trong khi phng trnh ban u l x3 . Do mp3 = 1

V u tin s nguyn nn ta ly m = p = 1. Tng t, VP th hng t bc nht l a 3 ax + b,


tng ng vi nh(x) trong f (h(x)), nn n = a.
Vy f (t) = t3 + at. Do ta cn a (2.1) v dng

3
(x + u)3 + a(x + u) = ax + b + a ax + b (2.2)
Tip tc phn tch, ta thy VT khng xut hin x2 nn c ngay u = 0, v nu u 6= 0 ta khng
th kh hng t 3ux2 .
Ngha l

3
(2.2) x3 + ax = ax + b + a ax + b
D thy ch cn cng ax + b vo 2 v ca (2.1) l ta c (2.2). Cng vic n y tr nn n
gin.
~ Li gii:

3
(2.1) x3 b + ax + b = ax + b + a ax + b

f (x) = f ( 3 ax + b) vi f (t) = t3 + at

x = 3 ax + b x3 = ax + b
y chnh l phng trnh bc 3 dng c bn. 2
~ Nhn xt: Bi ton trn cho ta mt cch nhn s lc v n iu hm s, trong phn
quan trng nht l xy dng hm v dng nhng nh gi thch hp tm ra h s. Chng
ta cng c th m rng hn mt cht:

Bi 2*: Gii phng trnh xn b = a n ax + b (n N ), n l v a > 0


Bi 3: Gii phng trnh 8x3 36x2 + 53x 25 =

3
3x 5 (3.1)

Gii
~ tng: Ta cn a 2 v v biu thc dng f (g(x)) = f (h(x)) trong f (t) = mt3 + nt

rng hng t 3 3x 5 VP c bc thp nht nn tng ng vi nh(x) trong f (h(x)),


vy n = 1.
Nh bi 2, ta xc nh g(x) = px + u. VT sau khi bin i s l m(px + u)3 + n(px + u). Xt
hng t bc 3 ta c mp3 x3 = 8x3 . Nh vy mp3 = 8. Tuy nhin n y li c 2 trng hp
m ta s phi ln lt xt: m = 8, p = 1 hoc m = 1, p = 2
> Nu m = 1 : f (t) = t3 + t. Do cn a (3.1) v dng
(2x + u)3 + (2x + u) = 3x 5 +

3x 5

8x3 + x2 (12u) + x(6u2 1) + u3 + u + 5 =

3x 5

108
ng nht h s vi VT ca (3.1) ta c

12u = 36

6u2 1 = 53

u3 + u + 5 = 25

u = 3

Vy trng hp m = 1 cho kt qu, do khng cn xt m = 8.


~ Li gii:
(3.1) 8x3 36x2 + 54x 27 + 2x 3 = 3x 5 +

(2x 3)3 + 2x 3 = 3x 5 + 3 3x 5

3
3x 5

f (2x 3) = f ( 3 3x 5)(2.2) vi f (t) = t3 + t


Ta cf (t) ng bin trn R do
(3.2) 2x 3 =

3x 5 (2x 3)3 = 3x 5

8x 36x + 51x 22 = 0

x=2

5 3
x=
4

5 3
Vy (3.1) c tp nghim S = {2;
}2
4
~ Nhn xt: i khi ta cn tinh trong vic xy dng hm, nh trong bi trn h s bc cao
nht c th l 8 hoc 1. Mt v d khc:

Bi 3*: Gii phng trnh 4x3 + 18x2 + 27x + 14 = 3 4x + 5


1
Lu rng 4x3 = 4(x3 ) = .(2x)3 do ta cng cn xt 2 trng hp.
2

Bi ton trn cng c th gii bng cch t 3 4x + 5 = 2y + 3 a v h i xng loi II.


Nhng bc phn tch trn nhn tuy di nhng khi quen ri th ta c th tnh rt nhanh.Tuy
nhin, trong mt s bi ton, hm f (t) ca ta khng ng bin trn R nhng ta c th ch cn
xt n iu trn min xc nh D.
Bi 4: Gii phng trnh 9x2 28x + 21 =

x 1 (4.1)

Gii

~ tng: Ta xy dng hm f (t) = mt2 +nt. rng hng t x 1 VP c bc thp nht


nn tng ng vi nh(x) trong f (h(x)), do n = 1. Nh bi 3, khi xc nh g(x) = px + u
v mp2 = 9, ta cng th xt 2 trng hp: m = 9, p = 1 hoc m = 1, p = 3.
> Nu m = 9 : f (t) = 9t2 + t. Vy cn a (3.1) v dng
9(x + u)2 + x + u = 9(x 1) +

x1

9x2 + x(18u 8) + u2 + u + 9 =

x1

109
ng nht h s ta c

18u 8 = 28
u2 + u + 9 = 21

u = 10
9

u {4; 3}

> Nu m = 1 : f (t) = t2 + t. Vy cn a (3.1) v dng


(3x + u)2 + 3x + u = x 1 +

x 1 9x2 + x(6u + 2) + u2 + u + 1 =

x1

ng nht h s ta c

6u + 2 = 28
u2 + u + 1 = 21

u = 5

n y c l bi ton c gii quyt nhng tht ra "chng gai" cn pha trc. Th


lm tip ta s c

(4.1) x2 30x + 25 + 3x 5 = x 1 + x 1

f (3x 5) = f ( x 1) (4.2) vi f (t) = t2 + t


1
1
(!) Lu rng f (t) = t2 + t ch ng bin trn ( ; +) v nghch bin trn (;
), hn
2
2

1
na x 1 > 0 >
.
2

1
3
Nh vy t (4.2) ta ch suy ra 3x 5 = x 1 khi 3x 5 >
x> .
2
2
3
Cn x [1; ] th sao? Li rng hm s bc 2 cng c ci hay ca n, l t2 = (t)2 .
2
trn, da vo h s ca x2 , ta ch mi xt g(x) = px + u vi mp2 = 9 v p N , nhng thc ra
vn cn trng hp m = 1, p = 3 . Ta s xt tip trng hp ny:
Cn a (4.1) v dng
(u 3x)2 + u 3x = x 1 +

x 1 9x2 + x(6u 4) + u2 + u + 1 = x 1

ng nht h s ta c

6u 4 = 28
u2 + u + 1 = 21

u=4

3
1
4 3x >
. Vy chn u = 4.
2
2
n y bi ton mi thc s c gii quyt.
~ Li gii:
KX: x > 1
1
3
> Nu x > 3x 5 >
2
2
Kim tra li: C x <

Ta c
(4.1) (3x 5)2 + (3x 5) = (x 1) +

x1

f (3x 5) = f ( x 1) (vi f (t) = t2 + t) 3x 5 = x 1

110

> Nu 1 6 x <

3x 5 > 0
(3x 5)2 = x 1

x > 5
3

13

x {2; }
9

x=2

3
1
: 4 3x >
2
2

Ta c
(4.1) (4 3x)2 + 4 3x = x 1 +

x1

f (4 3x) = f ( x 1) (vi f (t) = t2 + t) 4 3x = x 1

4 3x > 0
x 6
13
25

(chn)
x=
(4 3x)2 = x 1

18
x { 25 13 }
8

25 13
Vy (3.1) c tp nghim S = {2;
}2
18
~ Nhn xt: Cn linh hot trong vic xy dng hm s, nht l i vi hm bc chn.

Ta cng c th gii bi ton trn bng cch t x 1 = 3y 5 a v h i xng loi 2.


p
Bi 5: Gii phng trnh 3x3 6x2 3x 17 = 3 3 9(3x2 + 21x + 5) (5.1)

Gii
~ tng: Nh nhng bi trc, u tin ta th a 2 v v biu thc dng f (t) = 3t3 + 3t.
(5.1) tr thnh:
p
3(x u)3 + 3(x u) = 9(3x2 + 21x + 5) + 3 3 9(3x2 + 21x + 5)
p
3x3 + x2 (9u + 27) + x(9u2 186) + (3u3 3u 45) = 3 3 9(3x2 + 21x + 5)

9u + 27 = 6

ng nht h s vi VT(5.1) ta c

9u2 186 = 3

3u3 3u 45 = 17

D thy h ny v nghim. Vy ta khng th xy dng hm nh bnh thng. rng nguyn


nhn dn n vic ny l v h s ca 9(3x2 + 21x + 5) qu ln, cn tr vic ng nht h s.
Vy ta hy th xy dng hm theo mt hng khc:
Nhn 9 cho 2 v ca (5.1) ta c:
p
(5.1) 27x3 54x2 27x 153 = 27 3 9(3x2 + 21x + 5) (5.2)
By gi ta s a 2 v v biu thc dng f (t) = t3 + 27t (cch tm hm s tng t nhng bi
trn).
(5.2) tr thnh:
p
(3x u)3 + 27(3x u) = 9(3x2 + 21x + 5) + 27 3 9(3x2 + 21x + 5)
p
27x3 + x2 (27u + 27) + x(9u2 108) 27u u3 45 = 27 3 9(3x2 + 21x + 5)

111

27u + 27 = 54
ng nht h s ta c 9u2 108 = 27

27u u3 45 = 153

u=3

Bi ton c gii quyt.


~ Li gii: Nhn 9 vo 2 v ta c phng trnh:
p
(3x 3)3 + 27(3x 3) = 9(3x2 + 21x + 5) + 27 3 9(3x2 + 21x + 5)
p
f (3x 3) = f ( 3 9(3x2 + 21x + 5)) (vi f (t) = t3 + 27t)
p
3x 3 = 3 9(3x2 + 21x + 5) (3x 3)3 = 9(3x2 + 21x + 5)
3(x 1)3 = (3x2 + 21x + 5) 3x3 6x2 12x 8 = 0

2
3
x = (1 + 2)2 (tham kho cch gii PT bc 3 tng qut)
3
~ Nhn xt: Mt cu hi t ra l: Ti sao li nhn 9 m khng phi l s khc? Tht ra iu
ny c cp n ri. Khi xy dng hm f (t) = mt3 +3t, ta thng ngh ti g(x) = px+q
1
nn mp3 = 3, do m = 3, p = 1 m qun rng cn c m = , p = 3 (trng hp ny tht ra
9
t3
him gp, tr nhng bi h s ln nh bi ny). Nh vy f (t) cng c th l + 3t (v trong
9
bi ny th ng l vy). Vic nhn 9 ch n gin l kh mu s.
Lu rng vi nhng dng phng trnh nh trn, ta s khai trin v ng nht h s cc bc
3, 2, 1, 0. Ngha l ta c mt 4 h phng trnh, do s n ti a c th l 4. bi trn,
ta vn c th t f (t) = mt3 + t, khi trong qu trnh ng nht h s s xut hin thm 2
n m v p nhng vn gii c. Ta cng xem qua bi tng t:
Bi 6: Gii phng trnh x3 6x2 + 12x 7 =

3
x3 + 9x2 19x + 11 (6.1)

Gii
~ tng: Ta a hai v v hm s f (t) = mt3 + t. H s bc nht l 1, v n tng ng vi

3
x3 + 9x2 19x + 11 VP. VT th hng t bc 3 l x3 nn ta ngh ti m = 1, nhng vic
ng nht h s khng thnh cng ( bi ny nguyn nhn l do VP cng c x3 trong cn).
Vy, vi nhn xt rng VT th t = g(x) = px + u, ta s tm c p, u v m.
Cn a hai v v dng

3
m(px + u)3 + (px + u) = m(x3 + 9x2 19x + 11) + x3 + 9x2 19x + 11

3
x3 (mp3 + m) + x2 (3mup2 9m) + x(3u2 mp + p + 19m) + mu3 + u 11m = x3 + 9x2 19x +
ng nht h s VT ca (6.1) v (6.2) ta c

mp3 + m = 1

3mpu2 9m = 6

3u2 mp + p + 19m = 12

3
mu + u 11m = 7

m=

2
p=1

u = 1

112
~ Li gii:
Ta vit (6.1) di dng
(x 1)3
x3 + 9x2 19x + 11
3
+ (x 1) =
+ x3 + 9x2 19x + 11
2
2

t3
3
f (x 1) = f ( x3 + 9x2 19x + 11) (vi f (t) = + t)
2

3
3
2
x 1 = x + 9x 19x + 11 (do f(t) ng bin trn R)

x=1

3
3
2
(x 1) = x + 9x 19x + 11
x = 2
x=3
Vy (6.1) c tp nghim S = {1; 2; 3} 2
Chng ta lm quen vi mt s bi phng trnh tng. Hy xem qua nhng bi phng trnh
c tch.

Bi 7: Gii phng trnh x3 + 3x2 + 4x + 2 = (3x + 2) 3x + 1 (7.1)

Gii
3
~ tng: thot nhn th VT c bc 3, VP c bc nn kh c th dng n iu. Nhng
2

nu VP ta coi y = 3x + 1 l n th VP cng l bc 3 theo y. Nh vy cn phn tch


3x + 2 = m(3x + 1) + n(), khi VP c dng my 3 + ny. D thy t (*) c ngay m = n = 1.
Cng vic cn li l a VT v dng (x u)3 + x u l ta c th dng n iu. ng nht
h s ta c u = 1.
~ Li gii:
1
KX: x >
3
Ta c:

(7.1) (x + 1)3 + x + 1 = (3x + 1 + 1)( 3x + 1) = ( 3x + 1)3 + 3x + 1

f (x + 1) = f ( 3x + 1) (vi f (t) = t3 + t)

x + 1 = 3x + 1 x {0; 1} (tho KX)


Vy (7.1) c tp nghim S = {0; 1} 2.
~ Nhn xt: Vi nhng bi phng trnh tch cn linh hot trong vic i bin v xy dng
hm c th a hai v v hm c trng. Mt s bi nhn vo rt phc tp i hi ta phi
bnh tnh phn tch. Hy nh ta lun c gng phn tch biu thc bc ln theo biu thc bc nh.
Bi 8: Gii phng trnh 3x(2 +

9x2 + 3) + (4x + 2)( 1 + x + x2 + 1) = 0 (8.1)

Gii

113
~ tng: Nhn qua s sp xp ca bi ton, ta thy hai biu thc VT kh ging nhau v
hi vng c th tm hm c trng ca phng trnh t y.
u tin a mi biu thc v 1 v:

(8.1) (4x + 2)( 1 + x + x2 + 1) = 3x(2 + 9x2 + 3) (7.2)


Nh kinh nghim bi 7, ta s phn tch biu thc bc ln theo biu thc bc nh.
p
Ta c V P (8.2) = 3x(2 + (3x)2 + 3) nn hi vng V T (8.2) cng c th a v

f (t) = t(2 + t2 + 3). V xut hin s 2 trong f (t) ta bin i:

V T (8.2) = (2x + 1)( 4x2 + 4x + 4 + 2)


D thy 4x2 + 4x + 4 = (2x + 1)2 + 3. Vy ta xy dng hm thnh cng.
2t3 + 3t
Tuy nhin hm s f (t) c f 0 (t) = 2 +
nn c th i chiu n iu, do ta phi
t4 + 3t2
1
c thm mt nhn xt: (8.1) ch c nghim trong [ ; 0]. n y bi ton thc s c gii
2
quyt.
~ Li gii:
1
1
> Nu x > 0 hoc x <
th (8.1) v nghim. Vy ta xt x [ ; 0].
2
2
Ta c
p
p
(8.1) (8.2) (2x + 1)(2 + (2x + 1)2 + 3) = (3x)(2 + (3x)2 + 3)
f (2x + 1) = f (3x) (8.3) vi f (t) = t(2 +

t2 + 3)

1
2t3 + 3t
1
> 0 t [ ; 0] nn (8.3) 2x + 1 = 3x x =
Do f 0 (t) = 2 +
(chn)
2
5
t4 + 3t2
1
Vy (8.1) c tp nghim S = { } 2
5
~ Nhn xt: y l mt bi ton hay v kh, i hi phi c k nng bin i linh hot. Ta
cng c li gii gn gi hn, khng cn dng o hm nh sau:
p
p
(8.1) (2x + 1)(2 + (2x + 1)2 + 3) = (3x)(2 + (3x)2 + 3) ()
1 1
;
):
2 5
3x < 2x 1 < 0 (3x)2 > (2x + 1)2
p
p
2 + (3x)2 + 3 > 2 + (2x + 1)2 + 3 V T () < V P ()
1
> Nu x ( ; 0]: Chng minh tng t ta cng c () v nghim.
5
1
> Nu x =
: Ta c () nghim ng.
5
1
Vy (8.1) c tp nghim S = { } 2
5
> Nu x (

Bi tp t luyn
Gii cc phng trnh sau

Bi 1) x3 15x2 + 78x 141 = 5 3 2x 9 (1)


2
Bi 2) 2x1 2x x = (x 1)2 (2)
2x 1
Bi 3) log3 (
) = 3x2 8x + 5 (3)
(x 1)2

114

Bi 4) sin 2x + cos x = 1 + log2 sin x (x (0; ) (4)


p
p

2p
Bi 5) Chng minh phng trnh x 3 18 + x 5 = x 2 (5) c nghim duy nht.

PHNG PHP DNG BT NG THC


L thuyt
Ta gii phng trnh, h phng trnh bng bt ng thc da trn hai tng sau:
I 1) Bin i phng

trnh v dng f (x) = g(x) m


f (x) > a
f (x) 6 a
hay
vi a l hng s.
g(x) 6 a
g(x) > a

f (x) = a
Nghim ca phng trnh l cc gi tr x tha mn
.
g(x) = a
I 2) Bin i phng trnh v dng h(x) = m ( m l hng s) m ta lun c h(x) > m hoc
h(x) 6 m th nghim ca phng trnh l cc gi tr x lm cho du ca ng thc xy ra.
I Mt s phng php hay c s dng l a v bnh phng ng, s dng tnh n
iu ca hm s nh gi mt cch hp l, s dng mt s bt ng thc nh bt ng
thc AM-GM, BCS v bt ng thc Holder.

> Bt ng thc AM-GM : x1 + x2 + .... + xn > n n x1 .x2 ....xn , vi xi > 0 (i = 1, n)


ng thc xy ra khi v ch khi x1 = x2 = .... = xn .
> Bt ng thc Cauchy-Schwarz (BCS):

n
X

!
a2i

n
X

i=1

ng thc xy ra khi v ch khi

!
b2i

>

i=1

n
X

!2
ai b i

i=1

a2
an
a1
=
= ... =
b1
b2
bn

> Bt ng thc Holder: (dng m rng ca BCS): cho m b n s dng (m, n > 2):
(a11 , a12,...., a1n ) (a21 , a22,...., a2n ) ; ....; (am1 , am2,...., amn )
Ta lun c:
(a11 .a21 ....am1 + a12 .a22 ....am2 + ...a1n .a2n ....amn )m
6 (a11 m + a12 m + ... + a1n m )(a21 m + a22 m + ... + a2n m )...(am1 m + am2 m + ... + amn m )
ng thc xy ra khi v ch khi cc b s t l vi nhau.

Bi tp v d
Bi 1: Gii phng trnh

13 x 1 + 9 x + 1 = 16x

115
Gii
~ tng: Ta thy VT c bc thp hn nn ngh ti vic s dng bt ng thc BCS
chng minh V T 6 V P .
y nu k ta s cn chn im ri sao cho xut hin (16x a) t c th tip tc
s dng bt ng thc AM-GM.
~ Li gii:
iu kin: x > 1
p dng bt ng thc BCS:


2
V T 2 = ( 13. 13x 13 + 3 3. 3x + 3) 6 (13 + 27)(13x 13 + 3x + 3) = 40(16x 10)
p dng bt ng thc AM-GM:
10 + 16x 10 2
) = (16x)2 = V P 2
2

13x

13
3x + 3

=
13
3 3 x
V T 6 V P . ng thc xy ra

10 = 16x 10
Vy phng trnh v nghim. 2
~ Nhn xt: Ta cng c bi ton tng t:

Bi 1*: Gii phng trnh 13 x2 x4 + 9 x2 + x4 = 16


40(16x 10) = 4.10.(16x 10) 6 4.(

Bi 2: Gii phng trnh

x2 + 4x + 5 = 2 2x + 3
Gii

~ tng: Nh bi trn, do VT c bc nh hn VP nn nhiu kh nng c th dng bt ng


thc AM-GM.
~ Li gii:
3
iu kin x >
.
2
T phng trnh, p dng bt ng thc AM-GM:

(2x + 3) + 1 > 2 2x + 3 = x2 + 4x + 5
2x + 4 > x2 + 4x + 5 x2 + 2x + 1 6 0
(x + 1)2 6 0 x = 1 (chn)
Vy phng trnh c nghim duy nht x = 1 2.
Bi 3: Gii phng trnh

x2 + x 1 +

x2 + x + 1 = x2 x + 2
Gii

~ tng: rng VT c dng A + B nn ta ngh ti vic s dng bt ng thc BCS


tm gi tr ln nht. Ri sau ta s chng minh VP ln hn hoc bng gi tr ny.

116
~ Li gii:(
iu kin:

x2 + x 1 > 0
x2 + x + 1 > 0

p dng bt ng thc BCS c:


VT 6

2(x2 + x 1 x2 + x + 1) = 2 x

Mt khc:

2
V P 2 x = x2 x + 2 2 x = (x 1)2 + ( x 1) > 0 V P > 2 x > V T

2
2

x + x 1 = x + x + 1

x = 1 (tho KX)
ng thc xy ra (x 1)2 = 0

x = 1
Vy phng trnh c nghim x = 1 2.

Bi 4: Gii phng trnh


r
x=

1
x +
x

r
1

1
x

Gii
y l mt bi ton trong k thi v ch ton cng ha Yugoslavia (Nam T) nm 1977, a s
cc li gii thc hin php bin i tng ng hoc gii bng phng php a v h phng
trnh.Sau y l mt li gii khc s dng bt ng thc AM-GM:
iu kin x > 1.
p dng bt ng thc AM-GM ta c:
r
r
r
r
1
1
1
1
1
1
1
x + 1 = (x ).(1) + (x 1). 6 (x + 1 + x 1 + ) = x
x
x
x
x
2
x
x

x 1 = 1
x
Vy phng trnh cho tng ng:
1

x 1 =
x

1+ 5
Kt hp vi iu kin ta tm c x =
2.
2
Bi 5: Gii phng trnh

1 x2 +

1+x+

1x=3

Gii

~ tng: y ta thy rng VT l tng ca cc biu thc c dng 4 A, cn VP l hng


s. Do ta ngh ti vic s dng bt ng thc AM-GM cho tng biu thc VT chng
minh V T 6 V P .
~ Li gii:

117
iu kin: 1 6 x 6 1
p dng bt ng thc AM-GM ta c :

1
+
x
+
1x

4
4
1 x2 = (1 x)(1 + x) 6

(1)

1+ 1+x
4
(2)
1
+
x
6

4 1 x 6 1 + 1 x (3)
2
Cng theo v (1), (2) v (3) ta c:

1 x2 +

1+x+

1x61+

1+x+ 1x

p dng bt ng thc BCS:

1+x+

1x6

p
2(1 + x + 1 x) = 2 V T 6 1 + 2 = 3 = V P

1 x = 1 + x

x = 0 ( tha)
ng thc xy ra 1 + x = 1

1 x = 1
Vy phng trnh c nghim x = 0 2.
Bi 6: Gii phng trnh
p
4

(x 2)(4 x) +

4
x 2 + 4 4 x + 6x 3x = x3 + 30
Gii

p
~ tng: ta thy ngay phi p dng bt ng thc AM-GM cho 4 (x 2)(4 x) v

6x 3x v bt ng thc BCS cho ( 4 x 2 + 4 4 x). y vic kh khn ch l phi dng

AM-GM 6x 3x nh hn hoc bng biu thc dng x3 + a sao cho ph hp nht .


~ Li gii:
iu kin: 2 6 x 6 4
Ap dng bt ng thc AM-GM:

x2+4x
p
4
(x 2)(4 x) 6
= 1 (1)
2

6x3x = 227x3 6 x3 + 27 (2)


p dng bt ng thc BCS:

q p
q

4
x 2 + 4 x 6 2( x 2 + 4 x) 6 2. 2(x 2 + 4 x) = 2 (3)

Cng theo v (1), (2) v (3) ta c:


p

4
(x 2)(4 x) + 4 x 2 + 4 4 x + 6x 3x 6 x3 + 30 V T 6 V P

118

x2=4x

x = 3 (tha).
ng thc xy ra x3 = 27

x 2 = 4 x
Vy phng trnh c nghim x = 3 2.
Bi 7:Gii phng trnh

5
5
27x10 5x6 + 864 = 0
Gii

~ tng: H s ca bi kh xu ( 5 864 = 2 5 27) nn kh c th bin i ng thc.

Vy ta s c gng s dng bt ng thc chng minh 5 27x10 + 5 864 > 5x6 .


V vic ny d dng hn ta chia c 2 v ca phng trnh cho x6 .
~ Li gii:
D thy x = 0 khng phi l nghim ca phng trnh, chia 2 v ca phng trnh cho x6 , vit
li phng trnh di dng:

2
27
5
4
=5
27x +
6
x
p dng bt ng thc AM-GM c:

5
VT =

27x
+
3

27x
+
3

5
27x
27
27
+ 6 + 6 >5
3
x
x

275 x12
=5=VP
33 .x12

5
5

27x4
27
ng thc xy ra
= 6 x = 10 3
3
x
Vy phng trnh c nghim x = 10 3 2.
Bi 8: Gii phng trnh

r
2 x2 +

1
1
= 4 (x + )
2
x
x

Gii
~ tng: Hng i ca ta khi nhn vo bi ton l a ht cc bin sang mt v, t dng
BCS mt cch thch hp.
r kh c x khi dng BCS th ta s nhm

1
1
x vi 2 x2 v vi 2 2 .
x
x
~ Li gii:

2 x2 > 0

1
KX: 2 2 > 0

x 6= 0
r

1
1
2
Phng trnh cho tng ng vi x + 2 x + + 2 2 = 4.
x
x

119
p dng bt ng thc BCS:

(x + 2 x2 )2 6 2(x2 + 2 x2 ) = 4
r
1
1
1
1

( + 2 2 )2 6 2( 2 + 2 2 ) = 4
x
x
x
x

x + 2 x 2 6 2

1
1
r
1
x + 2 x2 + + 2 2 6 4
1

x
x
+ 2 2 62
x
x
Du "=" xy ra ti x = 1 (tha iu kin).
Vy phng trnh c nghim duy nht x = 1 2.
Bi 9: Gii phng trnh

1
3x2 1 + x2 x x x2 + 1 = (7x2 x + 4)
2 2
Gii
> tng: D on phng trnh c nghim duy nht x = 1, t ta ngh ti dng bt
ng thc v VT c nhiu cn thc phc tp.
~ Li gii:
1
iu kin x > 1 hoc x 6 .
3
p dng bt ng thc BCS:
p

3x2 1 + x2 x x x2 + 1 6 (x2 + 2)(3x2 1 + x2 x + x2 + 1)


p

3x2 1 + x2 x x x2 + 1 6 (x2 + 2)(5x2 x) (1)


p dng bt ng thc AM-GM:
p
1
1
1
(7x2 x + 4) = [5x2 x + 2(x2 + 2)] > .2 (5x2 x).2(x2 + 2)
2 2
2 2
2 2
p
1
(7x2 x + 4) > (5x2 x)(x2 + 2) (2)
2 2
p

1
T (1) v (2) ta c 3x2 1 + x2 x x x2 + 1 = (7x2 x + 4) = (x2 + 2)(5x2 x)
2 2
Du "=" xy ra khi x = 1 (tha iu kin).
Vy nghim ca phng trnh l x = 1 2.
Bi 10: Gii phng trnh
r

1x
2x + x2
=
x
1 + x2
Gii

iu kin: 0 6 x 6 1
d dng hn trong vic nh gi ta vit li phng trnh:

1
2x 1
1=1+
x
1 + x2

120
1
l mt nghim ca phng trnh. Ta chng minh nghim ny l duy nht:
2
(
VT >1
1
* Nu 0 6 x < , khi
, phng trnh v nghim.
2
VP <1
(
VT <1
1
* Nu < x 6 1, khi
, phng trnh v nghim.
2
VP >1
1
Vy phng trnh c nghim x = 2.
2
Nhn thy x =

> Nhn xt: li gii trn khc vi nhng bi trc, chng ta khng th chng minh V T > V P
hay V T 6 V P nu khng chia trng hp ra xt. Do i vi nhng bi phng trnh c
s dng phng php nh gi i hi ta phi bin i linh hot (nhn qua th c v VP l
hm tng cn VT l hm gim, nn nhiu kh nng PT c nghim duy nht) . K thut chia
trng hp ny cng rt quan trng trong gii h phng trnh.
Bi 11: Gii phng trnh

2
1x
3x
= (x 1)(1
x2 + 4x + 5 2
)
2
x +x+1
x +x+1
Gii
K: x 6 1.
Phng trnh cho tng ng vi:



x2 2x + 1
2 1x
(x + 2) + 2
= (x 1) 1
x +x+1
x2 + x + 1
2

hay
(x + 2)2 = (1 x)
t y =

1 x, z =


(1 x)2
2 1x

1 2
x +x+1
x2 + x + 1

x2 + x + 1 (y > 0, z > 0), phng trnh tr thnh:




y4
2y
2
2
(x + 2) = y
1 2
z
z

D thy V T > 0. Ta c:

 2



2
y4
y
2y
2y
2
2
2 y
VP =y
1 2 =y 2 +
1 = y
1 60
z
z
z
z
z
T suy ra V T > V P v phng trnh tng ng vi:
(
(
x
+
2
=
0
x+2=0
y


x = 2 (chn)
y
1 =0
y =0y =z
z
Vy phng trnh c nghim duy nht x = 2 2
Bi 12: Gii phng trnh
h
i
2
2
2
2
2
13 (x 3x + 6) + (x 2x + 7) = (5x2 12x + 33)

121
Gii
~ tng: thy rng 13 = 22 + 32 nn VT c dng (a2 + b2 )(x2 + y 2 ), t ta ngh ti
vic s dng BCS chng minh V T > V P .
~ Li gii:
Ap dng bt ng thc BCS c:
h
i
2
2
V T = (22 + 32 ) (x2 3x + 6) + (x2 2x + 7)

2
2
> 2(x2 3x + 6) + 3(x2 2x + 7) = (5x2 12x + 33) = V P
ng thc xy ra
3
2
= 2
2(x2 2x + 7) = 3(x2 3x + 6) x2 5x + 4 = 0
2
x 3x + 6
x 2x"+ 7
x=1
(x 1)(x 4) = 0
x=4
Vy phng trnh c tp nghim S = {1; 4} 2
Bi 13: Gii phng trnh
x3000 + 500x3 + 1500x + 1999 = 0
Gii
Vi x > 0, V T > 0 = V P (v l) x 6 0 |x| = x
Ap dng bt ng thc AM-GM c:

3000
x3000 .1.1...1 = 3000 |x| = 3000x (1)
x3000 + 2999 = x3000 + 1 + 1 + ... + 1 > 3000

1000
x3000 + 999 = x3000 + 1 + 1 + ... + 1 > 1000
x3000 .1.1...1 = 1000 |x3 | = 1000x3 (2)
Ly (1)+(2) ta c:
2x3000 + 3998 > (1000x3 + 3000x)
x3000 + 500x3 + 1500x + 1999 > 0 V T > V P
(
ng thc xy ra

x3000 = 1

x = 1
x60
Vy phng trnh c nghim S = {1} 2
Bi 14: Gii cc phng trnh sau:
1
a) 32x4 + (4x 1)4 =
27
(1 + x)8 + 16x4
1
b)
=
(1 + x2 )4
8
Gii

1
a) Ta chng minh bt ng thc ph sau: x4 + y 4 + z 4 > (x + y + z)4
27 

2
1
1
1
1
4
4
4
2
2
2 2
2
Tht vy: x + y + z > (x + y + z ) >
(x + y + z)
= (x + y + z)4
3
3 3
27

122
x4 + y 4 + z 4 >

1
(x + y + z)4
27

p dng ta c
32x4 + (4x 1)4 = (2x)4 + (2x)4 + (1 4x)4 >

1
1
(2x + 2x + 1 4x) =
27
27

1
Do phng trnh trn tng ng 2x = 1 4x x =
6
1
Vy phng trnh c nghim duy nht x = 2.
6
b) Ta chng minh bt ng thc ph sau: A4 + B 4 >
Vi A = (x + 1)2 v B = 2x th ta c:

(A2 + B 2 )2
(A + B)4
>
2
8

(1 + x)8 + 16x4
1 (1 + x2 )4
1
> .
=
2
4
2
4
(1 + x )
8 (1 + x )
8

Du "=" xy ra khi v ch khi (x + 1)2 = 2x x = 2 + 3

Vy phng trnh c nghim duy nht x = 2 + 3 2.

Bi tp t luyn

trnh 3x2 + 6x + 7 + 5x2 + 10x + 14 = 4 2x x2

trnh x2 6x + 11 + x2 6x + 13 + 4 x2 4x + 5 = 3 + 2

6 2
4 2
trnh 19 x1 + 5 x 1 + 95 x 3x+2 = 3
p
trnh x2 3x + 3.5 = (x2 2x + 2)(x2 4x + 5)

1
x2
Bi 5: Gii phng trnh 5x3 + 3x2 + 3x 2 =
+ 3x
2
2
Bi
6:
Gii
phng
trnh
p
p

x
x
2 8x + 7
2 8x 9) = 2x+1
x2 8x + 7 + x2 8x 9)
+
(
x
x
(
r
1
5
2
Bi 7: Gii phng trnh 8x +
=
x
2
1
2
2
Bi 8: Gii phng trnh
(3x3 + x2 + 9x 7) = (x2 + 2) + (x3 + 3x 3)
10
Bi
Bi
Bi
Bi

1:
2:
3:
4:

Gii
Gii
Gii
Gii

phng
phng
phng
phng

Chng IV: PHNG TRNH M-LOGARIT

Bi vit ny s gii thiu n cc bn mt s Phng php gii phng trnh m - Logarit, vi


mong mun t nhiu s l ti liu tham kho hu ch cho bn c, qua chun b hnh trang
cho cc bn hc sinh trong cc k thi tuyn sinh vo i hc.

L THUYT
Cho phng trnh ax = m (a > 0, a 6= 1), ta c:
Nu m > 0 th phng trnh cho c mt nghim duy nht x = loga m.
Nu m 6 0 th phng trnh cho v nghim.
Hm s logarit:
ac = b c = loga b.
aloga b = b.
loga (b1 .b2 ) = loga b1 + loga b2 .
loga b = . loga b.
loga c =

logb c
.
logb a

logb c = logb a. loga c.


loga b =

1
.
logb a

loga c =

1
. loga c.

Sau y l mt s phng php gii phng trnh m - logarit.

PHNG PHP T N PH
Phng php t n ph l mt phng php kh ph bin i vi cc bi ton phng trnh
v h phng trnh. Cc bi ton gii phng trnh m ta c th p dng phng php ny,
nu d, th ta s thy ngay du hiu l mt biu thc cha bin no lp i lp li nhiu
ln, cn nu kh hn, th ta cn phi c mt t bin i kho lo, ch yu l a v hnh
dng s khai ca bi ton, l mt phng trnh vi cc biu thc cha bin lp li. Cng c
trng hp, bi ton yu cu ta phi t thm nhiu n ph khc, nhm to ra mt phng
123

124
trnh hoc mt h phng trnh mi d dng gii quyt hn. Sau y, chng ta hy cng nhau
xt cc v d nh nhm lm sng t hn tng gii quyt cc bi ton dng ny.
2 x

Bi 1. Gii phng trnh 2x

+ 22+xx = 3

Gii
~ tng: Ta nhn thy rng biu thc x2 x c lp li trong phng trnh trn. V th,
ta s c ngay tng l phi t mt n mi thay th cho biu thc ny.
~ Li gii:
t t = x2 x, phng trnh cho tr thnh:
2t + 22t = 3 2t + 4.

1
= 3 (2t )2 + 4 = 3.2t
2t

n y, ta li thy phng trnh thu c c biu thc 2t c lp li, vy ta s t thm


mt n mi.
t u = 2t , ta c : u2 3u + 4 = 0. Phng trnh ny v nghim.
Do , ta c phng trnh cho v nghim 2
Bi 2. Gii phng trnh log2 x +

p
10 log2 x + 6 = 9

Gii
t t = log2 x, phng trnh c vit li thnh:
(

t69
t + 10t + 6 = 9
t=3x=8
10t + 6 = t2 18t + 81
Vy phng trnh cho c mt nhim duy nht x = 8 2
~ Nhn xt: hai bi ton trn, biu thc lp li c "phi by" ra trc mt, v chng
ta d dng nhn bit c phng php t n ph i vi bi ton. Nhng bi ton sau y,
cc biu thc cha bin lp li s b giu i, t n gin, n tinh xo, v kh ca vic tm
ra quy tc "b n" y, cng l nc thang quyt nh mc kh ca bi ton.
Bi 3. Gii phng trnh 34x+8 4.32x+5 + 27 = 0 ()

Gii
~ tng: Vi bi ton ny, ta nhn thy rng cc hng t trong phng trnh u c ly
tha ca 3, do , u tin ta s trit tiu cc ly tha ny.
~ Li gii:
Ta c:
() 34x+8 4.32x+5 + 33 = 0 34x+5 4.32x+2 + 1 = 0
3.34x+4 4.32x+2 + 1 = 0

125
n y, ta d dng c c cch t n ph hp.
t t = 32x+2 > 0. Phng trnh cho tr thnh:

"
t=1
32x+2 = 1
3t2 4t + 1 = 0
1
32x+2 = 31
t=
3


3
2
Vy phng trnh cho c tp nghim l S = 1;
2

x = 1
3
x=
2

Ta s luyn tp thm vi bi ton tip theo.


Bi 4. Gii phng trnh 22x+6 + 2x+7 17 = 0 ()

Gii
Ta c:
() 22x+6 + 16.2x+3 17 = 0
t t = 2x+3 > 0, ta c:
"
t2 + 16t 17 = 0

t=1
2x+3 = 1 x = 3
t = 17 (loi)

Vy phng trnh cho c tp nghim l S = {3} 2


Bi 5. Gii phng trnh 2 +

x
x
3 + 2 3 = 4 ()

Gii


~ tng: Vi bi ton ny, rng 2 + 3 . 2 3 = 1. Ta s nhn lng lin hp
lm xut hin biu thc chung.
~ Li gii:
Ta c:

x
1
x = 4
() 2 + 3 +
2+ 3
x
t t = 2 + 3 > 0. Phng trnh tr thnh:
"
"
"
x

2+ 3 =2+ 3
t=2+ 3
x=1
1

x
1
t+ =4
t
t=2 3
x = 1
2+ 3 = 2+ 3
Vy phng trnh cho c nghim l x = 1 2
Bi 6. Gii phng trnh 2.16x 15.4x 8 = 0

Gii

126
~ tng: Ta ch rng 16x = (4x )2 , do , ta nh hng c ngay phng php t
n ph.
~ Li gii:
t t = 4x > 0, ta c:

t=8
3
4x = 8 22x = 23 x =
2t2 15t 8 = 0
1
2
t = (loi)
2
3
2
2
x
x
Bi 7. Gii phng trnh 3 + 5 + 16 3 5 = 2x+3 ()

Vy phng trnh c mt nghim duy nht x =

Gii

~ tng: Vi bi ton ny, tng u tin chnh l nhn lng lin hp cho (3 5). Th
nhng, v phi ca phng trnh by gi l 2x+3 , do , ta cn phi bin i phng trnh
lm sao cho mt i bin x hng t 2x+3 .
~ Li gii:
Ta c:
!x
!x
3+ 5
3 5
()
+ 16.
=8
2
2
n y, ta s dng
lng lin hp nh bnh
thng.
!x
!x
1
3 5
3+ 5
>0 =
. Ta c:
t t =
2
t
2
!x
!
16
ln 4
3+ 5
3+ 5

t+
=8t=4
= 4 x. ln
= ln 4 x =
t
2
2
ln 3 + 5 ln 2
ln 4

2
ln 3 + 5 ln 2
x
x
Bi 8. Gii phng trnh 7 + 4 3 3 2 3 + 2 = 0

Vy phng trnh c mt nghim duy nht x =

Gii
Vit li phng trnh:

t t = 2 +

2+


2x
x
3
3 2 3 +2=0

x
x
1
3 > 0 = 2 3 . Ta c:
t
"

x
t=1
3
t2 + 2 = 0 2
2+ 3 =1x=0
t
t t+3=0

Vy phng trnh cho c nghim duy nht x = 0 2

Bi 9. Gii phng trnh 2 x 4 + x 6 = x 9

127
Gii
~ tng: Bi ton ny yu cu chng ra cn phi bin i mt hp l to ra biu thc
thch hp cho vic t n ph.
~ Li gii:
Phng trnh cho tng ng vi
 1  2
3 x
3 x
2+
=
2
2
 1
3 x
t t =
> 0, ta c:
2
"
2 + t = t2

t = 1 (loi)
t=2
t=2

 
 1
3
1
3 x
= ln 2 x = log2 3 1
= 2 . ln

2
x
2
Vy phng trnh cho c mt nghim duy nht x = log2 3 1 2
Bi 10. Gii phng trnh 3.16x + 2.81x = 5.36x
Gii
Phng trnh cho tng ng vi
 2x
 x
9
9
3+2
=5
4
4
 x
9
> 0. Phng trnh tr thnh:
t t =
4

t=1
x=0

3 + 2t2 = 5t

3
1
t=
x=
2
2


1
Vy phng trnh c tp nghim S = 0;
2
2
Bi 11. Gii phng trnh 125x + 50x = 23x+1


Phng trnh tng ng vi
 x
5
t t =
> 0. Ta c:
2

125
8

x


+

Gii
x
50
= 2.
8

t3 + t2 2 = 0 t = 1 x = 0
Vy phng trnh cho c nghim duy nht x = 0 2
1
2
Bi 12. Gii phng trnh
+
=1
4 log x 2 + log x

128
Gii
(
t t = log x, suy ra

t 6= 4
. Phng trnh tr thnh
t 6= 2
"
"
t=1
x = 10
2
1
+
=1

4t 2+t
t=2
x = 100

Vy phng trnh cho c hai nghim l x = 10 v x = 100 2


Bi 13. Gii phng trnh

p
p
1 + log0.04 x + 3 + log0.2 x = 1
Gii

~ tng: i vi cc biu thc loga f (x) vi a > 0, th iu kin hm logarit ny tn ti


l f (x) > 0. Khi tin hnh gii cc bi ton phng trnh logarit, ta phi c bit ch n
iu kin xc nh ca bi ton.
~ Li gii:
KX: log0.2 x > 2, x > 0.
t t = log0.2 x t > 2. Ta c:
r
r

3
1
1 2 5
t + t + 3 = 3 t
1+ t+ 3+t=12
2
2
2
2
3
Do t > 1 nn 3 t 6 0, suy ra t = 2, hay x = 25 2
2
Bi 14. Gii phng trnh logx2 16 + log2x 64 = 3
Gii
1
khng tha mn phng trnh cho.
2
Vi mt s bin i nh, phng trnh tng ng vi:
Ta nhn thy rng x = 1 v x =

2
6
+
=3
log2 x 1 + log2 x
(
t t = log2 x

t 6= 0
. Ta c:
t 6= 1

"
t=2
x=4
2
6
+
=3
1
t t+1
x = 21/3
t=
3


Vy phng trnh cho c tp nghim S = 4; 21/3 2
Bi 15. Gii h phng trnh sau:
(
log1+x (1 2y + y 2 ) + log1y (1 + 2x + x2 ) = 4 (1)
log1+x (1 + 2y) + log12y (1 + 2x) = 2 (2)
(H Quc gia TP HCM)

129
Gii
(
iu kin:

(
0 < 1 y 6= 1
x > 1

.
0 < 1 + x 6= 1
y<1

Ta c:
(1) logx+1 (1 y)2 + log1y (1 + x)2 = 4 log1+x (1 y) + log1y (1 + x) = 2 (3)
1
t t = log1+x (1 y), suy ra log1y (1 + x) = . Phng trnh (3) tr thnh:
t
t+

1
= 2 t2 2t + 1 = 0 t = 1
t

Suy ra
log1+x (1 y) = 1 1 y = 1 + x x = y
Thay vo phng trnh (2), ta c
log1+x (1 2x) + log1+x (1 + 2x) = 2 log1+x (1 4x2 ) = 2

x = 0 (loi)
1 4x2 = (1 + x)2 5x2 + 2x = 0
2
x=
5
2
Suy ra y = .
5



2 2
Th li nghim nhn c vo h phng trnh ban u, ta c c nghim ;
l nghim
5 5
duy nht ca h cho 2
Bi 16. Gii phng trnh log3 (3x 1). log3 (3x+1 3) = 6

Gii
t t = log3 (3x 1), phng trnh c vit li:
"
"
"
x
t=2
log3 (3 1) = 2
x = log3 10
t2 + t 6 = 0

x
t = 3
log3 (3 1) = 3
x = log3 28 3
Vy phng trnh cho c hai nghim l x = log3 10 v x = log3 28 3 2
Bi 17. Gii phng trnh log4 (x

x2 1). log5 (x +

x2 1) = log20 (x

x2 1)

Gii

1
x2 1 = .
t
Phng trnh cho tng ng vi

t t = x

x2 1, suy ra x +

"
log4 t = 0
1
log4 t. log5 = log20 t log4 t. log5 t = log20 4. log4 t
t
log5 t = log20 4

130
"

t=1
.
t = 5 log20 4

F Vi t = 1 suy ra x = 1. (


x x2 1 = 5 log20 4
1 log20 4
log20 4
log20 4

F Vi t = 5
suy ra
5
+
5

x
=
log
4
2
x + x2 1 = 5 20

1 log20 4
Th li cc gi tr nhn c, phng trnh cho c hai nghim l x = 1 v x =
5
+ 5log20 4 2
2
Bi 18. Gii phng trnh log2 (x2 + 3x + 2) + log2 (x2 + 7x + 12) = 3 + log2 3

Gii
(
Nghim ca phng trnh (nu c) phi tha mn iu kin xc nh

x2 + 3x + 2 > 0
x2 + 7x + 12 > 0

Phng trnh tng ng vi


log2 (x + 1)(x + 2)(x + 3)(x + 4) = log2 24 x + 1)(x + 2)(x + 3)(x + 4) = 24
t t = x2 + 5x + 4, ta thu c
"
t(t + 2) = 24

t = 6
t=4

"
x=0
x2 + 5x + 4 = 6
.

Suy ra
2
x = 5
x + 5x + 4 = 4
Nhn thy cc gi tr ny tha mn tp xc nh ban u. Vy, phng trnh cho c hai
nghim l x = 0 v x = 5 2
Qua cc bi ton trn, chng ta rt ra c mt s kinh nghim v mt s iu ng lu khi
gii phng trnh m - logarit:
Lu n iu kin xc nh ca bi ton a ra. Cc iu kin ny c th gip ta chn
c min nghim trn phng trnh h qu, gip ta thu c nghim chnh xc hn. Cn nu
nh chng ta khng xc nh iu kin bi ton, th sau khi gii quyt xong, chng ta cn mt
bc khng th b qua l th li nghim vo bi ton.
c c cc bc bin i mu mo trong cc bi ton phng trnh nh trn, i hi
chng ta phi c kinh nghim lm bi, luyn tp vi nhiu dng bi tp khc nhau. V gip
cho cc bn luyn tp nhiu hn vi phng php ny, chng ti xin ngh vi cc bn th
luyn tp vi mt s bi tp t luyn sau y:
"

Bi tp t luyn:
1/ 43+2 cos x 7.41+cos x 2 = 0.
2/ log22 x.logx (4x2 ) = 12.
log2 x
log8 4x
3/
=
.
log4 2x
log16 8x
4
4/ (2 log3 x) log9x 3
= 1.
1 log3 x
5/ 2xlog2 x + 2x3log8 x 5 = 0.

131
p
cos x p
cos x 5
6/
7+4 3
74 3
+
= .
2
7/ log2 (9 2x ) = 3 x.
2
8/ 7log25 (5x)1 xlog5 7 = 0.

PHNG PHP DNG N IU HM S


Bi 1. Gii phng trnh 22x1 + 32x = 52x+1 = 2x + 3x+1 + 5x+2

Gii
Nhn vo bi ton ny, chng ta s thy ngay rng phng php t n ph c v nh khng
c tc dng. Chng ta s c nhng nhn xt sau y:
Nu x > 1 th 22x1 + 32x = 52x+1 > 2x + 3x+1 + 5x+2
Nu x < 1 th 22x1 + 32x = 52x+1 < 2x + 3x+1 + 5x+2
Nu x = 1 th 22x1 + 32x = 52x+1 = 2x + 3x+1 + 5x+2 (= 136).
Vy phng trnh c 1 nghim duy nht x = 1 2
Bi 2. Gii phng trnh 32x + 42x = 2.12x

Gii
p dng BT AM-GM, ta c: 32x + 42x > 2.12x . Du ng thc xy ra, do : 3x = 4x .
Nu x > 0: 3x < 4x .
Nu x < 0: 3x > 4x .
Nu x = 0: 3x = 4x = 1.
Vy phng trnh cho c nghim duy nht x = 0 2
Bi 3. Gii phng trnh x + log(x2 x 6) = 4 + log(x + 2)

Gii
Phng trnh tr thnh:
(
x + log(x + 2) + log(x 3) = 1 + log(x + 2)
(

x>3
log(x 3) = 4 x

Nu x > 4 th log(x 3) > log 1 = 0 > 4 x.


Nu 3 < x < 4 th log(x 3) < log 1 = 0 < 4 x.
Nu x = 4 th log(x 3) = log 1 = 0 = 4 x.

x+2>0
log(x 3) = 4 x

132

Vy phng trnh c nghim duy nht x = 4 2


Bi 4. Gii phng trnh (1 + 2x )(1 + 3x )(1 + 36x ) = (1 + 6x )3

Gii
Ta c BT sau:
(1 + x)(1 + y)(1 + z) > (1 +

3
xyz)3 , x, y, z > 0

p dng vo bi ton, ta c:
(1 + 2x )(1 + 3x )(1 + 36x ) > (1 +

2x 3x 36x ) = (1 + 6x )3

Du ng thc xy ra, do 2x = 3x = 36x x = 0.


Vy phng trnh c nghim duy nht x = 0 2
Bi 5. Gii phng trnh x = 2log5 (x+3)

Gii
KX: x + 3 > 0 x > 3. t t = log5 (x + 3) x = 5t 3. Ta c:
 t
 t
5
1
t
t
5 3=2
3
=1
3
2
 t
 t
1
5 3
5
3
> = 1.
Nu t > 1 th
3
2
2 2
 t
 t
5
1
5 3
Nu t < 1 th
3
< = 1.
3
2
2 2
 t
 t
5
1
Nu t = 1 th
3
= 1.
3
2
Do t = 1, hay x = 2 (nhn).
Vy phng trnh cho c mt nghim duy nht x = 2 2
~ Nhn xt: Vi cc bi ton trn, c th thy c nhng bc bin i rt kinh in ca
phng php nh gi BT. Cn nhng bi ton sau y, cc bn cn phi c s kt hp
kho lo gia cc BT v vic xt tnh n iu ca hm s.
Bi 6. Gii phng trnh x.2x = x(3 x) + 2(2x 1)

Gii
Phng trnh cho tng ng vi:
"
(x 2)(2x + x 1) = 0

x = 2 (nhn)
2x + x 1 = 0

Xt phng trnh 2x + x 1 = 0. t f (x) = 2x + x 1.


Ta c f 0 (x) = ln 2.2x + 1 > 0 x R.

133
Nhn thy x = 0 tha phng trnh ny, do , phng trnh cho c 2 nghim l x = 2 v
x=0 2
~ Qua bi ton ny, ta rt ra c mt iu: i vi vic gii phng trnh 2x + x 1 = 0
nh trn, ta c th suy ra c iu kin l x > 1. Th nhng iu ny l khng nht thit
cn thit, v trong mt s trng hp, vic gii iu kin h qu l kh rc ri. Mt cch
n gin hn ta c th lm l tm ra cc nghim ca phng trnh cho, sau thay vo
li phng trnh ban u nhn nhng nghim tha mn. Nhng li trong nhiu trng hp
khc, nhng iu kin h qu li t ra rt hu hiu cho vic gii phng trnh nh chn min
nghim, s dng BT, xt du, tnh n iu, ...v.v...
Bi 7. Gii phng trnh 2x2 6x + 2 = log2

2x + 1
(x 1)2

Gii

x 1 6= 0
iu kin:
1 .
x>
2
Phng trnh c vit li thnh:




1
1
+2 x+
= log2 (x 1)2 + 2(x 1)2 ()
log2 x +
2
2
t f (x) = log2 x + 2x vi x > 0. Ta c: f 0 (x) = 2 +

1
> 0 x > 0.
x. ln 2

Do , hm f ngbin trn
 (0; +).
1
Li t (), ta c f x +
= f ((x 1)2 ), suy ra:
2

3+ 7
1
x =
2
x + = (x 1)2
3 7
2
x=
2
(
)

3+ 7 3 7
Vy phng trnh cho c tp nghim S =
;
2
2
2

Bi 8. Gii phng trnh (1 + x)(2 + 4x ) = 3.4x

Gii
D c x > 1. Xt hm s f (x) = (1 + x)(2 + 4x ) 3.4x vi x (1; +).
Ta c: f 0 (x) = (x2)4x ln 4+4x +2, f 00 (x) = (x4)(ln 4)2 +2.4x ln 4, f 000 (x) = (x2)4x (ln 4)3 +
3.4x (ln 4)2 .
3
3
f 000 (x) = 0 x = 2
. t x0 = 2
.
ln 4
ln 4
Ta thy hm f 00 (x) nghch bin trn (1; x0 ), ng bin trn (x0 ; +) v f 00 (1) < 0,
f 00 (x0 ) < 0, lim f 00 (x) = +, nn phng trnh f 00 (x) = 0 c ng 1 nghim. Do ,
x+
0

phng trnh f (x) = 0 c ti a 2 nghim, suy ra phng trnh f (x) = 0 c khng qu 3


nghim.

134
1
Ta nhn thy c ba gi tr x tha mn phng trnh cho l x = 0, x = , x = 1.
2


1
Vy phng trnh cho c tp nghim S = 0; ; 1
2
2
Bi 9. Gii phng trnh 12x + 13x + 14x = 2x + 3x + 4x + 870x 2400x2 + 1560x

Gii
Xt hm s f (x) = 12x + 13x + 14x (2x + 3x + 4x + 870x 2400x2 + 1560x).
Ta s chng minh o hm cp 4 ca hm s ny lun m hoc lun dng vi mi gi tr x
c th nhn c. Ta c:
f (4) (x) = 12x (ln 12)4 + 13x (ln 13)4 + 14x (ln 14)4 2x (ln 2)4 3x (ln 3)4 4x (ln 4)4
Ta xt cc trng hp sau:
Nu x < 0, p dng BT Bernoulli cho s x < 0, ta c 12x > x(12 1) + 1 = 11x + 1. Tng
t, ta cng c 13x > 12x + 1, 14x > 13x + 1. Suy ra:
12x + 13x + 14x > 36x + 3
Li c
2x + 3x + 4x + 870x 2400x2 + 1560x < 3 + 36x + x(870x2 2400x + 1524) < 36x + 3
Do
12x + 13x + 14x > 2x + 3x + 4x + 870x 2400x2 + 1560x
iu ny mu thun vi bi cho.
Nu x > 0, suy ra f (4) > 0, do phng trnh cho khng c qu 4 nghim.
Nhn thy cc gi tr x = 0, x = 1, x = 2, x = 3 tha mn phng trnh cho.
Vy phng trnh c tp nghim S = {0; 1; 2; 3} 2
Bi 10. Gii phng trnh 9x (3x + 2x ) = 2x (8x + 7x ) + 5x (5x 2x )

Gii
Ta c nhn xt rng, vi bi ton trn, ta khng thy cc c s c mi lin quan no n nhau,
cc hm s cha bin th ton l hm s m. Ta th bin i v dng phng trnh c li
f (a) = f (b). Ta c:
9x (3x + 2x ) = 2x (8x + 7x ) + 5x (5x 2x )
10x + 12x 16x 25x = 12x + 14x 18x 27x
Ta xt hm s f (t) = tx + (t + 2)x (t + 6)x (t + 15)x vi t > 0. Phng trnh cho tng
ng vi
f (10) = f (12)

135
Ta thy rng, hm f lin tc v kh o hm trn [10; 12], nn theo nh l Largrange, tn ti
f (10) f (12)
c (10; 12) sao cho f 0 (c) =
= 0. Do , ta c:
10 12


x cx1 + (c + 2)x1 (c + 6)x1 (c + 15)x1 = 0
"
x=0
x1
x1
c
+ (c + 2)
= (c + 6)x1 + (c + 15)x1
Xt phng trnh cx1 + (c + 2)x1 = (c + 6)x1 + (c + 15)x1 . Ta c:
Nu x > 1 th x 1 > 0, suy ra cx1 + (c + 2)x1 < (c + 6)x1 + (c + 15)x1 .
Nu x < 1 th x 1 < 0, suy ra cx1 + (c + 2)x1 > (c + 6)x1 + (c + 15)x1 .
Nu x = 1 th cx1 + (c + 2)x1 = (c + 6)x1 + (c + 15)x1 = 2.
Th li, ta c phng trnh ny c tp nghim l S = {0; 1} 2
Qua cc bi tp v d nu trn, cc bn s thy rng, ngi ra s thng hay bt u to
ra bi ton t mt hng ng thc, t mt phng trnh n gin, hay t trng hp xy ra
du bng ca mt bt ng no . H cng c cng ct giu biu thc ban u bao nhiu,
th mc kh ca bi ton cng theo m kh ln by nhiu. V cng vic ca chng ta l
tm ra con ng m ngi ra s dng. C th c bi ton, chng ta i theo mt hng
khc tc gi, nhng i vi mt s bi ton, dng nh l con ng tc gi xut pht cng l
con ng duy nht m ta c th s dng gii quyt c bi ton. nhun nhuyn hn,
cc bn hy luyn tp vi cc bi ton sau y:

Bi tp t luyn
Gii cc phng trnh sau:

1/ 2 3x = x2 + 8x 14.
8
.
log3
4x + 4)
3/ (x + 3) log23 (x + 2) + 4 (x + 2) log3 (x + 2) = 16.

3
4/ ex + (x3 x) ln(x2 + 1) = e x .
2/ 22x+1 + 232x =

(4x2

PHNG PHP BIN I NG THC


Bn cht ca phng php bin i ng thc l c gng chuyn i phng trnh cho tr
thnh mt phng trnh tng ng, mt phng trnh h qu, hay mt h phng trnh, h
bt phng trnh m ta c th d dng gii quyt. Thng thng, ta s bin i bi v dng
hai v c cng c s hoc tr thnh mt hng ng thc quen thuc. Chng ta s lm r hn
nhng iu trn bng cch xt qua cc v d sau y.
Bi 1. Gii phng trnh 2x

2 x+8

= 413x

Gii
~ tng: Ta c mt nhn xt kh th v: 4 l ly tha bc hai ca 2. Do , ta s bin i
c hai v v cng c s 2.

136
~ Li gii:
Phng trnh cho tng ng vi:
"
x2 x+8

= 226x x2 x + 8 = 2 6x

x = 2
x = 3

Vy phng trnh cho c tp nghim l S = {2; 3} 2


Bi 2. Gii phng trnh (x2 x + 1)x

2 1

=1

Gii
Phng trnh tng ng vi:
(
"
x2 x + 1 > 0
x=0
(x2 x)(x2 1) = 0
2
2
(x x + 1 1)(x 1) = 0
x = 1
Vy tp nghim ca phng trnh cho l S = {1; 0; 1} 2
~ Nhn xt: i vi phng trnh c dng f (x)g(x) = 1, u tin, ta c iu kin l f (x) > 0.
Tip n, biu thc f (x)g(x) bng 1, th hoc f (x) = 1, hoc g(x) = 0, (
do , ta suy ra phng
f (x) > 0
trnh [f (x) 1]g(x) = 0. Vy, phng trnh cho s tng ng vi
.
[f (x) 1]g(x) = 0

23x = 5y 2 4y
Bi 3. Gii h phng trnh sau: 4x + 2x+1

=y
2x + 2
Gii
H tng ng vi:

(
23x

(
x=0
(

y=1
= 5y 2 4y
2x = y
(

x=2
2x = y
y 3 5y 2 + 4y = 0

y=4

Vy h cho c tp nghim {(x; y)} = {(0; 1); (2; 4)}2


Bi 4. Gii phng trnh 2x + 2x1 + 2x2 = 3x 3x1 + 3x2

Gii
~ tng: Ta c ngay nhn xt l biu thc ly tha cha x ging nhau c hai v ca
phng trnh. Do , ta s c tng l rt ra phn t chung ca mi hai v.
~ Li gii:
Phng trnh cho tr thnh:




1 1
1 1
x
x
2 1+ +
=3 1+
2 4
9 3

137
7
7
2 . = 3x .
4
9
x

 x  2
2
2
=
x=2
3
3

Vy phng trnh cho c nghim duy nht x = 2 2


x1
Bi 5. Gii phng trnh 5 .8 x = 500
x

Gii
iu kin xc nh: x 6= 0.
Phng trnh tng ng vi:
3(x 1)
3x
1 x3

x
5x .2
= 53 .22 5x3 = 2 x 5x3 = (2 x )

5x3 = (

1 x3
)
1
2x

"
1 x3
x3=0
x=3

1
(5.2 x )

=1
x = log5 2
5.2 x = 1

Vy phng trnh cho c tp nghim l S = {3; log5 2} 2

log 1 (y x) log4 = 1
y
Bi 6. Gii h phng trnh sau:
4

x2 + y 2 = 25 ()
Gii
iu kin xc nh: y > x; y > 0. Ta c bin i sau:
log 1 (y x) log4

1
1
= 1 log4 (y x) log4 = 1
y
y

4
log4

yx
3y
=1x=
y
4

Th vo phng trnh (*) ta c:




3y
4

2

+ y 2 = 25 y = 4

T iu kin bi, ta c y = 4, suy ra x = 3. Vy h phng trnh c nghim duy nht


(x; y) = (3; 4) 2
Bi 7. Gii phng trnh

2x . 3 4x . 3x 0.125 = 4 3 2

Gii

138

~ tng: Chng ta cn ch rng, nu trong bi ton c cn thc dng x th cn phi c


(
x>1
iu kin ca x l
.
xN
~ Li gii:

x > 1
KX:
3 . Nhn thy rng cc c s ca cc ly tha u l ly tha ca 2, do ta
3x N
s bin i bi ton v dng cc ly tha ca 2.
Phng trnh cho tng ng vi:
r

1
1
x x 1
7
2x
x=3

+
x x
1
7
=
2x .2 3 .2 2x = 22 .2 3 2 2 3 2x = 2 3 +
1
2 3 2x
3
x=
5
Kt hp vi KX, ta c phng trnh ny c mt nghim duy nht x = 3 2
1
1
3 (x + 1) +
Bi 8. Gii phng trnh log
log81 (x 3)2012 = 5 log243 [4(x 2)]
3
3
503
Gii
~ tng: Nhn vo bi, ta thy ngay rng c s ca cc hm logarit u l ly tha ca
3. Do , ta s bin i chng v dng cc hm s logarit c s 3.
~ Li gii:
KX: x > 2, x 6= 3.
Phng trnh tng ng vi
log3 (x + 1) + log3 |x 3| = log3 (4x 8) (x + 1)|x 3| = (4x 8)
Nu x > 3, ta c:
"
x2 2x 3 = 4x 8

x = 1 (loi)
x=5

Nu 2 < x < 3, ta c:

x = 1 + 12

x + 2x + 3 = 4x 8
x = 1 12 (loi)
"



Vy phng trnh cho c tp nghim S = 5; 1 + 12 2
V cho kinh nghim lm bi c cng c thm, cc bn hy th t gii cc bi ton t
luyn sau:
Bi tp t luyn.
Gii cc phng trnh:
1/ 2x .3x1 .5x2 = 12.

2
2/ (x2 2x + 2) 4x = 1.
2
2
3/ 2x +x 4.2x x 22x + 4 = 0

139

BI TP TNG HP
Sau y, chng ta s cng nhau th sc mnh vi nhng bi ton kh hn v hay hn. Cc bi
ton ny i hi s vn dng linh hot cc k nng bin i, t n ph, cng nh xt n
tnh n iu ca cc hm s ph cn dng n.
Bi 1. 3x2 + 1 + log2006

4x2 + 2
= x6
x6 + x2 + 1
Gii

Vit li phng trnh:


4x2 + 2
= x6 3x2 1
x6 + x2 + 1
Lu rng (x6 + x2 + 1) (4x2 + 2) = (x6 3x2 1).
t a = 4x2 + 2 > 2, b = x6 + x2 + 1 > 1, phng trnh cho tr thnh:
log2006

log2006 a + a = log2006 b + b ()
1
+ 1 > 0 x > 0, li do hm
x ln 2006
f lin tc trn (0; +), nn t (*) suy ra a = b. Ta cn phi gii phng trnh:
Xt hm s f (x) = log2006 x + x vi x > 0, ta c: f 0 (x) =

x6 3x2 1 = 0
t u = x2 > 0. Phng trnh c vit li: u3 3u 1 = 0.
1
t u = 2v v > 0, ta c 4v 3 3v = .
2
1
3
Xt g(v) = 4v 3v lin tc trn R. Ta c g 0 (v) = 0 v = .
2
 
1
Ta c g(0) = 0, g
= 1, g(1) = 1, do , phng trnh ch c mt nghim duy nht
2

v0 > 0, vi v0 < 1. t v = cos t vi 0 < t < , phng trnh tr thnh:


2
r

1
cos 3t = v = cos u = 2 cos x = 2 cos
2
9
9
9
r
r


Vy phng trnh cho c tp nghim l S =


2 cos ; 2 cos
2
9
9
Bi 2. sin2 x.2cos 2x +

1 2
sin 2x + cos 2x = 1
2
Gii

Phng trnh cho tng ng vi:


1 cos 2x cos 2x 1
.2
+ (1 cos2 2x) + cos 2x = 1
2
2
t t = cos 2x, ta c:
"
(1 t)2t + 1 t2 + 2t = 2 (1 t)(2t + t 1) = 0

t=1
2 +t1=0
t

140
Vi t = 1, suy ra x = k vi k Z.
Xt phng trnh f (t) = 2t + t 1 = 0.
Ta c hm f l hm s tng trn R nn f (t) = 0 c nhiu nht mt nghim, li do f (0) = 0,
k
nn suy ra t = 0 l nghim ca phng trnh f (t) = 0, suy ra x = +
vi k Z.
4
2 

k
+
|k Z 2
Vy phng trnh cho c tp nghim S = {k|k Z}
4
2
(
71+xy + 31+xy + 71x+y = 101+xy + 101x+y (1)

(x, y
Bi 3. Gii h phng trnh:
4x2 + 2x + 3 = 8y + 1 (2)
R)
Gii
1
3
iu kin: x > , y > .
2
8
Ta c b sau: Vi mi a > b > 1 ax + ax > bx + bx , x R (3), du ng thc xy ra
khi x = 0. Tht vy, bt ng thc (3) tng ng vi:


1
x
x
(a b ) 1 x x > 0 (4)
a b
(


ax > b x
1
x
x
Nu x > 0
(a b ) 1 x x > 0. Du ng thc xy ra khi x = 0.
a b
(ab)x > 1
(
Nu x 6 0



ax 6 b x
1
x
x
a b ) 1 x x > 0. Du ng tc xy ra khi x = 0.
a b
(ab)x 6 1

p dng b trn vo bi ton, ta c:


(
(
7(10t + 10t ) > 71+t + 71t
10t + 10t > 7t + 7t
(t = x y)

3(10t + 10t ) > 31+t + 31t


10t + 10t > 3t + 3t
101+t + 101t > 71+t + 71t + 31+t + 31t
101+xy + 101x+y > 71+xy + 71x+y + 31+xy + 31x+y (5)
T (1) v (5), ta c ng thc xy ra, do t = x y = 0 x = y. Thay vo phng trnh
(2) ta c:

4x2 + 2x + 3 = 8x + 1

9
1
4x2 6x + = 2x + 3 2x + 3 +
4
4

2 
2

3
1
2x
=
2x + 3
2
2
"

2x 1 = 2x + 3

2x 2 = 2x + 3

3 + 17
x =
4

5 21
x=
4

141

Vy HPT cho c hai nghim (x; y) =

3+

17 3 +
;

17

; (x; y) =

2x2 6x.4y + 4x = 3.4y+1 (1)


1
Bi 4. Tm s nghim ca h phng trnh:

|y 2| = 0 (2)
(x + 3)2

21 5
;

!
21
2
4

( ngh OLP 30-04 THPT chuyn Lng Th Vinh, ng Nai)


Gii
Trc tin, ta hy tm cc nghim "p" (nu c) ca h.
iu kin xc nh: x 6= 3. t t = 4y > 0, phng trnh (1) tr thnh:
"
x = 2
2x2 6tx + 4x 12t = 0 2(x + 2)(x 3t) = 0
x = 3t
Khi x = 2, th vo phng trnh (2), ta c
"
1 |y 2| = 0

y=3
y=1

T suy ra hai nghim ca h l (2; 3) v (2; 1).


V ta c c hai nghim "p". Tip theo, ta s i tip theo con ng cn li tm s
nghim.
Vi x = 3t, th vo phng trnh (2), ta c:
1
|y 2| = 0
(3t + 3)3
Ta c t = 4y y = log4 t.
1
Xt hm s f (t) =
| log4 t 2| vi t > 0.
(3t + 3)3
Ta s tm cch gii quyt du gi tr tuyt i ny, bng cch xt cc khong c th ca t:
1
log4 t + 2.
(3t + 3)3
9
1
Ta c f 0 (t) =

< 0, t > 16.


4
(3t + 3)
t ln 4
Do hm f nghch bin trn [16; +).
Mt khc f (16) > 0 v f (64) < 0, suy ra tn ti duy nht s t1 (16; 64) l nghim ca
phng trnh f (t) = 0.
Do , nghim (3t1 ; log4 t1 ) l nghim th ba ca h.

Vi t > 16, khi f (t) =

1
+ log4 t 2.
(3t + 3)3
1
9
(3t + 3)4 9t ln 4
Ta c f 0 (t) =

=
.
t ln 4 (3t + 3)4
(3t + 3)4 .t. ln 4
t g(t) = (3t + 3)4 9t ln 4 vi t [0; 16).
Ta c: g 0 (t) = 12(3t + 3)3 9 ln 4 v g 00 (t) = 12.32 .(3t + 3)2 > 0.

Vi 0 < t < 16, khi f (t) =

142
Do g 0 (t) ng bin trn [0; 16), hay vi mi t thuc (0; 16), ta c g 0 (t) > g 0 (0). M
g 0 (0) = 12.33 9 ln 4 > 0 suy ra g 0 (t) > 0, t (0; 16) .
T y, suy ra hm g(t) ng bin trn [0; 16), nn g(t) > g(0) = 81 > 0. Suy ra
f 0 (t) > 0, t (0; 16), hay hm f (t) ng bin trn (0; 16).
V f (1) < 0, f (16) > 0 nn phng trnh c nghim duy nht t2 thuc khong (1; 16).
Do nghim (3t2 ; log4 t2 ) l nghim th t ca h.
Vy h phng trnh cho c bn nghim 2

Bi
tp t luyn

x y
+
y
x = 32
4
. (HV CNBCVT 1999)
1/

log (x y) = 1 log3 (x + y)
( 3
x y = (log2 y log2 x)(2 + xy)
2/
. (H Ngoi thng 1999)
x3 + y 3 = 16
( 2
p

2
4x 16 + 3 x + x2 + 1 = 4y 8y + 3 y 4 + y 2 8y + 17
3/
y(x2 1) 4x2 + 3x 8 + ln(x2 3x + 3) = 0

( ngh OLP 30-04 THPT chuyn Nguyn Bnh Khim, Qung Nam)
4/ log2


x2 3x + 2 + 1 +

 x2 +3x1
1
= 2.
3

( ngh OLP 30-04 THPT chuyn L Qu n, Khnh Ha)


(
x2 + y 3 = 29
5/ Xc nh s nghim ca h phng trnh (n x, y) sau:
log3 x. log2 y = 1

Chng V: H PHNG TRNH

CC LOI H C BN
L thuyt
F H phng trnh i xng loi I:
H phng trnh i xng loi I l h c dng

F (x; y) = 0
G(x; y) = 0
Trong F (x; y), G(x; y) l cc a thc i xng vi x, y.
Cch gii chung ca h l t S = x + y; P = xy (iu kin S 2 4P > 0). Dng tnh i xng
ta a h v dng

F (S; P ) = 0
1

G1 (S; P ) = 0
Tm c S v P. T , theo nh l Viete o, x v y l nghim ca phng trnh
t2 St + P = 0
F H phng trnh i xng loi II:
H phng trnh i xng loi II l h c dng

F (x; y) = 0
F (y; x) = 0
Trong F (x; y) l mt a thc khng i xng.
Tr hai phng trnh v theo v c F (x; y) F (y; x) = 0 ()
Coi x l n s, y l tham s v t F (x; y) = f (x) + g(y) (g(y) c lp vi x)
th F (y; x) = f (y) + g(x). Khi
() G(x) = f (x) + g(y) f (y) g(x) = 0
Xt G(x), thay x = y th G(y) = f (y) + g(y) f (y) g(y) = 0
y l nghim ca phng trnh G(x) = 0
G(x) c cha nhn t (x y) theo nh l Bezout.
Nh vy ta c cch gii h i xng loi II l: Tr hai phng trnh v theo v c
G(x; y) = (x y).M (x; y) = 0
143

144
Sau gii h trong tng trng hp x = y v
M (x; y) = 0.
f (x) = g(y)
Lu rng h i xng loi II cn c dng
v nu f (x), g(x) l hai hm s
f (y) = g(x)
n iu cng chiu th ta c th suy ra x = y m khng cn xt M (x; y) = 0 (v trng hp
ny s dn ti h v nghim). Nhng nu f (x), g(x) khng n iu cng chiu th trng hp
M (x; y) = 0 s cho nghim.
F H phng trnh bc hai 2 n:
H phng trnh bc hai 2 n c dng tng qut

a x2 + b xy + c y 2 + d x + e y + f = 0
1
1
1
1
1
1
a2 x2 + b2 xy + c2 y 2 + d2 x + e2 y + f2 = 0
Trong ai , bi , ci , di , ei , fi (i = 1, 2) l cc tham s v x, y l n s.
Ta cng xt mt s trng hp c bit ca h:
a) H cha mt phng trnh bc nht:
Ta tnh x theo y hoc ngc li t mt phng trnh, thay vo phng trnh cn li c mt
phng trnh bc 2 mt n.
b) H cha mt phng trnh thun nht bc 2:
Nu mt trong 2 phng trnh ca h khng cha hng t bc nht v s hng t do, chng
hn nu d1 = e1 = f1 = 0 th h c th a v phng trnh bc hai bng cch t y = tx ri
thay vo phng trnh thun nht tm c t, t thay vo phng trnh th 2 tm
x, y tng ng.
Phng php ny cng c th p dng gii h gm 2 phng trnh bn ng cp bc hai:

a x2 + b xy + c y 2 + f = 0
1
1
1
1
()
a2 x2 + b2 xy + c2 y 2 + f2 = 0
V r rng t h ta c th to mt phng trnh thun nht bc 2 l
f2 (a1 x2 + b1 xy + c1 y 2 ) f1 (a2 x2 + b2 xy + c2 y 2 ) = 0
Trong trng hp tng qut, php gii h bc 2 hai n s dn n gii mt phng trnh
bc cao (> 4). Nhng vi mt s h phng trnh, ta c th a v h (*) bng cch t
x = u + a; y = v + b trong u, v l n mi (phng php tnh tin nghim). Ta cn tm hng
s a, b hng t bc nht hai phng trnh b trit tiu, nh vy h thu c l h ng cp.

x2 + 3y 2 + 4xy 18x 22y + 31 = 0 (1)


V d: Gii h phng trnh (I)
2x2 + 4y 2 + 2xy + 6x 46y + 175 = 0 (2)
Gii
~ tng: t x = u + a; y = v + b ta c h phng trnh

u2 + 4uv + 3v 2 + u(2a + 4b 18) + v(6b + 4a 22) + 4ab + a2 + 3b2 18a 22b + 31 = 0


2u2 + 4v 2 + 2uv + u(4a + 2b + 6) + v(2a + 8b 46) + 2a2 + 4b2 + 2ab + 6a 16b + 175 = 0

145
h s ca u, v l 0 ta gii h phng trnh

2a + 4b 18 = 0

6b + 4a 22 = 0

4a + 2b + 6 = 0

2a + 8b 46 = 0

a = 5
b = 7

~ Li gii:
t x = u 5; y = v + 7 ta c h phng trnh

u2 + 3v 2 + 4uv = 1 (3)
(II)
2u2 + 4v 2 + 2uv = 1 (4)
H ny c th gii theo cch thng thng, nhng lu l tr 2 phng trnh v theo v ta c
ngay
u2 + v 2 2uv = 0 u = v

u
=
v
=
8u2 = 1

2 2
Ta c h (II)

1
u = v
u=v=
2 2
1
1
1
1
Vy (I) c nghim (x; y) = ( 5; + 7), ( 5; + 7) 2
2 2
2 2
2 2
2 2
Vi nhng h c cha tham s, cch gii hon ton tng t:

x2 + 2xy = m (1)
V d: Tm m h c nghim:
x2 + xy + y 2 = 1 (2)
Gii

x 2 = m
Nu y = 0 ta c h
x 2 = 1

ch c nghim khi m = 1.

y 2 (t2 + 2t) = m (1)


Nu m 6= 1 th y =
6 0, t x = ty ta c h (I)
y 2 (t2 + t + 1) = 1 (2)
Chia (1) cho (2) v quy ng ta c t2 + 2t = m(t2 + t + 1) (3).
H cho c nghim (x; y) (I) c nghim (t; y) (3) c nghim t (do t2 + t + 1 > 0 nn t
y lun tm c y tho (2)).
Vit li (3) di dng
(m 1)t2 + (m 2)t + m = 0
2
Ta c = 4 3m2 nn (3) c nghim > 0 |m| 6
3
2
2
Do m = 1 cng tho m 6 nn ta kt lun h c nghim |m| 6 2
3
3

H phng trnh 3 n bnh ng:


H phng trnh 3 n bnh ng l h c cc phng trnh u bnh ng vi 3 n, ngha l

146
khi hon v 2 n tu th mi phng trnh u khng i.
Phng php c bn gii h l a v h phng trnh

x + y + z = a (1)
() xy + yz + zx = b (2)

xyz = c (3)
Bng cch dng php th hoc nh l Viete o, ta a () v phng trnh mt n
x3 ax2 + bx c = 0

y + z = a x 0
Gii phng trnh trn, tm nghim x0 th vo (1) v (3) ta c
c
yz =
x0
c
2
= 0.
Nh vy y, z l nghim ca phung trnh t (a x0 )t +
x0

x + y + z = 1
V d: Gii h phng trnh

x2 + y 2 + z 2 = 9

x 3 + y 3 + z 3 = 1
Gii

T h phng trnh ta c:

2
2
2
2
xy + yz + zx = (x + y + z) (x + y + z )
2

3xyz = (x3 + y 3 + z 3 ) (x + y + z)(x2 + y 2 + z 2 xy yz zx) = 12

xy + yz + zx = 4

xyz = 4

x + y + z = 1 (1)
Ngha l ta a h v dng xy + yz + zx = 4 (2)

xyz = 4 (3)
Theo nh l Viete o, x, y, z l nghim ca phng trnh

t=1

t3 t2 4t + 4 = 0
t = 2
t = 2
T ta tm c nghim ca h l (x; y; z) = (1; 2; 2) v cc hon v.2
~ Nhn xt: nh l Viete o khng nm trong sch gio khoa, nn vi nhng k thi i tr,
ta c th chng minh li bng php th nh sau:
T (3) x, y, z 6= 0
Nhn hai v ca (2) cho x ta c
x2 y + xyz + x2 z = 4x x2 (y + z) = 4x xyz = 4 4x

147
Nhn hai v ca (1) cho x2 ta c
x3 + x2 (y + z) = x2 x3 + 4 4x = x2 x {1; 2; 2}
Sau y ta cng xem qua mt s bi tp tng hp:

Bi tp tng hp
(
Bi 1: Gii h phng trnh sau

x2 y 2 2x + 2y = 3
y 2 2xy + 2x = 4
Gii

t x = u + 1 v y = v + 1.H phng trnh tng ng:


(
(u + 1)2 (v + 1)2 2(u + 1) + 2(v + 1) = 3
(v + 1)2 2(u + 1)(v + 1) + 2(u + 1) = 4

u2 v 2 = 3
v 2 2uv = 5

u2 v 2 = 3

( 2
(

5u 5v 2 = 15
u2 v 2 = 3
4

u= v
2
2
2

5
3v 6uv = 15
5u + 6uv 8v = 0

u = 2v
4
> Vi u = v. Ta c:
5
5
v = ;u =

25
16 2
3
v v 2 = 3 v 2 =

5
25
3
v = ;u =
3

3
4

2
> Vi u = 2v ta c 4v 2 v 2 = 3


 v = 1 (vnghim)
4
5
5
4
Kt lun: H c nghim (x; y) = + 1; + 1 ; + 1; + 1 2
3
3
3
3
~ Nhn xt: Ngoi phng php tnh tin nghim, vi bi ny ta c th bin i ng thc
tm ra cch t n ph trn:
Xt h phng trnh tng ng:
(
(
(x 1)2 (y 1)2 = 3
(x 1)2 (y 1)2 = 3

(y 2 2y + 1) 2(xy y x + 1) = 5
(y 1)2 2(x 1)(y 1) = 5

T ta t x = u + 1 v y = v + 1
(
Bi 2: Gii h phng trnh sau

x2 2xy + 2y + 15 = 0
2x 2xy + y 2 + 5 = 0
Gii

t x = u + 1 v y = v + 1. H phng trnh tng ng:


(
(u + 1)2 2(u + 1)(v + 1) + 2(v + 1) + 15 = 0
2(u + 1) 2(u + 1)(v + 1) + (v + 1)2 + 5 = 0

u2 2uv = 16
v 2 2uv = 6

148

3u2 6uv = 48
8v 2 16uv = 48

v
u
=

3
3u2 + 10uv 8v 2 = 0

u = 4v

v 2 2uv = 6

2
v 2uv = 6

2
> Vi u = v ta c:
3

"

v
=
3
2
4
v 2 v 2 = 6 v 2 = 18

3
v = 3 2

> Vi v = 3 2 u = 2 2

> Vi v = 3 2 u = 2 2
> Vi u = 4v.Ta c: v 2 + 8v 2 = 6 9v 2 = 6 (v nghim)



Kt lun: H c nghim (x; y) l 2 2 + 1; 3 2 + 1 ; 2 2 + 1; 3 2 + 1 2
(
Bi 3: Gii h phng trnh sau:

x2 + y 2 = 8 x y
xy(xy + x + y + 1) = 12
Gii

~ tng: Nhn thy vai tr ca x v y l nh nhau nn ta s c gng phn tch ri t n


ph a h v dng i xng loi 1.
~ Li gii:
H phng trnh tng ng:
( 2
( 2
x + x + y2 + y = 8
(x + x) + (y 2 + y) = 8
(I)
xy(x + 1)(y + 1) = 12
(x2 + x)(y 2 + y) = 12
(
t x2 + x = a v y 2 + y = b. H (I) tr thnh

a+b=8
ab = 12

Nh vy a, b l nghim ca phng trnh bc hai


"
X 2 8X + 12 = 0

X=2
X=6

> Vi a = 2 ta c:

"
x2 + x 2 = 0

> Vi a = 6 ta c:

x = 2
"

x2 + x 6 = 0

x=1

x=2
x = 3

Tng t gii vi b = 2 v b = 6
Kt lun: H c nghim (x; y) l (1; 2); (1; 3); (2; 2); (2; 3) v cc hon v. 2

x3 + 1 = 2(x2 x + y)
Bi 4: Gii h phng trnh ()
y 3 + 1 = 2(y 2 y + x)

149
Gii
Ta c ()

x3 2x2 + 2x + 1 = 2y
y 3 2y 2 + 2y + 1 = 2x

Nhn thy f (t) = t3 2t2 + 2t + 1 c f 0 (t) = 3t2 4t + 2 > 0 t nn f (t) ng bin trn R.
Nu x > y 2x > 2y f (y) > f (x) y > x (mu thun). Tng t vi x < y. Vy x = y.
Ta c h tng ng

x = y
x = y

1 5
x3 2x2 + 1 = 0

x {1;
}
2

1+ 5 1+ 5 1 5 1 5
Vy h c nghim (x; y) = (1; 1), (
;
), (
;
)2
2
2
2
2
~ Nhn xt: Nu tr hai phng trnh v theo v ta s c
"
x=y
(x y)(x2 + xy + y 2 2x 2y + 2) = 2(y x)
x2 + xy + y 2 2(x + y) + 4 = 0
> Xt trng hp x2 + xy + y 2 2(x + y) + 4 = 0:
x3 2x2 + 2x + 1
Thay y =
ta c phng trnh bc 6 theo x:
2
x6 4x5 + 10x4 14x3 + 16x2 10x + 13 = 0
(x6 4x5 + 4x4 ) + (6x4 14x3 + 10x2 ) + (6x2 10x + 13) = 0

Phng trnh trn v nghim do

x6 4x5 + 4x4 = x4 (x 2)2 > 0

6x4 14x3 + 10x2 > 0

6x2 10x + 13 > 0

Nh vy cch dng o hm khin li gin n gin hn nhiu, nht l trong nhng bi h


phng trnh m - logarit.

Bi 5: Gii h phng trnh

2x 2 = 3y 3x
3y 2 = 3x 2y
Gii

Ta vit li h nh sau:

2x + 3x = 3y + 2
2y + 3y = 3x + 2

2
2
. Xt hm s f (t) = 2t +3t c f 0 (t) = 2x . ln 2+3x . ln 3 > 0 t ( ; +)
3
3
2
Vy hm s f (t) ng bin trn ( ; +). Suy ra x = y. Ta c h tng ng
3

x = y
2x + 3x = 3x + 2 ()
T suy ra x, y >

150
00

Xt g(t) = 2t + 3t 3x 2, ta c g (t) = 2t . ln2 2 + 3t . ln2 3 > 0 nn g 0 (t) = 0 c ti a 1 nghim,


suy ra g(t) = 0 c ti a 2 nghim. Nh vy (*) ch c 2 nghim x = 1 v x = 0.
Kt lun: H c nghim (x; y) = (0; 0), (1; 1) 2

Bi tp( t luyn

x2 + 2xy + 3y 2 2x 10y = 0

1) Gii h

2x2 + 2xy + y 2 2y = 0
(

2) Gii h

2x2 + 3xy + y 2 9x 6y = 6

x2 + xy + 2y 2 3x 12y + 10 = 0

3x2 + 2xy + y 2 = 11
3) Tm m h c nghim
x2 + 2xy + 3y 2 = 17 + m

H PHNG TRNH HON V


L thuyt
H phng trnh hon v l h c dng:

f (x1 ) = g(x2 )

f (x2 ) = g(x3 )

...

f (xn1 ) = g(xn )

f (xn ) = g(x1 )
Sau y l mt s nh l tng qut v h hon v:
F nh l 1: Nu hai hm s f (x), g(x) cng tng trn tp A v (x1 , x2 , ..., xn ) l nghim ca
h (trong xi A, i = 1, n) th x1 = x2 = .. = xn
~ Chng minh: Khng gim tng qut gi s x1 = max(x1 , x2 , ..., xn )
f (x1 ) > f (x2 ) g(x2 ) > g(x3 ) x2 > x3 ... xn > x1
x1 > x2 > x3 > ... > xn > x1 x1 = x2 = ... = xn 2
F nh l 2: Nu hm s f (x) gim v g(x) tng trn tp A v (x1 , x2 , ..., xn ) l nghim ca
h (trong xi A, i = 1, n) th vi n l, x1 = x2 = .. = xn .
~ Chng minh: Khng gim tng qut gi s x1 = max(x1 , x2 , ..., xn ).
x1 > x2 f (x1 ) 6 f (x2 ) g(x2 ) 6 g(x3 ) x2 6 x3 ... xn > x1
f (xn ) 6 f (x1 ) x1 6 x2 x1 = x2 x1 = x2 = ... = xn 2

151
F nh l 3: Nu hm s f (x) gim, g(x) tng trn tp A v (x1 , x2 , ..., xn ) l nghim ca h
(trong xi A, i = 1, n) th vi n chn, x1 = x3 = .. = xn1 hoc x2 = x4 = ... = xn
~ Chng minh: Khng gim tng qut gi s x1 = max(x1 ; x2 ; ...; xn )
x1 > x3 f (x1 ) 6 f (x3 ) g(x2 ) 6 g(x4 ) x2 6 x4
f (x2 ) > f (x4 ) g(x3 ) > g(x5 ) x3 > x5 ...
f (xn2 ) > f (xn ) g(xn1 ) > g(x1 ) xn1 > x1
f (xn1 ) 6 f (x1 ) g(xn ) 6 g(x2 ) xn 6 x2

Vy ta c

x > x > ... > x


1
3
n1 > x1
x2 6 x4 6 ... 6 xn 6 x2

x = x = ... = x
1
3
n1

x2 = x4 = ... = xn2

Trong gii ton, ta thng gp h hon v 3 n, trong a phn c th gii bng nguyn l
cc hn. Bi vit ny cng cp n mt s h c bit, c th gii bng cch bin i cc
phng trnh, dng bt ng thc hoc lng gic ho. Sau y l mt s bi tp v d.

Bi tp v d

2x(y 2 + 1) = y(y 2 + 9)

Bi 1: Gii h phng trnh (I) 2y(z 2 + 1) = z(z 2 + 9) .

2z(x2 + 1) = x(x2 + 9)
Gii

x=

H phng trnh tng ng y =

z =

y(y 2 + 9)
2(y 2 + 1)
z(z 2 + 9)
(z 2 + 1)
x(x2 + 9)
(x2 + 1)

t(t2 + 9)
(t R).
2(t2 + 1)
t4 4t2 + 9
Ta c f 0 (t) =
> 0 (t R). Suy ra hm s f (t) ng bin trn R.
2(t2 + 1)2
Khng mt tnh tng qut, gi s x = max{x; y; z}.
x > y f (x) > f (y) z > x z = x f (x) = f (y) x = y.
Thay vo phng trnh th nht ca h ta c:

x= 7

2x3 + 2x = x3 + 9x x3 7x = 0
0
x =
x= 7
Xt hm s f (t) =


Vy h phng trnh c ba nghim (x; y; z) l ( 7; 7; 7); (0;0;0) v ( 7; 7; 7). 2
~ Nhn xt: Bi 1 cho ta ci nhn s lc v nguyn l cc hn trong h hon v. Ta cng c

152
th tng qut bi trn nh sau:
Bi 1*:(Olympic 30-4-2009)

2x(y 2 + a2 ) = y(y 2 + 9a2 )

Gii v bin lun theo tham s a h phng trnh 2y(z 2 + a2 ) = z(z 2 + 9a2 )

2z(x2 + a2 ) = x(x2 + 9a2 )

3
2

y 6x + 12x 8 = 0
Bi 2: Gii h phng trnh: (II) z 3 6y 2 + 12y 8 = 0

x3 6z 2 + 12z 8 = 0
Gii

3
2

y = 6x 12x + 8 (1)
(II) z 3 = 6y 2 12y + 8 (2)

x3 = 6z 2 12z + 8 (3)
Khng gim tng qut gi s x = max{x; y; z}.
Ta c y 3 = 6(x 1)2 + 2 > 2 y > 1. Chng minh tng t c x, y, z > 1.
Xt f (t) = 6t2 12t + 8; f 0 (t) = 12t 12 > 0 t > 1 f(t) ng bin trn (1; +)
Ta c x > y f (x) > f (y) y 3 > z 3 y > z f (y) > f (z) z 3 > x3
x > y > z > x x = y = z. Khi :

x = y = z
(II)
x=y=z=2
x3 = 6x2 12x + 8
Vy (II) c nghim (x; y; z) = (2; 2; 2) 2
~ Nhn xt: bi ny ta thy f (t) khng n iu, khc vi bi 1. Do phi c thm nhn
xt x, y, z > 1. y chnh l mu cht ca bi ton.
Sau y ta s n vi mt cch nhn mi v nguyn l cc hn. Thay v nh gi x, y, z ring
l, ta s nh gi nhng biu thc hon v ca x, y, z l x + y, y + z, z + x:

3
2

x + 3x + 2x 5 = y
Bi 3: (VMO 2006) Gii h phng trnh (III) y 3 + 3y 2 + 2y 5 = z

z 3 + 3z 2 + 2z 5 = x
Gii
Ta cng xem qua 2 cch gii ca bi ton.
> Cch 1:
Cng 3 phng trnh ca h ta c:
x3 + 3x2 + x 5 + y 3 + 3y 2 + y 5 + z 3 + 3z 2 + z 5 = 0
(x 1)(x2 + 4x + 5) + (y 1)(y 2 + 4y + 5) + (z 1)(z 2 + 4z + 5) = 0

153
Nu x > 1 z 3 + 3z 2 + 2z 5 > 1 (z 1)(z 2 + 4z + 6) > 0 z > 1.
Li t z > 1, lp lun tng t suy ra y > 1. Khi :
(x 1)(x2 + 4x + 5) + (y 1)(y 2 + 4y + 5) + (z 1)(z 2 + 4z + 5) > 0 (v l)

Tng t, nu x < 1 ta suy ra y < 1, z < 1 cng suy ra iu v l.


Vy x = y = z = 1 chnh l nghim ca h phng trnh. 2
> Cch 2:
Ta c:

(x + 1) = y + x + 6 (1)
(III) (y + 1)3 = z + y + 6 (2)

(z + 1)3 = x + z + 6 (3)
Khng gim tng qut gi s x = max{x; y; z}.
x + z > y + z (z + 1)3 > (y + 1)3 z > y x + z > x + y (z + 1)3 > (x + 1)3
z > x z = x x + y = y + z (x + 1)3 = (y + 1)3 x = y
Khi :

x = y = z
(III)
x=y=z=1
x3 + 3x2 + x 5 = 0
Vy (III) c nghim (x; y; z) = (1; 1; 1) 2
~ Nhn xt: Cch 1 l phng php dng bt ng thc, s c cp sau. Xt cch 2,
ta thy vic nh gi x + y, y + z, z + x r rng c li th ca n. Ta vn c th so snh
x + y, y + z, z + x da vo mi quan h hon v gia x, y, z nhng v tri ca (1), (2), (3)
tr nn rt p. Ta cng xt mt bi tng t:

3
2

x = 3y + 2y (1)
Bi 4: Gii h phng trnh (IV ) y = 3z 3 + 2z 2 (2)

z = 3x3 + 2x2 (3)


Gii
Ta c

3
2

x + y = 3y + 2y + y
(IV ) y + z = 3z 3 + 2z 2 + z

z + x = 3x3 + 2x2 + x
Khng gim tng qut gi s x = max{x; y; z}.
Xt f (t) = 3t3 + 2t2 + t ta c f 0 (t) = 9t2 + 4t + 1 > 0 f (t) ng bin trn R
Li c x + y > y + z f (y) > f (z) y > z x + y > x + z f (y) > f (x) y > x
y = x f (y) = f (x) x + y = x + z y = z

154
Vy

x = y = z
(IV )
3x3 + 2x2 x = 0

x = y = z

x {0; 1; 1 }
3
1 1 1
Vy (IV ) c nghim (x; y; z) = (0; 0; 0), (1; 1; 1), ( ; ; ) 2
3 3 3
~ Nhn xt: Cu hi t ra l: khi no ta phi xt (x + y; y + z; z + x)? Mu cht y
chnh l ta cn xy dng mt hm n iu trn min xc nh. Nu nguyn (IV ), ta xt
f (t) = 3t3 + 2t2 th f 0 (t) = 9t2 + 4t c nghim t = 0 nn f (t) c th i chiu n iu. T
ta ngh ti vic bin i cc phng trnh.
c mt hm n iu, ta s cng hai v ca (1) cho ky, tng t vi (2) v (3):

3
2

x + ky = 3y + 2y + ky
(IV ) y + kz = 3z 3 + 2z 2 + kz

z + kx = 3x3 + 2x2 + kx
4
9
Nhng s k cn phi m bo ta
vn
c
th
so
snh
VT
ca
(1),
(2)
v
(3).
D
thy
hng
s

9t2 + 4t + 1 > 0 t
tt nht l k = 1. Khi ta c
x + y > x + z > y + z
Khi ta xt f (t) = 3t3 + 2t2 + kt c f 0 (t) = 9t2 + 4t + k. Ta cn f 0 (t) > 0 t, ngha l k >

T ta i n li gii nh trn.

x = 2y 1 (1)
Bi 5: Gii h phng trnh (V ) y = 2z 2 1 (2)

z = 2x2 1 (3)
Gii
Khng gim tng qut gi s x = max{x; y; z}.
1
(3) 2x2 1 = z 6 x
6x61
2
2
t x = cos t (t [0; ])
3
(3) z = 2 cos2 t 1 = cos 2t
(2) y = 2 cos2 2t 1 = cos 4t
(1) x = 2 cos2 4t 1 = cos 8t
k2
k2
cos t = cos 8t t =
t=
(k Z)
7
9
2 4 2 2 4
2
t {0; ; ; ; ;
} do t [0; ]
9 9 3 7 7
3
2
4
8 1 1 1
Vy (V I) c nghim (x; y; z) = (1; 1; 1); (cos ; cos ; cos ); ( ;
;
);
9
9
9
2 2 2
2
4
8
(cos ; cos ; cos ) v cc hon v. 2
7
7
7
~ Nhn xt: Phng php lng gic ho cn s kho lo v hiu bit r v cng thc lng
gic. Nh bi 5, t x = 2y 2 1 lm ta nh n cng thc cos2t = 2 cos2 t 1, do ta mi

155
t x = cos t. Ta xt tip Bi 6:

x = y + 2 (1)
Bi 6: Gii h phng trnh (V I) y 2 = z + 2 (2)

z 2 = x + 2 (3)
Gii
Khng gim tng qut gi s x = max{x; y; z}.
x2 = y + 2 6 x + 2 1 6 x 6 2.
2
t x = 2 cos t (t [0; ]).
3
(1) y = x2 2 = 2(2 cos2 t 1) = 2 cos 2t
(2) z = y 2 2 = 2(2 cos2 2t 1) = 2 cos 4t
(3) x = z 2 2 = 2(2 cos2 4t 1) = 2 cos 8t
k2
k2
t=
(k Z)
cos t = cos 8t t =
7
9
2 4 2 2 4
2
t {0; ; ; ; ;
} do t [0; ]
9 9 3 7 7
3
2
4
8
Vy (V I) c nghim (x; y; z) = (2; 2; 2); (2 cos ; 2 cos ; 2 cos ); (1; 1; 1);
9
9
9
2
4
8
(2 cos ; 2 cos ; 2 cos ) v cc hon v. 2
7
7
7
~ Nhn xt: Bi 6 r rng cn s kho lo. Nu t x = sin t; cos t hay tan t th biu thc x2 2
khng lin quan trc tip n cng thc lng gic no, nhng rng
(2 cos t)2 2 = 2(2 cos t 1) = 2 cos 2t. V vy ta phi t x = 2 cos t.

Bi 7: Gii h phng trnh (V II)

4
2
3

x y 6x y + 4x 4x + y = 0
y 4 z 6y 2 z + 4y 3 4y + z = 0

z 4 x 6z 2 x + 4z 3 4z + x = 0
Gii

~ tng: u tin ta bin i

4x 4x3

y
=
(1)

x4 6x2 + 1

4y 4y 3
(V II) z = 4
(2)

y 6y 2 + 1

x = 4z 4z
(3)
z 4 6z 2 + 1
Ta th xt o hm ca f (t) =

4t 4t3
. D thy
t4 6t2 + 1

4(t2 + 1)3
f (t) = 4
> 0 t 6=
(t 6t2 + 1)2
0

32 2

n y, nu cha nhn ra vn th ta s c nhn xt x = y = z


(V II) c nghim (x; y; z) = (0; 0; 0); ( 3; 3; 3); ( 3; 3; 3).

156
Thc ra y ta mc mt sai lm rt tinh vi l hm f (t) ch ng bin trn tng khong
24
24
xc nh ch khng ng bin trn min xc nh (v d f (2) =
> f (3) =
). V vy
7
7
bi 7 khng th dng hm n iu. Theo tng trn, ta ngh ti phng n 2: lng
gic ho.
Nhn vo phn thc, ta thng ngh ti cng thc v tan. rng f (tan a) = tan 4a do
ta t x = tan a.
~ Li gii:
p

KX: x, y, z 6= 3 2 2

t x = tan a (a [0; ) a 6= )
2
T (1), (2), (3) ta tnh c: y = f (x) = tan 4a; z = f (y) = tan 16a; x = f (z) = tan 64a
k
tan a = tan 64a a =
(k = 0, 62)
63
k
Vy (V II) c nghim (x; y; z) = (tan a; tan 4a; tan 16a) vi a =
, k = 0; 62
63
~ Nhn xt: Nguyn l cc hn thng c dng trong cc bi h hon v nhng khng cn
tc dng vi nhng bi t lng gic. Nu xt hm n iu theo cch u tin th ta lm
mt 60 nghim ca h!

x3 3x = y (1)

Bi 8: Gii h phng trnh: (V III) y 3 3y = z (2) (HSG Thi Bnh 2009)

z 3 3z = x (3)

Gii
h
i3
h
i
3
3
3
3
3
3
T h phng trnh suy ra: (x 3x) 3(x 3x) 3 (x 3x) 3(x 3x) = x.
y l phng trnh a thc bc 27 nn c nhiu nht 27 nghim. Do h cho c nhiu
nht 27 nghim.
> Xt x [2; 2]: Khi ta t x = 2 cos t (t [0; ]). Khi ta c:

(1) y = 2 cos 3t
(2) z = 2 cos 9t

(3) x = 2 cos 27t

13 (k Z)
cos 27t = cos t

t=k
14

t=k

(k Z), do t [0; ] nn k nhn cc gi tr 0;1;2;. . . ;12;13.


13

Vi t = k (k Z), do t [0; ] nn k nhn cc gi tr 0;1;2;. . . ;12;13;14.


14
Nghim ca h ng vi t = 0 v t = c hai trng hp trng nhau nn h c 27 nghim
k
k3
k9
phn bit l (2 cos
; 2 cos
; 2 cos
) vi k = 0, 13 v
13
13
13
k3
k9
k
(2 cos
; 2 cos
; 2 cos
)vi k = 1, 13 2
14
14
14
Vi t = k

Tip theo ta cng xem qua mt s bi h hon v gii bng Bt ng thc:

157

x2 + x 1 = y

Bi 9: Gii h phng trnh (IX) y 2 + y 1 = z

z2 + z 1 = x
Gii
Ta cng xem qua 2 cch gii cho bi ton:
> Cch 1:
Khng mt tnh tng qut gi s: x = min {x; y; z}
x2 1 = y x > 0 |x| > 1
z 2 1 = x z 6 0 |z| 6 1 1 6 y 2 + y 1 6 1 0 6 y 2 + y 6 2 y [2; 1] [0; 1].
* Nu x > 0 z(> x > 1 z = x = 1 y = 1
x 6 1 z 2 + z 1 6 1 z [0; 1]
* Nu x 6 0
x 6 1 x(x + 1) > 0 y > 1
* Nu y = 1 x = y = z = 1
* Nu y > 1 : y [0; 1]

0 6 y 6 5 1
1 > z 2 + z 1 = x > 5
4
Ta c:
|xyz| < 1 ().
2

y 2 + y 1 = z 6 0
1 6 z 6 0
M xyz(x + 1)(y + 1)(z + 1) = (x2 + x)(y 2 + y)(z 2 + z) = (x + 1)(y + 1)(z + 1)
> Nu (x + 1)(y + 1)(z + 1) = 0 th x = y = z = 1
> Nu (x + 1)(y + 1)(z + 1) 6= 0 th xyz = 1 (sai do (*))
Vy h c nghim (x; y; z) = (1; 1; 1), (1; 1; 1) 2
> Cch 2:
2
2
2
Cng v
theo v ta c: x + y + z = 3
x(x + 1) = y + 1

H y(y + 1) = z + 1

z(z + 1) = x + 1
Nu x = 1 y = z = 1
Nu x, y, z 6= 1 th nhn v theo v, ta c: xyz = 1
p
Li c: 3 = x2 + y 2 + z 2 > 3 3 x2 y 2 z 2 = 3 x2 = y 2 = z 2 = 1 x = y = z = 1 Vy h c
nghim (x; y; z) = (1; 1; 1), (1; 1; 1) 2

x 2x 2 = y
Bi 10: Gii h phng trnh (X) y 3 2y 2 = z

z 3 2z 2 = x
Gii
H (X) tng ng vi

(x 2)[(x + 1) + 1] = y 2 (1)
(y 2)[(y + 1)2 + 1] = z 2 (2)

(x 2)[(z + 1)2 + 1] = x 2 (3)

158

(1) |x 2| > |y 2|
rng (2) |y 2| > |z 2|

(3) |z 2| > |x 2|

|x 2| = |y 2| = |z 2|

Ta cng c x 2, y
2, z 2 cng du do x 2 = y 2 = z 2 x = y = z
"
x = y = z
x = y = z = 1
Vy (X) tr thnh

x3 3x 2 = 0
x=y=z=2
Kt lun: H phng trnh c nghim (x; y; z) = (1; 1; 1), (2; 2; 2) 2

4x2 3x + 1 = y

Bi 11: Gii h phng trnh (XI) 4y 2 + y + 1 = 5z

4z 2 z + 1 = 3x
Gii
Vit li h phng trnh di dng

(2x 1) = y x

x y > 0
(XI) (2y 1)2 = 5(z y) y z > 0

(2z 1)2 = 3(x z)

z x > 0

x = y = z
Vy (X)
4x2 4x + 1 = 0

x=y=z=

Kt lun: H c nghim duy nht x = y = z =

x=y=z

1
2

1
2
2

[f (x)]2 = (y x)p .

~ Nhn xt: Tng qut hn, ta c th gii h phng trnh


[f (y)]2 = (z y)m .

[f (z)]2 = (x z)n .
vi m, n, p l cc s l v , , > 0

x
=
cos(x2 )
1

x2 =
cos(x3 )
9
Bi 12: Gii h phng trnh (XII)

x3 =
cos(x4 )

x4 =
cos(x1 )
9

( Olympic 30-4-2007)

Gii

3
1
T h phng trnh ta c |xi | 6
< (i = 1, 4)
9
2

< xi < 0 < cos(xi ) 6 1 0 < xi < 0 < xi <


2
2
2
2
Khng gim tng
qut,
gi
s
x
=
max{x
;
x
;
x
;
x
}
1
1
2
3
4

3
3
x1 > x3
cos(x2 ) >
cos(x4 ) x2 6 x4 x2 6 x4
9
9

159

3
3

cos(x3 ) 6
cos(x1 ) x3 > x1 x3 > x1 x3 = x1
9
9
Khi suy ra x2 = x4 .


3
1
Xt hm s f (x) = x +
cos(x) vi x 0;
9
2



1
3
3
0
Ta c: f (x) = 1
sin(x) > 1
> 0. Suy ra hm s f (x) ng bin trn 0;
.
9
9
2

3
3
Li c: x1 + x4 = x1 + x2 x1 +
cos(x1 ) = x2 +
cos(x2 )
9
9
x1 = x2 (do hm s f (x) ng bin).

3
Do : x1 = x2 = x3 = x4 x1 =
cos(x1 ) cos(x1 ) 3 3x1 = 0
9



1
Xt hm s g(x) = cos(x) 3 3x vi x 0;
 2

1
. Vy hm s g(x) nghch bin trn khong
Ta c: g 0 (x) = sin(x) 3 3 < 0 x 0;
2


1
0;
.
2
Do phng trnh g(x) = 0 c nhiu nht 1 nghim trn on ny.
1
1
Mt khc, g( ) = 0 nn phng trnh g(x) = 0 c nghim duy nht x = .
6
6


1 1 1
Vy h phng trnh c nghim duy nht l
; ;
2.
6 6 6

x = (y 1)2

y = (z 1)2
Bi 13: Gii h phng trnh (XIII)

z = (t 1)2

t = (x 1)2
Gii
Gi s x = min {x; y; z; t}. T h phng trnh ta suy ra x, y, z, t > 0.
> Trng hp 1: 0 < x 6 1
1 < x 1 6 0 0 6 (x 1)2 < 1 0 < t 6 1 0 6 (t 1)2 < 1
0 < z 6 1 0 6 (z 1)2 < 1 0 < y 6 1.
Vy ta c: 0 < x, y, z, t 6 1.
Khi , t z > x suy ra (t 1)2 > (y 1)2 y > t (z 1)2 > (x 1)2
x > z (y 1)2 > (t 1)2 t > y.
T suy ra: x = z, y = t.

3+ 5
>1
x = y =
2

3 5

x = y =
(

2
(
x = (y 1)2

T h phng trnh ta c:
x=0
.
y = (x 1)2

y=1
(

x=1

y=0

> Trng hp 2: x > 1

160
x, y, z, t > 1. Khi ,t y > x suy ra (z 1)2 > (y 1)2 z > y
(t 1)2 > (z 1)2 t > z (x 1)2 > (t 1)2 x > t. Suy ra: x = y = z = t.
T h phng trnh suy ra:

3+ 5
x =
2
x = (x 1)2

3 5
<1
x=
2

3+ 5 3+ 5 3+ 5 3+ 5
;
;
;
);
Vy h phng trnh c cc nghim l: (
2
2
2
2

3 5 3 5 3 5 3 5
(
;
;
;
);(0; 1; 0; 1); (1; 0; 1; 0) 2.
2
2
2
2

x21 + x1 x2 1 = 0

x2 + x2 x3 1 = 0
2
. (Olympic 30-4-2011)
Bi 14: Gii h phng trnh (XIV )

x23 + x3 x4 1 = 0

x2 + x x 1 = 0
4
1
4
Gii
Khng gim tng qut gi s x1 = min{x1 ; x2 ; x3 ; x4 }.
Xt f (t) = t2 + t 1 (t R). H phng trnh tng tng

f (x1 ) = x2

f (x2 ) = x3

f (x3 ) = x4

f (x ) = x
4
1
1
1
Hm s f (t) tng trn khong ( ; +), gim trn khong (; ) v
2
2
1
5
5
f (t) > f ( ) =
t R nn xi >
i = 1, 4.
2
4
4
1
> Nu x1 > :
2

x4 >
1
4
Ta c:
x4 > .

2
f (x4 ) > 1 > 11 = f ( 5 )
2
6
4
1
Lp lun tng t ta cng c: x2 , x3 > .
2
Do x1 6 x2 f (x1 ) 6 f (x2 ) x2 6 x3 f (x2 ) 6 f (x3 ) x3 6 x4
f (x3 ) 6 f (x4 ) x4 6 x1 x1 6 x2 6 x3 6 x4 6 x1 x1 = x2 = x3 = x4 = 1.
1
> Nu x1 < :
2
1
1
Nu c k {1; 2; 3; 4} xk > th theo trng hp trn ta c: xi >
i = 1, 4
2
2
1
1
x1 > ( mu thun). Vy xi <
i = 1, 4.
2
2
Do x1 6 x3 f (x1 ) > f (x3 ) x2 > x4 f (x2 ) 6 f (x4 ) x3 6 x1 x3 = x1 .
Lp lun tng t ta c x2 = x4 .

161

x1 = x3

x2 = x4
H phng trnh tr thnh:

f (x1 ) = x2

f (x ) = x
2
3
"
x1 + f (x1 ) = x2 + f (x2 ) x21 + 2x1 1 = x22 + 2x2 1

x1 = x2
x1 = x2 2

x1 = x2 = 1

x1 = x2 = x3 = x4 = 1
Vy h phng trnh ban u c nghim l x1 = x2 = x3 = x4 = 1 2.

1
1

x1 = (x2 + ) (1)

2
x2

x2 = (x3 + 1 ) (2)
2
x3
Bi 15: Gii h phng trnh (XV )

...

x2012 = 1 (x1 + 1 ) (2012)


2
x1
Gii
Kx: xi 6= 0 (i = 1; 2012)
1
1
1
1
(1) |x1 | = |x2 + | = (|x2 | + | |) > 1
2
x2
2
x2
Chng minh tng t c |xi | > 1 (i = 1; 2012)
1
Li c: x1 .x2 = (x22 + 1) > 0 x1 , x2 cng du. Chng minh tng t c x1 , x2 , ..., x2012 cng
2
du.
2012
2012
2012
2012
X
X
X
X
1
1
|
xi | =
|xi | |
|=
| |()
x
xi
i=1
i=1
i=1 i
i=1
Cng (1), (2), ..., (2012) v theo v ta c:
2012
X
i=1

2012

2012

2012

2012

2012

2012

2012

X
X 1
X
X 1
X
X 1
1
1X
xi =
|
xi | = |
|
|xi | =
| |() (do())
xi =
(xi + )
2 i=1
xi
x
x
xi
i=1
i=1 i
i=1
i=1 i
i=1
i=1

Li c: |xi | > 1

2012
X
i=1

2012
X
1
|xi | > 2012 >
| |
xi
i=1

Vy () |x1 | = |x2 | = ...|x2012 | = 1 x1 = x2 = ... = x2012 = 1 x1 = x2 = ... = x2012 = 1


Th li ta thy (V III) c nghim (x1 ; x2 ; ...x2012 ) = (1; 1; ...; 1), (1; 1, ...; 1) 2
n
n
X
X
Nh vy, bng bin i kho lo, kt hp ng thc "|
xi | =
|xi | x1 , x2 , ...xn cng
i=1

i=1

du", ta gii quyt bi ton m khng cn ti nguyn l cc hn.

x21 = x2 + 1

x2 = x 3 + 1

2
Bi 16: Gii h phng trnh: (XV I) .................

x2n1 = xn + 1

x2 = x + 1
1
n
2010)

(n N , n > 2).(Olympic 30-4-

162
Gii
Khng mt tnh tng qut, gi s x1 = max{x1 , x2 ..., xn }(*).
Th th x2n = x1 + 1 = max {x1 + 1, x2 + 1, ..., xn + 1} = max {x21 , x22 , ..., x2n }.
|xn | = max{|x1 | , |x2 | ..., |xn | }(**)
T (*) v (**) suy ra xn 6 0.
> Nu x1 > 0 th x2n = x1 + 1 > 1. V xn < 0 nn xn < 1( v l v x2n1 = xn + 1 < 0).
Vy x1 6 0 xi 6 0 i = 1, n.
T (**) suy ra xn = min{x1 , x2 ..., xn }.
V xi 6 x1 6 0 i nn |x1 | = min {|xi |} hay x1 2 = min {xi 2 } .
2
2
Vy x2 = x21 1 = min{x2i
( 1} = min{xi }= xn . T x2 = x3 + 1 v xn = x1 + 1 suy ra x1 = x3 .
x1 = x3 = ... = max {xi }
C tip tc nh vy ta c
.
x2 = x4 = ... = min {xi }=xn
> Vi n l:

5
Ta c x1 = x2 = ... = xn . T h ta c: x21 x1 1 = 0 x1 =
.
2
1 5
Do h phng trnh c nghim l x1 = x2 = ... = xn =
.
2
> Vi n chn:

x1 = 0

2
T x22 = x3 + 1, x21 = x2 + 1 v x1 = x3 ta c (x21 1) = x1 + 1 x1 = 1

1 5
x1 =
2
Vy h phng trnh
c cc nghim

1 5 1 5
1 5
(x1 , x2 , ..., xn )= (
;
;. . . ;
); (1; 0; 1; 0. . . ; 1; 0); (0; 1; 0; 1; . . . ; 0; 1) 2
2
2
2
q

2009

x
+
4
x
=
x2008
30
1
2
3

30x + 4x = 2009 x2008


2
3
4
Bi 17: Tm nghim dng ca h phng trnh (XV II)

......................................

2009
30x
x2008
2008 + 4 x1 =
2
Gii
Gi s (x1 , x2 , x3 ....., x2008 ) l mt nghim ca h phng trnh.
t a = max {x1 , x2 , x3 ....., x2008 }, b = min {x1 , x2 , x3 ....., x2008 } a > b > 0.
Ta c:
q

2009

34 a > 30 x1 + 4 x2 =
x2008
3

34a > 30x + 4x = 2009 x2008


2
3
4

......................................

2009
34a > 30x
x2008
2008 + 4 x1 =
2
n p
o
p
p

2009
2009
2008
2008
34 a > max 2009 x2008
,
x
,
...,
x
1
2
2008

2009
4018 2009
4016
2007
2008
34 a >
a
34 a
>a
a
6 344018 a 6

344018

2007

163
Tng t ta c:

2009

34 b 6 30 x1 + 4 x2 =
x2008
3

34 b 6 30 x2 + 4 x3 = 2009 x2008
4

......................................

2009
34b 6 30x
+
4
x
=
x2008
2008
1
2
o
n p
p
p

, 2009 x2008
, ..., 2009 x2008
34 b 6 min 2009 x2008
1
2
2008

2007
2009 2008
4018 2009
4016
2007
34 b 6
b
34 b
6b
b
> 344018 b >
344018

2007
2007
Vy a = b =
344018 x1 = x2 = ...x2008 =
344018 .

2007
344018 2.
Th li ta thy nghim ca h phng trnh l x1 = x2 = ...x2008 =

Bi 18: Gii h phng trnh: (XV III)

3
2

x 12y + 48y 64 = 0 (1)

y 3 12z 2 + 48z 64 = 0 (2)

z 3 12x2 + 48x 64 = 0 (3)


Gii

Cng (1), (2), (3) v theo v ta c:


(x 4)3 + (y 4)3 + (z 4)3 = 0()
Suy ra trong 3 s hng trn c t nht 1 s khng m, gi s l (z 4)3 > 0 z > 4
(2) y 3 16 = 12(z 2)2 > 12.22 y > 4
(1) x3 16 = 12(y 2)2 > 12.22 x > 4
Vy x, y, z > 4 V T () > 0. Khi () x 4 = y 4 = z 4 = 0 x = y = z = 4
Th li ta thy (IX) c nghim (x; y; z) = (4; 4; 4) 2
Ta cng xem qua mt s bi h hon v lng gic:

cos 2x + cos x = 2 cos y

Bi 19: Gii h phng trnh (XIX) cos 2y + cos y = 2 cos z

cos 2z + cos z = 2 cos x


Gii
Ta c:

(2 cos x 1)(cos x + 2) = 2 cos y 1


|2 cos x 1| (cos x + 2) = |2 cos y 1|

(XIX)
(2 cos y 1)(cos y + 2) = 2 cos z 1
|2 cos y 1| (cos y + 2) = |2 cos z 1|

|2 cos z 1| (cos z + 2) = |2 cos x 1|


(2 cos z 1)(cos z + 2) = 2 cos x 1
|2 cos x 1| > |2 cos y 1| > |2 cos z 1| > |2 cos x 1| (do cos x + 2; cos y + 2; cos z + 2 > 1)
|2 cos x 1| = |2 cos y
1| = |2 cos z 1|

|2 cos x 1| (cos x + 2) = |2 cos x 1|

Khi (XIX) tr thnh:


|2 cos y 1| (cos y + 2) = |2 cos y 1|

|2 cos z 1| (cos z + 2) = |2 cos z 1|

164
"

2 cos x 1 = 2 cos y 1 = 2 cos z 1 = 0


cos x + 2 = cos y + 2 = cos z + 2 = 1

+ k|k Z}
3

x, y, z { 2k|k Z}

x, y, z {

tan2 x + tan2 y

2
cos
2x
+
=2

1 + tan2 x

tan2 y + tan2 z
Bi 20: Gii h phng trnh (XX) 2 cos 2y +
=2
2

1
+
tan

tan2 z + tan2 x

=2
2 cos 2z +
1 + tan2 z
Gii
1 tan2
T ng thc cos 2 =
, ta c
1 + tan2

tan2 y tan2 x

= 1 cos 2x

12+ tan x2
tan z tan y
(XX)
= 1 cos 2y

1 + tan2 y

2
2

tan x tan z = 1 cos 2z


1 + tan2 z
tan2 x > tan2 y > tan2 z > tan2 x tan2 x = tan2 y = tan2 z

1 cos 2x = 1 cos 2y = 1 cos 2z = 0

x = k

y = l

z = m

(k, l, m Z)

Th li ta thy (XX) c nghim (x; y; z) = (k; l; n) (k, l, m Z) 2

ex exy = y

Bi 21: Gii h phng trnh (XXI) ey eyz = z

ez ezx = x
Gii
Vi x = 0, ta c h

1 ey = y

ey eyz = z

ez ez = 0
Xt hm f (y) = 1 ey y (y R).
Ta c: f 0 (y) = ey 1 = 0 y = 0
Do f (y) ng bin trn (; 0] v nghch bin trn [0; +) nn Maxf (y) = 0 v do phng
trnh f (y) = 0 c nghim duy nht y = 0 z = 0.
Vy (0;0;0) l mt nghim ca phng trnh.

165
Vi x 6= 0, h phng trnh tng ng vi h

yey
x

e
=

ey 1

zez
ey = z

e 1

z
e = xe
ex 1
tet
(t 6= 0).
et 1
t t
e (e t 1)
Ta c: g 0 (t) =
> 0 t 6= 0( v et > t + 1 t 6= 0) lim g(t) = 1; lim+ g(t) = 1
2
t
t0
t0
(e 1)
D thy g(t) ng bin trn (; 0) v trn (0; +)
Suy ra: nu x > y g(x) > g(y) ez > ex z > x g(z) > g(x) ey > ez y > z
y > x(mu thun).
Tng t, nu x < y cng suy ra mu thun.
(
x = y = z 6= 0
Vy x = y y = z v h phng trnh tr thnh
x, y, z .
ex 1 = x
Vy (0;0;0) l nghim duy nht ca phng trnh.
Sau y ta cng xem qua mt s bi h phng trnh cha tham s:
Xt hm g(t) =

x2 yz = a

Bi 22: Cho a, b, c > 0. Gii h phng trnh: (XXII) y 2 xz = b

z 2 yx = c
Gii
2

Ta c: a2 bc = (x2 yz) (y 2 xz)(z 2 yx) = x(x3 + y 3 + z 3 3xyz)


b2 ac = y(x3 + y 3 + z 3 3xyz), c2 ab = z(x3 + y 3 + z 3 3xyz)
2
t k = x3 + y 3 + z 3 3xyz, ta c: (a2 bc) (b2 ac)(c2 ab) = a(a3 + b3 + c3 3abc).

Suy ra k 2 (x2 yz) = a(a3 + b3 + c3 3abc) k = a3 + b3 + c3 3abc.

a2 bc
b2 ac
c2 ab
Vy h phng trnh c cc nghim l: x =
,y =
,z =
vi k = a3 + b3 + c3 3a
k
k
k

mx = z +

1
Bi 23: Gii v bin lun h phng trnh theo m: (XXIII) my = x +

mz = y +
y
Gii
H phng trnh tng ng

mxz = z 2 + 1

myx = x2 + 1

mzy = y 2 + 1

x.y.z 6= 0

166
mxy > 0; myz > 0; mzx > 0 m3 x2 y 2 z 2 > 0 m > 0.
Ta thy x,y,z cng du v nu (x; y; z) l nghim ca h phng trnh th (x; y; z) cng
l nghim. Do ta ch cn xt x, y, z > 0. Gi s x = max {x; y; z}.
> Nu x > y > z > 0 :
y 2 + 1 > z 2 + 1 myz > mxz y > x y = x
y 2 + 1 = x2 + 1 myx = myz x = z x = y = z.
> Nu x > z > y > 0 :
x2 + 1 > z 2 + 1 myx > mxz y > z y = z
y 2 + 1 = z 2 + 1 myz = mxz x = y x = y = z.
C hai trng hp u suy ra x = y = z.
Do h phng trnh tr thnh
(
(
mx2 = x2 + 1
(m 1)x2 = 1

x=y=z>0
x=y=z>0
* Vi 0 < m 6 1, h phng trnh v nghim.
* Vi m > 1, h phng trnh tng ng x = y = z =
Tm li:
Vi m 6 1: h phng trnh v nghim.
Vi m > 1, h phng trnh c hai nghim l (
(

1
.
m1

1
1
1
;
;
) v
m1 m1 m1

1
1
1
;
;
) 2.
m1
m1
m1

x2 = y + a

Bi 24: Gii h phng trnh (XXIV ) y 2 = z + a

z2 = x + a

(0 < a < 1)

Gii
Gi s x = max {x; y; z}.
T h phng trnh suy ra z 2 = max {x2 ; y 2 ; z 2 }.
> Nu z > 0:
Ta c z 2 = max {x2 ; y 2 ; z 2 } z = max {x; y; z} x = z x2 = z 2
y + a = x + a x = y x = y = z.
r
1
1
T h phng trnh ta c: x2 = x + a x2 x a = 0 x = +
+ a. Suy ra:
2
4
r
1
1
x=y=z= +
+a
2
4
> Nu z < 0:

Gi s x > 0, khi z 2 = x + a > a z 6 a < a y 2 = z + a < 0 (v l).Do x < 0.


V x = max {x; y; z}, x < 0 nn y < 0. Khi ta c: z 6 y 6 x < 0 y 2 = z + a 6 x2 =
y + a x 6 y.
r
1
1
Suy ra x = y = z = x =
+ a.
2
4
r
r
r
1
1
1
1
1
1
Vy h phng trnh c cc nghim l ( +
+ a; +
+ a; +
+ a) v
2
4
2
4
2
4

167
r
r
r
1
1
1
1
1
1
(
+ a;
+ a;
+ a) 2.
2
4
2
4
2
4
Sau y l mt s h phng trnh hon v khng mu mc. Ta khng th dng nguyn l cc
hn, nhng vn c th dng lng gic ho, bt ng thc. Nhng cch gii ch yu vn l
bin i ng thc, da vo quan h gia cc phng trnh.






1
1
1

3 x +
=4 y+
=5 z+
(1)
x
y
z
Bi 25: Gii h phng trnh (XXV )

xy + yz + zx = 1 (2)
Gii
K: xyz 6= 0
Nu (x, y, z) l mt nghim ca h th (x, y, z) cng l mt nghim ca h v t (1)suy
ra x, y,
zcng du nn ta ch cn xt x, y, z dng l .

x = tan

 
ta c:
t: y = tan , ; ; 0;

z = tan

(I)

3 tan +

1
tan


= 4 tan +

1
tan


= 5 tan +

1
tan


(3)

tan tan + tan tan + tan tan = 1 (4)


Li c:
(3) 3

tan2 + 1
tan2 + 1
tan2 + 1
3
4
5
=4
=5

=
=
(5)
tan
tan
tan
sin 2
sin
sin

T (4) ta c:
(4) tan (tan + tan ) = 1tan tan tan =

1 tan tan
= cot ( + ) ++ = (6
tan + tan
2

T (5) v(6), suy ra 2, 2, 2l cc gc trong mt tam gic vung, c cc cnh l 3, 4, 5 Do


:

2 = = tan = 1 = z
2
4
T ta c:

tan = y =
2

tan = x = 1
3

 

1 1
1 1
Vy h phng trnh cho c nghim l: (x; y; z) =
; ; 1 , ; ; 1 2
3 2
3 2

ab(a + b) = 6
Bi 26: Gii h phng trnh (XXV I) bc(b + c) = 30

ac(a + c) = 12
Gii

168
R rng a, b, c khng th bng 0. t S = a + b + c, P = abc h phng trnh tr thnh

PS
P

c=

(S c) = 6

P +6
c

PS
P
(S a) = 30 a =

P + 30
a

b = P S
P (S b) = 12
b
P + 12


P
P
P
Nh vy S = S
+
+
P + 6 P + 12 P + 30
Nu S = 0 th a + b = c, khi phng trnh u tin tr thnh abc = 6 v hai pt cn li
l abc = 30 v abc = 12 iu ny khng th xy ra.
P
P
P
+
+
=1
Do
P + 6 P + 12 P + 30
Bin i v thu gn ta c P 3 + 24P 2 1080 = 0 (P 6)(P 2 + 30P + 180) = 0
> Vi P = 6 ta c a =

S
S
S
S3
, b = , c = ab(a + b) =
=6S=6
6
3
2
36

Vy (a, b, c) = (1, 2, 3).

> Vi P 2 + 30P + 180 = 0 P = 15 + 3 5; P = 15 3 5

> Xt P = 15 + 3 5 ta c

15
+
3
5
53

S=
a=
S

15
+
3
5

15 + 3 5
S = 5S
b=

3 + 3 5

15 + 3 5
5+5

c =
S=
S
2
9 + 3 5
r
3
36
Kt hp ab(a + b) = 6 ta c 5S = 6 S = 6
r
r
r5 !

3 5 6 36
36 5 + 5 6 36
Vy (a, b, c) =
, 56
,
2
5
5
2
5
r
r !
r

3 + 5 6 36 6 36 5 5 6 36
Tng t vi P = 15 3 5 ta tm c (a, b, c) =
, 5
,
2
5
5
2
5
r
r
r !
3 5 6 36 6 36 5 + 5 6 36
Kt lun: H c nghim (a; b; c) =
, 5
,
,
2
5
5
2
5
r
r
r !
3 + 5 6 36 6 36 5 5 6 36

, 5
,
2
2
5
5
2
5

3
2
2

2x = 2y(x + 1) (z + 1) (1)
Bi 27: Gii h phng trnh

2y 4 = 3z(y 2 + 1) 2(x2 + 1) (2)

2z 5 = 4x(z 2 + 1) 3(y 2 + 1) (3)


Gii

Xt 6 trng hp:
> Nu y 6 x 6 z:

169
T (1) ta c:
0 = 2y(x2 + 1) (z 2 + 1) 2x3
0 6 2x(x2 + 1) (z 2 + 1) 2x3
0 6 2x(x2 + 1) (x2 + 1) 2x3 (x 1)2 6 0
T suy ra x = y = z = 1
> Nu y 6 z 6 x :
2x3 + z 2 + 1 > 2x3 + z 2 + 1 ()
T (1) ta c 2x3 + z 2 + 1 = 2y(x2 + 1) = 2yx2 + 2y
Li c:

2
3

2yx + 2y 6 2x + 2y
2x3 + z 2 + 1 6 2x3 + z 2 + 1
2yx2 + 2y 6 2x3 + 2z

2yx2 + 2y 6 2x3 + z 2 + 1
Kt hp () suy ra x = z, ngha l x = y = z = 1.
> Nu z 6 y 6 x :
T (2) ta c
0 = 3z(y 2 + 1) 2y 4 2(x2 + 1)
0 6 3y(y 2 + 1) 2y 4 2(x2 + 1)
0 6 3z(y 2 + 1) 2y 4 2(y 2 + 1) = (2y 2 + y + 2)(y 1)2
T suy ra x = y = z = 1
> Nu z 6 x 6 y:
T (2) ta c
0 = 8[3z(y 2 + 1) 2y 4 2(x2 + 1)] 0 6 8[3x(y 2 + 1) 2y 4 2(x2 + 1)]
0 6 (3y 2 + 1 4x)2 16(y x) [7(y + 1)2 + 8](y 1)2
T suy ra y = 1 x = y = z = 1
> Nu x 6 y 6 z:
T (3) ta c
0 = 4x(z 2 + 1) 3(y 2 + 1) 2z 5
0 6 4y(z 2 + 1) 3(y 2 + 1) 2z 5
3
7
0 6 (2z 3 + 4z 2 + 2z + 3)(z 1)2 (z y)[(2z ) + ]
2
4
Do y = z x = y = z = 1
> Nu x 6 z 6 y:
T (3) ta c
0 = 4x(z 2 + 1) 3(y 2 + 1) 2z 5
0 6 4z(z 2 + 1) 3(y 2 + 1) 2z 5
0 6 4x(z 2 + 1) 3(z 2 + 1) 2z 5
0 6 (2z 3 + 4z 2 + 2z + 3)(z 1)2

170
T c x = y = z = 1.
Tm li h c nghim duy nht (x; y; z) = (1; 1; 1) 2.

x2 + y 2 + xy = 37 (1)

Bi 28: Gii h phng trnh (XXV III) x2 + z 2 + xz = 28 (2)

y 2 + z 2 + yz = 19 (3)
Gii
Ta c:
(1) (2) y 2 z 2 + x (y z) = 9 (y z) (x + y + z) = 9 (4)
(2) (3) x2 y 2 + z (x y) = 9 (x y) (x + y + z) = 9 (5)
Li c:
"
(4) (5) [(y z) (x y)] (x + y + z) = 0

x+y+z =0
yz =xy

> Nu : x + y + z = 0 z = (x + y) Thay vo h ta c:

x2 + y 2 + xy = 37

x2 + y 2 + xy = 28 (v nghim)

x2 + y 2 + xy = 19
(
> Nu y z = x y = t :

x=y+t
z =yt

Thay vo (4) ta c:

t (y + y + t + y t) = 9 ty = 3 t =

3
(6)
y

Thay vo (3) ta c:
y 2 + (y t)2 + y (y t) = 19 3y 2 3ty + t2 = 19 3y 2 + t2 = 28 (7)
Thay (6) vo (7) ta c:

y 2 = 9 y = 3 t = 1
9

3y 2 + 2 = 28 3y 4 28y 2 + 9 = 0

1
3
2
y
y = y=
t = 3 3
3
3

1
1
Xt 4 trng hp (y; t) = (3; 1), (3; 1), ( ; 3 3), ( ; 3 3) ta tm c nghim ca
3
3
h l

!

!
10 3 3 8 3
10 3
3 8 3
;
;
,
;
;
2
(x; y; z) = (4; 3; 2) , (4; 3; 2) ,
3
3
3
3
3
3

Bi tp t luyn

x = 3z 3 + 2z 2

Bi 1:. Gii h phng trnh: y = 3x3 + 2x2

z = 3y 3 + 2y 2

171
(
Bi 2: Gii h phng trnh:

2x 2 = 3y 3x
2y 2 = 3x 3y

( OLP 30-4-2008 T130)

x3 9(y 2 3y + 3) = 0

Bi 3: Gii h phng trnh: y 3 9(z 2 3z + 3) = 0 (OLP 30-4-2010)

z 3 9(x2 3x + 3) = 0

a1 4a2 + 3a3 > 0

a 4a3 + 3a4 > 0

2
.
Bi 4: Cho 100 s a1 , a2 , a3 , ..., a100 tha mn: ...........................

a99 4a100 + 3a1 > 0

a 4a + 3a > 0
100
1
2
Tnh cc s a2 , a3 , ..., a100 bit
a1 = 1
3x y

= x2

x 3y

3y z
Bi 5: Gii h phng trnh:
= y 2 (THTT 12-2010).

3z

3z x

= z2
z

3x

4x2

y
=

4x2 + 1

4y 2
Bi 6: Gii h phng trnh: z = 2

4y + 1

x = 4z
4z 2 + 1

PHNG PHP BIN I NG THC


Nhiu bi h phng trnh tuy nhn phc tp nhng c th gii bng nhng ng thc n
gin. Mu cht gii nhng bi ton dng ny l ta phi nhn ra quan h gia cc n s, t
t lp nn nhng hng ng thc thch hp. Mt s ng thc c th khng quen thuc, nn
phng php ny i hi kinh nghim v s tinh .

Bi tp v d
2
x + y 2 = 1 + xy
2
2
Bi 1: Gii h phng trnh
( x ) + ( y ) = 1
y+1
x+1

Gii
~ tng: H trn tuy l i xng loi I nhng bc kh cao, vic a v S = x + y v P = xy
c th gp nhiu kh khn. Nhng nu ta bit ng thc sau:
x
y
x2 xy + y 2 = 1
+
= 1 th bi ton tr nn n gin.
y+1 x+1
~ Li gii: KX: x, y 6= 1

172
(

x
y
= a,
= b ta c h phng trnh mi:
y+1
x+1

a=0

b=1
a+b=1

(
2
2
a=1
a +b =1

b=0

T tm ra nghim ca h l (x; y) = (0; 1); (1; 0) 2

x y
1
1
1

+
= 2 + 2 1

y x xy
x
y
Bi 2: Gii h phng trnh
2
y
x xy + y 2
x

+
=
x+1 y+1
xy
Gii
~ tng: Cng nh bi 1, vic a v S = x + y v P = xy cn nhiu bin i phc tp v
rc ri. Thc ra bn cht bi ton l t ng thc sau:
xy = 1

1
1
+
=1
x+1 y+1

Ta cng xem k hn v tng to ra bi ton:


u tin ta to ra 1 phng trnh cho tha mn xy = 1 chng hn nh
(xy 1)(x2 + xy + y 2 ) = 0 ()
thm phn tinh t cho bi ton ta bin i (*) thnh x2 + y 2 x2 y 2 = xy(x2 + y 2 1)
T lp c h phng trnh mi:
2
x + y 2 x2 y 2 = xy(x2 + y 2 1)
1 + 1 = 1 + (x y)2
x+1 y+1
Nhng mun che giu bn cht ca bi ton hn na ch cn thng qua php i bin n gin
1 1
P (x, y) P ( , )
x y

x y
1
1
1

y + x xy = x2 + y 2 1
Cui cng li h thnh
x
y
x2 xy + y 2

+
=
x+1 y+1
xy
y cng chnh l li gii cho bi 2! 2

1
2

y 2 = xy 2 xy (1)
Bi 3: Gii h phng trnh
1
4

= 1 (2)

2 +
(x + 1)
(y + 2)2
Gii
~ tng: T (1) ta c: (xy 2)(
Nhng nu t y th x =

1
+ 1) = 0 xy = 2
y2

2
vo (2) th s rt phc tp. Vy ta th tm mt quan h khc
y

173
gia

2
1
v
(v 2 biu thc ny xut hin (2)). Khi ta li n ng thc sau:
x+1
y+2
xy = 2

1
2
+
=1
x+1 y+2

n y th mi chuyn tr nn n gin.
~ Li gii:
KX: x 6= 1; y 6= 2
H cho tng ng:

xy = 2
1
4

=1

2 +
(x + 1)
(y + 2)2

1
2

+
=1

x+1 y+2

4
1

=1
2 +
(x + 1)
(y + 2)2

"
a + b = 1
a = 0; b = 1
1
2
t a =
; b=
(a, b 6= 0) ta c HPT:

(loi)
a2 + b 2 = 1
x+1
y+2
a = 1; b = 0
Vy h v nghim. 2
2
2
2
v2
1
( v + 1 1 + y + v 2 u2 )( u

+ 2
) = 9u2 (1)
2
2
2
2
u
v
1+v
1+u
v u2
Bi 4: Gii h phng trnh
2
u + v 2 = 1 (2)
Gii

~ tng:
Ta c mnh sau: (

bc ca ab
a
b
c
+
+
)(
+
+
) = 9 a + b + c = 0 ()
a
b
c
bc ca ab

* Chng minh:
(x y)(y z)(z x)
xy yz zx
+
+
=
z
x
y
xyz
t x = b c, y = c a, z = a b x y = 3c, y z = 3a, z x = 3b
Th th
3a
3b
3c
27abc
+
+
=
bc ca ab
(b c)(c a)(a b)

Xut pht t ng thc:

ng thc trn tng ng vi iu phi chng minh.


p dng mnh trn ta c li gii sau:
~ Li gii:
p dng () vi a = u2 , b =v 2 , c = 1 t (1) tasuy ra 9 = 9u2 .
9 = 9u2
u = 1
Vy ta c h tng ng:

u2 + v 2 = 1
v = 0
Vy h c nghim (u; v) = (1; 0); (1; 0) 2

x + y + xy = 3 (1)
Bi 5: Gii h phng trnh
4
4
1
x+1

+
+
=
(2)

5y + 9 x + 6 1 + (x + 1)(y + 2)
2
Gii

174
~ tng: T (1) ta n cch phn tch quen thuc l (x + 1)(y + 1) = 4.
(2) cng c x + 1 v y + 1 nn ta ngh ti t a = x + 1; b = y + 1. Tuy nhin mu cht ca
1
bi ton chnh l t c = t (1) c abc = 1.
4
1
1
1
Khi ta li c mnh sau: abc = 1
+
+
= 1.
1 + a + ab 1 + b + bc 1 + c + ca
T ta i n li gii:
~ Li gii:
1
t x + 1 = a, y + 1 = b, = c ta c h phng trnh
4

abc = 1
1
1
x+1
1

+
+
=
1 + a + ab 1 + b + bc 1 + c + ca
2
Li c:
1
1
1
1
a
ab
+
+
=
+
+
=1
1 + a + ab 1 + b + bc 1 + c + ca
1 + a + ab a + ab + abc ab + abc + a2 bc

x+1
x=1y=1
2
Vy h c nghim duy nht l (x; y) = (1; 1) 2

Thay vo phng trnh sau c 1 =

(
Bi 6: Gii h phng trnh sau:

x2 + y 2 = 2
(x + y)(1 + xy)4 = 32
Gii

~ tng: R rng khng th gi nguyn (1 + xy)4 . Mun vy ta ch ti ng thc


(x + y)2
2xy + 2
=
. n y bi ton tr nn n gin.
xy + 1 =
2
2
~ Li gii:
( 2
x + y2 = 2
H cho c vit li thnh:
(x + y)(2 + 2xy)4 = 29
Thay x2 + y 2 = 2 vo phng trnh th 2 ta c
4

(x + y)(x2 + y 2 + 2xy) = 29 (x + y)9 = 29 x + y = 2

x 2 + y 2 = 2
Vy ta c h
x + y = 2

x=y=1

Vy nghim ca h l (x; y) = (1; 1) 2


~ Nhn xt: tng chnh ca bi ton chnh l dng php th a v ng thc ng
bc. Khi vic gii s d dng hn. Ta xt mt s v d tng t:
(
Bi 7: Gii h phng trnh

x2 + y 2 = 2
(x + y)(4 x2 y 2 2xy) = 2y 5
Gii

175
~ tng: Xt phng trnh th hai. x + y c bc nht, 4 x2 y 2 2xy c bc 4 nhng
cc hng t cha ng bc. V vy ta ngh ti php th t phng trnh u to biu thc
thun nht.
~ Li gii: Thay phng trnh th nht vo phng trnh th 2 ta c:
h
i
2
(x + y) (x2 + y 2 ) xy(x2 + y 2 ) x2 y 2 = 2y 5


(x + y) x4 + y 4 + x2 y 2 xy(x2 + y 2 ) = 2y 5
x5 + y 5 = 2y 5 x = y
Kt hp vi phng trnh u ta tm c nghim ca h l (x; y) = (1; 1), (1; 1) 2

Bi 8: Gii h phng trnh sau

3x3 y 3 =

1
(1)
x+y

x2 + y 2 = 1 (2)
Gii

~ tng: Nhn vo h ta d dng ngh n vic th s 1 t phng trnh (2) vo phng


trnh (1) c phng trnh thun nht.
~ Li gii:
KX: x + y 6= 0
2
Phng trnh (2) tng ng vi (x2 + y 2 ) = 1
Th vo phng trnh (1) ta c:

2
3x3 y 3 (x + y) = x2 + y 2
3x4 y 4 + 3x3 y xy 3 = x4 + y 4 + 2x2 y 2
2x4 + 3x3 y 2x2 y 2 xy 3 2y 4 = 0

(x y) (x + 2y) 2x2 + xy + y 2 = 0

x=y

x = 2y
2x2 + xy + y 2 = 0
> Vi 2x2 + xy + y 2 = 0 x = y = 0 (khng tho x2 + y 2 = 1). Trng hp ny loi.
> Vi x = y ta c:

x
=
y
=

1
2
2x2 = 1 x2 =

1
2
x=y=
2
> Vi x = 2y ta c:

1
2
;x =
y
=

1
5
5
5y 2 = 1 y 2 =

1
2
5
y = ;x =
5
5

 
 
 

2 1
2 1
1 1
1 1
Kt lun: H c nghim (x; y) l ; ; ; ; ; ; ;
2
5 5
5 5
2 2
2 2

176
(
Bi 9: Gii h phng trnh sau

x3 + 4y y 3 16x = 0
y 2 = 5x2 + 4
Gii
(

~ tng: H phng trnh c th vit li nh sau:

x3 y 3 = 4(4x y) (1)

y 2 5x2 = 4 (2)
T ta ngh n vic th (2) vo (1) c phng trnh thun nht.
~ Li gii:
H phng trnh tng ng:
( 3
( 3

x y 3 = 4 (4x y)
x y 3 = y 2 5x2 (4x y)

y 2 5x2 = 4
y 2 5x2 = 4

(
(

3
2
2

21x 5x y 4xy = 0
x (7x 4y) (3x + y) = 0

y 2 5x2 = 4
y 2 5x2 = 4

"
> Vi x = 0

x=0

7x

y =

4
y = 3x
y 2 5x2 = 4

y=2

y = 2
 2
31 2
7x
7x
5x2 = 4

x = 4 (V nghim)
> Vi y =
4
4
15
"
x = 1; y = 3
2
> Vi y = 3x (3x) 5x2 = 4 x2 = 1
x = 1; y = 3
Kt lun: H c nghim (x; y) l (0; 2) ; (0; 2) ; (1; 3) ; (1; 3) 2
(
Bi 10: Gii h phng trnh sau

x3 8x = y 3 + 2y
x2 3y 2 = 6

( H Tnh TST 07-08)

Gii
(
H phng trnh c th vit li nh sau:

x3 y 3 = 2 (4x + y) (1)

x2 3y 2 = 6 (2)
Ta ngh n cch ng bc (1) bng cch dng php th t (2).Nhng trc khi th ta phi
lm xut hin s 6 (1) nn bin i:
(
(



3 x3 y 3 = 6 (4x + y)
3 x3 y 3 = x2 3y 2 (4x + y)

x2 3y 2 = 6
x2 3y 2 = 6

x=0

( 3
(

x = 3y
x + x2 y 12xy 2 = 0
x (x 3y) (x + 4y) = 0

2
2
2
2

x = 4y
x 3y = 6
x 3y = 6

x2 3y 2 = 6
> Vi x = 0 y 2 = 2 (v nghim)
"
> Vi x = 3y (3y)2 3y 2 = 6 y 2 = 6

y = 1; x = 3
y = 1; x = 3

177
r
6
6
; x = 4
y=

6
13
13
r
r
> Vi x = 4y (4y)2 3y 2 = 6 y 2 =

13
6
6
y=
;x = 4
13 !
r
r
r13
r !
6
6
6
6
Kt lun: H c nghim (x; y) l (3; 1) ; (3; 1) ; 4
;
; 4
;
2
13
13
13
13
( 2
x + y 2 + xy = 3
Bi 11: Gii h phng trnh
x5 + y 5 + 15xy(x + y) = 32

Gii
~ tng: Ta cn tm mi quan h gia cc hng t trong h. Mun vy ta ch ti hng
ng thc (x + y)5 = x5 + y 5 + 5xy(x + y)(x2 + xy + y 2 ).
~ Li gii:
H cho c vit li nh sau:

x2 + y 2 + xy = 3
x2 + y 2 + xy = 3

x5 + y 5 + 5xy(x2 + xy + y 2 )(x + y) = 32
x5 + y 5 + 5.3xy(x + y) = 32

( 2
2
x + y = 2
x + y + xy = 3

x=y=1
xy = 1
x+y =2
Vy h c nghim (x; y) = (1; 1) 2


x
Bi 12: Gii h phng trnh
1

+
x


1
= 2 y 4 x4
2y


1
= x2 + 3y 2 3x2 + y 2
2y
Gii

~ tng: Khai trinv phi ca hai phng trnh. gii bi ton ta ch cn ch cng
x5 + 5x4 y + 10x3 y 2 + 10x2 y 3 + 5xy 4 + y 5 = (x + y)5
thc khai trin bc 5:
x5 5x4 y + 10x3 y 2 10x2 y 3 + 5xy 4 y 5 = (x y)5
~ Li gii:
H tng ng

1
1

+
= 3(x4 + y 4 ) + 10x2 y 2
x 2y
1
1


= 2(y 4 x4 )
x 2y
Cng tr 2 PT ta c:

= 5y 4 + x4 + 10x2 y 2
x
1

= 5x4 + y 4 + 10x2 y 2
y

2 = 5xy 4 + x5 + 10x3 y 2
1 = 5yx4 + y 5 + 10x2 y 3

Li cng tr 2 PT ta c:

(x + y)5 = 3
(x y)5 = 1

178

5
Vy h phng trnh cho c 1 nghim l: (x; y) =

3+1
;
2

31
2

!
2

Sau y l mt s li gii bng cch ghp hng ng thc to ra nhng i lng khng m.
(
Bi 13:Gii h phng trnh

x2 4 3x 2 + 10 = 2y
p
y 2 6 4y 3 + 11 = x
Gii

H cho tng ng vi:

(x 2)2 + ( 3x 2 2)2 + x + 4 = 2y
p
(y 3)2 + ( 4y 3 3)2 + 2y = x + 4
Cng 2 phng trnh trn v theo v ta thu c:
p

2
2
(x 2)2 + (y 3)2 + ( 3x 2 2) + ( 4y 3 3) = 0
T tm c nghim ca h l (x; y) = (2; 3) 2

1 1 1

+ + = 2 (1)
x y z
Bi 14: Gii h phng trnh
2
1

2 = 4 (2)

xy z
Gii
Ta c:

2

2
1 1 1
1
1 1 1
2
(1)
+ +
+ +
2
=4
=
x y z
x y z
xy z
1
1
1
2
2
2
2
1
2+ 2+ 2+
+
+
=
2
x
y
z
xy yz zx
xy z

 

1
2
1
1
2
1

+
+
+
+
+
=0
x2 xz z 2
y 2 yz z 2
1
1


2 
2
=
1 1
1 1
x
z

+
+
+
=0 1
x = y = z
1

x z
y z
=
y
z


1 1 1
Th vo h ta tm c nghim (x; y; z) =
; ;
2
2 2 2
(

(2 x) (1 2x) (2 + y) (1 + 2y) = 4 10z + 1 (1)


Bi 15: Gii h phng trnh
x2 + y 2 + z 2 + 2xz + 2yz + x2 y 2 + 1 = 0 (2)


Gii
Ta c:

(2) (x + y + z)2 + (xy 1)2 = 0




z = (x + y)
z = x +
x+y+z =0
x

1
xy 1 = 0

y=
y =
x
x

179
Thay vo (1), ta c:
s




1
2
1
(2 x) (1 2x) 2 +
1+
= 4 1 10 x +
x
x
x
s





x+2
1
2x + 1
= 4 1 10 x +
(2 x) (1 2x)
x
x
x
s


(4 x2 ) (1 4x2 )
1

= 4 1 10 x +
x2
x
s




1
1
2
(3)
4 x + 2 17 = 4 1 10 x +
x
x


|t| > 2
1
t: t = x +
.
x2 + 1 = t2 2
x
x2
T ta c:


(3) 4 t2 2 17 = 4 1 10t

4t2 25 = 4 1 10t
2
4t2 25 16 (1 10t) = 0

4t2 20t + 29 (2t + 3) (2t + 7) = 0
7
1
7
t = (do |t| > 2) x + =
2
x 2
7 33
2x2 + 7x + 2 = 0 x =
4

7 + 33
7 33
7
~ Vi x =
y=
z=
4
4
2
7 33
7 + 33
7
~ Vi x =
y=
z=
4
4
2
Vy h phng trnh cho c 2 nghim
l
!
!

7 + 33 7 33 7
7 33 7 + 33 7
(x; y; z) =
;
;
,
;
;
2.
4
4
2
4
4
2

Bi tp t luyn
Gii
h phng trnh:

xy x = 2
1)
16
1

4 +
4 = 1
(x
+
1)
(y
+
1)
( 3
x + 5xy 2 3y 3 = 2x y
2)
x2 + 2xy = 1
5
5

x + y = 31
7
3) x3 + y 3

x2 + xy + y 2 = 3
( 3
x + y 3 xy 2 = 1
4)
4x4 + y 4 4x y = 0
( 2
x + y2 = 4
5)
8x2 + x5 = 8y 2 + y 5

180

PHNG PHP DNG N IU HM S


L thuyt
I nh l 1: Nu hm s y=f(x) lun ng bin (hoc lun ngch bin) th s nghim ca
phng trnh : f (x) = k khng nhiu hn mt v f (x) = f (y) x = y.
I nh l 2: Nu hm s y=f(x) lun ng bin (hoc lun nghch bin) v hm s y=g(x)
lun ngch bin (hoc lun ng bin) trn D th s nghim trn D ca phng trnh: f(x)=g(x)
khng nhiu hn mt.
I nh l 3: Cho hm s y=f(x) c o hm n cp n v pt f (k) (x) = 0 c m nghim, khi
pt f (k1) (x) = 0 c nhiu nht l m+1 nghim.
Nh vy, bn cht ca vn chnh l vic phn tch
g(x) = h(y) f (p(x)) = f (q(y))
Sau ta s xt tnh ng bin , nghch bin ca hm c trng f (x). Trong tin cho
vic nh gi th ta thng bin i hm c trng f (x) thnh mt hm a thc .V c th
lm c iu th chng ta cn s dng , kt hp kho lo 2 k thut t n ph v h
s bt nh.
~Ch : Hm s c trng f (x) khng c tnh duy nht , tc l nu t mt phng trnh no
ta rt ra c 1 hm s c trng th chc chn cn c th tm c hm s th 2, v vn
ca chng ta l lm sao c th tm c 1 hm s n gin v mt tng , cng nh n
gin trong vic nh gi ( xt tnh n iu ) ca n trong bi ton.

Bi tp v d

tan x tan y = y x (1)


Bi 1: Gii h phng trnh: (I)
p
y + 1 1 = x y + 8 (2)
2005)

( Olympic 30-4 nm

Gii
~ tng: Ta thy bi ton dng ny rt c trng cho phng php c nu v cch ra
ny cng thng hay c s dng trong cc thi C H, thi HSG. iu quan trng l
cn chng minh c x = y t h trn.
~ Li
( gii:
16y
K: p
y+86x
T phng trnh ban u ta c : tan x + x = tan y + y

181
V rt d dng tm c hm c trng ca phng trnh l f (t) = tant + t.
1
Li c f 0 (t) =
+ 1 > 0 nn f (t) l mt hm ng bin trn tp xc nh, suy ra x = y.
cos2 t
Th vo (2) ta c:
p
p

y+11= y y+
8

y
+
1
=
y

y
+
8
+
1

y
+
8
=
4y

4
y+8

8
y >

3
3y 8 = 4 y + 8
y=8
2
9y 48y + 64 = 16y + 128
Vy x = y = 8 l nghim duy nht ca (I) 2.
~ Nhn xt: bi ton trn bn cht ca vn c th hin rt r rng ngay phng
trnh u tin. Nn c th nhn xt rng y l mt bi ton d trong vic nhn ra tng
ca phng php.

(x + 1 + x2 )(y + p1 + y 2 ) = 1 (1)
Bi 2: Gii h phng trnh (II)
x6x 2xy + 1 = 4xy + 6x + 1 (2)
Gii
~ tng: Nhn qua ta thy (2) c v d khai thc hn thng qua vic bnh phng
tm quan h gia x, y. Nhng tht ra mu cht ca bi ton li l (1). Ch cn rng
p
1
p
= y + y 2 + 1 th ta gii quyt xong.
y + y2 + 1
~ Li gii:
KX: 6x 2xy + 1 > 0 ()
p

2 + 1 = y +
2 + 1 f (x) = f (y) vi f (t) = t +
Ta c: (1) x +
x
y
t2 + 1

t2 + 1 + t
|t| t
Li c: f 0 (t) =
>
> 0 do f (t) ng bin trn R.
t2 + 1
t2 + 1
Vy f (x) = f (y) x = y. Th vo (2) ta c:
"

2 25x2

2x2 + 6x + 1 = 3x
x
x 6x + 2x2 + 1 = 4x2 +6x+1
2x2 + 6x + 1
=

2
4
2x2 + 6x + 1 = 2x

> Nu

2x2 + 6x + 1 = 3x

2x2 + 6x + 1 = 9x2

x = 1 y = 1

x > 0

2x2 + 6x + 1 = 4x2

> Nu 2x2 + 6x + 1 = 2x
x 6 0

11

3 + 11
y=
2

11 3 + 11
;
)2
2
2

3x 1 + 4 (2x + 1) = y 1 + 3y (1)
Bi 3: Gii h phng trnh (III)
(x + y) (2x y) + 4 = 6x 3y (2)

Th li () ta thy (II) c nghim (x; y) = (1; 1); (

x=

Gii
~ tng: Ta thy (1) c dng n iu, nhng khng th tm c hm s c trng. Nh
vy r rng ta phi thay i x hoc y. Chuyn qua phn tch (2) th thy rng

182
(2) (x + y + 1)(2x y + 4) = 0. Vy ta tnh c y theo x, t vic xy dng hm c
trng cho (1) tr nn d dng v ch cn mt n.
~ Li gii:
1
KX: x > , y > 1 ().
3
Ta c:
(2) (x + y)(2x y) + 4 = 4(x + y) (2x y) (x + y + 1)(2x y + 4) = 0
7
y = 2x + 4 ( do () x + y + 1 > > 0). Thay vo (1) ta c:
3

3x 1 + 2x 8 = 2x + 3

2 (3x 1) + 3x 1 = 2 (2x + 3) + 2x + 3.
(1)
Xt hm s f (t) = 2t2 + t vi t > 0. Phng trnh (12) c th vit li thnh




f
3x 1 = f
2x + 3
Ta c f 0 (t) = 4t + 1 > 0 t > 0 nn hm s f ng bin trn (0; +). Do

3x 1 =

2x + 3 x = 4 y = 12.

Vy (III) c nghim (x, y) = (4, 12) 2

x3 + x 2 = y 3 + 3y 2 + 4y (1)
Bi 4: Gii h phng trnh (IV )
x5 + y 3 + 1 = 0 (2)
Gii
~ tng: Rt t nhin ta nhn vo tng PT nh gi vi mc ch tm mi quan h gia
hai bin.
T (1) ta thy rng 2 v l 2 a thc c lp ca 2 bin x ,y v cng bc. Nh vy vic p
dng phng php s dng tnh n iu c c hi thnh cng rt cao. V y cng l lc
chng ta dng ti k thut h s bt nh.
u tin, ta chn mt a thc bt k lm chun (1). D thy nn chn a thc bn v tri
v nhn n n gin hn. Vi tng ta c hm s c trng f (t) = t3 + t 2 , nh vy
vic ca chng ta cn lm l phn tch :
y 3 + 3y 2 + 4y = g 3 (y) + g(y) 2
R rng g(y) c dng g(y) = y + b t y ta khai trin v c b = 1
Nh vy ta c phng trnh x3 + x 2 = (y + 1)3 + (y + 1) 2 ti y th tng gii bi
ton c hon thin.
~ Li gii:
T PT (1) ta c: x3 + x 2 = (y + 1)3 + (y + 1) 2 (*)
Xt hm s c trng: f(t) = t3 + t 2 c f(t) = 3t2 + 1 > 0, t R
Suy ra f(t) ng bin trn R , Kt hp vi (*) ta c x = y + 1
Th vo PT (2):
x5 + (x 1)3 + 1 = 0 x5 + x3 3x2 + 3x = 0

183
(
x(x4 + x2 3x + 3) = 0

x=0

x4 + x2 3x + 3 = 0(V N )
Vy (IV ) c nghim duy nht (x; y) = (0, 1) 2
(
Bi 5: Gii h phng trnh:V

p
(4x2 + 1)x + (y 3) 5 2y = 0 (1)
(H khi A 2010)

4x2 + y 2 + 2 3 4x = 7 (2)
Gii

~ tng: Kh c th khai thc g t (2), do ta s bt u phn tch t (1). Phng trnh


ny c x, y tch bit nn kh nng dng n iu l cao. Vy ta bin i phng trnh v dng
p
g(x) = h(y) (4x2 + 1)x = (3 y) 5 2y
p
p
1 + a2
a3 a
5 a2
5 a2
v (3 y) 5 2y = (3
)a =
.a =
+
5 2y = a y =
2
2
2
2
2
3
t
t
Ta hi vng rng f (t) = + chnh l hm c trng m ta cn tm. Vy cn phi phn tch
2
2
3
p(x)
p(x)
(4x2 + 1).x = 4x3 + x =
+
. R rng p(x) c dng mx + n , dng h s bt nh ta
2
2
t3
t
thu c m = 2; n = 0 p(x) = 2x. Nh vy hm s c trng chnh l f (t) = +
2
2
3
5
~ Li gii: K: x 6 ; y 6 .
4
2
Ta c
q
3
(5 2y)3 5 2y
p
(2x)
2x
2
+
=
+
(1) (4x + 1)x = (3 y) 5 2y
2
2
2
2
t

t3
t
3t2 1
+ c f 0 (t) =
+ > 0, t R
2
2
2
2
(
x>0

Suy ra hm f(t) l hm ng bn trn R , hay : 2x = 5 2y


4x2 = 5 2y
Ti y ta th vo (2) ta c :
Xt hm s c trng : f (t) =

5
2
4x + ( 2x ) + 2 3 4x 7 = 0
2
2

3
5
Xt hm s g(x) = 4x2 + ( 2x2 ) + 2 3 4x 7 c g 0 (t) > 0, x 6 suy ra g(x) ng bin
2
4
1
1
trn TX , li c g( ) = 0 x = y = 2
2
2
1
Vy V c nghim (x; y) = ( ; 2) 2.
2
( 5
x + xy 4 = y 10 + y 6 (1)
p
Bi 6: Gii h phng trnh (V I)
4x + 5 + y 2 + 8 = 6(2)

Gii
~ tng: Nhn vo h trn ta thy rng khng th dng php th. Phng trnh 1 c 2 n
khng c lp vi nn ta ngh ti vic th nhm li v phn tch nhn t, li c x y 2 l mt

184
nhn t chung, tuy vy vic phn tch ra l tng i phc tp khi phi dng hng ng thc
lin quan ti A5 B 5 .
Ta cng xt cch khc n gin hn. Vi d on x = y 2 l mi quan h duy nht ca x ,y ta
ngh ti dng n iu. Hy th nh gi:
(1) c hng t xy 4 m trong vic xt hm c trng ta li ch c th xt hm mt n nh vy
mt iu rt t nhin l ta tm cch bin i sao cho (1) tr thnh PT c 2 n tch bit. V
chia 2 v cho y 5 l cch duy nht gip chng ta gii quyt c tt c vn nu trn. Ta
cng i n li gii chi tit:
~ Li gii:
5
K: x >
4
D thy y = 0 khng tho h. Vi y6= 0 chia c 2 v ca PT(1) cho y 5 ta c:
(1)

x5 x
+ = y 5 + y ()
y5 y

Xt hm s c trng f (t) = t5 + t c f 0 (t) = 5t4 + 1 > 0, t R


x
Suy ra f(t) ng bin trn R , kt hp vi (*) ta c: = y x = y 2
y

Th vo PT(2) ta c 4x + 5 + x + 8 = 6 ()
5
Ta c x =
khng tho (**). t V T () = g(x).
4
2
1
5
5
Ta c g 0 (x) =
+
> 0 x >
do g(x) ng bin trn ( ; +)
4
4
4x + 5 2 x + 8
Li c g(1) = 6 nn (**) c nghim duy nht x = 1. Suy ra y = 1.
Vy (V I) c nghim (x; y) = (1 : 1); (1; 1) 2
~ Nhn xt: Vi nhng phng trnh c bc cao th vic h bc v t n ph rt hu ch. Ta
cng xem qua mt s bi tng t:
(
Bi 7: Gii h phng trnh (V II)

2
x3 (4y
+ 2 (x2 +
1) x = 6 (1)

 + 1)
p

x2 y 2 + 2 4y 2 + 1 = x + x2 + 1 (2)
Gii

KX: x > 0.
Nu x = 0, t phng trnh th hai ca h ta c 0 = 6 (sai). Vy x > 0, chia c hai v ca (2)
cho x2 ta thu c
!
r

 1
p
1
2y 1 + 4y 2 + 1 =
1+
+ 1 (3)
x
x2


Xt hm s f (t) = t 1 + 1 + t2 vi t R, phng trnh (3) c th vit li thnh
1
f (2y) = f ( )
x
2t2 + 1
1
Ta c f 0 (t) = 1 +
> 0 t R, do 2y = .Thay vo (1) ta c
x
t2 + 1


x3 + x + 2 x2 + 1 x 6 = 0 x3 + x 6 = 2 x2 + 1 x (4)

185

Xt cc hm s g (x) = x3 + x 6, h (x) = 2 (x2 + 1) x vi x (0; +).


D thy g(x), h(x) n iu ngc chiu trn (0; +) v g(1) = h(1) nn (4) c nghim duy
1
nht x = 1 y = .
2


1
Vy (V II) c nghim (x, y) = 1,
2.
2

 p


x2 + 1 3x2 y + 2
4y 2 + 1 + 1 = 8x2 y 3
Bi 8: Gii h phng trnh sau (V III)
x2 y x + 2 = 0
Gii
Vi x = 0 hoc y = 0 th h v nghim.
Vi x =
6 0 v y 6= 0.Phng trnh (1) tng ng vi

x2 + 1 4x2 y + x 2
x2 + 1 4x2 y + x
p
p
.4y = 8x2 y 3
= 2x2 y
4y 2 + 1 1
4y 2 + 1 1
p

p

2
2
2
2
2
2
2
2
4y + 1 + 1
x + 1 4x y + x = 2x y 4y + 1 2x y x + 1 + x = 2x y
!
r
q

1
1
2

+ 1 + 1 = 2y
(2y) + 1 + 1 (3)
x
x2


t
t
+
t
+
1
+
t2 + 1
0

+1 =
>0
Xt hm s: f (t) = t t2 + 1 + 1 c f (t) = t2 + 1+
t2 + 1
t2 + 1


Nn hm s f (t) = t t2 + 1 + 1 ng bin trn R
T phng trnh (3) ta c
 
1
1
f
= f (2y) = 2y 2xy = 1
x
x
Th vo phng trnh (2) ta c:

x = 4
2x2 y 2x + 4 = 0 x 2x + 4 = 0
y = 1
8


1
Kt lun: H c nghim (x; y) = 4;
2
8
p
( 3
x 3x2 + 2 = y 3 + 3y 2 (1)
p
Bi 9: Gii h phng trnh :(IX)

3 x 2 = y 2 + 8y (2)
Gii
~ tng: Chng ta li bt u tm ti t ci n gin ti phc tp
T PT(1): rng ta c dng g(x) = h(y) nh mong mun nh vy tng dng tnh
n iu xt hm c trng xut hin. V s tt hn nu g(x), h(y) l hm a thc. Vy
ta th bnh phng loi b cn thc:
2

(1) (x3 3x2 + 2) = y 3 + 3y 2

186
Cng vic tip theo l tm hm c trng. D thy h(y) = y 3 + 3y 2 l la chn tt v y l
hm s n gin v ng bin trn [0; +].
2
Ta s c gng phn tch (x3 3x2 + 2) = q 3 (x) + 3q 2 (x) . ng nht h s s tm c
q(x) = x2 2x 2. Suy ra x2 2x 2 = y (ch iu kin c nghim l
x3 3x2 + 2 = (x 1)(x2 2x 2) > 0 x2 2x 2 > 0 do x > 2).
Nhng cu hi t ra l, vic khai trin v ng nht h s vi (x3 3x2 + 2)2 kh phc tp.
Li ch rng hm s c trng khng phi l duy nht. Liu c mt hm s no n gin
hn? Vy 1 iu t nhin l ta s i tm cch t n ph : 1 hm cha cn no khng
phi lu tha.
p

rng (1) x3 3x2 + 2 = y 3 + 3y 2 x3 3x2 + 2 = y y + 3

Nh vy ta s t a = y + 3 y = a2 3 , y y + 3 = (a2 3)a = a3 3a

3
Phn tch hon ton tng t nh cc v d trc , ta c (x 1)3 3(x 1) = ( y + 3)

3 y + 3 v hm c trng f (t) = t3 3t l hm ng bin trn [1; +] .Nh vy tng


r rng.
~ Li gii:
KX: x > 2; y > 0
Ta c:
p
p
p
3
y + 3 (x 1)3 3(x 1) = ( y + 3) 3 y + 3

Xt hm c trng f (t) = t3 3t c f 0 (t) = 3t2 3> 0, t > 1 ( v y + 3 > 3; x 1 > 1 )

Suy ra hm s ng bin trn [1; +] , hay x 1 = y + 3 y = x2 2x 2


Th vo (2):
(1) x3 3x2 + 2 = y

(2) 9(x2) = y 2 +8y 9(x2) = (x2 2x 2) +8(x2 2x2) x4 4x3 +8x2 17x+6 = 0
"
x=3
(x 3)(x3 x2 + 5x 2) = 0
x3 x2 + 5x 2 = 0
Xt Q(x) = x3 x2 + 5x 2 c Q0 (x) = 3x2 2x + 5 > 0, x suy ra y l mt hm ng bin
trn R
Li c x > 2 Q(x) > Q(2) = 13 > 0 suy ra phng trnh Q(x) = 0 v nghim.
Vy (IX) c nghim (x; y) = (3; 1) 2.
~ Nhn xt: Nhng bi ton trn cho thy c nhiu cch a hai v ca mt phng
trnh v hm c trng. Tuy nhin mt s bi ton kh hn s i hi phi bin i cc phng
trnh ca h tm ra hm c trng.
(
Bi 10: Gii h phng trnh (X)

x3 (2x + 3y) = 1
x(y 3 2) = 1
Gii

> Vi x = 0 d thy khng phi l nghim ca h trn.


> Vi x 6= 0 ta c
( 3
x (2 + 3y) = 1
1
3
(X)
x3 (2 + 3y + y 3 2) = 3x2 + 1 y 3 + 3y = 3 +
3 3
2
x
x
x (y 2) = 3x

187

Ti y hm s c trng l r: f (t) = t3 + 3t
C f 0 (t) = 3t2 + 3 > 0, t R suy ra f (t) l hm s ng bin trn R y =

1
ti y ta ch
x

vic th vo mt PT bt k trong 2 PT th bi ton s c hon tt :

y = 1
y = 1
x = 1, y = 1
x

x
(X)

1
3

2x3 + 3x2 1 = 0
x3 (2 + ) = 1
x = ,y = 2
2
x
1
Th li ta thy (X) c nghim (x; y) = (1; 1); ( ; 2) 2.
2
Sau y ta cng xem qua mt s bi h phng trnh logarit:
Bi 11:
Gii h phng trnh
log (2x + 1) log (x y) + 1 = 4x2 + 4x + 2 p(x y)2 + 1 + (x y)2 4x(x + 1) (1)
3
3
(XI)

log (2y 2) + 4x2 4x2 + 1 = 1 2 (2)


3
Gii
~ tng: Phng trnh (1) nhn phc tp hn nhng kh nng nhm nhn t li cao, nn
ta s phn tch (1). D thy biu thc trong log l khng th thay i, do ta s ly 2x + 1
v x y lm chun. Vic xy dng hm c trng bng h s bt nh ging nh nhng bi
trn, nn y ta s khng nhc li.
~ Li gii:
Ta c:
(1)

p
p
(2x + 1)2 + 1 (2x + 1)2 log3 (2x + 1) = (x y)2 + 1 (x y)2 log3 (x y) ()

t
1
1
t2 + 1 t2 log3 t (t > 0) ta c f 0 (t) =
(2t + ) 6 2 2 < 0 nn
t
t2 + 1
2
f (t) nghch bin trn (0; +).
Do () 2x + 1 = x y x = y 1

Khi (2) log3 (2x) + 4x2 4x2 + 1 = 1 2.

1
1
Xt g(x) = log3 (2x) + 4x2 4x2 + 1 (x > 0) ta c g 0 (x) = 4x(2
) + > 0 nn
x
4x2 + 1

1
1
3
g(x) ng bin. Li c g( ) = 1 2 nn (2) c nghim duy nht x = y =
2
2
2
1 3
Vy (IX) c nghim (x; y) = ( ;
) 2.
2 2

ey2 x2 = x + 1
y2 + 1
Bi 11: Gii h phng trnh

3log (x + 2y + 6) = 2log (x + y + 2) + 1
Xt f (t) =

Gii
(
KX:

x + 2y + 6 > 0

x+y+2>0
Xt hm s: f (t) = et (t + 1), t [0, +). Ta c f 0 (t) = et (t + 1) + et = et (t + 2) > 0 nn y

188
l hm ng bin.
Do :
ey

2 x2

x2 + 1
2
2
ex (x2 + 1) = ey (y 2 + 1) f (x2 ) = f (y 2 ) x2 = y 2 x = y
2
y +1

Phng trnh th hai ca h tng ng vi:


3log2 (x + 2y + 6) = 2log2 (x + y + 2) + 1




log2 (x + 2y + 6)3 = log2 2(x + y + 2)2
(x + 2y + 6)3 = 2(x + y + 2)2 ()
Xt hai trng hp:
> Nu x = y th thay vo (*), ta c: (3x + 6)3 = 2(2x + 2)2 .
Theo KX ta c x > 1 .Li c
(3x + 6)3 2(2x + 4)2 = (x + 2)2 (27x + 46) > 0 (3x + 6)3 > 2(2x + 4)2
Do : (3x + 6)3 > 2(2x + 4)2 > 2(2x + 2)2 nn (*) v nghim.
> Nu x = y, thay vo (*), ta c:
(x + 6)3 = 2(2)2 (6 x)3 = 8 6 x = 2 x = 4 y = 4
Th li ta thy h cho c nghim duy nht l (x, y) = (4, 4) 2
~ Nhn xt: Trong bi ny nn ch cc nh gi trong trng hp x = y, bi v khi
phng trnh bc ba thu c phi gii theo cng thc tng qut, iu thng b trnh cc
k thi HSG. Do , vic tm mt nh gi thch hp chng minh nghim khng tha bi
l mt cch rt hay.

Bi tp t luyn
Gii cc(h phngtrnh sau:
(8x 3) 2x 1 y 4y 3 = 0
Bi 1)
4x2 8x + 2y 3 + y 2 2y + 3 = 0
( 3
x (3y + 55) = 64
Bi 2)
xy(y 2 + 3y + 3) = 12 + 51x

2x3 4x2 + 4x 1 = 2x3 (2 y)3 2y


Bi 3)
p
x + 2 = 3 14 x3 2y + 1
(

x+y+1+ 3x+y =5
p
Bi 4) p 2
x + xy + 4 + y 2 + xy + 4 = 12
(
x3 y 3 2 = 3x 3y 2
p

Bi 5)
x2 + 1 x2 3 2y y 2 + 2 = 0

PHNG PHP H S BT NH
H s bt nh l ngun gc cho nhiu li gii p. Bi vit ny s cp n nhng bin i
trong h phng trnh hu t da vo h s bt nh. Mc tiu ca chng ta l sau khi bin i

189
thu c mt phng trnh c th phn tch thnh nhn t hoc c chnh phng
(nu l bc 2). Ta cng xem qua mt s v d:

Bi tp v d

x3 y 3 = 35 (1)
Bi 1: Gii h phng trnh (I)
2x2 + 3y 2 = 4x 9y (2)
Gii
~ tng: Khng th dng php th gii h trn. V th ta hi vng c th t hai phng
trnh ca h a v dng (x + a)3 = (y + b)3 ( rng x, y c lp vi nhau). Mun vy ta
nhn (2) cho mt s . Cng vic ca ta l tm a, b, (lu phng trnh (1) c bc 3 (cao
nht) nn ta mc nh h s nh c, cc s a, b, s c chn ph hp)
Ly (1) + .(2) ta c:
x3 y 3 35 + (2x2 + 3y 2 4x + 9y) = 0
x3 + 2x2 4x y 3 + 3y 2 + 9y 35 = 0 ()
Ta cn tm a, b, tho:

= 3
a = 2

b = 3

3
3

a b = 35
V T () = (x + a)3 (y + b)3 3a = 2

3a2 = 4
Do () tr thnh: (x 2)3 (y + 3)3 = 0.
~ Li gii: Ta c h phng trnh:

x3 y 3 = 35
x = y + 5

x 2 = y + 3
(y + 5)3 y 3 = 35

x = 2; y = 3
x = 3; y = 2

Vy (I) c nghim (x; y) = (2; 3); (3; 2) 2


~ Nhn xt: Bi ton trn cho ta mt ci nhn tng quan v h s bt nh trong h phng
trnh hu t. y l mt bi c bn v x, y ng c lp. Ta cng xem qua mt bi tng t:
(
Bi 2: (VMO 2010) Gii h phng trnh

x4 y 4 = 240 (1)
x3 2y 3 = 3(x2 4y 2 ) 4(x 8y) (2)

Gii
~ tng: Nh bi trn, do x, y tch bit nn ta hi vng t hai phng trnh ca h a v
dng (x + )4 = (y + )4 . Mun vy ta nhn phng trnh th hai cho mt s k. Cng vic
ca ta l tm cc s , , k.
H phng trnh cho tng ng:
( 4
x + a = y 4 + a + 240
x3 3x2 + 4x = 2y 3 12y 2 + 32y

190
Suy ra:
x4 + a + k(x3 3x2 + 4x) = y 4 + a + 240 + k(2y 3 12y 2 + 32) ()
Cn chn k sao cho :
(x + )4 = (y + )4 x4 + 4x3 + 62 x2 + 43 x + 4 = y 4 + 4y 3 + 6 2 y 2 + 4 3 y + 4 ()
T (*) v (**) ng nht h s ta c:

a = 4

k = 4

k = 8
3k = 6

3
= 2
4k = 4

a = 16
a + 240 = 4

= 4
2k = 4

12k = 6 2

32k = 4 3
~ Li gii:
H cho tng ng:
(

x4 + 16 = y 4 + 256
x3 3x2 + 4x = 2y 3 12y 2 + 32y

Ly phng trnh th hai nhn cho (-8) cng vi phng trnh th nht ta c:
x4 + 16 8(x3 3x2 + 4x) = y 4 + 256 8(2y 3 12y 2 + 32y)
"
"
x2=y4
x=y2
4
4
(x 2) = (y 4)

x2=4y
x=6y
> Nu x = y 2 thay vo (1) ta c:
8y 3 24y 2 + 32y + 224 = 0 (y + 2)(8y 2 40y + 112) = 0 y = 2 x = 4
> Nu x = 6 y thay vo (1) ta c:
y 3 9y 2 + 36y 44 = 0 (y 2)(y 2 7y + 22) = 0 y = 2 x = 4
Vy h phng trnh cho c nghim (x; y) = (4; 2), (4; 2) 2
~ Nhn xt: bi ny hoc cc bi tng t vi s m ln hn hay nh hn nu khng cha
cc hng t c dng xm .y n ta u c th s dng phng php trn. Tuy nhin ch cn qua
mt php i bin h s khng cn p na. Chng ta cng xt bi ton sau:

x2 + y 2 = 1 (1)
5
Bi 3: Gii h phng trnh (II)
57

4x2 + 4x
= y(3x + 1) (2)
25
Gii

191
~ tng: rng 2 phng trnh ca h u c bc hai v xut hin c hng t xy nn
vic dng HSB nh bi 1 s gp nhiu kh khn. Mt hng i thng dng ca ta vi h
loi ny l a v phng trnh bc hai theo (ax + by). lm iu , ta nhn (1) cho , (2)
cho v cng li:
57
1
=0
(1). + (2). (x2 + y 2 ) + (4x2 + 3xy + 3x + y
5
25
4 2 3
57
[(1 +
)x +
xy + y 2 ] + (3x + y)
=0

5
25
xut hin hng t ax + by chnh l (3x + y). Do ta hi vng c
(1 +

4 2 3
)x +
xy + y 2 = k(3x + y)2

rng h s ca y 2 l 1 nn k = 1. Khai trin v ng nht h s ta c

1 + 4 = 9
= 1

= 2

= 6

Vy ta tm c , .
~ Li gii: Ly (1) + (2).2 ta c

7
7
17
119
3x + y = 5
= 0 (3x + y )(3x + y + ) = 0
(3x + y) + 2(3x + y)
17
25
5
5
3x + y =
5
7
> Nu 3x + y = :
5

2
1
x= ; y=

x 2 + y 2 = 1
5
5
17

5
:

> Nu 3x + 7 =
Ta c h phng trnh

y = 3x +
2
11
5
x= ; y=
25
25

1
17

2
2
x + y =
y = 3x
5
5
Ta c h phng trnh

(v nghim)
17
102x 284

y = 3x
10x2 +
+
=0
5
5
25
2 1 11 2
Vy (II) c nghim (x; y) = ( ; ); ( ; ) 2.
5 5 25 25
~ Nhn xt: Nhng h phng trnh cha hng t x2 , xy, y 2 phn ln c th a v phng
trnh bc hai theo ax + by. Bi h trn cn c 2 cch gii khc, l dng php th hoc t n
ph tng - hiu. Ta cng xem qua mt bi tng t:
2

(
Bi 4: Gii h phng trnh

x2 + 2xy + 2y 2 + 3x = 0 (1)
xy + y 2 + 3y + 1 = 0 (2)
Gii

~ tng:
Nh bi trn ta s bin i a v phng trnh bc hai theo mx + ny. lm iu ta
nhn phng trnh th nht vi v phng trnh th cho ri cng li:
(1). + (2). (x2 + 2xy + 2y 2 + 3x) + (xy + y 2 + 3y + 1) = 0

192
[x2 + (

+ 2)xy + ( + 2)y 2 ] + 3(x + y) + = 0

Ta cn chn v sao cho:


x2 + (
x2 + (

+ 2)xy + ( + 2)y 2 = (x + y)

2
+ 2)xy + ( + 2)y 2 = x2 + 2 xy + 2 y 2

ng nht h s ta c:

+2=

+2=

2
= 2 = 2
2

cho n gin chn = 1 v = 2.


~ Li gii:
Ly phng trnh th nht cng vi phng trnh th hai nhn (2) ta c:
x2 + 4xy + 4y 2 + 3x + 6y + 2 = 0
(x + 2y)2 + 3(x + 2y) + 2 = 0 (x + 2y + 1)(x + 2y + 2) = 0
"
x + 2y + 1 = 0

x + 2y + 2 = 0
> Nu x + 2y + 1 = 0 x = 2y 1 thay vo phng trnh th hai ca h ta c:
"

x
=
3

2
2
y
=
1
+
y 2 + 2y + 1 = 0

y = 1 2 x = 3 + 2 2
> Nu x + 2y + 2 = 0 x = 2y 2 thay vo phng trnh th hai ca h ta c:

1+ 5
x = 3 + 5
y=

2
y 2 + y + 1 = 0

1 5
y=
x = 3 5
2

Vy h phng
trnh cho c 4 nghim
(x, y) l (3 2 2; 1 + 2); (3 + 2 2; 1 2);

1+ 5
1 5
(3 + 5;
), (3 5;
)2
2
2
~ Nhn xt: Nhng h phng trnh c cha cc phn t x2 , xy, y 2 phn ln c th a v
phng trnh bc hai theo mx + ny, t c th tnh x theo y hoc y theo x ri th ngc lai
vo mt trong hai phng trnh ca h s ra c nghim.

x4 4x2 + y 2 6y + 9 = 0 (1)
Bi 5: Gii h phng trnh
x2 y + x2 + 2y 22 = 0 (2)
Gii
~ tng: H phng trnh trn c bc kh cao (bc 4) nhng c th gim bc bng cch t
t = x2 . Vy cch t nhin nht chnh l ta a v phng trnh bc 2. m bo chnh

193
phng ta s dng h s bt nh nh sau:
Ta c:
(1) + (2).a x4 + x2 (4 + ay + a) + y 2 6y + 9 + 22ay 22a = 0
Coi y l phng trnh bc 2 theo x2 ta c:
= y 2 (a2 4) + y(16a + 2a2 + 24) + a2 + 80a 20
l mt bnh phng th trc ht h s ca y 2 phi l s chnh phng, ngha l ta gii
PT nghim nguyn a2 4 = k 2

a 4 = 0
Tm c cc nghim ca PT ny, ln lt th li. D thy a = 2 th 16a + 2a2 + 24 = 0

a2 + 80a 20 = 144
Vy ta chn a = 2.
~ Li gii:
Xt (1) + (2).2 ta c:
(1) + (2).2 x4 4x2 + y 2 6y + 9 + 2(x2 y + x2 + 2y 22) = 0 (x2 + y + 5)(x2 + y 7) = 0
> Nu y = x2 5, thay vo (1) ta c phng trnh
x4 4x2 + (x2 + 5)2 + 6(x2 + 5) + 9 = 0 x4 + 6x2 = 32(v nghim)
> Nu y = x2 + 7, thay vo (1) ta c phng trnh

x4 4x2 + (x2 7)2 + 6(x2 7) + 9 = 0 x4 6x2 + 8 = 0 x {2; 2}

T ta tm c nghim ca h l (x; y) = (2; 3), (2; 3), ( 2; 5), ( 2; 5) 2


(
Bi 6: Gii h phng trnh

x3 + 3xy 2 = 49 (1)
x2 8xy + y 2 = 8y 17x (2)
Gii

~ tng: Do phng trnh th nht ca h c bc cao nht nn ta s nguyn, nhn phng


trnh th hai cho ri cng vi phng trnh th nht ta c :
x3 + 3xy 2 + 49 + (x2 8xy + y 2 + 17x 8y) = 0 (1)
Mt khc ta c th nhm c nghim ca h l (x; y) = (1, 4), do ta mong mun (1)
phn tch c thnh:
(x + 1)(ax2 + bx + cy 2 + dy + 49) = 0 (d thy h s ca xy trong ngoc bng 0)
ax3 + bx2 + cxy 2 + dxy + 49x + ax2 + bx + cy 2 + dy + 49 = 0
ax3 + (a + b)x2 + cxy 2 + dxy + cy 2 + (b + 49)x + dy + 49 = 0 (2)

194
T (1) v (2) ng nht h s ta c:

a=1

a+b=

a=1

c==3
c=3

d = 8

b=2

c
=

d = 24

b + 49 = 17

d = 8
~ Li gii:
Nhn phng trnh th hai ca h cho 3 ri cng vi phng trnh th nht c :
x3 + 3xy 2 + 49 + 3x2 24xy + 3y 2 24y + 51x = 0
(x + 1)(x2 + 2x + 3y 2 24y + 49) = 0
(x + 1)((x + 1)2 + 3(y 4)2 ) = 0
"
"
x+1=0
x = 1

x + 1 = 0, y 4 = 0
x = 1, y = 4
> Nu x = 1 thay vo phng trnh th nht ca h c y 2 = 16 y = 4
> Nu x = 1, y = 4 thay vo h thy tha.
Vy h phng trnh cho c nghim (x, y) l (1, 4), (1, 4) 2

Ta cng xem qua mt cch gii khc cho bi ton:


~ tng: rng h c bc ba nhng l bc ba theo x, v ch c bc hai theo y. Vy ta
s ly (1) cng vi (2).a v a v mt phng trnh bc hai theo y:
y 2 (3x + a) + y(8ax 8a) + x3 + ax2 + 17ax + 49 = 0
Ta cn chn a l mt bnh phng ca mt biu thc theo x. C th ta c:
= 3x4 + x3 (4a) + x2 (15a2 51a) + x(15a2 147a) + 16a2 49a
Nu l bnh phng th n s phi c dng 3[f (x)]2 . Mun vy trc ht 16a2 49a phi
l
(3) ln ca mt s chnh phng. n gin ta chn a Z trc, ngha l cn phi gii
phng trnh nghim nguyn
16a2 49a = 3b2
D thy c 16a2 49a 6 0 a(16a 49) 6 0 a {0; 1; 2; 3}
Ln lt th ta thy a = 3 th = 3(x + 1)4
Li rng > 0 x = 1. Vy ta c ngay nghim x = 1 (trng hp tng qut th ta
phi tnh y theo x).
~ Li gii: Xt (1) + 3.(2) ta c phng trnh
y 2 (3x + 3) 24y(x + 1) + x3 + 3x2 + 51x + 49 = 0 ()

195
() c = 3x4 12x3 18x2 12x3 3 = 3(x + 1)4 > 0 x = 1 Vy ta c h phng
trnh

x = 1
x = 1
x = 1

x3 + 3xy 2 = 49
y 2 = 16
y = 4
Do h c nghim (x; y) = (1; 4), (1; 4) 2
~ Nhn xt: Hai cch trn u xt (1) + (2).a v u ra a = 3 tuy tng khc nhau.
Cch 1 rt hiu qu trong nhng bi c th tm c nghim. Cch 2 mang tnh tng qut ,
v tng gii phng trnh nghim nguyn kh th v. Bi ny cn mt cch gii khc bng
phng php t n ph tng - hiu.
Sau y ta s khai thc su v cch 1. Lu rng vi nhng bi cng on c nhiu nghim
th cng d dng h s bt nh.

x4 + 2(3y + 1)x2 + (5y 2 + 4y + 11)x y 2 + 10y + 2 = 0 (1)


Bi 7: Gii h phng trnh
y 3 + (x 2)y + x2 + x + 2 = 0 (2)
Gii
~ tng: gii bi trn ta s tin hnh 4 bc:
Bc 1: tm nghim ca h. Nu bit c nghim th tng ca ta s r rng hn
nhiu. y ln lt th x = 2, 1, 0, 1, 2, 3, ... ta tm c 2 nghim ca h l (x; y) =
(1; 1), (2; 2) 2
Tuy nhin lu rng bc ca hai phng trnh l kh cao (4 v 3). Nu phn tch nh bi 5
th ta vn phi gii quyt mt h phng trnh hai n bc 3 v 2 (sau khi gim bc). V vy
y, do bit 2 nghim, ta s tin hnh bc 2:
Bc 2: tm quan h tuyn tnh gia 2 nghim ny: D thy l y = x.
Bc 3: thay vo h v phn tch thnh nhn t: Ta thay x bi y hoc y bi x (ty
trng hp xem cch no c li), vi bi ny ta thay y = x vo hai phng trnh ca h v
thu c

x4 + 2(3x + 1)x2 + (5x2 4x + 11)x x2 10x + 2 = 0


(x + 1)2 (x 1)(x 2) = 0

x3 (x 2)x + x2 + x + 2 = 0
(x + 1)2 (x 2) = 0
Vic phn tch trn l khng kh v ta bit trc nghim.
Bc 4: La chn biu thc thch hp:
Nh th, so vi phng trnh th nht va nhn c th phng trnh th hai thiu i mt
biu thc l x 1 , nhng ch rng biu thc ny cng tng ng vi y 1. Ta s chn
mt trong hai biu thc ny nhn vo.
R rng nu chn y 1 th vic nhn s to ra mt a thc c cha bin y ng bc vi a
thc phng trnh th nht ban u. Vy ta s nhn phng trnh sau cho y 1.
~ Li gii:
Nhn phng trnh th hai cho y + 1, ri ly phng trnh th nht, tr phng trnh va
nhn c, ta c:
(x + y)(x y + 2)(x2 2x + y 2 + 3y + 5) = 0]

196
- Vi x = y, ta a phng trnh th hai ca h v (y + 2)(y 1)2 = 0
- Vi x = y 2, ta a phng trnh th hai ca h v (y 1)2 (y + 4) = 0
D thy khng xy ra x2 2x + y 2 + 3y + 5 = 0
Th li ta thy h c nghim (x, y) = (1, 1), (2, 2), (6, 4) 2
~ Nhn xt: Vn dng linh hot 4 bc trn c th cho ta li gii rt p v ngn gn. Ta
cng xem qua mt bi tng t:

(x y)2 + x + y = y 2 (1)
Bi 8: Gii h phng trnh
x4 4x2 y + 3x2 = y 2 (2)
Gii
~ tng: u tin ta thy (1) x2 2xy + x + y = 0.
C th tm c 3 nghim nguyn ca
h ny l (x; y) = (0, 0), (2; 2), (1; 2). T 2 nghim u
x2 2x = 0
ta thy x = y. Thay vo h ta c
[x(x 2)]2 = 0
Nh vy ta s phi nhn x(x 2) hoc y(y 2) vo (1). rng nu chn x(x 2) ta s lm
mt bc 4. Vy khu chun b hon tt.
~ Li gii:
Xt x = 0 v x = 2 ta thy h c nghim (x; y) = (0; 0), (2; 2).
Vi x
/ {0; 2} xt (1).[x(x 2)] + (2) ta c phng trnh
x(x 2)(x2 2xy + x + y) + (x4 4x2 y + 3x2 + y 2 ) = 0 (x y)(2x3 x2 + x y) = 0
> Nu x = y thay vo (1) ta c 2x x2 = 0. Trng hp ny loi do x
/ {0; 2}
3
2
> Nu 2x x + x y = 0 ta c h phng trnh

2x3 x2 + x y = 0
()
x2 2xy + x + y = 0
Cng hai phng trnh ta c
2x3 2xy + 2x = 0 2x(x2 + 1 2y) = 0 x2 + 1 = 2y (do x 6= 0)
Thay vo (1) ta c phng trnh
2x3 + x2 + 1 = 0 x = 1
T tm c (x; y) = (1; 2).
Vy h cho c nghim (x; y) = (1; 2), (0; 0), (2; 2) 2
M RNG:
~ tng: Nh cp, nu bit cng nhiu nghim th ta c li gii cng p. Sau y l
mt cch phn tch khi ta bit c 3 nghim. Ta s ln lt lp 3 quan h tuyn tnh gia x v

197

y = x
y. Trong bi ny, l y = 2x

y = 2
Do (2) c bc cao hn nn ta xt
(1). + (2) x4 + x2 (4y + + 3) + x( 2y) + y + y 2 = 0 (3)
Ta s chn quan h no d s dng nht, chnh l y = 2. Nh vy (3) c nghim y = 2,
ngha l
x4 + x2 ( 5) + x(3) + 2 + 4 = 0 =

x4 + 5x2 4
= (x + 1)(x + 2)
x2 3x + 2

Khi (3) tr thnh 2x3 + x2 + x + y = 0


Ta tip tc khai thc mt trong hai quan h cn li, rng nh phn tch trn, vi
= x(x 2) ta thu c 2x3 x2 + x y = 0
Vy ta i n li gii ngn gn nh sau:
~ Li gii:
Th vi x M = {0; 2; 1; 2} ta tm c nghim (x; y) = (0; 0), (2; 2), (1; 2). Xt x
/ M ta
c
(1). + (2) y 2 + y( 2x 4x2 ) + x + x2 ( + 3) + x4 = 0 ()
Chn = x(x 2) ta c 2x3 x2 + x y = 0
Chn = (x + 1)(x + 2) ta c 2x3 + x2 + x + y = 0
Cng li ta c 4x3 + 2x = 0 (sai do x
/ M ). Vy trng hp ny loi.
Tm li h c nghim (x; y) = (0; 0), (2; 2), (1; 2) 2

Bi tp t luyn
Giicc h phng trnh sau:
x3 + y 3 = 9 (1)
1)
x2 + 2y 2 = x + 4y (2)
( 3
x y 3 + 3y 2 + 2x 5y + 3 = 0(1)
2)
x2 + 2y = 1(2)
( 2
x + y2 = 1
3)
21x + 3y + 96x2 + 28xy = 117
( 2
6x y + 2y 3 + 35 = 0
4)
5x2 + 5y 2 + 2xy + 5x + 13y = 0
( 4
x y 4 = 1215
5)
2x3 4y 3 = 9(x2 4y 2 ) 18(x 8y)

PHNG PHP T N PH TNG - HIU


L thuyt
Trong vic gii h phng trnh th t n ph l cch hiu qu v rt a dng. Phng php

198
t n ph tng - hiu l mt cch t d nhn bit, tuy khng phi ti u nhng cng c th
a v mt h phng trnh gii bng phng php th.
Nidung phng php chnh l,t mt h phng trnh theo 2 n x, y ta t
a = mx + ny
x = ma + nb
()
hoc ()
(a, b l n mi) .
b = ux + vy
y = ua + vb
Hai php t trn thc ra tng ng, v ta c th tnh x, y theo a, b v ngc li. Lu rng:
F Php t () c dng vi nhng h m ta c th nhm cc hng t mt cch thch hp
to ra phng trnh n gin hn theo mx + ny v ux + vy.
F Php t () c dng vi nhng h phng trnh m ta hi vng c th kh mt hay mt
s hng t no sau khi khai trin. (lu rng h phng trnh hu t m 2 n tch bit th
c th gii quyt bng h s bt nh).
Khi gp mt bi ton ta nn th () trc,
v php t ny cho h n gin hn.
a = mx + ny
Mt trng hp c bit hn l php t
dng cho h phng trnh c mt
b = nx my
phng trnh i xng, c th a mt s bi v h i xng loi I.
Phng php ny c th gii quyt nhiu h phng trnh hu t, c bit l h bc hai. V t
mt phng trnh bc hai 2 n m1 x2 + m2 xy + m3 y 2 + m4 x + m5 y + m6 = 0 ta c th a v
dng n1 ab + n2 a + n2 b + n3 = 0, t tnh c a theo b.
Sau y l cc v d c th:

Bi tp v d

x2 + y 2 = xy + x + y
Bi 1: Gii h phng trnh
x 2 y 2 = 3
Gii
~ tng: D thy nu t a = x + y, b = x y th t phng trnh sau c ab = 3 nn mc
tiu dng php th thnh cng. Vic cn li l a phng trnh u v bin a, b (d thy
1
x2 xy + y 2 = [3(x + y)2 + (x y)2 ] )
4
~ Li gii:
t a = x + y; b = x y ta c h phng trnh

3a2 + b2 = 4b
a = 3
b = 3
b
27

ab = 3

a = 1
+ b2 = 4b
b2
T tm c (x; y) = (2; 1) 2
2
x + y2 + x = 3
Bi 2: Gii h phng trnh sau
2xy
x2 4y 2 +
= 1
x+y1
Gii

199
KX: x + y 6= 1
Phng trnh (2) tng ng:

(x + y 1) x2 4y 2 + x + y + 2xy + 1 = 0

x3 4y 3 4xy 2 + x2 y x2 + 4y 2 + x + y + 2xy 1 = 0



(x + 2y) x2 xy 2y 2 + xy + x + 2y 2 + 2y x2 xy 2y 2 + y + 1 = 0


(x + 2y) x2 xy 2y 2 + (x + 2y) (y + 1) x2 xy 2y 2 + y + 1 = 0

(x + 2y 1) x2 xy 2y 2 + y + 1 = 0
"
x + 2y = 1

x2 xy 2y 2 + y + 1 = 0
> Nu x + 2y = 1 thay vo phng trnh (1) ta c

x = 1 2y

14
14
1

2
3
+

;x =
3 + 14
y =
x = 1 2y
5
5
y =

5
2

1 + 2 14
3 14

5y 6y 1 = 0

;x =
y=

14

y=
5
5
5
> Nu x2 xy 2y 2 + y + 1 = 0 ta c h phng trnh:

x 2 + y 2 + x = 3
()
x2 xy 2y 2 + y + 1 = 0
Nh vy mu cht ca bi ton li l h phng trnh ()
~ tng: s dng php th, ta phi a mt phng trnh v dng p.ab + q.a + r.b = m
(n a, b). Nh vy cc hng t bc 2 trong phng trnh phi c a v tch ca a v b (
2
a, b l bc nht). Ngha l ta phi phn tch x2 xy
2y thnh nhn t.
a = x + y
D thy x2 xy 2y 2 = (x + y)(x 2y), vy ta t
b = x 2y
~ Li gii:

x = 2a + b
3
t a = x + y; b = x 2y
ab

y =
3
Ta c HPT:

(2a + b)2 + (a b)2 + 3(2a + b) = 27 (1)


3ab + a b + 3 = 0 (2)
b3
. Th vo (1) c phng trnh bc 4 theo b:
3b + 1
9
54b4 + 135b3 648b2 729b = 0 b { ; 1; 0; 3}
2
Nh vy h tng ng

9
3
b = 2 ;a = 5

b = 1; a = 2

b = 0; a = 3

b = 3; a = 0

T (2) c a =

200
11 17
; ); (1; 1); (2; 1); (1; 1);
T tm c nghim (x; y) = (

10 10
1 2 14 3 + 14 1 + 2 14 3 14
;
), (
;
)2
5
5
5
5
~ Nhn xt: Nh vy t mt h khng th dng php th, qua php t n ph a v h
gii bng php th. D phng trnh bc 4 c h s cao nhng tng gii l r rng.
H () cng c th gii bng h s bt nh. C th, ly phng trnh u nhn vi 2 v
cng vi phng trnh sau ta thu c
3x2 x(y 2) + y 5 = 0 (x 1)(3x y + 5) = 0

x2 + y 2 = 2 (1)
5
Bi 3: Gii h phng trnh
57

4x2 + 3x
= y(3x + 1) (2)
25
Gii
~ tng: Xt cc hng t bc 2 c hai phng trnh, ta thy khng th phn tch thnh
nhn t. Tuy nhin lu rng ta c th thay i h s ca x2 v y 2 (2) bng php th (1).
Nh vy ta hng ti cch t (), nhng y kh khn hn v ta khng bit biu thc cn
phn tch l g. Tip tc khai thc bi ton:
I Ta s th (1) vo (2) bin i biu thc bc 2. Ngha l ta xt:
(1). + (2) x2 (4 + ) + 3xy + y 2 + 3x + y =

57
+
25 5

Mc ch ca ta l phn tch f (x) = x2 (4 + ) + 3xy + y 2 thnh nhn t. Mun vy th


ca f (x) phi l mt bnh phng. Ta xt
= 9y 2 4(4 + )y 2 = y 2 (9 16 42 )
Nh vy ta phi tm 9 16 42 l s chnh phng, ni cch khc l gii phng trnh
nghim nguyn
9 16 42 = k 2
T tm c {4; 2; 0} (tht ra ch cn tm nghim nguyn ca 9 16 42 > 0 ri
th li). D thy ch nhn = 2. Khi ta c phn tch 2x2 + 3xy 2y 2 = (2x y)(x + 2y).
Vy t a = 2x y; b = x + 2y.
~ Li gii:
Ly (2) 2.(1) ta c h tng ng

x 2 + y 2 = 2
5
47

2
2x + 3xy 2y 2 + 3x + y =
25
t a = x + 2y, b = 2x y ta c h i xng :
3
a = ;b =

47
4
a + b + ab
=0
a = ;b =
25
5

a2 + b2 = 1

4
5
3
5

201
2 1 11 2
Vy ta tm c (x; y) = ( ; ); ( ; ) 2
5 5 25 25
~ Nhn xt: Bi ton trn cng c th gii bng h s bt nh. C th ta xt
7 17
(1).25 + (2).50 25(3x + y)2 + 50(3x + y) 119 = 0 3x + y { ;
}
5 5
T d dng lm tip.

x3 + 3xy 2 = 49
Bi 4: Gii h phng trnh
x2 8xy + y 2 = 8y 17x
Gii
~ tng: Th xt hng t bc 2 trc, ta thy khng th phn tch x2 8xy+y 2 thnh nhn t
v cng khng c cch no bin i biu thc ny. n hng t bc 3, d x3 +3xy 2 = x(x2 +3y 2 )
nhng khng d a v bc 3 theo a, b vi php t (). Vy ta chuyn qua php t ().
Trc tin ta th cch t x = ua + vb; y = va ub v mc ch ca ta chnh l kh ab, a2 b, ab2
sau khi a v h bc 3 theo a, b.
Gi ta
( s tm u, v:
x = ua + vb
t
thay vo h ta c:
y = va ub
(

a3 (u3 + 3uv 2 ) + b3 (v 3 + 3u2 v) + a2 b(3v 3 3u2 v) + ab2 (3u3 3uv 2 ) = 49


a2 (u2 8uv + v 2 ) + b2 (v 2 + 8uv + u2 ) + 8ab(u2 v 2 ) = 9ua 25vb

Ta s ng nht h s sao cho h s ca a2 b, ab2 v ab bng 0 :

"
3v 3 3u2 v = 0

u=v
3u3 3uv 2 = 0

u = v

u2 v 2 = 0
Do u = v hay u = v u c cng cch t nn ta chn u = v = 1 (tht ra c th ly u, v tu
, v v tri ca phng trnh u l thun nht).
~ Li gii:
t x = a + b, y = a b ta c h:
( 3
( 3
4a + 4b3 = 49
8a + 8b3 = 98

6a2 + 10b2 = 9a 25b


6a2 9a = 10b2 + 25b
Ti y c th gii tip bng h s bt nh. C th, ly phng trnh th nht cng phng
trnh th hai nhn (-6) ta c
(2a 3)3 = (2b 5)3 2a 3 = 2b 5 a = b 1
5 3 3 5
; ); ( ;
)
2 2 2 2
Nh vy h c nghim (x; y) = (1; 4), (1; 4) 2
~ Nhn xt: T bi trn c th rt ra mt kinh nghim nh l nu trong mt phng trnh c
T tm c (a; b) = (

202
mx3 v 3mxy 2 , hoc my 3 v 3mx2 y th ta c th t x = a + b; y = a b.Sau y ta xt mt
bi tng t:

6x2 y + 2y 3 + 35 = 0 (1)
Bi 5: Gii h phng trnh
5x2 + 5y 2 + 2xy + 5x + 13y = 0 (2)
Gii
~ tng: Theo nhn xt bi 4, do phng trnh u c 2y 3 +6x2 y nn ta t x = a+b; y = ab.
~ Li gii:
t x = a + b; y = a b ta c (1) tr thnh:


6(a + b)2 (a b) + 2(a b)3 + 35 = 0 6 a2 + 2ab + b2 (a b) + 2 a3 3a2 b + 3ab2 b3 + 35 = 0

35
6 a3 a2 b + 2a2 b 2ab2 + ab2 b3 + 2a3 6a2 b + 6ab2 2b3 + 35 = 0 a3 b3 +
=0
8
Tng t, (2) tr thnh: 6a2 + 9a + 4b2 4b = 0
Vy ta c h phng trnh

a3 b3 = 35
8
6a2 + 9a + 4b2 4b = 0
3 3
Gii h ny bng h s bt nh, ta tm c (a; b) = (1; ); ( ; 1)
2
2
1 5 1 5
); ( ;
)2
Vy ta c (x; y) = ( ;
2 2
2 2
~ Nhn xt: Nu bit nghim ca h, ta cng c th trnh by ngn gn hn bng h s bt
nh nh sau:
1
5
(1)+3.(2) (6y+15)x2 +3(2y+5)x+2y 3 +15y 2 +39y+35 = 0 (2y+5)[3(x+ )2 +(y+ )2 ] = 0
2
2
(
Bi 6: Gii h phng trnh

x4 2x = y 4 y
3

(x2 y 2 ) = 3

( kim tra i d tuyn trng THPT Chuyn HSP H Ni)


Gii
t x + y = a, x y = b, 3 = c3 .
T phng trnh th hai ca h, ta c: (ab)3 = c3 ab = c.
a+b
ab
Ta c: x =
,y =
. Suy ra:
2
2
"
2 
2 #
a
+
b
a

b
ab
x4 y 4 = (x y)(x + y)(x2 + y 2 ) = ab
+
= (a2 + b2 )
2
2
2
Li c na:
a + 3b
a + c3 b
(a b)
2x y = (a + b)
=
=
2
2
2

203
Do , phng trnh th nht ca h cho tng ng vi:
ab 2
a + c3 b
(a + b2 ) =
c(a2 + b2 ) = a + c3 b
2
2
Ta c h mi l:
( 2
c(a + b2 ) = a + c3 b

c(a2 +

ab = c

c2
c4
ca4 + c3 = a3 + ac4
)
=
a
+
a2
a

(ca 1)(a3 c3 ) = 0 a =

1
a=c
c

1
Suy ra h ny c hai nghim l: (a, b) = (c, 1); ( , c2 ).
c
Xt hai trng hp:

3
3
3+1
31
c+1
> Nu a = c, b = 1 th x =
=
,y =
.
2
2
2



3
1+c
1 c3
1 1
2
1
1
1 1
2
2
2
=
+c =
=

c
=
> Nu a = , b = c th x =
,
y
=
3
3
c
2 c
2c
2c
3 ! 2 c
3



3
3
3+1
31
2 1
,
2.
Vy h cho c hai nghim l: (x, y) =
,
,
3
2
2
3 33

Bi tp t luyn
Gii
h phng trnh:
x 2 y 2 = y
1)
x2 + xy + x = 6

=7
4(x2 + y 2 ) + 4xy +
2
(x
+
y)
2)
1

2x +
=3
x+y

= 2(10 xy)
3(x2 + y 2 ) +
2
(x

y)
3)
1

2x +
=5
xy

x2 + 2x + 6 = y + 1
4)
x2 + xy + y 2 = 7

x2 y 2 = 4x + 6y 1
5)
x4 + y 4 5x2 5y 2 = 2x2 y 2 10xy 1

x 2 + y 2 = 1
6)
21x + 3y + 48x2 48y 2 + 28xy = 69

PHNG PHP DNG BT NG THC


V tng, dng bt ng thc trong phng trnh v h phng trnh l tng t. Nhng
trong nhiu bi h, vic nh gi cc n s phc tp hn nhiu. Ta cng xem qua mt s bi

204
tp:

Bi tp v d
(
Bi 1: Gii h phng trnh sau:

x6 + y 8 + z 10 = 1 (1)
x2007 + y 2009 + z 2011 = 1 (2)
Gii

T (1) ta c: 1 6 x, y, z 6 1
T (1) v (2) ta c:
x2007 + y 2009 + z 2011 > x6 + y 8 + z 10



x6 1 x2001 + y 8 1 y 2001 + z 10 1 z 2001 6 0 (3)
T 1 6 x, y, z 6 1 ta thy:



x6 1 x2001 , y 8 1 y 2001 , z 10 1 z 2001 > 0
Do :



(3) x6 1 x2001 = y 8 1 y 2001 = z 10 1 z 2001 = 0 x, y, z = 1 x, y, z = 0
Kt hp vi (1)suy ra h c cc nghim l: (x; y; z) = (1; 0; 0) , (0; 1; 0) , (0; 1; 0) , (0; 0; 1) 2

Bi 2: Gii h phng trnh

3(a + b) = 2 |ab + 1|
9(a3 + b3 ) = |a3 b3 + 1|
Gii

Ta c:
3 3






a b + 1 = (ab + 1)(a2 b2 ab + 1) = |(ab + 1)| (a2 b2 ab + 1) = |(ab + 1)| (ab + 1)2 3ab

3
9
9
|(ab + 1)| (ab + 1)2 3ab = (a + b)[ (a + b)2 3ab] = (a + b)(3a2 + 2ab + 3b2 )
2
4
8
9
Li c: 5(a + b)(a b)2 > 0 9(a3 + b3 ) > (a + b)(3a2 + 2ab + 3b2 ) 9(a3 + b3 ) > |a3 b3 + 1|
8
ng thc xy ra nn a = b.

3+ 5
Vy h c nghim duy nht a = b =
2.
2

a + b = 3 24

Bi 3: Gii h phng trnh

1
1
( a + b)(
+
)=2
a + 3b
b + 3a
Gii
p dng bt ng thc AM-GM ta c:

a 6 1( a + a + b )

2 a + b a + 3b
a+ 3b
b
1 1
2b

6 ( +
)
2 2 a + 3b
a + 3b

a+ b
1 a
3

6 (
+ )
2 a+b 2
a + 3b

205

a+ b
1 b
3
Chng minh tng t ta cng c:
6 (
+ )
2 a+b 2
b + 3a
Cng lai ta c

1
1
( a + b)(
+
)=2
a + 3b
b + 3a
ng thc xy ra nn a = b.

Vy h c nghim duy nht a = b =

24
2.
2

x 4 + y 4 = 2
Bi 4: Gii h phng trnh:
x3 2x2 + 2x = y 2
Gii
Vit li h cho di dng:

x 4 + y 4 = 2
x3 2x2 + 2x 1 = y 2 1

x 4 + y 4 = 2

(x 1)(x2 x + 1) = y 2 1

Xt cc kh nng sau:
> Nu x > 1 (x 1)(x2 x + 1) > 0 y 2 > 1 y 4 > 1 x4 + y 4 > 2
Khi y h cho v nghm.
> Nu 0 < x < 1 y 2 < 1 y 4 < 1
Trong trng hp ny h cng v nghim.
2
2
> Nu x < 0 th x3 2x
+ 2x < 0 y < 0 : v l.
y 4 = 2
> Ti x = 0, ta c h
(v nghim)
y 2 = 0

y 4 = 1
> Ti x = 1, h tr thnh:
y = 1
y 2 = 1
Vy h cho c nghim (x; y) = (1; 1), (1; 1) 2
Bi 5: Gii h phng trnh

4
x +
32 x y 2 = 3

x + 32 x + 6y = 24
Gii
iu kin 0 6 x 6 32.
Cng hai phng trnh v theo v:

( x + 32 x) + ( 4 x + 4 32 x) = y 2 6y + 21 (1)
Li c:

y 2 6y + 21 = (y 3)2 + 12 > 12 (2)

x + 32 x 6 2(x + 32 x) = 8
q

4
4

x + 32 x 6 2( x + 32 x) 6 2.8 = 4

206

T suy ra ( x + 32 x) + ( 4 x + 4 32 x) 6 12 (3)


Kt hp (1),(2) v (3) ta c ( x+ 32 x)+( 4 x+ 4 32 x) = y 2 6y+21 = 12

x = 16
y = 3

Vy h cho c nghim x = 16, y = 3 2.

2xy

= x2 + y (1)
x +
3
2
x 2x + 9
Bi 6: Gii h phng trnh
2xy

= y 2 + x (2)
y + p
3
2
y 2y + 9
Gii
~ tng: y l h phng trnh i xng loi II, nhng nu lm theo cch thng thng
th s rt kh khn do c s xut hin ca cn bc ba. rng khi ta cng 2 phng trnh
li hng t (x + y) mi v s c n gin, khi VT xut hin 2xy, cn VP xut hin
x2 + y 2 , v do y l h phng trnh i xng loi II nn s c nghim x = y. T ta ngh
ti vic nh gi 2xy v x2 + y 2 .
~ Li gii:
Vi x = 0 y = 0. Xt x, y 6= 0 :
Cng (1) v (2) v theo v:
x + y + 2xy(
3

2xy(
3

x2

1
1
+p
) = x2 + y 2 + x + y
3
2
2x + 9
y 2y + 9

1
1
) = x2 + y 2 (3)
+p
3
2
2
x 2x + 9
y 2y + 9

Suy ra 2xy > 0. Mt khc ta c:

1
1
1
1

= q
6
=

3
3

2
3
8
x2 2x + 9
(x 1)2 + 8
1
1
1
1

p
q

=
6
=

2
3
3 y 2 2y + 9
8
(y 1)2 + 8

3

x2

1
1
1
1
+p
6 1 2xy(
+p
) 6 2xy 6 x2 +y 2 (4)
3
3
3
2
2
2
2x + 9
x 2x + 9
y 2y + 9
y 2y + 9

T (3) v (4) suy ra x = y, thay vo (1) c:


2x2
2
= x2 + x x2 (
1) = 0
3
2
2
x 2x + 9
x 2x + 9

2
3

1 2 = x2 2x + 9
3
2
x 2x + 9
8 = x2 2x + 9 (x 1)2 = 0 x = 1

x+
3

207
Th li cc nghim (0;0), (1;1) u tha.
Vy h phng trnh c nghim (x; y) l (0;0), (1; 1) 2.
( p
3 + 2x2 y x4 y 2 + x4 (1 2x2 ) = y 4
p
Bi 7: Gii h phng trnh
1 + 1 + (x y)2 = x3 (x3 x + 2y 2 )
Gii
~ Nhn thy cc biu thc trong cn rt phc tp. Nhng phng trnh sau gi cho ta
a v cc bnh phng, t hi vng c th nh gi bt ng thc mt cch hp l.
~ Li gii:
p
p
p
Nhn thy 3 + 2x2 y x4 y 2 = 4 (1 x2 y)2 6 2 v 1 + 1 + (x y)2 > 2
y 4 + 2x6 x4 6 x6 x4 + 2x3 y 2 (x3 y 2 )2 6 0

1x y =0
x=y=1
Suy ra x3 = y 2
xy =0

3
2
x =y
Th li ta thy h c nghim (x; y) = (1; 1) 2
(
Bi 8: Gii h phng trnh

y 3 x3 = 7
x3 y 2 + x = 2
Gii

H phng trnh tng ng


( 3
( 3
( 3
y 8 = x3 1
y 8 = x3 1
y 8 = x3 1 (1)

(y 3 7) y 2 + x = 2
y3 y2 4 = 1 x
(y 2)(y 2 + y + 2) = 1 x (2)
1 2 7
Ta c y 2 + y + 2 = (y + ) + > 0, y R.
2
4
Xt cc trng (
hp:
(1) x > 1
>y > 2 : Ta c
H v nghim
(2) x < 1
(
(1) x < 1
>y < 2 : Ta c
H v nghim
(2) x > 1
>y = 2 : x = 1
Vy h phng trnh c nghim (1;2) 2.
~ Nhn xt: Mu cht ca li gii trn chnh l on c nghim (x; y) = (1; 2), t c
nhng bin i ng thc hp l chng minh y l nghim duy nht. Ta cng xem qua mt
bi tng t:
(
Bi 9: Gii h phng trnh

y = x3 + 3x + 4
x = 2y 3 6y 2
Gii

()

208
~ tng: d dng nhm c nghim ca h phng trnh l x = y = 2.
Ta hi vng
( y l nghim duy nht ca h, v th ta s c gng a h phng trnh ban u
y 2 = (x 2).f (x)
v dng:
x 2 = (y 2).g(y)
Ri t hai phng trnh trn da vo du ca f (x) v g(y) ta s nh gi gia x v y, v
lm c iu ny ta s chia trng hp xt.
~ Li gii:
( 3
(
x 3x 2 = 2 y
(x 2)(x + 1)2 = 2 y (1)
H (*)

2y 3 6y 4 = x 2
2(y 2)(y + 1)2 = x 2 (2)
> Nu x > 2 :
(
(1) y < 2
VN
(2) y > 2
>:Nu x < 2:

(1) y > 2
(2) y < 2

VN

x = 2, t suy ra y = 2
Th li ta thy x = y = 2 tha h.
Vy h phng trnh c nghim (2;2) 2.

(
Bi 10: Gii h phng trnh

x5 + y 5 + z 5 = 3
x6 + y 6 + z 6 = 3
Gii

~ tng: t hai phng trnh trn ta suy ra x6 + y 6 + z 6 = x5 + y 5 + z 5 .


Ta ngh ti vic nh gi gia x6 + y 6 + z 6 v x5 + y 5 + z 5 , c th l ta s i chng minh
x6 + y 6 + z 6 > x5 + y 5 + z 5 .
D nhin chng minh c bt ng thc ny ta phi ngh ti vic s dng cng c mnh
l bt ng thc Holder.
~ Li gii:
T gi thit ta suy ra x6 + y 6 + z 6 = x5 + y 5 + z 5
Ap dng bt ng thc Holder cho 6 b ba s dng c :
6

(x6 + y 6 + z 6 )5 .(1 + 1 + 1) > (x5 + y 5 + z 5 ) x6 + y 6 + z 6 > x5 + y 5 + z 5


ng thc xy ra khi x6 = y 6 = z 6 , kt hp vi gi thit d dng suy ra x = y = z = 1.
Th li ta thy h phng trnh cho c nghim (1; 1; 1) 2

x2 y 2 2x + y 2 = 0 (1)
Bi 11: Gii h phng trnh
7x2 14x + 3y 3 + 10 = 0 (2)

209
Gii
T (1) suy ra:
y2 =

x > 0

2x
61
1 6 y 6 1
+1

y 3 > 1 ()

x2

T (2) suy ra 7(x 1)2 + 3(y 3 + 1) = 0


M 7(x 1)2 > 0; 3(y(3 + 1) > 0 (do (*))
7(x 1)2 + 3(y 3 + 1) > 0
(
(x 1)2 = 0
x=1
ng thc xy ra

y3 + 1 = 0
y = 1
Th li ta thy h c nghim (x; y) = (1; 1) 2
(
Bi 12: Gii h phng trnh ()

x2 + y 2 + z 2 + 2xy xz yz = 3
x2 + y 2 + yz zx 2xy = 1
Gii

~ tng: Ta thy rng phng trnh th nht c th phn tch thnh mt bnh phng
ca mt tng theo x, y, z v cn d mt lng theo z 2 , phng trnh th hai cng c th phn
tch thnh bnh phng ca mt tng c cha x, y, z nhng li thiu mt lng theo z 2 .
T ta ngh ti vic nh gi z 2 t mi phng trnh.
~ Li gii:
Vit li h (*) di dng

z 2 3z 2

( 2
2

2
2

(x
+
y)

z(x
+
y)
+
+
=3
(x + y + 2xy) z(x + y) + z = 3
4
4

z2 z2
(x2 + y 2 2xy) z(x y) = 1

(x y)2 z(x y) +

= 1
4
4

z 2
z2
z2

(x + y ) = 3(1 )
1
>0
2
4 ()
4

z 2 z2
z2

(x y ) =
1>0
1
2
4
4

"
z 2 6 4
z=2

z2 = 4
z 2 > 4
z = 2
(
> z = 2, ()
> z = 2, ()

(x + y 1)2 = 0
2

(x y 1) = 0
(
(x + y + 1)2 = 0

x+y1=0

xy1=0
(
x+y+1=0

x=1

y=0
(
x = 1

(x y + 1)2 = 0
xy+1=0
y=0
Th li ta thy h c nghim (x; y; z) = (1; 0; 2), (1; 0; 2) 2.
(
Bi 13: Gii h phng trnh

2x + 4y = 32
xy = 8
Gii

210
~ tng: Ta thy phng trnh th nht ca h c dng tng, phng trnh th hai c dng
tch nn ngh ti vic s dng bt ng thc AM-GM nh gi.
~ Li gii:
Do xy = 8 > 0 x, y cng du
Nu x < 0, y < 0 V T < 2 < V P phng trnh v nghim.
Nu x > 0, y > 0:
p dng bt ng thc AM-GM c:
p
p

2x + 4y = 2x + 22y > 2 2x 22y = 2 2x+2y > 2 22 2xy = 2 22 2.8 = 32

(
2x = 22y

x=4
ng thc xy ra x = 2y

y=2

xy = 8
Th li ta thy nghim (4;2) tha h.
Vy h phng trnh c nghim (4;2) 2

2x2

=y

2+1

2y 2
Bi 14: Gii h phng trnh
=z

y2 + 1

2z = x
z2 + 1
(Thi v ch ton Bungari 1977)
Gii
Ta thy
x = y = z = 0 l mt nghim ca h.
x 6= 0

Nu
y 6= 0 th x, y, z > 0. Khi , nhn 3 v ca h phng trnh ta c
z 6= 0
8x2 y 2 z 2
= xyz (x2 + 1)(y 2 + 1)(z 2 + 1) = 8xyz
(z 2 + 1)(y 2 + 1)(z 2 + 1)
Mt khc p dng bt ng thc AM-GM c:
p

(x2 + 1)(y 2 + 1)(z 2 + 1) > 2 x2 .2 y 2 .2 z 2 = 8 |xyz| = 8xyz (do xyz > 0)

x, y, z > 0
ng thc xy ra
x=y=z=1
x 2 = y 2 = z 2 = 1
Th li ta thy x = y = z = 1 tha h.
Vy h phng trnh c nghim (0;0;0), (1;1;1) 2.
~ Nhn xt:

2t2
1) Bi h trn cng c th gii bng hm n iu. C th xt hm f (t) = 2
, ta c
t +1
4t
f 0 (t) = 2
nn f (t) ng bin trn (0; +).
(t + 1)2

211
D thy x, y, z > 0 do khng gim tng qut gi s x = max{x; y; z} th
f (x) > f (y) y > z f (y) > f (z) z > x x = y = z.
Ta cng xem qua bi ton tng qut hn:

2x2

=y

x + 13
3y
Bi 15: Gii h phng trnh sau:
=z
4

y + y2 + 1

4z 4

=x
z6 + z4 + z2 + 1
Gii
R rng nu mt trong s cc n x, y, z bng 0 th cc x = y = z = 0.
Nh vy x = y = z = 0 l nghim ca h cho.
Nu x, y, z 6= 0 x, y, z > 0
Ta c:
2x2
2x
61y= 2
6 x (1)
x2 + 1 > 2x 2
x +1
x +1
y 4 + y 2 + 1 > 3y
z 6 + z 4 + z 2 + 1 > 4z 3

3y 3
3y 2
6
1

z
=
6 y (2)
y4 + y2 + 1
y4 + y2 + 1

4z 3
4z 4
6
1

x
=
6 z (3)
z6 + z4 + z2 + 1
z6 + z4 + z2 + 1

T (1),(2),(3) ta suy ra x = y = z = 1.
Vy h cho c nghim x = y = z = 0 hoc x = y = z = 1 2.
(
Bi 16: Gii h phng trnh

x4 + y 2 = 9
x2 + y 2 + xy 3x 4y + 4 = 0
Gii

Gi s h phng trnh c nghim (x, y)


Vit li phng trnh th hai theo x:
x2 + x(y 3) + (y 2)2 = 0
phng trnh c nghim x th
x = (y 3)2 4(y 2)2 > 0 3y 2 + 10y 7 > 0 1 6 y 6

7
(1)
3

Vit li phng trnh th hai theo y:


y 2 + y(x 4) + x2 3x + 4 = 0
phng trnh c nghim y th:
y = (x 4)2 4(x2 3x + 4) > 0 3x2 + 4x > 0 0 6 x 6

4
(2)
3

212
 4  2
7
697
4
+
=
< 9 (mu thun)
T (1) v (2) x + y 6
3
3
81
Vy h phng trnh cho v nghim 2
4

Bi 17: Gii h phng trnh:

x + y + z = 2010
1
1
1
1
1
1
+
+
=
+
+
3x + 2y 3y + 2z 3z + 2x
x + 2y + 2z y + 2z + 2x z + 2x + 2y
Gii
~ tng: Do s n v phng trnh khng bng nhau nn ta s dng bt ng thc. Kh c

th so snh x + y + z vi 2010, nn ta s khai thc phng trnh sau.


~ Li gii:
KX: x, y, z khng m v khng ng thi bng 0.
Theo BT Cauchy-Shwarz ta c:
1
2(2x + 2y + z)
2(2x + 2y + z)
1
+
=
>
3x + 2y x + 2y + 2z
(3x + 2y)(x + 2y + 2z)
(2x + 2y + z)2

1
1
2
+
>
3x + 2y x + 2y + 2z
2x + 2y + z

Tng t ta cng c:
1
1
2
+
>
3y + 2x y + 2x + 2z
x + 2y + 2z
1
1
2

+
>
3z + 2x z + 2x + 2y
y + 2x + 2z

Suy ra:
1
1
1
1
1
1
+
+
>
+
+
3x + 2y 3y + 2x 3z + 2x
2x + 2y + z x + 2y + 2z y + 2x + 2z
Du "=" xy ra x = y = z.

670
Thay vo h ta c: 3 x = 2010 x =
3
670 670 670
Vy h phng trnh c nghim duy nht (x; y; z) = (
,
,
). 2
3
3
3
Bi 18: Gii h phng trnh:

a2 + 1

x+a+ y+b+ z+a=3


r a
2

ax+ ay+ az =3 a 1
a
Vi a>1 v a l hng s.

Gii

213

x+a+ y+b+ z+
a, B = a x + a y + a z
A2 6 3(3a + x + y + z)
p dng BT Cauchy-Schwarz:
B 2 6 3(3a x y z)

t A =

Cng v theo v ta c: A2 + B 2 6 18a ()


Mt khc theo gi thit ta li c:
A2 + B 2 = 9(

a2 + 1 a2 1
+
) = 18a
a
a

Vydu "=" trong (*) xy ra


r

a2 + 1

a+x= a+y = a+z =


1
r a

x=y=z=
2

a 1

ax= ay = az =
a
1 1 1
Vy h phng trnh c nghim duy nht (x, y, z)=( , , ). 2
a a a

2011

1 + x1 + 1 + x2 + ... + 1 + x2010 = 2010


r 2010
Bi 19: Gii h phng trnh

2009

1 x1 + 1 x2 + ... + 1 x2010 = 2010


2010
Gii
iu kin: -1 6 xi 6 1 (i= 1, 2, 3, ..., 2010). Ta c:

2011
20102 .
= ( 1 + x1 + 1 + x2 + ... + 1 + x2010 )2 6 2010(2010 + x1 + x2 ... + x2010 )
2010
Suy ra
x1 + x2 + ... + x2010 > 1(1)
Li c:

2011
20102 .
= ( 1 x1 + 1 x2 + ... + 1 x2010 )2 6 2010(2010 x1 x2 ... x2010 )
2010
nn x1 + x2 + ... + x2010 6 1(2)
T (1) v (2) suy ra x1 + x2 + ... + x2010 = 1
Do
h phng trnh cho tr thnh:

1 + x1 = 1 + x2 = ... = 1 + x2010
1
1 + x1 = 1 + x2 = ... = 1 + x2010 x1 = x2 = ... = x2010 =

2010

x1 + x2 + ... + x2010 = 1
1
Vy h phng trnh c nghim xi =
(i = 1, 2010) 2
2010

1
1
1

+ + =3 3

y
z
x
Bi 20: Gii h phng trnh: x + y + z = 1

xy + yz + zx = 7 + 2xyz()
27
Gii
~ tng: Bi ny c th gii bng cch bnh thng vn cho ta kt qu,nhng phng
trnh (*) chnh l BT IMO 1984. Nh th h c lin quan n BT. Phng php BT s

214
cho ta li gii p v gn hn trong bi ton ny.
Ta c:
() xy + yz + zx 2xyz =

7
27

Li c:
xyz > (x + y z)(y + z x)(z + x y) = (1 2x)(1 2y)(1 2z)
9xyz > 4(xy + yz + zx) 1 2xyz >

8(xy + yz + zx) 2
9

(a + b + c)2
1
(ng)
Ta phi chng minh xy + yz + zx 6 =
3
3
1
Du "=" xy ra x=y=z=
3
Th li vo h phng trnh thy tha mn.
1 1 1
Vy h phng trnh c nghim duy nht (x, y, z)=( , , ). 2
3 3 3

xy(x + y) = x2 xy + y 2
Bi 21: Gii h phng trnh
1 + 1 = 16
x3 y 3
Gii
iu kin x, y 6= 0.
1
1
t a = v b =
x
y
1 1 1
1
1
1
T gi thit ta suy ra ( + ) = 2
+ 2 a + b = a2 ab + b2
ab a b
a
ab b
1
1
3
3
2
3 + 3 = a + b = (a + b)(a ab + b2 ) = (a + b)2
x
y
(a + b)2
(a + b)2
Ta c bt ng thc sau: a2 ab + b2 >
a+b>
06a+b64
4
4
1
1
3 + 3 = (a + b)2 6 16
x
y
1
ng thc xy ra nn a = b x = y = .
2
1
Vy h c nghim duy nht x = y = 2.
2
2009
X

xi = 2009

i=1
Bi 22: Gii h phng trnh
2009
2009

X
X

xi =
x6i

i=1

i=1

Gii
Gi s (x1 , x2 , . . . , x2009 ) l mt nghim ca h.
Khng gim tng qut gi s x21 > x22 > ... > x22009 .
p dng bt ng thc BCS ta c
2009
X
2009
x2i >

2009
X

i=1

i=1

!2
xi

2009
X
i=1

x2i > 2009 (1)

215
p dng bt ng thc Chebysev cho cc b s (x21 , x22 , . . . , x22009 ) v (x61 , x62 , . . . , x62009 ) c
sp th t ta c
! 2009 !
2009
2009
X
X
X
2
6
xi
xi 6 2009
x8i (2)
i=1

i=1

i=1

T (1) v (2) ta suy ra


2009
X
i=1

x8i

>

2009
X

x6i (3)

i=1

ng thc xy ra x1 = x2 = ... = x2009 = 1


Th li ta thy xi = 2009 (i = 1, 2009) l nghim duy nht ca h phng trnh 2

Bi tp t luyn
(
Bi 1: Gii h phng trnh

2x2 4x + 3 + y 3 = 0
(

Bi 2: Gii h phng trnh


Bi 3: Gii h phng trnh

x2 y 2 2x + y 2 = 0
xy 3 = 9

x + 3y = 6
(

x+ y+ z =3
(1 + x)(1 + y)(1 + z) = (1 +
(

Bi 4: Gii h phng trnh

Bi

Bi

Bi

Bi

Bi

Bi

xyz)3

3(x2 + y 2 + z 2 ) = 1

x2 y 2 + y 2 z 2 + z 2 x2 = xyz(x + y + z)3

1
1
1

+ + =3 3
y
x
z
5: Gii h phng trnh

x + y + z = 1
( 7
y + 1 = (x + 1)(x2 + 1)(x4 + 1)
6: Gii h phng trnh
x7 + 1 = (y + 1)(y 2 + 1)(y 4 + 1)
y

30 2 + 4y = 2007

xz
7: Gii h phng trnh 30 2 + 4z = 2007
y

30 x + 4x = 2007
z2
2x2

=y

2+1

3y 3

=z
4
y + y2 + 1
8: Gii h phng trnh

4z 4

=t

z6 + z4 + z2 + 1

5t5

8
=x
6
4
2
t +t +t +t +1

x3 + y = 3x + 4

9: Gii h phng trnh 2y 3 + z = 6y + 6

3z 3 + x = 9z + 8
( 2
x + y2 = 1
10: Gii h phng trnh

125y 5 125y 3 + 6 15 = 0

216

TNG HP CC BI H PHNG TRNH


H phng trnh hu t
(
Bi 1: Gii h phng trnh

x2 4xy + x + 2y = 0
x4 8x2 y + 3x2 + 4y 2 = 0
Gii

Nhn thy phng trnh (1) c hng t x2 +2y v phng trnh (2) li c hng t x4 +4y 2 ,Nn
ta ngh n vic phn tch c 2 phng trnh u c hng t x2 + 2y
H phng trnh tng ng
( 2
( 2
x + 2y + x 4xy = 0
(x + 2y) + x(1 4y) = 0

2
x4 + 4x2 y + 4y 2 + 3x2 12x2 y = 0
(x2 + 2y) + 3x2 (1 4y) = 0
t x2 + 2y = a v 1 4y = b ta c HPT:

x=0

x2 b2 + 3x2 b = 0 x2 b(b + 3) = 0
b = 0
2
2
a + 3x b = 0
b = 3
a + xb = 0

> Vi x = 0 y = 0
1
1
> Vi b = 0 y = x2 x + x + = 0 (V nghim)
4
2
> Vi b = 3 y = 1 x2 4x + x + 2 = 0 x2 3x + 2 = 0

"

x=2
x=1

Kt lun: H c nghim (x; y) l (0; 0); (2; 1); (1; 1) 2


(
Bi 2: Gii h phng trnh sau

81x3 y 2 81x2 y 2 + 33xy 2 29y 2 = 4


25y 3 + 9x2 y 3 6xy 3 4y 2 = 24
Gii

Nhn thy VT(1) c nhn t y 2 chung.V th chia c hai v phng trnh (1) cho y 2 (y 6= 0)
ri bin i phng trnh (1) v dng f (t) = 0.T ta s "g" phng trnh (2) v dng
f (t0 ) = 0.
Nhn thy (0; 0) khng phi l nghim

 2

2
4
3

3
2

81x 81x + 33x 29 = 2


3(3x 1) + 2(3x 1) = 24 + y
y
H tng ng vi

 3
24 4

2
2
2

3 + = 25 + 9x 6x
+ 2. = 24 + (3x 1)2
3
y
y
y
y

217
2
t 3x 1 = a v = b, ta c h i xng loi 2:
y

3a3 + 2a = 24 + b2
3b3 + 2b = 24 + a2

Tr v theo v ca h ta c
3(a b)(a2 + ab + b2 ) + 2(a b) + (a b)(a + b) = 0
(a b)(3a2 + 3ab + 3b2 + 2 + a + b) = 0
a=b
Thay vo mt trong hai phng trnh ca h ta c

(
3x 1 = 2
x=1
3a3 a2 + 2a 24 = 0 (a 2)(3a2 + 5a + 8) = 0 a = 2 2

=2
y=1
y
Kt lun: H c nghim (x; y) = (1; 1) 2
(
Bi 3: Gii h phng trnh

x2 + y 2 + xy + 1 = 4y
y(x + y)2 = 2x2 + 7y + 2
Gii

(
H trn c th vit li l

x2 + 1 + y(x + y) = 4y

2 x2 + 1 y(x + y)2 = 7y


~ tng: 2 phng trnh u c nhn t x2 + 1; x + y v y nn ta nhn nh s chia hai


v ca hai phng trnh cho y t n ph,a h phng trnh v dng n gin hn.
~ Li gii:
( 2

x + 1 + y(x + y) = 4y
H phng trnh tng ng vi

2 x2 + 1 y(x + y)2 = 7y
> Vi y = 0 h v nghim
> Vi y 6= 0.Chia c hai v ca hai phng trnh cho y ta c
2
x +1

+ (x + y) = 4

2 (x2 + 1)

(x + y)2 = 7
y
t a =

x2 + 1
v b = x + y h tr thnh
y
(

a+b=4
2a b2 = 7

a=4b
8 2b b2 = 7

a=1

b=3
a=4b

(
2
a=9
b + 2b 15 = 0

b = 5

> Vi a = 1v b = 3 ta c
(
x=1
2

(
(
x
+
1

y=2
=1
y =3x
y =3x

(
2
2
x = 2

x +1=3x
x +x2=0
x + y = 3

y=5

218
> Vi a = 9 v b = 5 ta c
2
(
(

x +1 =9
y = 5 x
y = 5 x
y

x + 1 = 45 9x
x2 + 9x + 46 = 0(v nghim)
x + y = 5

Kt lun: H c nghim (x; y) {(1; 2) ; (2; 5)} 2


Ta cng c mt cch khc cho bi 3:
~ tng: Nhn thy phng trnh (1) c hng t x2 + 1 cn phng trnh (2) c hng t
2x2 + 2 nn ta ngh n vic nhn hai v phng trnh (1) vi 2 ri tr i s kh 2x2 + 2
~ Li gii:
> Vi y = 0 h v nghim
( 2
2x + 2 + 2y 2 + 2xy 8y = 0
> Vi y 6= 0 H phng trnh tng ng vi
2x2 + 2 y(x + y)2 + 7y = 0
Tr v theo v hai phng trnh ca h ta c:
"
x+y =3
2y 2 + 2xy + y(x + y)2 15y = 0 (x + y)2 + 2(x + y) 15 = 0
x + y = 5
> Vi x + y = 3 thay vo h cho ta c
(

x2 + 3y + 1 = 4y
x+y =3

y =3x
x2 + 1 = 3 x

x=1

y=2
y =3x

(
2
x = 2
x +x2=0

y=5

> Vi x + y = 5 thay vo h cho ta c:


( 2
(
(
x + 1 5y = 4y
y = 5 x
y = 5 x

x + y = 5
x2 + 1 = 45 9x
x2 + 9x + 46 = 0(v nghim)

Kt lun: H c nghim (x; y)l (1; 2) ; (2; 5) 2


(
Bi 4: Gii h phng trnh sau:

9y 3 (3x3 1) = 125
45x2 y + 75x = 6y 2
Gii

> Vi x = 0 hoc y = 0 h v nghim


> Vi x 6= 0 v y =
6 0 ta c h tng ng vi:

125
125
3

27x + 3 = 9

27x3 + 3 = 9

y
y



2
5
5
x
x

3.x.
45. + 75. 2 = 6
3x +
=6
y
y
y
y

219
t a = 3x v b =

5
( b 6= 0).H tr thnh:
y
(

a3 + b 3 = 9

ab(a + b) = 6

(a + b)3 3ab(a + b) = 9

ab(a + b) = 6


T tm c nghim ca h l (x; y) l
(
Bi 5: Gii h phng trnh sau

1 5
;
3 2

a=1

b=2
a+b=3

(
a=2
ab = 2

b=1

 

2
;
;5 2
3

x2 + y 2 + 2x = 3


2 x3 + y 3 + 6x2 = 3 x2 + y 2 + 5
Gii

H phng trnh tng ng:


(
(x + 1)2 + y 2 = 4

2x3 + 6x2 + 2y 3 = 3 (3 2x) + 5

(x + 1)2 + y 2 = 4

2x3 + 6x2 + 6x + 2 + 2y 3 = 16

(x + 1)2 + y 2 = 4
(x + 1)3 + y 3 = 8

t a = x + 1 v b = y, H tr thnh:
(
()

a2 + b 2 = 4
a3 + b 3 = 8

n y ta s lp phng trnh ng bc hai v t h trn:


2

3
() a2 + b2 = a3 + b3 3a2 b2 a2 + b2 2a3 b3 = 0
"

a=0
a2 b2 3a2 + 3b2 2ab = 0
b=0
> Nu a = 0 : x = 1 y = 2
> Nu b = y = 0 : x = 1
Vy h phng trnh c nghim(x; y) = (1; 2), (1; 0) 2
2y(x2 y 2 ) = 3x
Bi 6: Gii h phng trnh
x(x2 + y 2 ) = 10y
Gii
Nu x = 0 hoc y = 0 th h c nghim (x; y) = (0; 0). Xt xy 6= 0, chia hai phng trnh v
theo v ta c:
2y(x2 y 2 )
3x
=
20y 2 (x2 y 2 ) = 3x2 (x2 + y 2 )
2
2
x(x + y )
10y

x2 = 4y 2

3x 17x y + 20y = 0 (x 4y )(3x 5y ) = 0

5y 2
x =
3

2 2

220
> Nu x2 = 4y 2 ta c h

2y.3y 2 = 3x
x.5y 2 = 10y
> Nu x2 =

2y 3 = x

xy = 2

"

x = 2; y = 1
x = 2; y = 1

5y 2
ta c h
3

4
135
15

;
y
=
x
=

4
2

2 135

4 135
15

; y=
x= 4
2
2 135

4
15
135
15 4 135
Vy h phng trnh c nghim (x; y) = (0; 0), (2; 1), (2; 1), (
;
), (
;
)2
2
2
2 4 135
2 4 135

z 2 + 2xyz = 1 (1)

Bi 7: Gii h phng trnh 3x2 y 2 + 3xy 2 = 1 + x3 y 4 (2)

z + zy 4 + 4y 3 = 4y + 6y 2 z (3)

2y. 2y = 3x
32
8y

x.
= 10y
3

4y 3 = 9x

4xy = 15

Gii
1 z2
V z = 0 khng l nghim ca h phng trnh nn (1) xy =
2z
 
\ {0} (a) cos 6= 0, sin 6= 0
t z = tan () vi ,
2 2
1 z2
1 tan2
Ta c: xy =
=
= cot 2
2z
2 tan
Thay vo (2) ta c :
3cot2 2 + 3y cot 2 = 1 + ycot3 2 y =

1
3cot2 2 1
=
= tan 6
3
cot 2 3 cot 2
cot 6

Ta suy ra: x = cot 2. cot 6


Thay vo (3) ta c :
z=

4 tan 6 4tan3 6
= tan 24 ()
1 6tan2 6 + tan4 6

T ()v ()ta c:
tan 24 = tan 24 = + k, k Z =

k
,k Z
23

 
Vi ,
\ {0}ta thu c:
2 2
=

2 3 4 5 6 7 8 9 10 11
, , , , , , , , ,
,
23 23
23
23
23
23
23
23
23
23
23

Vy h phng trnh c cc nghim l:


(x; y; z) = (cot 2. cot 6; tan 6; tan ) vi

2 3 4 5 6 7 8 9 10 11
= , , , , , , , , ,
,
2
23 23
23
23
23
23
23
23
23
23
23

221

x(y 2 + 1)
3

2
=
2
x +y
5
Bi 8: Gii h phng trnh
2
y(x

1)
4

=
2
2
x +y
5
Gii
T h suy ra xy 6= 0 v x 6= 1. Bnh phng hai phng trnh v cng li ta c


x(y 2 + 1) 2
y(x2 1) 2
]
+
] =1
x2 + y 2
x2 + y 2
x2 (y 2 + 1)2 + y 2 (x2 1)2 = (x2 + y 2 )2
(x2 y 2 + 1 x2 y 2 )(x2 + y 2 ) = 0
(x2 1)(y 2 1) = 0 y 2 1 = 0 y = 1

2x
3

2
=
+1
5 x=3
> Vi y = 1 ta c h x
2
4
x

=
2
5
x +1
3
2x

2
=
1
x +1
5

x
=
> Vi y = 1 ta c x
2

3
1 = 4
2
x +1
5
1
Th li ta thy h c nghim (x; y) = (3; 1), ( ; 1) 2
3

2003
2002

x + y + xy = z 2
+ 2z 2

2004
Bi 9: Gii h phng trnh
(I)
x4 + y 4 = 2z 2

(x + y)z1 = (z + 2004)xy
Gii
T h ta c:
2z 2

2004

2003

= x4 + y 4 > 2x2 y 2 xy 6 z 2

(1)

Li c
(x + y)4 6 4 x2 + y 2

2


2004
2002
6 4.2 x4 + y 4 = 16z 2
x + y 6 2z 2 (2)

(1) v (2) cho ta:


2003

x + y + xy 6 z 2
Du

00

2002

=00 xy ra khi v ch khi x = y = z 2

2002

+ 2z 2

. Khi ta c:

(
(I)

x=y=z

22002

(2x)z1 = (z + 2004)xy


Vy h c 3 nghim: (x; y; z) = (1; 1; 1) ,

1 1
; ;
2 2

22002

x=y=z=1

x = y =
2
1

z = 22002


2

222

x + y + z + t = 15 (1)

x2 + y 2 + z 2 + t2 = 65 (2)
(I)
Bi 10: Gii h phng trnh
3
3
3
3

x
+
y
+
z
+
t
=
315
(3)

xt = yz (4)
Gii
Ta c

(2) (x + t)2 + (y + z)2 2xt 2yz = 65


(x + y + z + t)2 2(x + t)(y + z) 4xt = 65(do(4))
(x + y + z + t)2 2(x + t) [15 (x + t)] 4xt = 65(do(1))
152 2(x + t) [15 (x + t)] 4xt = 65
(x + t)2 15(x + t) 2xt = 80(5)

Li c:
(3) (x + t)3 + (y + z)3 3xt(x + t) 3yz(y + z) = 315
(x + t)3 + (y + z)3 3xt(x + y + z + t) = 315(do(4))
(x + y + z + t)3 3(x + t)(y + z)(x + y + z + t) 45xt = 315(do (1))
153 45(x + t) [15 (x + t)] 45xt = 315
(x + t)2 15(x + t) xt = 68(6)
Ta c: (6) (5) xt = 12 (5)
Thay vo (5) ta c:
"
2

(x + t) 15(x + t) + 56 = 0

x+t=8
x+t=7

Kt hp (5) ta tm c (x; t) = (2; 6), (3; 4). T nghim ca h l


(x; y; z; t) = (6; 4; 3; 2), (6; 3; 4; 2), (2; 4; 3; 6), (2; 3; 4; 6), (4; 6; 2; 3), (4; 2; 6; 3), (3; 6; 2; 4), (3; 2; 6; 4) 2

x3 + y 3 + x2 (y + z) = xyz + 14 (1)

Bi 11: Gii h phng trnh: y 3 + z 3 + y 2 (x + z) = xyz 21 (2)

z 3 + x3 + z 2 (x + y) = xyz + 7 (3)
Gii

(1) + (2) + (3) x3 + y 3 + z 3 + x2 + y 2 + z 2 (x + y + z) = 3xyz

(x + y + z)3 3 (x + y + z) (xy + yz + zx) + x2 + y 2 + z 2 (x + y + z) = 0


(x + y + z) x2 + y 2 + z 2 (xy + yz + zx) + x2 + y 2 + z 2 = 0
" 2
x + y 2 + z 2 (xy + yz + zx) + x2 + y 2 + z 2 = 0 ()

x + y + z = 0 ()
> T () ta c:
(

x2 + y 2 + z 2 (xy + yz + zx) > 0


x2 + y 2 + z 2 > 0

V T(5) > 0

223
Du = xy ra x = y = z = 0
> T (**) ta c: z = (x + y)
Thay vo (1) v (3) ta c h phng trnh sau:
( 3
y + xy (x + y) = 14
x3 + xy (x + y) = 7
(
* Xt x = 0 (I)

(I)

y 3 = 14

(v nghim)
0=7
Xt x 6= 0. t: y = kx ta c:
( 3 3

x k + k 2 + k = 14 (4)
k3 + k2 + k

(I)
= 2 k 3 k 2 k 2 = 0 k = 2 y = 2x

2
2
3
k +k+1
x k + k + 1 = 7 (5)

Thay vo (5) ta c: x = 1 y = 2 z = 3
Vy h phng trnh c 1 nghim l: (x; y; z) = (1; 2; 3) 2

x3 + x(y z)2 = 2

Bi 12: Gii h phng trnh sau: y 3 + y(z x)2 = 30

z 3 + z(x y)2 = 16
Gii
Ta a h v dng:

x(x2 + y 2 + z 2 2yz) = 2 (1)

y(x2 + y 2 + z 2 2xz) = 30 (2)

z(x2 + y 2 + z 2 2xy) = 16 (3)


Ly (1) + (2) 2(3) ta c:
"
(x + y 2z) x2 + y 2 + z


2

=0

x + y 2z = 0 y = 2z x
x2 + y 2 + z 2 = 0 x = y = z = 0 (l)

Thay y = 2z x vo phng trnh (1) v (3) ta c:


x(2x2 + z 2 2xz) = 2 (4)
z(4x2 + 5z 2 4xz) = 16(5)
t z = kx ta tm c k = 2
Vy h phng trnh cho c 1 nghim l: (x, y, z) = (1, 3, 2) 2
Bi 13: Gii h phng trnh

x2 + y + x3 y + xy 2 + xy = 5 (1)
4
x4 + y 2 + xy(1 + 2x) = 5 (2)

4
(H khi A - 2008)

224
Gii
H phng trnh tng ng:

x2 + y + xy + xy(x2 + y) = 5
4
5

(x2 + y)2 + xy =
4

u + v + uv = 5
2
4
t u = x + y; v = xy ta c h
5

u 2 + v =
4

5
v = 5 u2
u = 0, v =

4
4

3
1
u3 + u2 + u = 0
,
v
=
u
=
4
2
2
r
r
25
5
3 5
ta tm c (x; y) = (
; 3
)
> Vi u = 0, v =
4
4
16
1
3
3
> Vi u =
,v =
ta tm c (x; y) = (1;
)
2
2
2
r
r
5
25
3
Vy h c nghim (x; y) = ( 3 ; 3
), (1;
)2
4
16
2

4yz

y 2 + z 2 x2 =

4zx
2
2
2
Bi 14: Gii h phng trnh z + x y =

x2 + y 2 z 2 = 4xy
z
Gii
XK: x, y, z 6= 0
Nhn hai v ca cc phng trnh ln lt vi x2 , y 2 , z 2 ta c:

2 2
2 2
4

x y + x z x = 4xyz
y 2 z 2 + x2 y 2 y 4 = 6xyz

x2 z 2 + y 2 z 2 z 4 = 8xyz

Li c:
(1) 2 (2) + (3) 2y 4 y 2 z 2 x2 y 2 x4 + 2x2 z 2 z 4 = 0
y 2 (2y 2 x2 z 2 ) (x2 z 2 )2 = 0



6xz
2
2
2
y 2y y +
(x2 z 2 )2 = 0
y
y 4 6xyz (x2 z 2 )2 = 0
(y 2 x2 + z 2 )(y 2 + x2 z 2 ) 6xyz = 0
4yz 8xy

6xyz = 0
x
z
32y 2 6xyz = 0 16y 3xz = 0

225
Thay xz =

16y
vo (2) v y =
3

3xz
16

vo (1) v (3) ta c:

x2 + y 2 + z 2 = 0

4
x2 y 2 + z 2 = 32

1 x2 + y 2 z 2 = 0
2



128
288
,
y l h phng trnh tuyn tnh theo (x2 ; y 2 ; z 2 ), ta tm c (x , y , z ) = 64,
5
5
yz zx xy
2 2 2
T h ban u ta thy vi nhng gi tr (x ; y ; z ) trn, cc hng t
, ,
u dng.
x y z
Gi s x < 0, t (1) ta c yz < 0. Tng t vi y!< 0 v z < 0.
r
r
r
r !
2
2
2
2
Vy nghim ca h l (x, y, z) = 8, 12
,8
, 8, 12
, 8
,
5
5
5
5
r
r !
r
r !
2
2
2
2
8, 12
, 8
, 8, 12
,8
v cc hon v 2
5
5
5
5

2z (x + y) + 1 = x2 y 2 (1)

Bi 15: Gii h phng trnh y 2 + z 2 = 1 + 2xy + 2zx 2yz (2)

y 3x2 1 = 2x x2 + 1 (3)
2

Gii
1
V x = khng tha phng trnh (3)nn:
3
(3) y =

2x (x2 + 1)
3x3 x 2x (x2 + 1)
x3 3x

x
+
y
=

x
+
y
=
3x2 1
3x2 1
3x2 1

  n o
t x = tan , ;
\ ;
cos 6= 0, cos 3 6= 0
2 2
6 6
Ta c:
tan + y =

(1) z =

tan3 3 tan
y = tan 3 tan
3tan2 1

x2 y 2 1
(do x = ykhng tha phng trnh (1) tan 3 6= 0)
2 (x + y)

(2 tan tan 3) . tan 3 1


2 tan . tan 3 tan2 3 1
=
2 tan 3
2 tan 3


tan 3 + cot 3
1 sin 3 cos 3
z = tan
= tan
+
2
2 cos 3 sin 3
1
z = tan
sin 6
z=

226
(2) x2 + y 2 + z 2 2xy 2zx + 2yz = 1 + x2 (y + z x)2 = 1 + x2

2
1
tan 3 tan + tan
tan = 1 + tan2
sin 6

2
sin 3
1
1

tan =
cos 3 2 sin 3. cos 3
cos2

2

2
2sin2 3 1
cos 6
1
1

tan =

+ tan =
2
2 sin 3. cos 3
cos
sin 6
cos2
2

2

1
cos 5
1
cos 6. cos + sin 6. sin
=

2
sin 6. cos
cos
sin 6. cos
cos2


cos 5 = sin 6 cos 5 = cos
6
2





6
5 =
6 + k2
cos 5 = cos

2
2





cos 5 = cos
+ 6
5 =
+ 6 + k2
2
2

k2

=
+
, = k2

22
11
2

(k Z)

k2

= +
, = k2
22
11
2
  n o

3 5 7 9
Vi: ;
\ ;
= ; ; ; ;
2 2
6 6
22 22
22
22
22
Vy h phng
trnh

cho
c
nghim
l:


1

3 5 7 9
(x; y; z) = tan ; tan 3 tan ; tan
, = ; ; ; ;
2
sin 6
22 22
22
22
22

(x + 2)2 + (y + 3)2 = (y + 3) (x + z 2)

(I)
Bi 16: Gii h phng trnh x2 + 5x + 9z 7y 15 = 3yz

8x2 + 18y 2 + 18xy + 18yz = 84x 72y 24z 176


Gii
t:a = x + 2; b = y + 3 ta c:

a2 + ab + b2 + bz 4b = 0 (1)

(I) a2 + a 7b + 3bz = 0 (2)

8a2 2a + 18 b2 + ab + bz 4b 30z + 94 = 0 (3)


Li c:
(1) b2 + ab + bz 4b = a2 ()
Thay ()vo (3) ta c:
8a2 2a 18a2 30z + 94 = 0
10a2 + 2a + 30z 94 = 0
z=
5a2 + a 47
Thay z =
vo (2)ta c:
15

5a2 + a 47
15

227

 2

5a2 + a 47
5a + a 12
a + a 7b b
=0
b = a2 + a
5
15

5 (a2 + a)
1 241
b= 2
(V a =
khng l nghim ca phng trnh)
5a + a 12
10


Nhn 2 v ca phng trnh (1)vi 3 ri tr cho phng trnh (2)v theo v, ta c:


2a2 a + 3ab + 3b2 5b = 0 (4)
Thay b =

5 (a2 + a)
vo (4) ta c:
5a2 + a 12


2
15a (a2 + a)
5 (a2 + a)
25 (a2 + a)
2a a + 2
+3
=0

5a + a 12
5a2 + a 12
5a2 + a 12

2 



2
2a2 a 5a2 + a 12 + 15a a2 + a 25 a2 + a
5a2 + a 12 + 75 a2 + a = 0
2

50a6 + 70a5 208a4 94a3 + 482a2 + 156a = 0


25a6 + 35a5 104a4 47a3 + 241a2 + 78a = 0

a (a + 2) 25a4 15a3 74a2 + 101a + 39 = 0


a (a + 2) 5a2 14a + 13 5a2 + 11a + 3 = 0

11 61
a = 0 a = 2 a =
10

b = 0
x = 2
> TH1: a = 0 :

z = 47
y = 3
15

b =

3 x = 4
> TH2: a = 2 :

y = 4
z = 29
3
15

17 2 61

31
+
61

b =
x =
11 + 61
15
10
:
> TH3: a =

28 + 2 61
10
39
+
61

z =
y=
15
15

17 + 2 61

31
+
61

b =
x =
11 61
15
10
> TH4: a =
:

28
+
2 61
10

y =
z = 39 61
15
15
Vy h phng
 trnh cho
 c 4 nghim
 l:
47
4 29
(x; y; z) = 2; 3;
, 4; ;
,
15
3 15!

!
31 + 61 28 + 2 61 39 + 61
31 + 61 28 + 2 61 39 61
,
2
;
;
;
;
10
15
15
10
15
15
Bi
17: Cho cc tham
(
x + y + z = a + b + c (1)

dng

a, b, c.

Tm

nghim

4xyz a2 x b2 y c2 z = abc (2)


Gii
Ta c:
(2)

a2
b2
c2
abc
+
+
+
= 4 (3)
yz xz xy xyz

dng

ca

228
b
c
a
t: x1 = ; y1 = ; z1 = ta c:
yz
xy
xz
(3) x21 + y12 + z12 + x1 .y1 .z1 = 4 (4)

D thy: 0 < x1 , y1 , z1 < 2 nn tn ti cc gi tr u, vtha: 0 < u, v < v x1 = 2 sin u; y1 =


2
2 sin v
. Khi :
(4) z12 + 4z1 . sin u. sin v + 4sin2 u + 4sin2 v 4 = 0



C 0 = (2 sin u. sin v)2 4sin2 u + 4sin2 v 4 = 4 1 sin2 u 1 sin2 v = 4cos2 u.cos2 v > 0
Vy
"
z1 = 2 sin u. sin v 2 cos u. cos v < 0
(4)
z1 = 2 sin u. sin v + 2 cos u. cos v > 0

Do : a = 2 yz. sin u; b = 2 zx. sin v; c = 2 xy (cos u. cos v sin u. sin v)


Thay vo (1) ta c:

x + y + z = 2 yz. sin u + 2 zx. sin v + 2 xy (cos u. cos v sin u. sin v)


2
 2 

x cos v y cos u +
x sin v + y sin u 2 = 0

x cos v y cos u = x sin v + y sin u 2 = 0

a y

a+b
b x
a+b

Ta tnh c: z = x sin v + y sin u = + = z =


2
2 zx 2 yz
2 z
b+c
c+a
;x =
Tng t, ta cng c: y =
2
2


b+c c+a a+b
;
;
Vy h phng trnh cho c 1 nghim l: (x; y; z) =
2
2
2
2

xz = x + 8 (1)

Bi 18: Gii h phng trnh 4y 2 = 7xz 3x 28 (2)

x2 + 4z 2 = 140 4y 2 (3)
Gii
Ta c (1) x = xz 8
Thay vo (2) ta c: 4y 2 = 7xz 3 (xz 8) 28 y 2 = xz 1 (y 2 > 0 xz > 1)
Thay vo (3) ta c:
"
"
x
+
2z
=
12
x = 2z + 12 (4)
2
x2 + 4z 2 = 140 4 (xz 1) (x + 2z) = 144

x + 2z = 12
x = 2z 12 (5)
Thay (4) vo (1) ta c:
"
2

z (2z + 12) = 2z + 12 + 8 z 7z + 10 = 0

z = 2 x = 8 y 2 = 15 y = 15
z = 5 x = 2 y 2 = 9 y = 3

Thay (5) vo (1) ta c:

5 + 33
z=
x = 7 33 ()

2
z (2z 12) = 2z 12 + 8 z 2 + 5z 2 = 0

5 33
z=
x = 7 + 33 y 2 = 33 y =
2

229


Loi (*) do xz < 0. Vy h phng
trnh
c
6
nghim
l:
(x;
y;
z)
=
8;

15;
2
, (2; 3; 5) ,
!

5 33
7 + 33; 4 33;
2
2

x + y + z = 0(1)

Bi 19: Gii h phng trnh x2 + y 2 + z 2 = 10(2)

x7 + y 7 + z 7 = 350(3)
Gii
T (1), (2) ta c xy + yz + zx = 5 (3)
Li c: (1) x3 + y 3 + z 3 = 3xyz(4)
(2) (4) x2 + y 2 + z 2


x3 + y 3 + z 3 = 30xyz

x5 + y 5 + z 5 + x2 y 2 (x + y) + y 2 z 2 (y + z) + x2 z 2 (x + z) = 30xyz
x5 + y 5 + z 5 x2 y 2 z xy 2 z 2 x2 yz 2 = 30xyz
x5 + y 5 + z 5 xyz(x + y + z) = 30xyz
x5 + y 5 + z 5 + 5xyz = 30xyz x5 + y 5 + z 5 = 25xyz


(2) (5) x2 + y 2 + z 2 x5 + y 5 + z 5 = 250xyz
x7 + y 7 + z 7 + x2 y 2 (x3 + y 3 ) + y 2 z 2 (y 3 + z 3 ) + z 2 x2 (z 3 + x3 ) = 250xyz
x7 + y 7 + z 7 + x2 y 2 (3xyz z 3 ) + y 2 z 2 (3xyz x3 ) + z 2 x2 (3xyz y 3 ) = 250xyz
x7 + y 7 + z 7 + 3xyz(x2 y 2 + y 2 z 2 + z 2 x2 ) x2 y 2 z 2 (x + y + z) = 250xyz


x7 + y 7 + z 7 + 3xyz (xy + yz + xz)2 2xyz(x + y + z) = 250xyz
x7 + y 7 + z 7 + 45xyz = 250xyz
x7 + y 7 + z 7 = 135xyz(6)
Thay (6) vo (3) ta c: 135xyz = 350 xyz = 2. Vy ta c:

x+y+z =0

xy + xz + yz = 5

xyz = 2
Suy ra x, y, zl nghim ca phng trnh t3 5t 2 = 0 t = 1 +


Vy h c nghim l 1 + 2; 2; 1 2 v cc hon v 2.

2, t = 2, t = 1 2

30x 4y +
= 2012

xy

1
Bi 20: Gii h phng trnh () 4y 2012z +
= 30

zx

2012y 30z + 1 = 4
yz
Gii
~ tng: R rng cc s 30, 4, 2012 c chn ngu nhin do ta c th t 30 = a; 4 =
b; 2012 = c thy r hn tng bi ton. Khi , nh ni trn, ta cn tnh a, b, c

230
theo x, yz. D thy h thu c l mt h tuyn tnh 3 n vi x, y, z l tham s. C th dng
cng thc Cramer tnh nghim, nhng do trong chng trnh ph thng khng c cng thc
Cramer cho h 3 n nn ta s a v h 2 n.
~ Li gii:
t 30 = a; 4 = b; 2012 = c ta c HPT:

1
1

c = ax by +
ax by +
= c (1)

xy

xy

1
1
1
0
= a (4)
(II ) by cz +
= a (2) bz (ax by + ).x +

xy
zx
zx

1
1

cy az + 1 = b (3)

(ax by + ).y az +
= b (5)
yz
xy
yz
Xt (4), (5) l 1 h 2 n theo a, b. Dng cng thc Cramer ta tnh c:



x
+
1
xy

= (1 + x2 + y 2 + z 2 ) 6= 0

D =

xy z y 1

1
1


(1 + x2 + y 2 + z 2 )

xy

xy y

Da = 1 1
=


xz
2

yz
x

1
1
2


(x2 + y 2 + z 2 + 1)
x
+
1



xy
y

Db =
=

1
1

xy z
yz



xy x

Da
1

a
=
=

D
xz

1
Db
=
b=
D
yz

c =
xy

Tr li HPT u ta c

1
x=

r15090

30

15
1
y=

r 4024

503

z =
2012
30
r
r
1
15
503
Vy () c nghim (x; y; z) = (
;
;
)2
4024
30
15090

x4 + 4x2 + y 2 4y = 2
Bi 21: Gii h phng trnh ()
x2 y + 2x2 + 6y = 23

xz =

yz =

xy =

Gii

t 4y = 2 x4 4x2
H tng ng
y(x2 + 6) = 23 2x2

D = x 2 + 6

Ta tnh c

vi t = y 2

Dt = x6 10x4 30x2 + 104

Dy = 23 2x2

Dt
Dy
= ( )2 (x2 + 6)(x6 10x4 30x2 + 104) = (23 2x2 )2
D
D
(1 x)(1 + x)(1 + x2 )(x4 + 16x2 + 95) = 0 x = 1
Do t = y 2 nn

231
T ta tm c nghim ca h l (x; y) = (1; 3), (1; 3) 2
(
Bi 22: Gii h phng trnh

x4 + x3 y + 9y = y 3 x + x2 y 2 + 9x
x(y 3 x3 ) = 7
Gii

Ta c:
x4 + x3 y + 9y = y 3 x + x2 y 2 + 9x (x4 xy 3 ) + (x3 y x2 y 2 ) 9(x y) = 0




(x y) x(x2 + xy + y 2 ) + x2 y 9 = 0 (x y) x(x + y)2 9 = 0
T phng trnh th hai ca h, ta thy x 6= y nn t bin i trn, suy ra:
x(x + y)2 9 = 0 x(x + y)2 = 9 ()
r
7
7
Ta c: x(y 3 x3 ) = 7 y 3 x3 = y = 3 x3 + .
x
x
Thay vo (*), ta c:
r
2
7
3
x(x + x3 + ) = 9
x
Ta s chng minh rng v tri l mt hm ng bin theo bin x. Tht vy:

s
r
r
2
2
7
7
7
3
3
3
x(x + x3 + ) = x x2 + 2x x3 + +
x3 +
x
x
x
s
r
2
q

7
7
3
3
3
3
2 3
3
3
6
2
3
= x + 2x . x + + x
= x + 2x x + 7x + x(x4 + 7)2
x +
x
x
T (*) suy ra x > 0v trong biu thc trn, cc s m ca bin x u dng nn y l hm
ng bin; suy ra n c khng qu mt nghim.
Thay trc tip x = 1 vo biu thc, ta thy tha.
Vy h cho c ng mt nghim l: (x, y) = (1, 2) 2.
~ Nhn xt: im c bit ca bi ny l x l c h phng trnh mi sau khi bin i, nu
nh ta dng cch i s trc tip, phn tch ra c mt nghim x = 1 th phng trnh bc
cao cn li kh m gii c. Cch lp lun theo tnh n iu ca hm s th ny va trnh
c iu va lm cho li gii nh nhng hn.
Bi 23 (China TST 2006) : Gii h phng trnh

xy+zw =2

x 2 y 2 + z 2 w 2 = 6

x3 y 3 + z 3 w3 = 20

4
x y 4 + z 4 w4 = 66
(China TST 2006)
Gii

232

x + z = a

xz = b

y+w =c

yw = d
T (1) v (2) ta c

a c = 2 (1)

a2 2b c2 + 2d = 6 (2)
ta c HPT:
a3 3ab c3 + 3cd = 20 (3)

4
a 4a2 b + 2b2 c4 + 4c2 d 2d2 = 66 (4)

c = a 2
d = 2a + b + 5

T (3) v (4) c

5a 2b 14 = 0
5a2 2ab 10a b 13 = 0

5a 2b = 14

4a b = 13

a = 4

b = 3

c = 2
d = 0

Th ngc li ta tm c nghim (x; y; w; z) ca h l (1; 0; 3; 2) v cc hon v. 2


(
Bi 24: Gii h phng trnh

(2x2 3x + 4)(2y 2 3y + 4) = 18
x2 + y 2 + xy 7x 6y + 14 = 0
Gii

Xt x2 + y 2 + xy 7x 6y + 14 = 0 () l phng trnh bc hai theo bin x, vit li l:


x2 + x(y 7) + y 2 6y + 14 = 0
Phng trnh ny c nghim khi:
y = (y 7)2 4(y 2 6y + 14) > 0 3y 2 + 10y 7 > 0 1 6 y 6

7
3

Hon ton tng t, xem (*) l phng trnh bc hai theo bin y, vit li l:
y 2 y(x 6) + (x2 7x + 14) = 0
Phng trnh ny c nghim khi:
x = (x 6)2 4(x2 7x + 14) > 0 3x2 + 16x 20 > 0 2 6 x 6

10
3

3
< 1.
4
Suy ra, trn [1, +), hm s ny ng bin. Li c f (x) > f (2) = 6, f (y) > f (1) = 3
f (x).f (y) > 3.6 = 18.
T phng trnh th nht ca h th ta thy ng thc phi xy ra, tc l x = 2, y = 1.
Thay hai gi tr ny vo (*), ta thy khng tha.
Vy h phng trnh cho v nghim. 2
~ Nhn xt: tng gii ca bi ny khng kh v cng kh quen thuc khi ch cn tm
min xc nh ca bin thng qua vic tnh Delta ca mt phng trnh bc hai; tuy trong
li gii trn c kho st hm s nhng thc ra cc kt qu c th chng minh bng bt
ng thc i s thun ty nn cng c gii chnh ca bi ny l i s. V do vic hai

Ta xt hm s: f (t) = 2t2 3t + 4(t R) f 0 (t) = 4t 3 = 0 t =

233
biu thc ca x v y phng trnh u ca h ging nhau c th dn n nh gi sai hng
m dng gii tch, xt hm s khai thc phng trnh u tin trong khi iu khng
em li kt qu g. Cc h s c chn ra s rt p chnh l u im ni bt ca bi ton ny.

x4 + y 4 + xy = 2xy 2 + 7 (1)
Bi 25: Gii h phng trnh
x2 y + 4xy + xy 3 + 11(x y 2 ) = 28 (2)
Gii
Xt (1).4 (2) x2 (y + 4) x(y 3 + 8y 2 + 11) + 4y 4 + 11y 2 = 0 ().
Nu y = 4 ta c phng trnh 27x + 1200 = 0 x = 16. Th li thy (x; y) = (16 4)
khng tho h.
Vy y 6= 4. Khi , coi (*) l phng trnh bc 2 theo x, ta tm c nghim
p
y 3 + 8y 2 + 11 (y 3 + 8y 2 + 11)2 4(y + 4)(4y 4 + 11y 2 )
x1,2 =
2(y + 4)
3
2
3
y + 8y + 11 (y 11)
x1,2 =
2(y + 4)
4y 2 + 11
x1 = y 2 ; x2 =
y+4
Vi x = y 2 thay vo (1) ta c y 3 = 7 y =

7 = x =

4y 2 + 11
3
49. Vi x =
thay vo
y+4

phng trnh u v rt gn ta c
y 6 +4y 4 6y 3 +4y 2 12y+9 = 0 (y 3 +2y3)2 = 0 (y1)(y 2 +y+3) = 0 = y = 1 = x = 3

Vy h c nghim (x; y) = ( 3 49; 3 7), (3; 1) 2
(
Bi 26: Gii h phng trnh

x2 + 3y 2 = 4
x4 + 9y 4 = 10
Gii

T h phng trnh ta c
(x2 + 3y 2 )2 (x4 + 9y 4 ) = 6x2 y 2 = 6 = x2 y 2 = 1
> Nu xy = 1 ta c:
q

x + 2 3xy + 3y = 4 + 2 3 = x + y 3 = 4 + 2 3 = ( 3 + 1)
2

Li c:
y=

1
1
= x2 ( 3 + 1)x + 3 = 0 x = 1, x = 3; y =
x
x

> Nu xy = 1 ta c
q

x + 2 3xy + 3y = 4 2 3 = x + y 3 = 4 2 3 = ( 3 1)
2

234
Li c
y=

1
1
= x2 ( 3 1)x 3 = 0 x = 1, x = 3; y =
x
x

1
1
Vy h phng trnh c nghim (x; y) = (1, 1), (1, 1) and ( 3, ), ( 3, ) 2
3
3
Bi 27: Gii h phng trnh
()(x + y)(z + 1) = (x + z)(y + 1) = (y + z)(x + 1) = 2a(a + 1)
(2012 Philippine Math Olympiad National Finals Oral Round)

Gii

x + yz = u

y + zx = v

z + xy = w

ta c h phng trnh

u + v = v + w = w + u = 2a(a + 1)
H ny c nghim u = v = w = a(a + 1) do h (*) tng ng
x + yz = y + zx = z + xy = a(a + 1)

(x y) = z(x y)

(y z) = x(y z)

x + zx = a(a + 1)

x = y z = 1
y =zx=1

x + yz = a(a + 1)

Tm li ta c s sau:

x=y

z=1







& x = 1 = x = y = 1 ; z = a2 + a 1




2
% y = z = y = z = 1 ; x = a + a 1




2
& x = 1 = x = z = 1 ; y = a + a 1
% y=z

= x = y = z

{a, (a + 1)}

Vy h c nghim (x; y; z) = (a; a; a), (a 1; a 1; a 1), (1; 1; a2 + a 1) v cc hon v. 2

235
Bi 28: Cho h phng trnh

x + y + z = a (1)
x2 + y 2 + z 2 = b2 (2)

xy = z 2 (3)
. a) Gii h trn vi n (x; y).
b) Cc s a, b phi tho iu kin g cc nghim x, y, z ca h dng v khc nhau?
Gii
Cu a) Bnh phng 2 v ca (1): a2 = x2 + y 2 + z 2 + 2xy + 2yz + 2zx
x + y = a z, kt hp (2) v (3) ta c:
a2 = b2 + 2z 2 + 2(a z).z
hay
a2 = b2 + 2az
T :
z=

a2 b 2
2a

T y tm c x, y bi cc h thc: x + y = a z =

(a2 b2 )2
a2 + b 2
; xy = z 2 =
2a
4a2

2
2
a
+
b
10a2 b2 3a4 3b4

x =

4a
4a

2
2
2 2
4
4

y = a + b 10a b 3a 3b
4a
4a
Cu b) x, y > 0 th x + y =
z > 0

a2 + b 2
>0a>0
2a

a2 b 2
> 0 a > |b|
2a

x 6= y cn c 10a2 b2 3a4 3b4 > 0 1 >


(*) chnh l iu kin cn c. 2

1
a
|b|
> a > |b| > ()
a
3
3

Bi 29: Gii h phng trnh


3
uv + vy 3 = 14

ux2 + vy 2 = 5
()

ux + vy = 2

u+v =1
Gii
t ak = uxk + vy k . rng
ak+1 = (x + y)ak xyak1

236
t A = x + y; B = xy. Ln lt xt k = 2, 3 t h (*) ta c:

5 = 2A B

14 = 5A 2B

A = 4
B = 3

Do x, y l nghim ca phng trnh t2 4t + 3 = 0.Nh vy (x; y) = (1; 3), (3; 1).


Th vo h (*) ta c

u + v = 1
1
u=v=
u + 3v = 2
2
1 1
1 1
Vy h (*) c nghim (x; y; u; v) = (1; 3; ; ), (3; 1; ; ) 2
2 2
2 2
Bi 30: Gii h phng trnh a2 + bc ab = b2 + ca bc = c2 + ab ca

Gii
Vi c = 0 ta tm c nghim (a; b; c) = (0; 0; 0)
a b
Vi c 6= 0, nu (a; b; c) l nghim ca h th ( ; ; 1) cng l nghim (do cc phng trnh l
c c
thun nht). Vy khng gim tng qut gi s c = 1.
Khi ta c h phng trnh
a2 + b ab = b2 + a b = 1 + ab a
a2 + a 1
1
T a + b ab = 1 + ab a, ta ca + a 1 = b(2a 1) b =
(d thy a 6= )
2a 1
12
Thay vo phng trnh a2 + b ab = b2 + a b ta c (a 1)(a3 6a2 + 5a 1) = 0
3
2
Nu a = 1 ta c nghim (a; b; c) = (1; 1; 1). Trng hp
cn li, a l nghim ca x 6x +5x1 =
2 7
0. C th gii phng trnh ny bng cch t t = y + 2 (xem cch gii tng qut Phng
3
trnh bc 3). T ta tm c nghim
2

2 7
t + 2k
3 3
2 + cos
vit = arccos vk = 1, 2
3
3
2 7

2 7
t + 2k
Nh vy h phng trnh c nghim (c, c, c) vi c R v (c (2 + cos
), c f (2 +
3
3

t + 2k
x2 + x 1
3 3
2 7
cos
), c) (trong c R, t = arccos , k = 0, 2 v f (x) =
)2
3
2x 1
3
2 7

H phng trnh v t

x + x2 2x + 2 = 3y1 + 1
Bi 1: Gii h phng trnh
p
y + y 2 2y + 2 = 3x1 + 1
Gii

237
Do nhm c nghim x = y = 1 nn ta s gii bng hm s v chng minh n l nghim duy
nht
ca h.
x 1 = a
t
, khi h phng trnh tng ng
y 1 = b

a + a2 + 1 = 3b

b + b2 + 1 = 3a
Nhn thy y l h i xng theo 2 n a v b, tr v theo v ta c

a + a2 + 1 + 3a = b + b2 + 1 + 3b

t2 + 1 + t
+ 3t ln 3
Ta xt hm s c trng f (t) = t + t2 + 1 + 3t c f 0 (t) =
2+1
t

V t2 + 1 > t2 > t nn f (t) > 0t Vy hm s ng bin trn R nn a = b

Thay vo phng trnh ta c a + a2 + 1 = 3a ln( a2 + 1 + a) a ln 3

Xt hm s G(a) = ln( a2 + 1 + a) a ln 3
1
Ta c G0 (a) =
ln 3 < 1 ln 3 < 0a R
a2 + 1
Nn hm s nghch bin v ta nhn thy a = 0 l nghim nn phng trnh c duy nht.
Vy h phng trnh cho c nghim l (x; y) = (1; 1) 2

Bi 2: Gii h phng trnh

x + y xy = 3
x + 1 + y + 1 = 4
( thi H khi A - 2006)
Gii
KX: x > 1, g > 1, xy > 0

t t = xy > 0. T phng trnh u ta c x + y = 3 + t ()


Bnh phng hai v ca phng trnh sau ta c
p
x + y + 2 + 2 xy + x + y + 1 = 16 ()
Thay (*) vo (**) ta c

3 + t + 2 + 2 t2 + 3 + t + 1 = 16 2 t2 t + 4 = 11 t

0 6 t 6 11
0 6 t 6 11

t=3
4(t2 + t + 4) = (11 t)2
3t2 + 26t 105 = 0
T tm c nghim ca h l (x; y) = (3; 3) 2
(
Bi 3: Gii h phng trnh

p
x + x2 + y + 3 = 2
p

2 x + 4 + 3 y + 8 = 13
Gii

238
K: x > 0; x2 + y + 3 > 0; y + 8 > 0
T phng trnh (2) s dng BT Cauchy-Schwarz ta c:

2
p
13 = 2 x + 4 + 3 y + 8 6 13 (x + 4 + y + 8) x + y > 1 ()
2

T phng trnh (1), bnh phng hai v ta c:


x + x2 + y + 3 + 2

p
x (x2 + y + 3) = 4 x + y = 1 x2 2 x (x2 + y + 3) x + y 6 1 ()

T (*) v (**) suy ra x + y = 1

x = 0
x=0

T du bng xy ra cc BT nn x + 4

y
+
8

y=1

=
2
3
Th li ta thy h c nghim duy nht (x; y) = (0; 1) 2
Bi
4:
Gii
h
phng
trnh
2003

(3 x)
=y+2

1
1 p

log
+
log
(y
+
2)
=
log
9 + 4y
3
1
()
2z y
3

log x2 + z 2  = 2 + log x
2
2

sau

vi

>

Gii
K: x > 0, 2z > y, y > 2
Ta c:

(3 x)2003 = y + 2

()
log3 (2z y) log3 (y + 2) = log3 (9 + 4y)

log x2 + z 2  = log 4x
2

(3 x)2003 = y + 2

(2z y) . (y + 2) = 9 + 4y

x2 + z 2 = 4x

(3 x)2003 = y + 2

y 2 + 9 + z 2 + 6y 2yz 6z = z 2 2z

x2 4x + 4 = 4 z 2

(3 x)2003 = y + 2 (1)

(y + 3 z)2 = z 2 2z (2)

(x 2)2 = 4 z 2 (3)
Nu (x0 , y0 , z0 ) l nghim ca h ta c:
(x0 2)2 = 4 z0 2 4 z0 2 > 0 2 6 z0 6 2 (4)
(y0 + 3 z0 )2 = z0 2 2z0 z0 2 2z0 > 0 z0 6 0 z0 > 2 (5)
Kt hp vi iu kin bi ton l(z0 > 0vi (4)(v (5) ta c: z0 = 0 z0 = 2
x0 = 0
x0 = 4
> Vi z0 = 0 t (2) v (3) ta c

khng tha iu kin bi ton


y0 = 3
y0 = 3

0:

239
(
> Vi z0 = 2 t (2) v (3) ta c

x0 = 2

y0 = 1
Th li ta thy h phng trnh c nghim duy nht l: (x; y; z) = (2; 1; 2) 2

1 + y = 2 x + 2
y
x x
Bi 5: Gii h phng trnh

y(x2 + 1 1) = 3x2 + 3
Gii
iu kin xc nh: x > 0, y 6= 0.
Phng trnh th nht ca h tng ng vi:

1
y
2 x
+ =
+ 2 y x + y 2 = 2x x + 2xy y 2 + y( x 2x) 2x x = 0
y
x x
Xem y l phng trnh bc hai theo bin y, ta c:

2
2
x = ( x 2x) + 8x x = x + 4x x + 4x2 = ( x + 2x) > 0
Do , phng
trnhny c hai nghim l:

(2x x) ( x + 2x)
(2x x) + ( x + 2x)
y1 =
= x, y2 =
= 2x,
2
2
Xt hai trng hp:

> Nu y = x, thay vo phng trnh th hai ca h, ta c:


x( x2 + 1 1) = 3x2 + 3


D thy: x( x2 + 1 1) < 0 < 3x2 + 3 nn phng trnh ny v nghim.
> Nu y = 2x, thay vo phng trnh th hai ca h, ta c:

2x( x2 + 1 1) = 3x2 + 3 x2 + 1.(2x 3) = 2x x2 + 1 =

2x
()
2x 3

3
3
(d thy x =
khng tha mn ng thc nn ch xt x 6=
v php bin i trn l ph
2
2
hp).

2x
, x > 0.
Xt hai hm s: f (x) = x2 + 1, x > 0 v g(x) =
2x 3

x
2 3
0
0
Ta c: f (x) =
> 0 nn l hm ng bin, g (x) =
2 < 0 nn l hm nghch
x2 + 1
(2x 3)
bin. Suy ra phng trnh (*) c khng qu mt nghim.

Nhm thy x = 3 tha mn (*) nn y cng chnh l nghim duy nht ca (*).

Vy h cho c nghim duy nht l (x, y) = ( 3, 2 3) 2.
~ Nhn xt: Quan h ca x v y c che giu ngay trong phng trnh u tin, nu nhn
thy iu th cc bc tip theo s rt d nhn bit.
p

3 + 2x2 y x4 y 2 + x4 1 2x2 = y 2
q
Bi 6: Gii h phng trnh sau
1 + 1 + (x y)2 = x3 x3 x + 2y 2 

240
Gii
H phng trnh tng ng:
q

4 (1 x2 y)2 = 2x6 x4 + y 2
q

1 + (x y)2 = 1 x6 + x4 2x3 y 2
Cng v theo v hai phng trnh ca h trn ta c:
q
q
2
2
4 (1 x y) 1 + (x y)2 = x6 2x3 y + y 2 + 1
q
q
2
2
2
4 (1 x y) = 1 + (x y)2 + x3 y + 1 ()
Ta thy:
(

1 x y = 0

V T () 6 2
V P () = V T () = 2 x y = 0 x = y = 1

V P () > 2

x3 y = 0

Th x = y = 1 vo h thy tha mn.Vy h cho c nghim duy nht (x; y) = (1; 1) 2


s


2
2

2x
+
4y
2
3

=4

(x + y) 1
xy
y x
Bi 7: Gii h phng trnh
q

p
p

(x + 1)2 + xy + 3x + 2y + 5 2x x (y + 3) = x + y + 3
Gii

x > 0

y > 3; y 6= 0


KX:
2 3

(x + y) > 0

y x

(x + 1)2 + xy + 3x + 2y + 5 2x x (y + 3) > 0
Bin i phng trnh (1):
r
x
y
x
y
(1) 2 + 4 = 4 2 3 1 1 (3)
y
x
y
x
t

x
= t (t 6= 0) , phng trnh (3) tr thnh:
y
r
4
3
2t + + 1 = 4 2t 1
t
t

Li c:
16
8
48
+ 1 + 16 + + 4t = 32t
16
2
t
t
t




16
56
4
2
2
2
4t + 2 28t +
+ 33 = 0 4 t + 2 28 t
+ 33 = 0 (5)
t
t
t
t
(4) 4t2 +

241
t t

2
= u, phng trnh (4) tr thnh:
t

7
4 u2 + 4 28u + 33 = 0 4u2 28u + 49 = 0 u =
2

t=4
7
2
2

t = 2t 7t 4 = 0
1
t
2
t=
2

Ly hai gi tr ca t va tm c thay vo phng trnh (4) th t = 4 tha mn.Suy ra x = 4y


Bin i phng trnh (2) kt hp vi h qu bin i ca phng trnh (1) l x = 4y ta c:
p
p
(2) (x + 1)2 + xy + 3x + 2y + 5 2x x (y + 3) = x + y + 2 x (y + 3)
p
p
x2 + xy + 3x 2x x (y + 3) + x + y + 3 x (y + 3) = 0
h
i
p
(x + 1) x + y + 3 2 x (y + 3) = 0
(
(

2
p
x=y+3
x=4
(x + 1)
x y+3 =0

(chn)
x = 4y
y=1
Vy h c nghim duy nht (x; y) = (4; 1) 2

log x = 2y+2 (1)


2
Bi 8: (THTT T8/415) Gii h phng trnh
p

4 1 + x + xy 4 + y 2 = 0 (2)
Gii
K: x > 0. T (2) suy ra y < 0 v nu y > 0 V T (2) > 0.
Ta bin i phng trnh (2):
p

(2) 4 1 + x = xy 4 + y 2 > 0 16(1 + x) = x2 y 2 (4 + y 2 )


4x2 y 2 16x + x2 y 4 16 = 0
(xy 2 4)(4x + xy 4 + 4) = 0
4
xy 2 4 = 0 x = 2 (3)
y
4
= 2y+2 4.2y + 2 log2 (y) 2 = 0 (5) (do y < 0)
y2
Nhn thy y = 1 l nghim ca (5). Xt f (y) = 4.2y + 2 log2 (y) 2 c f 0 (y) = 4.2y . ln 2 +
2
> 0 y < 0
y ln 2
Do hm s f (y) ng bin trn (; 0). Vy (5) c nghim duy nht y = 1. Th li ta
thy h c nghim (x; y) = (4; 1) 2.
Thay (3) vo (1) ta c log2

1
1
35

+p
=

2
12
1x
1 y2
Bi 9: Gii h phng trnh
x
y
7

p
=

12
1 x2
1 y2
Gii

242
r

r
1x
1y
t
= a v
= b.
1+x
1+y
Cng hai phng trnh v theo v ta c
r
r
1+x
1y
7
1
7
+
= + b = (1)
1x
1+y
2
a
2
Tr hai phng trnh v theo v ta c:
r
r
7
1
7
1x
1+y
+
= a + = (2)
1+x
1y
3
b
3
T ta c h phng trnh

1 + b = 7
2 + 2ab = 7b
a
2

1
7

3ab + 3 = 7b
a + =
b
3

3a2 7a + 2 = 0

b = 7 1
3 a

T tm c nghim ca h l (x; y) = (

a = 2, b = 3
1
1
a= , b=
3
2

3 4 4 3
,
), ( , ) 2
5 5
5 5

2x + 2y = 4 (1)
Bi 10: Gii h phng trnh
2x + 5 + 2y + 5 = 6 (2)
Gii
KX: x, y > 0

Cng hai phng trnh ta c 2x + 5 + 2x + 2y + 5 + 2y = 10

5
2
+
=
Tr (2) cho (1) ta c 2x + 5 2x+ 2y + 5 2y = 2
2y
+
5
+
2y
2x
+
5
+
2x

2 t a = 2x + 5 + 2x, b = 2y + 5 + 2y ta c h phng trnh:

a + b = 10
a=b=5
5 + 5 = 2
a b

Ta cn gii phng trnh 2x + 5 + 2x = 5 x = 2 (chn). Tng t c y = 2


Vy h c nghim (x; y) = (2; 2) 2

x x y 1 = 1 (1)
Bi 11: Gii h phng trnh
y 2 + x + 2y x y 2 x = 0 (2)
Gii
KX: x > 0; x y 1 > 0
T (1) ta c:

x=

p
p
p
xy1+1x=xy1+2 xy1+1y =2 xy1

y 2 = 4(x y 1) (y + 2)2 = 4x y + 2 = 2 x

243
T (2) ta c:
(y +
Ta c h mi l

x)2 = xy 2 y +

y + 2 = 2x
y + x = y x

x=y x

y = 1; x =

1
4

y = 2; x = 4

1
Th li ta thy h c nghim (x; y) = ( ; 1), (2; 4) 2
4
Bi 12: Tm a h sau c nghim thc x, y, z:

x 1 + y 1 + z 1 = a 1
x + 1 + y + 1 + z + 1 = a + 1
Gii
KX: x > 1, y > 1, z > 1
H cho tng ng
(




x1+ x+1 +
y1+ y+1 +
z 1 + z + 1 = 2a




x+1 x1 +
y+1 y1 +
z+1 z1 =2

t u = x 1 + x + 1; v = y 1 + y + 1; s = z 1 + z + 1

Do x > 1, y > 1, z > 1 nn u > 2, v > 2, s > 2 . Ngc li nu u > 2, v > 2, s >

2, ta c





2
2
1
2
1
4
2

x + 1 x 1 =
= x+1=
u+
x=
u + 2 >1x>1
u
2
u
4
u
x+1+ x1

Tng t vi y, z. Vy ta cn tm a h sau c nghim u > 2, v > 2, s > 2 :

u + v + s = 2a
() 1 1 1
(1)
+ + =1
u v s
> iu kin cn: Gi s h (*) c nghim. Theo BT Cauchy-Schwarz ta c


1 1 1
9
2a = (u + v + s)
+ +
>9a>
u v s
2
9
> iu kin : Gi s a > . Ta chng minh h (*) c nghim.
2

Ly s = 3 ( tho s > 2) . Khi (*) tr thnh

u + v = 2a 3
u.v = 3 (2a 3)
2
u, v l 2 nghim ca phng trnh t2 2 (2a 3) t +
p
2a 3 (2a 3) (2a 9)
u, v =
2

3 (2a 3)
2

244
2
t h = 2a 9 > 0 h + 6 2 2 > (h + 3)2 > h (h + 6).
p

Tc l (2a 3) 2 2 > (2a 3) (2a 9) u > 2, v > 2.

Nh vy h phng trnh c nghim u > 2, v > 2, s > 2.


9
Tm li a > l gi tr cn tm 2
2

1 2 x+ y

x( +
)=2
4
x
+
y

Bi 13: Gii h phng trnh


1 2 x+ y

4
y(
)=1
4
x+y
Gii
KX: x, y > 0
Nu x = 0 hoc y = 0 th h v nghim. Xt x, y > 0.

t u = x; v = y ta c h phng trnh

1
2u + v

u( +
)=2
4 u2 + v 2
1
2u + v

v(
)=1
4 u2 + v 2

2
1
2u + v

= + 2
4 u + v2
u

1
2u + v
1

=
4 u2 + v 2
v

2
1
1

+ =
2
u
v

2
1
4u + 2v

=
u2 + v 2
u
v

Nhn hai phng trnh ta c


4 1
2u + v
= 2
(4v u)(u2 + v 2 ) = (2u + v).uv (2v u)(u2 + 2v 2 ) = 0 u = 2v
u v
u + v2

T tm c nghim ca h l (x; y) = (64(17 + 12 2); 16(17 + 12 2)) 2

5x
+
7
5y

x +
=7
2
2
x +y

Bi 14: Gii h phng trnh

7 5x 5y

y +
=0
x2 + y 2
Gii

> x = 0 : y = 5
> y = 0 : v nghim
> x, y =
6 0:

5x2 + 7 5xy

x +
= 7x
2 + y2
x

Nhn (1) cho x v (2) cho y ta c:

7 5xy 5y 2

y 2 +
=0
x2 + y 2
2
2
Tr 2 phng trnh v theo v ta c
()
x y + 5= 7x
2
5xy + 7 5y

xy +
= 7y
2
2
x 2+ y
Nhn (1) cho y v (2) cho x ta c:

7 5x 5xy

xy +
=0
2 + y2
x
Cng 2 phng trnh v theo
v c: 2xy + 7 5 = 7y ()
x2 y 2 + 5 = 7x (3)
T (*) v (**) ta c HPT:

2xy + 7 5 = 7y (4)

245

7 2x 6= 0

Ta c: (4) y(7 2x) = 7 5


7 5

y =
7 2x
Th vo (3) ta c phng trnh
x2

245
x(4x3 28x2 + 69x 140)
+
5
=
7x

=
7x

x
=
7
(do
x
=
6
0)

y
=

5
(7 2x)2
(2x 7)2

Kt lun: H c nghim (x; y) = (0;

5), (7; 5) 2

p
x + y px y = 4x y (1)
Bi 15: Gii h phng trnh
x2 16 = 2 + y 3x (2)
Gii
Xt (1) ta c
"
p
p
(1) 2x 2 x2 y = 4x y y 2x = 2 x2 y

y = 0 (loi)
y = 4x 4

Thay y = 4x 4 vo (2) ta c

x2 16 = 2 + x 4 x2 16 3 = x 4 1

x2 25
x5
x5
1
=
(x 5)(

) = 0 ()
2
2
x4+1
x4+1
x 16 + 3
x 16 + 3

x+5
x+5
1
nn () x = 5 y = 16
>
>1>
x4+1
x2 16 + 3
x2 + 3
Th li ta thy h c nghim (x; y) = (5; 16) 2

Do

5x2 + y 2 6x 11 = 0
x4 + 2x3 p
Bi 16: Gii h phng trnh
3 y2 7 6

x 2 + x = p 2
y 7
Gii
KX: y 2 7 > 0
Xt (1) ta c:
(1) x2 (x2 + x) + x(x2 + x) 6(x2 + x) + y 2 7 4 = 0 (x2 + x 6)(x2 + x) + (y 2 7) = 4
t a = x2 + x; b =

y 2 7 ta c h phng trnh

a(a 6) + b2 = 4
a = 3b 6
b

a = 0; b = 2

a = 1; b = 3

a = 5; b = 3 (loi)

a = 6; b = 2 (loi)

246

1
T tm c nghim ca h l (x; y) = (1; 11), (0; 11), (
2

x + y + x + 3 = y 3 (1)
x
Bi 17: Gii h phng trnh
x + y + x = x + 3 (2)

51
); 4), (
; 4) 2
2

Gii
Ta c iu kin x > 0, y > 3

> Nu y = 3: thay vo (2) ta c x + 3 + x = x + 3 (v nghim)


> Nu y > 3:
Xt (1) ta c:

x + y (x + 3)
(1) x + y + x + 3 =
x
(x + y (x + 3)
x + y (x + 3)

=
x
x+y x+3
1
1

(y 3)(
)=0
x+y x+3 x

1
1

=0 x+y x+3=x
x+y x+3 x
T y ta c h phng trnh

x + y x + 3 = x
x + y + x = x + 3

Tr hai phng trnh ta c x + x + 3 = 3 x = 1 y = 8


Vy h c nghim (x; y) = (1; 8) 2

(x y)4 = 13x 4,
Bi 18: Gii h phng trnh

x + y + 3x y = 2
Gii

3x y = a
iu kin: x + y > 0 v 3x y > 0 t:
x+y =b

(a, b > 0)

phng trnh tr thnh:


2
2 4
2
2
(a b ) = 13(a + b ) 4(1)
16
4

a+b= 2
Phng trnh (1) tng ng:
(a b)4 .(a + b)4
13(a2 + b2 )
=
4
16
4
Th a + b =

2 vo phng trnh trn


= (a b)4 = 13(a2 + b2 ) 16(2)

a2 b 2
= x y H
2

247
Vi tng a (2) v mt phng trnh ng cp, ta s vit li (2) thnh:
(a b)4 =
T a + b =

13 2
(a + b2 ).2 4.4
2

2 (a + b)2 = 2 v (a + b)4 = 4. Th vo phng trnh trn:


= (a b)4 =

13 2
.(a + b2 )(a + b)2 4(a + b)4
2

= 3a4 + 2a3 b 34(ab)2 + 2ab3 + 3b4 = 0(3)

Nu b = 0 3a4 = 0 a = 0 m a + b = 2 b 6= 0 Chia 2 v ca (3) cho b4 :


a
a
a
a
(3) 3( )4 + 2( )3 34( )2 + 2 + 3 = 0
b
b
b
b
a
t t = (t > 0). Phng trnh trn tr thnh:
b
3t4 + 2t3 34t2 + 2t + 3 = 0

t=3
2
(t 3)(3t 1)(t2 + 4t + 1) = 0
1 (t + 4t + 1 > 0t > 0)
t=
3

a
Nu t = 3 = 3 kt hp vi a + b = 2
b

3x y =
x = 5
8
16

1
3

x + y =
y =
8
16

a
1
1
= kt hp vi a + b = 2
3
b
3

3x y =
x = 5
8
16

9
13

x + y =
y =
8
16
(

2x + 2 4 6 x y 2 = 2 2 (1)
Bi 19: Gii h phng trnh

4
2x + 2 6 x + 2 2y = 8 + 2 (2)

Nu t =

Gii
KX: x [0; 6]
Cng v theo v hai phng trnh ca h ta c

2

4
2x + 2 6 x + 2x + 2 4 6 x = y 2 + 6 + 3 2 (3)


Li c: V T (3) = 1. 2x + 2. 12 2x + 1. 4 2x + 2. 4 24 4x
p dng BT Cauchy-Schwarz cho V T (3) ta c
r 

p

V T (3) 6 3. (2x + 12 2x) + 3


2x + 2 6 x 6 6 + 3.6 = 6 + 3 2 ()

248

D dng nhn thy V P (3) > 6 + 3 2 () T (*), (**) v (3) suy ra

(
2x = 6 x

x=2

4
V T (3) = V P (3) = 6 + 3 2

2x = 4 6 x

y
=
2

y = 2
Th li ta thy h c nghim duy nht (x; y) = (0; 6) 2
p
x2 x y.
3
xy =y
Bi 20: Gii h phng trnh
2(x2 + y 2 ) = 11 + 32x 1
Gii
1
> 0.
2
Trong phng trnh th nht, x y v y cng du. Nu y > x th y > 0 v x y 6 0, mu
thun.
Do , x > y > 0.

t 3 x y = a > 0 x y = a3 .
Ta c (x2 x y)a2 = y 2 . Thay y = x a3 , ta c:
Ta thy 2x 1 > 0 x >

(x2 2x + a3 )a2 = (x a3 )2
ng thc ny tng ng vi:
a2 x2 2a2 x + a5 = x2 2a3 x + a6
R rng a = 1 th ng thc ng nn khi phn tch thnh nhn t, ta c:
(a 1)(x2 + 2a2 x + ax a5 ) = 0
D thy rng x2 + 2a2 x + ax a5 khng th xy ra v a5 6 max{a3 , a6 } = max{x y, (x y)2 }.
> Nu max{x y, (x y)2 } = (x y)2 th d thy a5 6 (x y)2 < x2 < x2 + 2a2 x + ax.
> Nu max{xy, (xy)2 } = xy th d thy a5 6 xy, suy ra 0 < xy 6 1 v t PT th hai,
11
vi x2 + y 2 > . Nu x 6 1 th x2 + y 2 6 2, mu thun. Suy ra x > 1 x2 > x > 1 > x y.
2
Trong c hai trng hp, ta u c x2 + 2a2 x + ax a5 > 0.
Vy a = 1 y = x 1, thay vo phng trnh th hai, ta c:

2(2x2 2x + 1) = 11 + 3 2x 1 (2x 1)2 10 = 3 2x 1


t

2x 1 = b > 0 th ta c phng trnh sau:


b4 10 = 3b (b 2)(b3 + 2b2 + 4b + 5) = 0

D thy PT ny c nghim khng m duy nht l b = 2, tng ng vi nghim ca h cho


5 3
l (x, y) = ( , ) 2
2 2

x3 3x2 + 2 = py 3 + 3y 2
Bi 21: Gii h phng trnh
p
3x 2 = y 3 + 8y

249
Gii
iu kin: x > 2, y 2 (y + 3) > 0, y(y + 8) > 0 y > 0.
Phng trnh th nht vit li l:
p
x3 3x2 + 2 = y y + 3

t x 1 = a > 1, y + 3 = b > 3, ta c:
a3 3a = b3 3b (a b)(a2 + b2 + ab 3) = 0

D thy a2 + b2 + ab > 1 + 3 + 3 > 3 nn ta c a = b hay


p
a = y + 3 y = a2 3
Bnh phng hai v ca PT th hai, ta c: 9(x 2) = y 2 + 8y hay 9(a 1) = (a2 3)(a2 + 5)
on nghim l a = 2 v phn tch thnh nhn t, ta c:
(a 2)(a3 + 2a2 + 6a + 3) = 0
Suy ra phng trnh trn c nghim duy nht l a = 2 tng ng vi nghim ca h l
(x, y) = (3, 1).
Th li ta thy tha.Vy h c nghim duy nht l (x, y) = (3, 1) 2

x + 3 = y3 6

p
Bi 22: Gii h phng trnh (I)
y + 2 = z 3 25

z + 1 = x3 + 1
Gii
t a =

x + 3, b = y + 2, c = z + 1 (a, b, c > 0) ta

3
2

a
=
b

6
ab=


3
(I) b = c2 1 25 b c =

3

c = a2 3 + 1
ca=

Li c:
(

a>0
b>0

c:
3
b2 2 b 6 = f (b)
3
c2 1 c 25 = g(c)
3
a2 3 a + 1 = h(a)

3
b2 2 > 6 > 13
b> 3

3
c> 3
c2 1 > 25 > 23

Suy ra:

3
a2 3 + 1 > 3

a > 3

a2 3 >

q
3

r ()
3 1
31>
2

Ta c:
0

2
2

f
(b)
=
3
b

2
.2b

1
>
3.1.2
3

1
>
0
b
>
3

g 0 (c) = 3 c2 1 2 .2c 1 > 3.22 .2 3 1 > 0 c > 3

 2

1 3
1
0
2

.2 3 1 > 3. .2 3 1 > 0 a()


h (a) = 3 a 3 .2a 1 > 3.
2
2

250
Suy ra: f (b), g(c), h(a) l hm ng bin v f (2) = g(2) = h(2) = 0.
> Nu a > 2 :
h(a) > h(2) = 0 c > a > 2 g(c) > g(2) = 0 b > c > 2
f (b) > f (2) = 0 a > b > 2 a > b > c > a
Vy trng hp ny loi. L lun tng t vi a < 2.
Vy ta c:
a = 2 c = a + h(a) = 2 b = c + g(c) = 2

x
+
3
=
2

x = 1

p
a=b=c=2
y+2=2 y =2

z+1=2
z = 3
Th li ta thy h phng trnh c 2 nghim l: (x; y; z) = (1; 2; 3) 2
~ Nhn xt: Nu c th on nghim ca h, ta cng c li gii p nh sau:
Ta c:

x1

= (y 2)(y 2 + 2y + 4)

x + 3 2 = y 8
xy+32+ 2
3
= (z 3)(z 2 + 3z + 9)
(I)
y + 2 2 = z 27
y
+
2
+
2

z + 1 2 = x3 1
z3

= (x 1)(x2 + x + 1)
z+1+2

x 1 = (y 2)(y + 2y + 4)(x + 3 + 2) (1)


y 2 = (z 3)(z 2 + 3z + 9)( y + 2 + 2) (2)

z 3 = (x 1)(x2 + x + 1)( z + 1 + 2) (3)


>Nu x 6= 1 : y6= 2, z 6= 3

(1) |x 1| > |y 2|
T h trn ta c: (2) |y 2| > |z 3|

(3) |z 3| > |x 1|

(mu thun)

Vy x = 1 y = 2, z = 3

2x + 2y xy = 3
Bi 23: Gii h phng trnh
3x + 1 + 3y + 1 = 4
Gii
Nnhn thy phng trnh th nht ca h c h s bc nht ca x; y t l vi nhau nn ta
thm bt a v ng h s.
3
3
9

T phng trnh th nht ta c 2x + 2y xy = 3 (2x + 2y)


xy =
2
2
2

2
a

3x + 1 = a
x =
3
t

2
3y + 1 = b

y = b 1
3
(3x + 1) + (3y + 1) 3 xy = 13
2
2
H phng trnh cho tng ng
3x + 1 + 3y + 1 = 4

251
Thay a; b vo h ta c

p
a2 + b2 1 (a2 1)(b2 1) = 13
2
2
a + b = 4

S = a + b
t
P = ab

v th (2) vo (1) ta c

P 2 + 2P 15 = 19 4P (vi P 6

19
)
4

94
(loi) S = P = 4
15

3x + 1 = 2
2
Vy a, b l nghim ca phng trnh X 4X + 4 = 0 a = b = 2
3y + 1 = 2
P 2 154P + 376 = 0 P = 4 P =

Gii h ny ta thu c (x; y) = (1; 1) 2


~ Nhn xt: Ngoi cch gii nh trn ta vn c th lm nh sau: Nhn phng trnh th nht
cho 2, nhn phng trnh th hai cho 4, v tr v theo v ta s a c v dng tng cc bnh
phng khng m A2 + B 2 + C 2 = 0.

x3 xy 2 + 2000y = 0
Bi 24: Gii h phng trnh
y 3 x2 y 500x = 0
Gii
T phng trnh (1) ta phn tch nh sau x(x2 y 2 ) + 2000y = 0 x2 y 2 = 2000
M phng trnh (2) ta cng phn tch c y(y 2 x2 ) 500x = 0
y2
Vy th (1) vo (2) ta c 2000 500x = 0 x = 4y 2 x = 2y
x
Vi x = 2y thay vo (1) ta c 6y 3 + 2000y = 0 y(6y 2 + 2000) = 0
y = 0 x = 0 V 6y 2 + 2000 > 0y R
3
Vi x = 2y
thay vo (1) ta
c 6y + 2000y = 0(y 6= 0)
10 30
20 30
y=
x=
3
3

!
20 30 10 30
20 30 10 30
Vy h cho c nghim (x; y) = (0; 0);
;
;
;
2
3
3
3
3

33x2y 5.6x + 4.23x2y = 0


Bi 25: Gii h phng trnh
x y = y + (2y x)(2y + x)2
Gii
iu kin x > y > 0

T phng trnh (2) ta c x y y = ( 2y x)( 2y + x)2

x 2y
2y x
( 2y + x)2

=
x y + y
2y + x


1
(x 2y)
+ 2y + x = 0
xy+ y

y
x

252
Vi x = 2y thay vo (1) ta c 34y 5.62y + 4.24y = 0 92y 5.62y + 4.42y = 0
2
Chia 2 v ca phng trnh cho 92y v t a = ( )2y (0 < a < 1) ta c:
3

1
a=
4
4a2 5a + 1 = 0
a = 1 (loi)
 2y
1
1
2
= y = log 3 4 x = log 3 4
Vy ta c
3
4
2
2
2
1
Vy h cho c nghim (x; y) = (log 3 4; log 3 4) 2
2
2
2
Bi 26: Gii h phng trnh
(

x11 + xy 10 = y 22 + y 12 (1)
p
7y 4 + 13x + 8 = 2y 4 3 x (3x2 + 3y 2 1) (2)

( thi chn i tuyn TP HCM 2009-2010)

Gii
8
13
Xt x = 0 t (1) suy ra y = 0 nhng (x; y) = 0 khng tho (2).
Xt xy 6= 0. T phng trnh (1) ta c
 1
x
x
1 + = y11 + y
y
y
Xt y = 0, thay vo h tm c x =

Xt hm s f (t) = t11 + t ta c f 0 (t) = 11t10 + 1 > 0 t R f (t) ng bin trn R. T


ta c:
x
(1) f ( ) = f (y) x = y 2 > 0
y
Khi (2) tr thnh:
7x2 + 13x + 8 = 2x2

1
3
3
x(3x2 + 3x 1) 7t + 13t2 + 8t3 = 2 3 + 3t t2 vi t = > 0 (3)
x

Phng trnh ny c th gii bng phng php dng hm s n iu (xem chng Cc


phng php gii phng trnh). Li gii c th nh sau:

3
(3) (2t + 1)3 + 2(2t + 1) = 3 + 3t t2 + 2 3 + 3t t2

3
g(2t + 1) = g( 3 + 3t t2 ) vi g(t) = t3 + 2t
Do g 0 (t) = 3t2 + 2 > 0 t > 0 nn ta suy ra

t = 1 (loi)

89

3
(loi)
2t + 1 = 3 + 3t t2 (2t + 1)3 = 3 + 3t t2 t =
16

89 5
t=
(chni)
16

253
r
16
16
Suy ra x =
y=
89 5
89 5
r
8
16
16
Vy h c nghim (x; y) = ( ; 0), (
;
)2
13
89 5
89 5

(1 + 4xy )51x+y = 1 + 3xy+2


r
Bi 27: Gii h phng trnh
1
2

x 3y y = 1 2y
x
Gii
K:y > x > 0 hoc y 6 x 6 0
t x y = u. Phng trnh th nht tr thnh:
(1 + 4u )51u = 1 + 9.3u (1)
> Nu u > 0 th V T < 5 + 5 = 10 cn V P > 1 + 9 = 10 (VN)
> Nu u < 0 th V T > 5 + 5 = 10 cn V P < 1 + 9 = 10 (VN)
> Nu u = 0 th x = y
Thay vo phng trnh sau ta c:
r
1
2
x 3x x = 1 2x (K: x > 1 hoc 1 6 x < 0)
x
p
3x2 3 x(x2 1) = 1 2x ()

t a = x2 1(a > 0), b = x th (*) tr thnh:

a=b
x2 1 = x
1 5

x=
a2 3ab + 2b2 = 0

x
=
2

5
2
a = 2b
x 1=2 x

1 5
Vy h c nghim x = y =
v x = y = 2 5 2
2

x2 + 91 = y 2 + y 2 (1)
Bi 28: Gii h phng trnh p
y 2 + 91 = x 2 + x2 (2)
Gii
K: x, y > 2
Ly (1) tr (2) ta c:

x2 + 91

y 2 + 91 =

y 2 + y 2 x 2 x2

x2 y 2
yx
p

=
+ (y 2 x2 )

2
2
x2+ y2
x + 91 + y + 91
x+y
1
p
(x y)(
+
+ x + y) = 0

2
2
x2+ y2
x + 91 + y + 91
x=y

254
Thay vo h ban u ta c:

x2 + 91 = x 2 + x2

x2 + 91 10 = x 2 1 + (x2 9)
x3
x2 9
=
+ (x 3)(x + 3)

2
x2+1
x + 91 + 10

x=3

1
x+3

=
+ (x + 3)
x2+1
x2 + 91 + 10
x+3
x+3
1
<x+3
<x+3+
x2+1
x2 + 91 + 10
x2 + 91 + 10
Do x = 3 (tho x > 2). Vy h c nghim x = y = 3 2
D thy

p
x4 y 4 (x2 + y 2 ) + 1 = 1 xy (1)
Bi 29: Gii h phng trnh
p

1 x2 + 1 y 2 = 2 (2)

Gii

1 6 x, y 6 1
KX:
x4 y 4 (x2 + y 2 ) + 1 > 0

a > 2b
2
2
t a = x + y ; b = xy
1 6 b 6 1

T (1) ta c: b4 a + 1 = 1 b b4 a + 1 = b2 2b + 1 a = b4 b2 + 2b
Li c a > 2b b4 b2
+ 2b > 2b b2 (b2 1) > 0 () Do b [1; 1] b2 (b2 1) 6 0 nn
a = 2b
2 2
() b (b 1) = 0
T a = 2b x = y. Vy ta c:
b {0; 1}
> Nu b = 0 x = y = 0 (tho (2)).
> Nu b = 1 x = y = 1 (khng tha (2)).
> Nu b = 1 x2 = 1 (v nghim).
Kt lun: H c nghim (x; y) = (0; 0) 2

2 xy + x + y
2xy
x2 + y 2

+
=
2
2
Bi 30: Gii h phng trnh x + y

2012x+y1 3x + y + 1 = p(2x 1)2 + x y + 1

Gii

255
iu kin: xy > 0
Phng trnh (1) vit li:
x2 + y 2
xy
2xy
x+y
x +y
4xy (x + y)

+
xy = 0
+r 2
=0
x+y
2
2
2(x + y)
x2 + y 2
+ xy
2

x=y
2
2

1
1
(x y)
(x y)
=r
(3)
+ r

=0

2
2
2
x+y
2(x + y)
x + y2
x +y

+ xy
2(
+ xy)
2
2
r

Xt (3) ta c:
r

p
x2 + y 2

+ xy 2(x + y) = 2(x2 + y 2 ) + 2 xy
2

2(x2 + y 2 ) (x + y)2
2 p 2

( x y) = 2(x + y 2 ) (x + y) ( x y)2 = p
2(x2 + y 2 ) + x + y

(x y)2
2
2 ( x y)2 .( x + y)2
( x y) = p
( x y) = p
2(x2 + y 2 ) + x + y
2(x2 + y 2 ) + x + y

x=y

( x + y)2

1= p
(4)
2(x2 + y 2 ) + x + y

(3) x + y =

Xt (4) ta c:
(4)

p
p

2(x2 + y 2 ) + x + y = ( x + y)2 2(x2 + y 2 ) = 2 xy (5)

T (3) x + y > 0 m xy > 0 x, y > 0


p dng bt ng thc Cauchy cho 2 s dng x,y ta chng minh c (5) xy ra khi v ch
khi x = y. Hp tt c cc trng hp, kt lun (1) x = y
p
Th y = x vo phng trnh (2), ta c: 20122x1 2x + 1 = (2x 1)2 + 1.
t 2x 1 = t. Phng trnh trn tr thnh:
2012t = t +

t2 + 1 ln(t +

t2 + 1) t ln 2012 = 0 (6)

1
+

2
1
t + 1 ln 2012 =
Xt f (t) = ln(t+ t2 + 1)t ln 2012. Ta c f 0 (t) =
ln 2012 <
t + t2 + 1
t2 + 1
0 vi mi t R f (t) nghch bin trn R
M f (0) = 0 (6) c duy nht mt nghim t = 0
t+1
1
1
Ta c t = 0 x =
= y=x=
2
2
2
1 1
Vy h c duy nht 1 nghim (x; y) = ( ; ) 2
2 2

Chng VI: SNG TO PHNG TRNH - H


PHNG TRNH

Xy dng mt s phng trnh c gii bng cch a


v h phng trnh
V d 1. Xt h i xng loi hai
(

x = 2 3y 2
2 2
.

x
=
2

3
2

3x
y = 2 3x2
Ta c bi ton sau
Bi 1 (THTT, s 250, thng 04/1998). Gii phng trnh
x + 3 2 3x2

2

= 2.

Gii
t y = 2 3x2 . Ta c h
(

x + 3y 2 = 2

y = 2 3x2

x = 2 3y 2 (1)
y = 2 3x2 (2)

Ly (1) tr (2) ta c
"
x y = 3(x2 y 2 )

y=x
xy =0

1 3x
3(x + y) = 1
y=
.
3

Vi y = x, thay vo (1) ta c


3x + x 2 = 0 x
Vi y =

2
1,
3


.

1 3x
, thay vo (2) ta c
3

1 3x
1 21
2
2
= 2 3x 9x 3x 5 = 0 x =
.
3
6

2 1 21
Kt lun: Phng trnh cho tp nghim S = {1; ;
}2
3
6
2
~ Nhn xt: T li gii trn ta thy rng nu khai trin (2 3x2 ) th s a phng trnh
cho v phng trnh a thc bc bn, sau bin i thnh
(x + 1)(3x 2)(9x2 3x 5) = 0.
256

257
Vy nu khi xy dng bi ton, ta c lm cho phng trnh khng c nghim hu t th
phng php khai trin a v phng trnh bc cao, sau phn tch a v phng trnh
tch s gp nhiu kh khn.
V d 2. Xt mt phng trnh bc hai c c hai nghim l s v t
5x2 2x 1 = 0 2x = 5x2 1.
(
Do xt

2y = 5x2 1
2x = 5
2x = 5y 2 1

5x2 1
2

2
1. Ta c bi ton sau.

Bi 2. Gii phng trnh


2
8x 5 5x2 1 = 4.
Gii
t 2y = 5x2 1. Khi
(

2y = 5x2 1

8x 5.4y 2 = 4

2y = 5x2 1 (1)
2x = 5y 2 1. (2)

Ly (1) tr (2) theo v ta c


"
2(y x) = 5(x2 y 2 )

y=x
yx=0

5x + 2
2 = 5(x + y)
y=
.
5

Vi y = x, thay vo (1) ta c

6
.
5x2 2x 1 = 0 x =
5
Vi y =

5x + 2
, thay vo (1) ta c
5

10x + 4
5 50
2
2

= 5x 1 25x + 10x 1 = 0 x =
.
5
25

1 6 1 2
Phng trnh cho c tp nghim S = {
,
}2
5
5
~ Nhn xt: Php t 2y = 5x2 1 c tm ra nh sau: Ta t ay + b = 5x2 1, vi a, b tm
sau. Khi thu c h
(
(
ay + b = 5x2 1
ay + b + 1 = 5x2

8x 5 (ay + b)2 = 4
8x + 4 5b2 = 5a2 y 2 + 10aby.

(
a = 5 = b+1
b=0
h trn l h i xng loi II th
8
5a2
4 5b2
10ab = 0
a = 2.
Vy ta c php t 2y = 5x2 1. Sau y l mt bi tng t:
Bi 3. Gii phng trnh 5(5x2 17)2 343x 833 = 0
Gii

258

ay + b = 5x2 17
2
~ tng: t ay + b = 5x 17 (a 6= 0)
5(ay + b)2 343x 833 = 0 ()
T (*) c
5(ay)2 + 10aby + b2 343x 833 = 0
5(ay)2 + 10aby + b2 833
343
5a3 .y 2 + 10a2 .by + b2 .a 833a
ax + b =
+ b ()
343
Ta hi vng c ax + b = 5y 2 17, kt hp vi (**) suy ra
x=

5a3 .y 2 + 10a2 .by + b2 .a 833a


+b
343
343.5y 2 5831 = 5a3 .y 2 + 10a2 .by + b2 .a 833a + 343b

343 = a3

a = 7

ng nht h s ta c a b = 0
b = 0

833a + 343b = 5831


5y 2 17 =

Vy ta c li gii sau:
~ Li gii:
t 7y = 5x2 + 17 ta c h phng trnh

7y = 5x2 17
245y 2 343x 833 = 0

7y = 5x2 17 (1)

7x = 5y 2 17 (2)

Ly (1) tr (2) ta c
"
7(y x) = 5(x + y)(x y)

x=y
5x + 5y = 7

> Nu x = y, thay vo (1) c


2

5x 7x 17 = 0 x =

389
10

> Nu 5x + 5y = 7, kt hp (2) ta c

5(5x2 17)
35 5 193
2
7 + 5x +
= 0 25x + 35x 36 = 0 x =
7
50

7 389 35 5 193
Kt lun: Phng trnh c tp nghim S = {
;
}2
10
50
V d 3. Xt mt phng trnh bc ba

3
3
4x 3x =
8x3 6x = 3 6x = 8x3 3
2
Do ta xt
(

6y = 8x3 3
6x = 8
6x = 8y 3 3

8x3
6

!3
3

259


3
1296x + 216 3 = 8 8x3 3

3

162x + 27 3 = 8x3 3 .
Ta c bi ton sau

3
Bi 4. Gii phng trnh 162x + 27 3 = 8x3 3 .
Gii
t 6y = 8x3

3. Ta c h
(
(

6y = 8x3 3
6y = 8x3 3 (1)

162x + 27 3 = 216y 3
6x = 8y 3 3 (2)

Ly (1) tr (2) theo v ta c





6(y x) = 8(x3 y 3 ) (x y) 8 x2 + xy + y 2 + 6 = 0.

(3)

V x2 + xy + y 2 > 0 nn 8 (x2 + xy + y 2 ) + 6 > 0. Do t (3) ta c x = y.


Thay vo (1) ta c
3

6x = 8x

3 4x 3x =

5
3
4x3 3x = cos
2
6

(4)

3 cos , ta c
3
3

S dng cng thc cos = 4 cos3

5
5
5
= 4 cos3
3 cos ,
6
18
18
17
17
17
cos
= 4 cos3
3 cos
,
6
18
18
7
7
7
cos
= 4 cos3
3 cos .
6
18
18

cos

5
17
7
, x = cos
, x = cos
l tt c cc nghim ca phng trnh (4) v cng l
18
18
18
tt c cc nghim ca phng trnh cho 2

~ Nhn xt: Php t 6y = 8x3 3 c tm ra nh sau :

Ta t ay + b = 8x3 3. Khi t phng trnh cho c h


(

ay + b = 8x3 3

162x + 27 3 = a3 y 3 + 3a2 by 2 + 3ab2 y + b3 .


Vy x = cos

Cn chn a v b sao cho

(
8
b+ 3
a
b=0

= 3 =
162
a
27 3 b3
2
a = 6.
3a b = 3ab2 = 0
Vy ta c php t 6y = 8x3

3.

260
V d 4. Ta s xy dng mt phng trnh v t c t nht mt nghim theo mun. Xt
x = 3. Khi
do x=3
2x 5 = 1 (2x 5)3 = 1 = x 2.

Ta mong mun c mt phng trnh cha (ax + b)3 v cha 3 cx + d, hn na phng trnh
ny c gii bng cch a v h "gn" i xng loi hai (ngha l khi tr theo v hai phng
trnh ca h ta c tha s (x y)). Vy ta xt h
(
(2y 5)3 = x 2
(2x 5)3 = x + 2y 2.
Nu c php t 2y 5 =

x 2, th sau khi thay vo phng trnh


(2x 5)3 = x + 2y 2

ta c
8x3 60x2 + 150x 125 = x +

x 2 + 5 2.

Ta c bi ton sau
Bi 5. Gii phng trnh

x 2 = 8x3 60x2 + 151x 128.


Gii

Cch 1:
Tp xc nh R. Phng trnh vit li

3
t 2y 5 =

x 2 = (2x 5)3 + x 3.

x 2. Kt hp vi (1) ta c h
(
(2y 5)3 = x 2
(2)
(2x 5)3 = x + 2y 2 (3)

Ly (3) tr (2) theo v ta c




2 (x y) (2x 5)2 + (2x 5) (2y 5) + (2y 5)2 = 2(y x)
"
xy =0
(4)

2
(2x 5) + (2x 5) (2y 5) + (2y 5)2 + 1 = 0. (5) (v nghim)
Ta c (4) y = x. Thay vo (2) ta c
(2x 5)3 = x 2 8x3 60x2 + 149x 123 = 0
(x 3)(8x2 36x + 41) = 0 x = 3.
Vy phng trnh c nghim duy nht x = 3 2
~ Nhn xt: T nghim duy nht trn, ta ngh n cch dng n iu hm s nh sau:

(1)

261
Cch 2.

Tp xc nh R. t y = 3 x 2. Ta c h
(
8x3 60x2 + 151x 128 = y
x = y3 + 2
Cng v theo v hai phng trnh ca h ta c
8x3 60x2 + 152x 128 = y 3 + y + 2
8x3 60x2 + 150x 125 + 2x 5 = y 3 + y
(2x 5)3 + (2x 5) = y 3 + y.

(*)

Xt hm s f (t) = t3 + t. V f 0 (t) = 3t2 + 1 > 0, t R nn hm f ng bin trn R. Do


() vit li
f (2x 5) = f (y) 2x 5 = y.
Bi vy
(2x 5) =

x 2 (2x 5)3 = x 2

8x3 60x2 + 149x 123 = 0


(x 3)(8x2 36x + 41) = 0 x = 3.
Phng trnh c nghim duy nht x = 3 2

V d 5. Xt mt phng trnh bc ba no , chng hn xt 4x3 + 3x = 2. Phng trnh ny


tng ng

8x3 + 6x = 4 8x3 = 4 6x 2x = 3 4 6x.


Ta "lng ghp" phng trnh cui vo mt hm n iu nh sau
(2x3 ) + 2x =

4 6x + 4 6x 8x3 + 8x 4 =

4 6x.

Ta c bi ton sau:
Bi 6. Gii phng trnh
8x3 + 8x 4 =

3
4 6x.

Gii
Tp xc nh ca phng trnh l R.
Cch 1:
Phng trnh cho tng ng
(2x)3 + 2x =

3
4 6x + 4 6x.

(1)

Xt hm s f (t) = t3 + t, t R. V f 0 (t) = 3t2 + 1 > 0, t R nn hm s f (t) ng bin trn




R. M (1) vit li f 3 4 6x = f (2x) nn n tng ng

3
4 6x = 2x 8x3 + 6x = 4 4x3 + 3x = 2.

(2)

262
V hm s g(x) = 4x3 + 3x c g 0 (x) = 12x2 + 3 > 0, x R nn PT (2) c khng qu mt
nghim. Xt



1
1
3
2=
3 (3 )2 43 1 3 = 2 5.
2



p

1
1
3
3
Do , nu t = 2 + 5 th 2 =
3 . Ta c
2



 

 
3
1
1
1
1
1
1
3
3 =3

+4

.
2



p


1
1
1 p
3
3
Vy x =

=
2 + 5 + 2 5 l nghim duy nht ca (2) v cng l
2

2
nghim duy nht ca phng trnh cho 2
Cch 2:
Phng trnh vit li thnh
(2x)3 =

6x + 4 8x + 4.

t 2y = 3 4 6x. Ta c h
(
(
8y 3 = 4 6x
8y 3 = 6x + 4
(a)

8x3 + 8x 4 = 2y
8x3 = 2y + 4 8x. (b)
Ly (b) tr (a) theo v ta c
8(x3 y 3 ) = 2(y x) (x y)[4(x2 + xy + y 2 ) + 1] = 0 y = x.
Thay y = x vo (a) ta c
8x3 = 6x + 4 4x3 + 3x = 2.
n y lm ging cch 1.
Bi 7. Gii phng trnh

6x + 1 = 8x3 4x 1

( ngh Olympic 30-4-2006)


Gii

Tp xc nh ca phng trnh l R. t 3 6x + 1 = 2y. Ta c h


(
(
8x3 4x 1 = 2y
8x3 = 4x + 2y + 1 (1)

6x + 1 = 8y 3
8y 3 = 6x + 1.
(2)
Ly (1) tr (2) theo v ta c
8(x3 y 3 ) = 2(y x) (x y)[4(x2 + xy + y 2 ) + 1] = 0 y = x.
Thay y = x vo (2) ta c

8x3 6x = 1 4x3 3x = cos .


3

(3)

263

3 cos , ta c
3
3

cos = 4 cos3 3 cos ,


3
9
9
7
7
7
cos
= 4 cos3
3 cos ,
3
9
9
5
5
5
cos
= 4 cos3
3 cos .
3
9
9

S dng cng thc cos = 4 cos3

5
7
Vy x = cos , x = cos , x = cos
l tt c cc nghim ca phng trnh (3) v cng l
9
9
9
tt c cc nghim ca phng trnh cho 2
~ Nhn xt: Ta cn c th gii cch khc nh sau : Phng trnh vit li

6x + 1 + 3 6x + 1 = (2x)3 + 2x.
(3)
Xt hm s f (t) = t3 + t, t R. V f 0 (t) = 3t2 + 1 > 0, t R nn hm s f (t) ng bin trn


R. M (2) c th vit li thnh f 3 6x + 1 = f (2x) nn tng ng

1
6x + 1 = 2x 8x3 6x = 1 4x3 3x = .
2

V d 6. Xt mt tam thc bc hai lun nhn gi tr dng : x2 + 2. Khi


Z

x3
2
+ 2x + C.
x + 2 dx =
3
x3
+ 2x. Ta c h(3) = 15.
3
Vy ta thu c mt hm s a thc bc ba ng bin g(x) v tho mn g(3) = 0 l
Ch cn chn C = 0 ta c mt a thc bc ba ng bin l h(x) =

g(x) =

x3
+ 2x 15.
3

Ta s tm mt a thc bc ba ng bin k(x) sao cho k(x) = x g(x) = 0, mun vy ta xt


x3
x3
+ x 15 = x
+ ( 1)x 15 = 0.
3
3
Do chn sao cho 1 = 2 = 3, khi k(x) =

x3
+ 3x 15 v k(x) = y tng ng
3

vi

x3
+ 3x 15 = y x3 + 9x 45 = 3y.
3
T phng trnh cui ny thay x bi y ta thu c h i xng loi hai
(
x3 + 9x 45 = 3y
y 3 + 9y 45 = 3x.
T h trn, s dng php th ta thu c phng trnh
 3
3
 3

x + 9x 45
x + 9x 45
+9
45 = 3x
3
3
3

x3 + 9x 45 + 81 x3 + 9x 45 = 1215 + 81x.

Vy ta thu c bi ton sau.

264
Bi 8. Gii phng trnh
3

x3 + 9x 45 + 81 x3 + 9x 45 = 1215 + 81x (1)
Gii
Tp xc nh R. t x3 + 9x 45 = 3y. Kt hp vi (1) ta c h
(
x3 + 9x 45 = 3y (2)
y 3 + 9y 45 = 3x. (3)
Ly (2) tr (3) theo v ta c
x3 y 3 + 9x 9y = 3y 3x x3 y 3 + 12(x y) = 0
(x y)(x2 + xy + y 2 + 12) = 0 x = y.
Thay vo (2) ta c

x3 + 9x 45 = 3x (x 3) x2 + 3x + 15 = 0 x = 3.
Phng trnh cho c nghim duy nht x = 3. 2
~ Nhn xt: Php t x3 + 9x 45 = 3y c tm ra nh sau: Ta t x3 + 9x 45 = ay. Khi

(
(
3
x + 9x 45 = ay
x3 + 9x 45 = ay

a3 y 3 + 81ay = 1215 + 81x


a3 y 3 + 81ay 1215 = 81x.
a3
81a
1215
81
=
=
=
a = 3.
1
9
45
a
3
Do t x + 6x 45 = 3y, ta s thu c mt h i xng loi hai.
Vy cn chn a tho mn iu kin

V d 7. Chn mt phng trnh ch c hai nghim l 0 v 1 l 11x = 10x + 1. T phng


trnh ny ta thit lp mt h i xng loi hai, sau li quay v phng trnh nh sau :
(
(
11x = 10y + 1
y = log11 (10x + 1)
11x 1

= log11 (10x + 1)
10
11y = 10x + 1
11x = 10y + 1
11x = 10 log11 (10x + 1) + 1 11x = 2 log11 (10x + 1)5 + 1.
Ta c bi ton sau.
Bi 9. Gii phng trnh 11x = 2 log11 (10x + 1)5 + 1.
Gii
1
. t y = log11 (10x + 1), khi 11y = 10x + 1. Kt hp vi phng trnh
10
cho ta c h
(
11x = 10y + 1 (1)
11y = 10x + 1 (2)
iu kin x >

Ly (1) tr (2) theo v ta c


11x 11y = 10y 10x 11x + 10x = 11y + 10y.

(3)

265
Xt hm s f (t) = 11t + 10t. Ta c f 0 (t) = 11t ln 11 + 10 > 0, t R. Vy hm s f ng bin
trn R. M (3) chnh l f (x) = f (y) nn x = y. Thay vo (1) ta c
11x = 10x + 1 11x 10x 1 = 0.


1
x
Xt hm s g(x) = 11 10x 1 trn khong ; + . Ta c
10

(4)

g 0 (x) = 11x ln 11 10, g 00 (x) = 11x (ln 11)2 > 0.




1
Vy hm s g c th lun lm trn khong ; + , suy ra th ca hm g v trc
10
honh c vi nhau khng qu hai im chung, suy ra (4) c khng qu 2 nghim. M g(1) = 0,
g(0) = 0 nn x = 0 v x = 1 l tt c cc nghim ca (4).
Vy nghim ca phng trnh cho l x = 0 v x = 1. 2

V d 8. Ta s s dng phng php lp sng tc phng trnh t h phng trnh i


xng loi hai. Xut pht t
(

4x = 30 + u

4u = x + 30,
r
1
x + 30. T phng trnh ny ta li
s dng php th ta c phng trnh 4x = 30 +
4
thu c h i xng loi hai

r
1

4u = 30 +
x + 30
4
r

4x = 30 +
u + 30.
4
T h ny, tip tc s dng php th ta thu c phng trnh
v
s
u
r
u
1
1
1
t
4x = 30 +
30 +
30 +
x + 30.
4
4
4
Ta c bi ton sau.
Bi 10 ( ngh Olympic 30/04/2010). Gii phng trnh
v
s
u
r
u
1
1
1
t
4x = 30 +
30 +
30 +
x + 30.
4
4
4
Gii
x l nghim th x > 0. t u =

1
4

r
30 +

1
x + 30, t phng trnh cho ta c h
4

4u = 30 +
r

4x = 30 +

1
x + 30
4
1
u + 30.
4

(1)

266
Gi s x > u. Khi
r
4u =

1
30 +
x + 30 >
4

r
30 +

1
u + 30 = 4x u > x x = u.
4

Vy t h (1) ta c x = u v
r
4x =
t v =

30 +

1
x + 30.
4

(2)

1
x + 30, t (2) ta c h
4
(

4x = 30 + v

4v = x + 30.

(3)

Gi s x > v. Khi

x + 30 > v + 30 = 4x 4v > 4x v > x v = x.


(

x>0
1 + 1921
Vy v = x v 4x = x + 30
x=
. Phng trnh cho c
32
16x2 = x + 30

1 + 1921
.2
nghim duy nht x =
32
4v =

S dng cng thc lng gic sng tc cc phng


trnh a thc bc cao
Trong mc ny ta s dng mt s cng thc lng gic sng tc ra cc phng trnh a
thc bc cao. Vic gii cc phng trnh a thc bc cao l rt phc tp, trong nhiu trng
hp l khng th. Tuy nhin s dng tnh cht phng trnh a thc bc n (n = 1, 2, . . . ) c
khng qu n nghim, v mt s nh hng trong qu trnh sng tc ton, ta c c li
gii rt ngn gn v n tng cho cc phng trnh dng ny.
V d 9. T cng thc
cos 6 = 32 cos6 48 cos4 + 18 cos2 1
Ly cos = x ta c
cos 6 = 32x6 48x4 + 18x2 1
Chn 6 =

ta c
3
32x6 48x4 + 18x2 1 =

1
2

Ta c bi ton sau.
Bi 11 ( ngh OLYMPIC 30/04/2009). Gii phng trnh
64x6 96x4 + 36x2 3 = 0.
Gii

267
Ta c
cos 6 = 2 cos2 3 1 = 2 4 cos3 3 cos

2

= 32 cos6 48 cos4 + 18 cos2 1.

(1)

Phng trnh cho tng ng


1

32x6 48x4 + 18x2 1 = cos .


(2)
2
3


k2

+
, k = 1, 5. 2
T (1) suy ra (2) c 6 nghim l x = cos
3.6
6
~ Nhn xt: Vic s dng cng thc biu din cos n theo cos , sin n theo sin s gip ta
gii c nhng phng trnh dng ny.
32x6 48x4 + 18x2 1 =

x
V d 10. T cos 5 = 16 cos5 20 cos3 + 5 cos , t cos = ta c
2 3
16x5
20x3
5x
x5
5x3
5x
x5 15x3 + 45x

cos 5 =

+
=

+
=
.
288 3 24 3 2 3
18 3 6 3 2 3
18 3

3
x5 15x3 + 45x

=
x5 15x3 + 45x 27 = 0. Ta c bi ton sau.
Chn 5 = c
6
2
18 3
Bi 12 ( ngh Olympic 30/04/2011). Gii phng trnh
x5 15x3 + 45x 27 = 0.
Gii

Tp xc nh R. t x = 2 3t, thay vo phng trnh cho ta c

288 3t5 360 3t3 + 90 3t 27 = 0




2 16t5 20t3 + 5t = 3 16t5 20t3 + 5t = cos .


6

(1)

Mt khc ta c
cos 5 + cos = 2 cos 3 cos 2


cos 5 = 2 4 cos3 3 cos 2 cos2 1 cos

cos 5 = 2 8 cos5 10 cos3 + 3 cos cos
cos 5 = 16 cos5 20 cos3 + 5 cos .
(2)



k2
T (2) suy ra (1) c 5 nghim l t = cos
+
, k = 0, 4 Vy phng trnh cho c 5
6.5
5



k2
nghim l x = 2 3 cos
+
, k = 0, 4. 2
30
5

~ Nhn xt: Trong li gii trn, php t x = 2 3t tm ra nh sau : Do cng thc


cos 5 = 16 cos5 20 cos3 + 5 cos ,
nn t x = at, vi a s tm sau. Thay x = at vo phng trnh cho ta c
a5 t5 15a3 t3 + 45at 27 = 0.

a5
15a3
45a
a4
3a2
Ta tm a tho iu kin
=
=

=
= 9 a = 2 3. Vy ta c php t
16
20
5
16
4

x = 2 3t.

268
V d 11. T sin 5 = 16 sin5 20 sin3 + 5 sin , ly sin = 2x ta c sin 5 = 512x5

160x3 + 10x. Chn 5 = , ta c


3

3
= 512x5 160x3 + 10x 1024x5 320x3 + 20x 3 = 0.
2
Ta c bi ton sau.
Bi 13. Gii phng trnh 1024x5 320x3 + 20x

3 = 0.

Gii
t
t x = , thay vo phng trnh cho ta c
2

32t5 40t + 10 = 3 16t5 20t3 + 5t = sin .


3

(1)

Ta c
sin 5 + sin = 2 sin 3 cos 2


sin 5 = 2 3 sin 4 sin3 1 2 sin2 sin

sin 5 = 2 8 sin5 10 sin3 + 3 sin sin
sin 5 = 16 sin5 20 sin3 + 5 sin .


k2

+
3.5
5

(2)

, k = 0, 1, 2, 3, 4. Phng trnh cho


T (2) suy ra (1) c 5 nghim l t = sin



k2
1
+
c 5 nghim l x = sin
, k = 0, 1, 2, 3, 4. 2
2
15
5

V d 12. Ta xt phng trnh x3 3x = x + 2. D thy x = 2 l nghim ca phng trnh


ny. Ta khng i theo con ng lng gic m s bin i i s ngay t phng trnh ban
u, c th l bnh phng hai v ta c
x6 6x4 + 9x2 x 2 = 0

(x 2) x5 + 2x4 2x3 4x2 + x + 1 = 0.
Ta ch xt x5 + 2x4 2x3 4x2 + x + 1 = 0. Li t thm n ph che lp vn k hn:
thay x bi 2x, ta c 32x5 + 32x4 16x3 16x2 + 2x + 1 = 0. Ta c bi ton sau.
Bi 14 ( ngh Olympic 30/04/2011). Gii h phng trnh
(
x2 + 4y 2 = 1

16x5 20x3 + 5x + 512y 5 160y 3 + 10y + 2 = 0.


Gii
t t = 2y, thay vo h ta c
(
x2 + t 2 = 1
(1)

5
5
3
3
16(x + t ) 20(x + t ) + 5(x + t) = 2. (2)

269
(
t

x = sin
vi [0; 2]. Thay vo (2) ta c
t = cos



16 sin5 20 sin3 + 5 sin + 16 cos5 20 cos3 + 5 cos = 2



3 k2
= 1 =
+
.
sin 5 + cos 5 = 2 sin 5 +
4
20
5


13 21 29 37
V [0; 2] nn
,
,
,
,
, t y suy ra x, y. 2
4 20 20 20 20
~ Nhn xt: Li gii trn tn dng "mt s trng lp th v" ca hm sin v cos :
sin 5 = 16 sin5 20 sin3 + 5 sin ; cos 5 = 16 cos5 20 cos3 + 5 cos .

S dng cc hm lng gic hyperbolic


S dng cc ng nht thc i s c xut s t cc hm lng gic hypeblic ta c th sng
tc c mt s phng trnh a thc bc cao c cch gii c th.




1
1
1
1
5
5
3
a 5 = 16m + 20m + 5m, trong m =
a
.
V d 13. Ta c
2
a
2
a
x
t m = , khi
2 2


1
20x3
5x
x5
10x3
20x
1
16x5
5
+ + = + + .
a 5 =
2
a
128 2 16 2 2 2
8 2 8 2 8 2


1
18
1
5
Ly
a 5 = , ta c bi ton sau.
2
a
8 2
Bi 15. Gii phng trnh x5 + 10x3 + 20x 18 = 0.
Gii
Ta thy rng




2
1
x x2 + 8

.
x= 2 a
2a xa 2 = 0 a =
a
2 2



1
Do ta c quyn t x = 2 a
. Khi
a



10
5
1
5
5
3
x = 4 2 a 5a + 10a
+ 3 5
a
a
a





3
1
1
3
3
10x = 20 2 a 3a + 3 , 20x = 20 2 a
.
a a
a

Thay vo phng trnh cho ta c

113
9
+



a5 =

1
4 2
4 2 a5 5 18 = 0 4 2(a5 )2 18a5 4 2 = 0

113
a

a5 =
.
4 2

270
s
!

9
+
113
4
2

Phng trnh c nghim duy nht x = 2 5


.2
5
4 2
9 + 113



1
~ Nhn xt: Trong li gii trn, php t x = 2 a
c tm ra nh sau :
a




1
1
1
1
5
5
3
Do cng thc
a 5 = 16m + 20m + 5m, trong m =
a
nn ta t x = pm,
2
a
2
a
thay vo phng trnh cho ta c

p5 m5 + 10p3 m3 + 20pm 18 = 0
Ta tm p tho mn iu kin

p5
10p3
20p
p4
p2
=
=

=
= 4 p = 2 2.
16
20
5
16
2



1
Vy ta c php t x = 2 a
.
a




1
1
1
1
5
5
3
V d 14. T ng nht thc
a + 5 = 16m 20m + 5m, trong m =
a+
.
2
a
2
a
Ly m = x ta c


1
1
5
a + 5 = 16x5 20x3 + 5x
2
a


1
1
a5 + 5 = 7 ta c phng trnh
Ly
2
a
16x5 20x3 + 5x + 7 = 0
T phng trnh ny ta c phng trnh
(x 1)(16x5 20x3 + 5x + 7) = 0
Vy ta c bi ton sau.
Bi 16 ( ngh OLYMPIC 30/04/2008). Gii phng trnh
16x6 16x5 20x4 + 20x3 + 5x2 + 2x 7 = 0.
Gii
Ta c

"
(1)

x=1

16x5 20x3 + 5x + 7 = 0

"

x=1
16x5 20x3 + 5x = 7 (2)

Tip theo ta gii phng trnh (2).


Nu |x| 6 1 th t x = cos t, vi t [0; ]. Thay vo (2) ta c
6 cos5 t 20 cos3 t + 5 cos t = 7 cos 5t = 7 (v nghim).
Nu |x| > 1 th xt phng trnh
1
x=
2

1
a+
a

"
a2 2xa + 1 = 0

a = x + x2 1

a = x x2 1.

(1)

271



1
1
2
a+
. Ta c
Vy nu t a = x + x 1 th x =
2
a
 
5


1
1
10
5
1
1
5
5
3
16x = 16
a+
a + 5a + 10a +
+ 3+ 5
=
2
a
2
a
a
a
 
3




1
1
3
1
5
1
5
3
3
20x = 20
a+
a + 3a + + 3 ; 5x =
a+
.
=
2
a
2
a a
2
a


1
1
5
3
5
Suy ra 16x 20x + 5x =
a + 5 . Thay vo (2) c
2
a


2
1
1
5
a + 5 = 7 a5 + 14a5 + 1 = 0
2
a
"
"
p

5
a = 7 48
a5 = 7 48
p

5
a5 = 7 + 48
a = 7 + 48.
Do (7

48)(7 +

48) = 1 nn (2) c nghim duy nht


q

q

1 5
5
x=
7 48 + 7 + 48 .
2
p


1 p
5
5
Nghim ca (1) l x = 1, x =
7 48 + 7 + 48 . 2
2
~ Nhn xt: Khi gii phng trnh, hy nhm nhng nghim "p" ca phng trnh bt c
lc no c th.

Sng tc mt s phng trnh ng cp i vi hai biu


thc
Ta bit rng nu mt phng trnh ng cp bc k i vi hai biu thc P (x) v Q(x) th c
P (x)
Q(x)
gii bng cch chia c hai v cho [P (x)]k (hoc [Q(x)]k ), sau t t =
(hoc t =
),
Q(x)
P (x)
a v phng trnh a thc bc k theo t. Vn dng iu ny ta c mt phng php n gin
to ra nhiu phng trnh th v.
V d 15. Xt mt phng trnh bc hai
7t2 + 13t 2 = 0.
Ly t =

x2

x1
ta c
+x+1

7.

x1
2
x +x+1

2
+ 13.

x2

x1
2 = 0.
+x+1

Quy ng b mu ta c bi ton sau


Bi 17 ( ngh OLYMPIC 30/04/2009). Gii phng trnh
2(x2 + x + 1)2 7(x 1)2 = 13(x3 1).
Gii

272
Tp xc nh R. Do x2 + x + 1 > 0 nn chia c hai v phng trnh cho (x2 + x + 1)2 > 0 ta
c
2

x1
x1
2 7.
= 13. 2
.
2
x +x+1
x +x+1
t t =

x2

x1
. Khi
+x+1

2 7t2 = 13t 7t2 + 13t 2 = 0

t = 2
1
t= .
7

Khi t = 2 ta c

x = 1
x1
2

=
2

2x
+
3x
+
1
=
0

1
x2 + x + 1
x= .
2
Khi t =

1
ta c
7
1
x1
= x2 6x + 8 = 0
2
x +x+1
7

"

x=2
x = 4.

1
Phng trnh cho c bn nghim x = 1, x = , x = 2, x = 4. 2
2
~ Nhn xt: Phng trnh ny c nhiu hn mt nghim, v cc nghim ca phng trnh ny
u l s nguyn v s hu t, do ta c th gii nhanh chng bng cch khai trin a v
phng trnh bc bn, sau nhm nghim, a v phng trnh tch.
V d 16. Xt mt phng trnh bc hai c nghim
2t2 7t + 3 = 0.
r
Ly t =

x2 + x + 1
ta c
x1
2

x +x+1
7
x1

x2 + x + 1
+ 3 = 0.
x1

Quy ng b mu ta c
2(x2 + x + 1) + 3(x 1) = 7

p
(x 1)(x2 + x + 1).

Ta c bi ton sau
Bi 18 ( ngh OLYPIC 30/04/2007). Gii phng trnh

2x2 + 5x 1 = 7 x3 1.

(1)

Gii
iu kin x > 1.
p
(1) 3(x 1) + 2(x2 + x + 1) = 7 (x 1)(x2 + x + 1).

(2)

273
V x = 1 khng phi l nghim nn chia c hai v ca (2) cho x 1 > 0 ta c
s
2
x +x+1
x2 + x + 1
3+2
=7
.
(3)
x1
x1
r
x2 + x + 1
x2 + (1 t2 )x + 1 + t2 = 0. iu kin ca t l
t t =
x1
(
q

t>0
t > 3 + 2 3.
4
2
x = t 6t 3 > 0


1
2
Phng trnh (3) tr thnh 2t 7t + 3 = 0 t 3,
. Kt hp vi iu kin ca t ta c
2
t = 3. Vy
s

x2 + x + 1
= 3 9x 9 = x2 + x + 1 x2 8x + 10 = 0 x = 4 6.
x1

Kt hp vi iu kin ta c x = 4 6 l tt c cc nghim ca phng trnh (1). 2


~ Nhn xt: Gi Q(x) = x 1, P (x) = x2 + x + 1. Mu cht ca li gii l phn tch v tri
ca PT (1) thnh
V T = 2P (x) + 3Q(x).
Tinh ta s thy 2 l h s ca x2 trong v tri ca (1). Cng t suy ra 3. Tuy nhin d
dng tm c cc s 2 v 3 bng phng php h s bt nh
2x2 + 5x 1 = p(x2 + x + 1) + q(x 1)
2x2 + 5x 1 = px2 + (p + q)x + p q.
ng nht h s ta c

p=2
p=2

p+q =5

q = 3.

p q = 1
V d 17. Xt x = 2. Khi
(x2 + 2x + 2) = 10, x + 1 = 3,
3(x2 + 2x + 2) 8(x + 1) = 6,
(x + 1)(x2 + 2x + 2) = 30,
(x + 1)(x2 + 2x + 2) = x3 + 3x2 + 4x + 2.
Vy vi x = 2 th
3(x2 + 2x + 2) 8(x + 1) =

6
6 3
30. =
x + 3x2 + 4x + 2
30
30

Ta c bi ton sau
Bi 19. Gii phng trnh
6 3
3x2 2x 2 =
x + 3x2 + 4x + 2
30

274
Gii
iu kin
x3 + 3x2 + 4x + 2 > 0 (x + 1)(x2 + 2x + 2) x > 1.
Phng trnh cho vit li
6 p
3(x2 + 2x + 2) 8(x + 1) =
(x + 1)(x2 + 2x + 2).
30
D thy x = 1 khng l nghim ca (1).
Tip theo xt x 6= 1. Chia c hai v ca (1) cho x + 1 > 0 ta c
s
6
x2 + 2x + 2
x2 + 2x + 2
8=
.
3.
x+1
x+1
30
r
x2 + 2x + 2
t t =
> 0. Khi
x+1

6
6
3t2 8 = t 3t2 t 8 = 0 3 30t2 6t 8 30 = 0.
30
30
r
10
. Vy
Nhn xt rng t l nghim dng ca phng trnh (3), hay
3

s
r
2
x=2
x + 2x + 2
10
=
3x2 + 6x + 6 = 10x + 10
2
x+1
3
x= .
3

(1)

(2)

(3)

Kt hp vi iu kin ta thy x = 2 l nghim duy nht ca phng trnh cho. 2


Bi 20. Gii phng trnh

x 3x + 1 =

3 4
x + x2 + 1.
3

Gii
Tp xc nh R. V
x4 + x2 + 1 = (x2 + 1)2 x2 = (x2 + x + 1)(x2 x + 1)
nn

3p 2
(1) 2(x x + 1) (x + x + 1) =
(x + x + 1)(x2 x + 1)
3
s 2
2
x x+1
3 x x+1
2 2
1=
.
x +x+1
3
x2 + x + 1
2

r
t t =

x2 x + 1
> 0. Khi
x2 + x + 1
1
t=

3
2t2 1 +
t = 0 2 3t2 + t 3 = 0
3
3
t = (loi)
2 3

(1)

275
Vy
s

x2 x + 1
1
= 2x2 4x + 2 = 0 x = 1.
2
x +x+1
3

Phng trnh cho c nghim duy nht x = 1. 2

Bi 21. Gii phng trnh 2(x2 3x + 2) = 3 x3 + 8.


Gii
iu kin : x > 3. Phng trnh tng ng
p
2(x2 2x + 4) 2(x + 2) = 3 (x + 2)(x2 2x + 4)
r
x+2
x+2
2 2. 2
=3
.
2
x 2x + 4
x 2x + 4
r
t t =

x2

x+2
> 0. Khi
2x + 4

1
t
=
2 2t = 3t 2t + 3t 2 = 0
2
t = 2 (loi).
2

Vy

"

x = 3 13
1
x+2
2

= x 6x 4 = 0
x2 2x + 4
2
x = 3 + 13.

Phng trnh cho c hai nghim x = 3 13 v x = 3 + 13. 2


r

Bi 22 ( ngh OLYMPIC 30/04/2009). Gii phng trnh

x2 + x 6 + 3 x 1 3x2 6x + 19 = 0.

Gii

x +x6>0
x > 2. Phng trnh tng ng
iu kin
x1>0

2
3x 6x + 19 > 0

x2 + x 6 + 3 x 1 = 3x2 6x + 19
p
x2 + x 6 + 6 (x2 + x 6)(x 1) + 9x 9 = 3x2 6x + 19
p
3 (x 2)(x + 3)(x 1) = x2 8x + 17
p
3 (x2 + 2x 3)(x 2) = (x2 + 2x 3) 10(x 2)
s
x2 + 2x 3
x2 + 2x 3
3
=
10.
x2
x2
r
x2 + 2x 3
(Do x = 2 khng l nghim ca (2)). t t =
> 0. Thay vo (2) ta c
x2
"
t = 2 (loi)
3t = t2 10 t2 3t 10 = 0
t = 5.

(1)
(2)

276
Vy

x2 + 2x 3
23 341
2
= 5 x 23x + 47 = 0 x =
.
x2
2

23 341
Kt hp vi iu kin ta thy phng trnh cho c hai nghim x =
.2
2
s

Bi 23. Gii phng trnh


r
x4 + 2x3 + 2x2 2x + 1 = x3 + x

1 x2
.
x

(1)

Gii
Do x4 + 2x3 + 2x2 2x + 1 = x2 (x + 1)2 + (1 x)2 > 0, x R nn nu x l nghim ca (1) th

x>0
2
0 < x < 1.
1x >0
x
Vi iu kin th
(1) x2 (x + 1)2 + (1 x)2 = (x2 + 1)

(1 x)[x(1 + x)].

(2)

t u = x(1 + x), v = 1 x (iu kin u > 0, v > 0). Khi u + v = x2 + 1. Vy (2) tr thnh


 u 2

u u
u
2
2
+
+1=
.
(3)
u + v = (u + v) uv
v
v v
v

u
t t = , thay vo (3) ta c
v
t4 + 1 = t3 + t (t 1)(t3 1) = 0 t = 1.

u
Vy = 1 u = v. Do
v
"
x(1 + x) = 1 x x2 + 2x 1 = 0
Phng trnh cho c nghim duy nht x = 1 +

x = 1 + 2

x = 1 2 (loi)

2. 2

Bi 24. Gii phng trnh

5x2 + 14x + 9

x2 x 20 = 5 x + 1.

Gii
iu kin

5x
+
14x
+
9
>
0
(x + 1)(5x + 9) > 0

x x 20 > 0
(x + 4)(x 5) > 0 x > 5.

x+1>0
x+1>0
Ta c
(1)

5x2 + 14x + 9 =

x2 x 20 + 5 x + 1

(1)

277
p
2x2 5x + 2 = 5 (x + 4)(x 5)(x + 1)
p
2(x2 4x 5) + 3(x + 4) = 5 (x2 4x 5)(x + 4).

(2)

Vi iu kin x > 5, chia c hai v ca (2) cho x + 4 > 0 ta c


s
2
x 4x 5
x2 4x 5
2.
+3=5
x+4
x+4
r
x2 4x 5
t t =
> 0, thay vo (3) ta c
x+4

t=1
2t2 5t + 3 = 0
3
t= .
2

(3)

Khi t = 1, ta c
s

3
Khi t = , ta c
2

x2 4x 5
5 6
2
= 1 x 5x 9 = 0 x =
.
x+4
2

x2 4x 5
3
= , ngha l
x+4
2

4(x2 5x 5) = 9x + 36 4x2 35x 56 = 0

x=8
7
x= .
4

Kt hp vi iu kin ta c cc nghim ca phng trnh cho l x = 8, x =

5+

61

.2

Xy dng phng trnh t cc ng thc.


Xut pht t mt ng thc no , chng ta c th xy dng ln cc phng trnh v t.
Chng hn t hng ng thc
(a + b + c)3 = a3 + b3 + c3 + 3(a + b)(b + c)(c + a)
ta c
(a + b + c)3 = a3 + b3 + c3 (a + b)(b + c)(c + a) = 0.
Bng cch chn a, b, c sao cho (a + b + c)3 = a3 + b3 + c3 ta x to ra c phng trnh v t
cha cn bc ba.

V d 18. Cho
a=

3
3
7x + 1, b = x2 x 8, c = x2 8x 1

th a3 + b3 + c3 = 8. Ta c bi ton sau
Bi 25 ( ngh OLYMPIC 30/04/1999). Gii phng trnh

7x + 1

x2 x 8 +

x2 8x 1 = 2.

278
Gii
Tp xc nh R. t
a=

3
3
7x + 1, b = x2 x 8, c = x2 8x 1.

Khi

a3 + b3 + c3 = 8 (1)
a+b+c=2
(2)

Mt khc ta c hng ng thc


(a + b + c)3 = a3 + b3 + c3 + 3(a + b)(b + c)(c + a).

(3)

Thay (1), (2) vo (3) ta c

a = b

(a + b)(b + c)(c + a) = 0 b = c
c = a.
Vy

3
7x + 1 = 3 x2 x 8
7x + 1 = x2 x 8

3 2
3
x x 8 = x2 8x 1 x2 x 8 = x2 8x 1

3
x2 8x 1 = 7x 1
x2 8x 1 = 3 7x + 1

x = 1
x2 8x 9 = 0
x=9

7x = 7

x=1
x2 x = 0
x = 0.
Thay cc gi tr 1, 0, 1, 9 vo phng trnh cho thy tho mn. Vy phng trnh c tp
nghim S = {1; 0; 1; 9}. 2

V d 19. Cho
a=

3
3x2 x + 2001, b = 3x2 7x + 2002, c = 3 6x 2003

th a3 + b3 + c3 = 2002. Ta c bi ton sau


Bi 26. Gii phng trnh

3x2 x + 2001

3x2 7x + 2002

3
3
6x 2003 = 2002.

Gii
t
a=

3
3x2 x + 2001, b = 3x2 7x + 2002, c = 3 6x 2003.

Khi
(a + b + c)3 = a3 + b3 + c3 (a + b)(b + c)(c + a) = 0.

279
Vic gii phng trnh cho c quy v gii

1
3
3x2 x + 2001 = 3 3x2 7x + 2002
x=

3 2

3x 7x + 2002 = 3 6x 2003

1 13

3
x=
6x 2003 = 3 3x2 x + 2001.
6

1 1 13
}. 2
Vy phng trnh c tp nghim S = { ;
6
6
V d 20. Cho
a=

1945x + 1975, b =

60x + 15, c =

3
15 x

th a3 + b3 + c3 = 2004x + 2005. Ta c bi ton sau


Bi 27. Gii phng trnh

1945x + 1975 +

60x + 15 +

3
15 x 3 2004x + 2005 = 0.

Gii
Tp xc nh R. t
a=

1945x + 1975, b =

60x + 15, c =

15 x.

Khi a3 + b3 + c3 = 2004x + 2005. Thay vo PT cho ta c

3
3
a + b + c a3 + b 3 + c 3 = 0 a + b + c = a3 + b 3 + c 3
(a + b + c)3 = a3 + b3 + c3

(1)

Mt khc ta c hng ng thc


(a + b + c)3 = a3 + b3 + c3 + 3(a + b)(b + c)(c + a).

(2)

T (1) v (2) suy ra

a = b

(a + b)(b + c)(c + a) = 0 b = c
c = a.
Vy
1990
x=

1945x + 1975 = 60x 15


2005

30

x=
60x + 15 = x 15
59

1990
15 x = (1945x + 1975)
x=
.
1944
1990
30
1990
Vy phng trnh c ba nghim x =
,x= ,x=
.2
2005
59
1944

V d 21. Cho a = 3 3x + 1, b = 3 5 x, c = 3 2x 9 th a3 + b3 + c3 = 4x 3. Ta c bi
ton sau

Bi 28. Gii phng trnh

3
3x + 1 + 3 5 x + 3 2x 9 3 4x 3 = 0.

280
V d 22. T hng ng thc
a3 + b3 ab(a + b) = (a + b)(a b)2 ,
ly a =

x + 1, b = 3 x + 2. Khi
a3 + b3 ab(a + b)



3
=x + 1 x 2 + x2 + 3x + 2 3 x + 1 3 x + 2



3
3
3
2
= 1 + x + 3x + 2
x+1 x+2 .

Bng cch cho a3 + b3 ab(a + b) = 0 ta c bi ton sau:


Bi 29. Gii phng trnh



3
3
3
2
x + 3x + 2
x + 1 x + 2 = 1.
Gii
Tp xc nh R. Phng trnh vit li



3
(x + 1) + (x 2) + x2 + 3x + 2 3 x + 1 3 x + 2 = 0.
t a =

x + 1, b = 3 x + 2. Thay vo (*) ta c
"
a3 + b3 ab(a + b) = 0 (a + b)(a b)2 = 0

Vy

a=b
a = b.

"
"

3
x+1=3x+2
x + 1 = x 2
3

x= .
3
3
2
x+1= x+2
0x = 1 (v)

Th li ta thy x =

3
l nghim duy nht ca phng trnh. 2
2

Xy dng phng trnh t cc h i xng loi II.


Xt h phng trnh
(

T (2) suy ra
"

Thay vo (1) ta c

(x + )2 = ay + b (1)
(y + )2 = ax + b (2)

ax + b

y + = ax + b
y =

ax + b
y + = ax + b
y=
.

a ax + b a

+ b (F)
(x + ) =

a ax + b a
(x + )2 =

+ b.

(*)

281
n y bng cch chn , , a, b ta s xy dng c cc phng trnh v t. Cch gii cc

phng trnh dng ny l t y + = ax + b (hoc ax + b) a v h i xng loi


II trn bit cch gii. By gi ta s i xy dng mt s phng trnh dng ny

V d 23. Cho = 3, = 2, a = 3, b = 8 thay vo (F) ta c

(3x + 2)2 = 3x + 8 + 6.
Ta c bi ton sau
Bi 30. Gii phng trnh
9x2 + 12x 2 =

3x + 8.

Gii
8
iu kin x > . Phng trnh vit li
3
(3x + 2)2 6 =
t 3y + 2 =

3x + 8.

3x + 8, suy ra (3y + 2)2 = 3x + 8. Kt hp vi (1) ta c h


(
(3x + 2)2 = 3y + 8 (2)
(3y + 2)2 = 3x + 8. (3)

x, y tho mn (1) v (2) th x >

8
8
v y > . Ly (2) tr (3) ta c
3
3

3(x y)(3x + 3y + 4) = 3(y x) (x y)(3x + 3y + 5) = 0


"
"
xy =0
y=x

3x + 3y + 5 = 0
3y = (3x + 5).
Vi y = x, thay vo (2) ta c

1
x
=
(nhn)

3
(3x + 2)2 = 3x + 8 9x2 + 9x 4 = 0
4
x=
(loi).
3
Vi y = (3x + 5), thay vo (2) ta c

5 + 21
(loi)
x=
6
(3x + 2)2 = 3x + 3 9x2 + 15x + 1 = 0
5 21
x=
(nhn).
6

1
5 21
Cc nghim ca phng trnh cho l x = v x =
.2
3
6

1
3
V d 24. Cho = 1, = 1, a = , b = thay vo (F) ta c
2
2
r
x 3
r
+
1 3
x 3
2 2
2
2
(x + 1) =
+ 2 (x + 1) =
+ + 2.
2
2 2
2 2
Ta c bi ton sau

(1)

282
r
2

Bi 31. Gii phng trnh 2x + 4x =

x+3
.
2

V d 25. Cho = 2, = 1, a = 8000, b = 1 thay vo (F) ta c

(2x 1)2 = 4000 8000x + 1 + 4001


Ta c bi ton sau
Bi 32. Gii phng trnh

x2 x 1000 8000x + 1 = 1000

Sau y l mt cch xy dng h khc:


Nu xt h
(
(x + )3 = ay + b
(y + )3 = ax + b.
T phng trnh di ta c
y + =

3
ax + b y =

ax + b
.

Thay vo phng trnh trn ca h :

a 3 ax + b a
(x + ) =

+ b.

V d 26. Chn = 1, = 1, a = 3, b = 5, ta c

(x + 1)3 = 3 3 3x + 5 + 2.
Ta c bi ton sau
Bi 33 ( ngh OLYMPIC 30/04/2009). Gii phng trnh

x3 + 3x2 3 3 3x + 5 = 1 3x.
Gii
Tp xc nh R. Phng trnh cho tng ng

(x + 1)3 = 3 3 3x + 5 + 2.
t y + 1 =

3
3x + 5. Ta c h
(

(x + 1)3 = 3y + 5 (1)
(y + 1)3 = 3x + 5 (2)

Ly (1) tr (2) theo v ta c


(x + 1)3 (y + 1)3 = 3(x y)
(x y)[(x + 1)2 + (x + 1)(y + 1) + (y + 1)2 + 3] = 0

(1)

283

x = y do (x + 1)2 + (x + 1)(y + 1) + (y + 1)2 > 0 .
Thay vo (1) ta c
"
3

(x + 1) = 3x + 5 x + 3x 4 = 0

x=1
x = 2.

Phng trnh cho c hai nghim l x = 1 v x = 2. 2

V d 27. Cho = 2, = 0, a = 4004, b = 2001 ta c

(2x)3 = 2002 3 4004x 2001 2001.


Ta c bi ton sau

Bi 34. Gii phng trnh

8x3 + 2001
2002

3
= 4004x 2001.

Xy dng phng trnh v t da vo tnh n iu ca hm s.


Da vo kt qu "Nu hm s y = f (x) n iu th f (x) = f (y) x = y" ta c th xy dng
c nhiu phng trnh, h phng trnh.

V d 28. Xt hm s f (t) = t3 + t ng bin trn R. Cho f (x + 1) = f ( 3 7x2 + 9x 4) ta


c

2(x + 1)3 + (x + 1)2 + 1 = 2(3x 1) 3x 1 + 3x 1 + 1.


Ta c bi ton sau
Bi 35. Gii phng trnh
x3 4x2 5x + 6 =

7x2 + 9x 4.

Gii
Tp xc nh R. t y =

7x2 + 9x 4. Ta c h
(
x3 4x2 5x + 6 = y (1)
7x2 + 9x 4 = y 3
(2)

Cng (1) v (2) theo v ta c


x3 + 3x2 + 4x + 2 = y 3 + y (x + 1)3 + (x + 1) = y 3 + y.

(3)

Xt hm s f (t) = t3 + t. V f 0 (t) = 3t2 + 1 > 0, t R nn hm s f ng bin trn R. Do


(3) vit li
f (x + 1) = f (y) x + 1 = y.
Bi vy

7x2 + 9x 4 = x + 1 7x2 + 9x 4 = (x + 1)3

284

x 4x 6x + 5 = 0 (x 5)(x + x 1)
1
Kt lun: Phng trnh c tp nghim S = {5;
2

x=5

1 5
x=
.
2

}. 2

V d 29. Xt hm s f (t) = t3 + 2t ng bin trn R. Cho




3
f
x3 + 9x2 19x + 11 = f (x 1).
ta c

3
x3 + 9x2 19x + 11 + 2 x3 + 9x2 19x + 11 = (x 1)3 + 2(x 1).
Khai trin v rt gn ta c bi ton sau
Bi 36 ( ngh OLYMPIC 30/04/2009). Gii phng trnh
x3 6x2 + 12x 7 =

3
x3 + 9x2 19x + 11.
Gii

t y =

3
x3 + 9x2 19x + 11. Ta c h
(
(
y 3 = x3 + 9x2 19x + 11
y 3 = x3 + 9x2 19x + 11

y = x3 6x2 + 12x 7
2y = 2x3 12x2 + 24x 14

Cng hai phng trnh vi nhau ta c


y 3 + 2y = x3 3x2 + 5x 3 y 3 + 2y = (x 1)3 + 2(x 1).
Xt hm s f (t) = t3 + 2t. Vi mi t1 6= t2 , ta c
f (t1 ) f (t2 )
= t21 + t1 t2 + t22 + 2 =
t1 t2


2
t2
3t2
t1 +
+ 2 + 2 > 0.
2
2

Vy hm s f (t) ng bin trn R. Do


() f (y) = f (x 1) y = x 1

3
x3 + 9x2 19x + 11 = x 1
x3 + 9x2 19x + 11 = x3 3x2 + 3x 1
x3 6x2 + 11x 6 = 0 x {1, 2, 3} .
Phng trnh cho c ba nghim x = 1, x = 2, x = 3. 2
V d 30. Xt hm s f (t) = 2t3 + t2 + 1 n iu trn [0; +). Cho


f (x + 1) = f
3x 1
ta c

2(x + 1)3 + (x + 1)2 + 1 = 2(3x 1) 3x 1 + 3x 1 + 1.

Ta c bi ton sau

(*)

285
Bi 37. Gii phng trnh

2x3 + 7x2 + 5x + 4 = 2(3x 1) 3x 1.


Gii

1
iu kin x > . t y = 3x 1, y > 0. Ta c h
3
(
2x3 + 7x2 + 5x + 4 = 2y 3
3x 1 = y 2 .
Cng theo v hai phng trnh trn ta c
2x3 + 7x2 + 8x + 3 = 2y 3 + y 2 2(x + 1)3 + (x + 1)2 = 2y 3 + y 2 .

(*)

Xt hm s f (t) = 2t3 +t2 . V f 0 (t) = 6t2 +2t > 0, t > 0 nn hm s f ng bin trn [0; +).
Do

() f (x + 1) = f (y) x + 1 = y x + 1 = 3x 1

"
(

x
" > 1
x=0
x+1>0

x
=
0

x = 1.
3x + 1 = x2 + 2x + 1

x=1
Kt hp vi iu kin ta c x = 1 l nghim duy nht ca phng trnh cho. 2

V d 31. Xt hm s f (t) = t3 + t n iu trn R. Nu cho f (2x) = f ( 3 6x + 1) th c


8x3 + 2x = 6x + 1 +

3
6x + 1 3 6x + 1 = 8x3 4x 1.

Ta c bi ton sau
Bi 38. Gii phng trnh

6x + 1 = 8x3 4x 1.

V d 32. Xt hm s f (t) = t 1 +
f 0 (t) = 1 +
Vy hm s f (t) = t 1 +


t2 + 2 . Ta c

t2
t2 + 2 +
> 0, t R.
t2 + 2


t2 + 2 ng bin trn R. Cho f (2x + 3) = f (3x) ta c






(2x + 3) 1 + 4x2 + 12x + 11 = 3x 1 + 9x2 + 2 .


Ta c bi ton
Bi 39. Gii phng trnh
(2x + 3)

4x2

+ 12x + 11 + 3x 1 +

9x2

+ 2 = 5x 3.

286
V d 33. Xt hm s ng bin trn khong (0; +) l
f (t) = log2 t 2t + t2 , t > 0.
Cho f

x+2 =f

1
2+
x


ta c




 
2

1
1
1
log2 x + 2 2 x + 2 + x + 2 = log2 2 +
2 2+
+ 2+
x
x
x

2

1
2x + 1
1
log2 (x + 2) + x + 3 = log2
+ 1+
+ 2 x + 2.
2
x
x

Ta c bi ton sau
Bi 40. Gii phng trnh

2

1
2x + 1
1
log2 (x + 2) + x + 3 = log2
+ 1+
+ 2 x + 2.
2
x
x
Gii
iu kin



x (2;
 +) 
1
(0; +) .
x 2;
1
x ;
2
(0; +)
2
Khi phng trnh vit li
log2




 
2

1
1
1
x + 2 2 x + 2 + x + 2 = log2 2 +
2 2+
+ 2+
.
x
x
x

(1)

Xt hm s f (t) = log2 t 2t + t2 , t > 0. Ta c


r
r
1
1
2
f (t) =
+ 2t 2 > 2
.2t 2 = 2
2>0
t. ln 2
t. ln 2
ln 2
0

Vy hm s f (t) ng bin trn khong (0; +), do


(1) f




1
1
x+2 =f 2+
x+2=2+ .
x
x




1
Vi iu kin x 2;
(0; +), bnh phng hai v phng trnh (2) ta c
2

x = 1
4
1

+ 2 x3 2x2 4x 1 = 0
3 13
x x
x=
.
2

3 + 13
Kt hp vi iu kin, ta thy PT cho c hai nghim x = 1 v x =
.2
2
x+2=4+

(2)

287

Xy dng phng trnh v t da vo cc phng trnh lng gic.


T mt phng trnh lng gic n gin no , kt hp vi cc php bin i lng gic th
s tm ra cc phng trnh v t hay. T mt phng trnh lng gic n gin no , kt hp
vi cc php bin i lng gic th s tm ra cc phng trnh v t hay.
t
V d 34. T phng trnh cos 3t = cos , vi t [0; ], ta thy phng trnh ny tng
2
p
ng vi 8 cos3 t 6 cos t = 2(1 + cos t). t x = 2 cos t ta c bi ton sau.
Bi 41 ( ngh OLYMPIC 30/04/2006). Gii phng trnh
x3 3x =

x + 2.

(1)

Gii
iu kin x > 2. Nu x > 2 th
x3 3x = x + x(x2 4) > x >

2x =

x+x>

x + 2.

Vy x > 2 khng tho mn (1), do gii phng trnh (1), ch cn xt 2 6 x 6 2. Khi


t x = 2 cos t, iu kin t [0; ]. Thay vo (1) ta c
p
t
8 cos3 t 6 cos t = 2 (1 + cos t) 4 cos3 t 3 cos t = cos
2

k4
t
t=
3t = + k2
t

2
5
cos 3t = cos

t
k4 (k Z) .
2
3t = + k2
t=
2
7
4
4
,t=
. Phng trnh cho c ba nghim
Do t [0; ] nn ch ly cc nghim t = 0, t =
5
7
4
4
x = 2, x = 2 cos , x = 2 cos . 2
5
7
V d 35. T phng trnh cos 3t = sin t, vi t [0; ], ta thy phng trnh ny tng ng

vi 4 cos3 t 3 cos t = 1 cos2 t. t x = cos t ta c bi ton sau:


Bi 42 ( ngh Olympic 30/04/2003-ton 10). Gii phng trnh
4x3 3x =

1 x2 .

Nu thay x bi x 1 ta c bi ton kh hn:

Bi 43. Gii phng trnh 4x3 12x2 + 9x 1 =

2x x2 .

V d 36. T phng trnh sin 3t = cos t,vi t [0; ], ta thy phng trnh ny tng ng
vi

3 sin t 4 sin3 t = cos x sin t 3 4 sin2 t = cos t


cos t
1 cos2 t 4 cos2 t 1 = cos t 1 cos2 t =
.
4 cos2 t 1
Ly x = cos t ta c bi ton sau.

288
Bi 44. Gii phng trnh

1 x2 =

x
.
1

4x2

Gii
T iu kin |x| 6 1, x 6=

1
1
v x 6= , ta t
2
2
x = cos t, t [0; ] , x 6=

, x 6=
.
3
3

Thay vo phng trnh cho ta c


cos t

1 cos2 t 4 cos2 t 1 = cos t


2
4 cos t 1


2
sin t 4 4 sin t 1 = cos t sin t 3 4 sin2 t = cos t


t
3 sin t 4 sin3 t = cos x sin 3t = cos t sin 3t = sin
2

k
3t = t + k2
t= +
2 

8
2 (k Z).2

3t =
t + k2
t = + k
2
4
1 cos2 t =

Trn on [0; ], ta nhn c cc nghim t1 = , t2 =


, t3 = . Nghim ca phng trnh
8
8
4

cho l cos , cos , cos . 2


8
8
4
V d 37. Ta c sin 5 = 16 sin5 20 sin3 + 5 sin . Xt sin 5t = cos t,vi t [0; ], ta thy
phng trnh ny tng ng vi
16 sin5 t 20 sin3 t + 5 sin t = cos t

sin t 16 sin4 t 20 sin2 t + 5 = cos t
i
h

2
sin t 16 1 cos2 t 20 1 cos2 t + 5 = cos t

sin t 16 cos4 t 12 cos2 t + 1 = cos t

cos t
1 cos2 t =
.
4
16 cos t 12 cos2 t + 1
Ly x = cos t ta c bi ton sau.

Bi 45. Gii phng trnh


1 x2 =

16x4

x
.
12x2 + 1

Gii
T iu kin |x| 6 1 v 16x4 12x2 + 1 6= 0, ta t
x = cos t, t [0; ] , 16 cos4 t 12 cos2 t + 1 6= 0.
Thay vo phng trnh cho ta c

cos t
t 12 cos2 t + 1

sin t 16 cos4 t 12 cos2 t + 1 = cos t
1 cos2 t =

16 cos4

289
h
i
2

sin t 16 1 sin2 t 12 1 sin2 t + 1 = cos t

sin t 16 sin4 t 20 sin2 t + 5 = cos t
16 sin5 t 20 sin3 t + 5 sin t = cos t sin 5t = cos t

k


t
=
+

12
3
sin 5t = sin
t
k (k Z).2
2
t= +
8
2
V d 38. Xt sin 5t = cos 3t,vi t [0; ], phng trnh ny tng ng vi
16 sin5 t 20 sin3 t + 5 sin t = cos 3t

sin t 16 sin4 t 20 sin2 t + 5 = 4 cos3 t 3 cos t
h
i
2

sin t 16 1 cos2 t 20 1 cos2 t + 5 = 4 cos3 t 3 cos t

sin t 16 cos4 t 12 cos2 t + 1 = 4 cos3 t 3 cos t


1 cos2 t 16 cos4 t 12 cos2 t + 1 = 4 cos3 t 3 cos t.
Ly x = cos t ta c bi ton sau.
Bi 46. Gii phng trnh

1 x2 (16x4 12x2 + 1) = 4x3 3x.


Gii

T iu kin |x| 6 1, ta t x = cos t, t [0; ]. Thay vo phng trnh cho ta c


1 cos2 t 16 cos4 t 12 cos2 t + 1 = 4 cos3 t 3 cos t
h
i
2

sin t 16 1 sin2 t 12 1 sin2 t + 1 = cos 3t

sin t 16 sin4 t 20 sin2 t + 5 = cos 3t
16 sin5 t 20 sin3 t + 5 sin t = cos 3t sin 5t = cos 3t



t=
+
16
4 (k Z).2
sin 5t = sin
3t

2
t = + k
4





V d 39. T sin 3t
= sin t +
, vi t [0; ], ta c
4
4
sin 3t cos 3t = sin t + cos t
3 sin t 4 sin3 t 4 cos3 t + 3 cos t = sin t + cos t
2 cos t + 3 sin t 4 sin3 t = 4 cos3 t + sin t

2 cos t + sin t 3 4 sin2 t = 4 cos3 t + sin t

2 cos t + sin t 4 cos2 t 1 = 4 cos3 t + sin t


2 cos t + 1 cos2 t 4 cos2 t 1 = 4 cos3 t + 1 cos2 t.
Ly x = cos t ta c bi ton sau.

Bi 47. Gii phng trnh 2x + (4x2 1) 1 x2 = 4x3 + 1 x2 .


Gii

290
T iu kin |x| 6 1, ta t x = cos t, t [0; ]. Thay vo phng trnh cho ta c


2 cos t + 4 cos2 t 1 1 cos2 t = 4 cos3 t + 1 cos2 t

2 cos t + 4 cos2 t 1 sin t = 4 cos3 t + sin t

sin t 4 cos2 t 2 = 4 cos3 t 2 cos t


sin t 2 cos2 t 1 = cos t 2 cos2 t 1
"

cos 2t = 0
2 cos2 t 1 (sin t cos t) = 0
sin t = cos t

"
k

t= +
2t = + k
k
4
2 t= +
2
.

4
2
tan t = 1
t = + k
4
Bi 48. Gii bt phng trnh

x2
1+x+ 1x62 .
4

(1)

Gii
iu kin 1 6 x 6 1. t x = cos , [0; ]. Thay vo (1) ta c

cos2
1 + cos + 1 cos 6 2
4
 

 2  
2
2 cos

6 2 sin

cos

2
4
2 4
2
4
h
i




2
2

6 2 1 cos

cos

2 cos
4
2
4
 2 4 
  2


4
2
cos

cos

2 cos

+2>0
2
4
2
4
2
4
h  
i2 h
 
 
i
cos

1
cos2

+ 2 cos

+ 2 > 0.
2
4
2
4
2
4

(2)

V (2) lun ng vi mi [0; ] nn nghim ca (1) l 1 6 x 6 1.

S dng cn bc n ca s phc sng to v gii h phng trnh.


Cho s phc z = r (cos + i sin ) , r > 0. Khi cc cn bc n ca z l



+ 2k
+ 2k
n
zk = r cos
+ i sin
, k = 0, 1, 2, . . . , n 1.
n
n
Cc cn bc hai ca s phc z = r (cos + i sin ) , r > 0 l




z0 = r cos + i sin
, z1 = r cos + i sin
.
2
2
2
2
Cc cn bc ba ca s phc z = r (cos + i sin ) , r > 0 l



3
z0 = r cos + i sin
,
3
3



+ 2
+ 2
3
z1 = r cos
+ i sin
,
3
3



+ 4
+ 4
3
z2 = r cos
+ i sin
.
3
3

291
Mt phng trnh nghim phc f (z) = 0, vi z = x + iy, ta bin i thnh
(
h(x, y) = 0
h(x, y) + ig(x, y) = 0
g(x, y) = 0.
Ngha l mt phng trnh nghim phc, bng cch tch phn thc v phn o lun c th
a v h phng trnh. 1. Sng tc cc h phng trnh bng cch lu tha ba mt
s phc cho trc. tm cn bc ba ca s phc 1 + i, ta tm s phc z = x + iy, x R,
y R sao cho
(x + iy)3 = 1 + i x3 + 3x2 yi + 3xy 2 i2 + y 3 i3 = 1 + i
(
x3 3xy 2 + (3x2 y y 3 )i = 1 + i

x3 3xy 2 = 1
3x2 y y 3 = 1.

Gii h ny ta tm c x v y, t c z. Tuy nhin, c th tm z bng cch khai cn bc ba


ca 1 + i nh sau : Ta c



.
1 + i = 2 cos + i sin
4
4
Vy cc cn bc ba ca 1 + i l
q 




3
6
z0 =
+ i sin
= 2 cos
+ i sin
,
2 cos
12
12
12
12



q
+ 2
+ 2

3
3
3
6
4
4
= 2 cos
z1 =
+ i sin
+ i sin
2 cos
,
3
3
4
4



q
+
4
+
4

17
17
3
6
4
4
= 2 cos
z2 =
2 cos
+ i sin
+ i sin
.
3
3
12
12
(

x3 3xy 2 = 1
l
3x2 y y 3 = 1

6
6
x = 2 cos
x = 2 cos
x = 6 2 cos 17
12 ;
4
12

3 ;
17
6
6
y = 6 2 sin

y = 2 sin
y = 2 sin
.
12
4
12
!



1
3
V d 40. Xt s phc z = 5 cos + i sin
=5
+
i . Gi s x + yi l s phc tho
3
3
2
2
T y, ngc li ta tm c nghim ca h

mn iu kin
!

1
3
5 5 3
3
3
2
2 2
3 3
(x + yi) = 5
+
i x + 3x yi + 3xy i + y i = +
i
2
2
2
2
(

2x3 6xy 2 = 5

2x3 6xy 2 + (6x2 y 2y 3 )i = 5 + 5 3i


6x2 y 2y 3 = 5 3.
Ta c bi ton sau
(
Bi 49. Gii h phng trnh

2x3 6xy 2 = 5
(1)

2
3
6x y 2y = 5 3 (2).

292
Gii
Nhn hai v ca (2) vi i ri cng vi (1) ta c

2x3 6xy 2 + (6x2 y 2y 3 )i = 5 + 5 3i

5
5
3
i
x3 3xy 2 + (3x2 y y 3 )i = +
2
2
5 5 3
x3 + 3x2 yi + 3xy 2 i2 + y 3 i3 = +
i
2
2
!
3
1
(x + yi)3 = 5
+
i .
2
2
Vy x + yi l mt cn bc ba ca s phc
!

3
1


z=5
+
i = 5 cos + i sin
.
2
2
3
3
M z c cc cn bc ba l



3
z0 = 5 cos + i sin
,
9
9

+ 2

3
+ i sin
z1 = 5 cos 3
3

+ 4

3
z2 = 5 cos 3
+ i sin
3



+ 2

7
7
3
3
= 5 cos
+ i sin
,
3
9
9



+ 4

13
13
3
3
= 5 cos
+ i sin
.
3
9
9

Vy cc nghim ca h phng trnh l

x = 3 5 cos 7
x = 3 5 cos 13
x = 3 5 cos
9 ;
9
9

7 ;
13
3
3
y = 3 5 sin
y = 5 sin
y = 5 sin
.
9
9
9
2. Sng tc cc h phng trnh t hai s phc cho trc.
V d 41. Xt hai s phc z1 v z2 nh sau
(
(

z1 = 7 5i
z1 + z2 = 7

z2 = 5i
z1 z2 = 7 5i + 5
Vy z1 v z2 l nghim ca phng trnh

5 + 7 5i
z 7z + 5 + 7 5i = 0 z 7 +
=0

z
5 7 5i
5z 7 5iz
z+ +
=7z+
+
=7
z
z
zz
zz
2

Gi s z = x + yi vi x, y R. Khi phng trnh trn vit li

5(x yi) 7 5(xi + y)


x + yi + 2
+
=7
x + y2
x2 + y 2

293

5x + 7 5y 7 5x 5y
x + yi +
+
i=7
x2 + y 2
x2 + y 2
!

7 5x 5y
5x + 7 5y
i=0
7+ y+
x +
x2 + y 2
x2 + y 2

5x
+
7
5y

x+
7=0
2 + y2
x

7 5x 5y

y+
= 0.
x2 + y 2
Ta c bi ton sau

5x + 7 5y

x+
=7
2 + y2
x

Bi 50. Gii h phng trnh

7 5x 5y

y+
= 0.
x2 + y 2
Ta xt tip mt bi tng t:

3x y

x+ 2
=3
x + y2
Bi 51. Gii h phng trnh
x + 3y

y
= 0.
x2 + y 2
Gii
iu kin x2 + y 2 6= 0. Xt s phc z = x + iy. Khi
iz = ix y, x2 + y 2 = |z|2 = z.z
H phng trnh cho vit li

3x y

x+ 2
=3
(1)
x + y2
x + 3y

yi
i = 0. (2)
x2 + y 2
Ly (1) tr (2) theo v ta c
x yi +

3x y
x + 3y
3(x + yi)
y xi
+
i
=
3

yi
+

= 3.
x2 + y 2 x2 + y 2
x2 + y 2
x2 + y 2

Hay
z+3

z
iz
1 i
+
= 3 z + 3 + = 3 (z)2 3z + 3 + i = 0.
z.z z.z
z z

Ta c = 9 4(3 + i) = 3 4i. Xt s phc a + bi tho mn iu kin

(
a2 b2 = 3
2
2
a b = 3
2
3 4i = (a + bi)

2
b= .
2ab = 4
a
Khi
a2

4
= 3 a4 + 3a2 4 = 0 a2 = 1 a = 1.
2
a

294
Vy (a; b) = (1; 2) ; (a; b) = (1; 2) . Do = 3 4i c hai cn bc hai l (1 2i). Suy
ra

3 + (1 2i)
=2i
z=
2

3 (1 2i)
z=
= 1 + i.
2
Ngha l
"
"
x yi = 2 i
(x; y) = (2; 1)

x yi = 1 + i
(x; y) = (1; 1).
(
(
x=2
x=1
Vy h cho c hai nghim l
v
.2
y=1
y = 1.
V d 42. Xt hai s phc
(

z1 + z2 = 3 2 + i
z1 = 2 i

z2 = 2 2 + 2i
z1 z2 = 6.

Khi z1 v z2 l hai nghim ca phng trnh





6
6z
= 3 2 + i.
z2 3 2 + i z + 6 = 0 z + = 3 2 + i z +
z
z.z
Gi s z = u + vi, khi phng trnh vit li

6(u vi)
=
3
2+i
u2 +v 2


6u
6v
2+i
u + 2
+
v

=
3
u + v2
u2 + v 2



6u

=3 2
u+
u 1+ 2
=
3
2
2
u +v 
u2 + v 2


6v
6

=1
= 1.
v 1 2
2
2
u +v
u + v2

Ly u = x, v = y, ta c



x
6

1+
= 2

3 
x + y
6

= 1.
y 1
x+y
u + vi +

Ta c bi ton sau




x
6

1+
= 2

3 
x + y
Bi 52. Gii h phng trnh
6

= 1.
y 1
x+y
Mt bi tng t tng xut hin trong k thi VMO 1996:
Bi 53. Gii h phng trnh




=2
3x 1 +
x + y


= 4 2.
7y 1
x+y

295
Gii

iu kin x > 0 v y > 0. t u = x, v = y. H phng trnh tr thnh




1
2

=
u 1+ 2
2
u +v 
3

2
1
4

v 1 2
=
2
u +v
7
V u2 + v 2 l bnh phng ca mun ca s phc z = u + iv nn ta bin i h phng trnh
thnh

u
u
2
2

=
=
(1)
u+ 2
u+ 2
2
2
u +v
u +v
3
3

v
4 2
iv
4 2i

(2)
=
=
v 2
iv 2
2
2
u +v
u +v
7
7
Cng (1) v (2) theo v ta c

u iv
4 2i
2
u + iv + 2
= + .
u + v2
3
7
V

z
z
1
u iv
= 2 =
= nn (3) vit li
2
2
u +v
z.z
z
|z|

2
1
4 2i
z + = + z2 2
z
3
7

Ta c
0 =

!
2 2i
1
+
z + 1 = 0.
3
7

!2
1
2 2i
38 4 2i
1 8 4 2i
+
1= + 1= + .
3 7
21
3
7
21
21

Xt s phc a + bi tho mn iu kin

38

a2 b 2 =

38 4 2i
21
+
= (a + bi)2
4
2

21
21
2ab =
21
Khi

8
38
19 + 23
4
a2
=
21a4 + 38a2 8 = 0 a2 =
= .
2
21a
21
21
21





2
2
Vy (a; b) = ; 2 ; (a; b) = ; 2 . Do
21
21
!




1
2
2 2
z = u + iv =
+ 2 i.
3
21
7

2
1

u= +
21
3
V u > 0 v v > 0 nn
Do

2
2

v = + 2.
7


2
1
2
11
4

=
+

x = 3 + 21
21 3 7
!2

y = 2 2 + 2 = 22 + 8 .

7
7
7

(3)

(4)

296
H phng trnh cho c nghim duy nht l

11
4

+
x=
21 3 7 . 2
22
8

y=
+ .
7
7
Bi 54 (VMO 2006). Gii h phng trnh



12

x=2
1
y + 3x 

12

y = 6.
1+
y + 3x
Gii

iu kin x > 0, y > 0, y + 3x 6= 0. t u = 3x > 0, v = y > 0. Thay vo h ta c





12
u

12u

=
2
1 2
u
3 (1)
=
2
2
u +v  3
u2 + v 2


12v
12

v+
= 6.
(2)
v=6
1+ 2
2
2
u + v2
u +v
Nhn PT (2) vi i, sau cng vi PT (1) ta c
u + vi

12(u vi)
=
2
3 + 6i.
u2 + v 2

Xt s phc z = u + vi, vi u > 0, v > 0. Khi (3) vit li

12z
12
z
= 2 3 + 6i z
= 2( 3 + 3i)
z.z
z
2
z 2( 3 + 3i)z 12 = 0.
Ta c

0 = ( 3 + 3i)2 + 12 = 6 + 6 3i + 12 = 6 + 6 3i
!

1
3


= 12
+
i = 12 cos + i sin
.
2
2
3
3
Vy 0 c hai cn bc hai l


12 cos + i sin
= 12
6
6
T (4) ta c
"

!


3 1
+ i = 3 + 3i .
2
2


z = 3 + 3i + 3 + 3i = 3 + 3 + 3 + 3 i


z = 3 + 3i 3 3i = 3 3 + 3 3 i.

Do u > 0 v v > 0 nn

2
(

3
+
3

u= 3+3
x=4+2 3
x
=


v = 3+3
y =3 4+2 3 .
y = 3 + 32
(

x=4+2 3
 .2
H cho c nghim duy nht
y =3 4+2 3 .
(

(3)

(4)

297

S dng bt ng thc lng gic trong tam gic


Trong thi vo i hc Nng nghip 1, nm 1995 c bi ton sau:
Bi 55. Gii phng trnh
9
sin2 x + sin2 y + sin2 (x + y) = .
4

(1)

Phng trnh ny khin ta lin tng n mt bt ng thc c bn trong tam gic : Vi mi


tam gic ABC ta c
9
(2)
sin2 A + sin2 B + sin2 C 6 ,
4

du bng xy ra khi v ch khi A = B = C = .


3
Trong v tri ca bt ng thc (2), ly A = x, B = y, C = (x + y), ta thu c v tri
ca phng trnh (1). Li gii ca phng trnh (1) cng thu c da trn c s php chng
minh bt ng thc lng gic (2).

Gii
Ta c
(1) 1

cos 2x + cos 2y
9
+ sin2 (x + y) =
2
4

1 cos(x + y) cos(x y) + 1 cos2 (x + y) =


cos2 (x + y) + cos(x y) cos(x + y) +

9
4

1
= 0.
4

(3)

Phng trnh (3) l phng trnh bc hai i vi cos(x + y), ta c


= cos2 (x y) 1 > 0.
V cos2 (x y) 6 1 nn phng trnh c nghim th cos2 (x y) = 1. Do


x y = k2
cos(x y) = 1

x + y = 2 + l2
cos(x + y) = cos(x y) = 1

3
2
2


cos(x y) = 1
x y = + k2

cos(x

y)
1
cos(x + y) =
x + y = + l2.
=
2
2
3
Xt (4) ta c:

2
x=
x y = k2; x + y =
+ l2

3
(4)

2
x=
x y = k2; x + y =
+ l2
3

(4)

(5)

+ (k + l); y = + (l k)
3
3

+ (k + l); y =
+ (l k)
3
3

Xt (5) ta c:

+ k2; x + y = + l2
x=
xy =

2
3
(5)

xy =
+ k2; x + y =
+ l2
x=
2
3

5
+ (k + l); y =
+ (l k)
12
12
5

+ (k + l); y =
+ (l k)
12
12

298
Theo trn ta suy ra mt hng sng tc mt lot cc phng trnh lng gic hai n kh
th v l : T mt bt ng thc c bn trong tam gic, trong bt ng thc cha cc gc
A, B, C ca mt tam gic, ta ln lt ly
A = f (x, y), B = g(x, y), C = [f (x, y) + g(x, y)],
v thay du bt ng thc bi du ng thc, ta s c mt phng trnh lng gic hai n
x v y tng ng. Cn vic gii phng trnh th thng c tin hnh tng t nh hai cch
gii trnh by trn.
Ch rng cch lm nh trn khng phi lc no cng thnh cng, c th ta xy dng c
nhng phng trnh lng gic hai n nhng ta li khng gii c nhng phng trnh m ta
va xy dng nn. Tuy nhin y l mt phng php sng tc bi ton rt ng quan tm,
n cho ta nhiu phng trnh th v c bi ln li gii.

V d 43. Xt bt ng thc
3
cos A + cos B + cos C 6 , ABC,
2
du bng xy ra khi v ch khi A = B = C =

. Ly A = x, B = y, C = (x + y), ta c
3

bi ton sau
Bi 56. Gii phng trnh
3
cos x + cos y cos(x + y) = .
2
Gii
Phng trnh cho tng ng


x+y
xy
3
2 x+y
2 cos
cos
2 cos
1 =
2
2
2
2
x+y
x+y
xy 1
2 cos2
2 cos
cos
+ =0
2
2
2
2
x
+
y
x
+
y
x

y
1
cos
cos
+ =0
cos2
2
2
2
4

2
x+y 1
xy
1
xy
cos
cos
+ sin2
=0
2
2
2
4
2

cos x + y = 1 cos x y
cos x + y = 1 cos x y
2
2
2
2
2
2

sin x y = 0
cos x y = 1
2
2

x

y
cos
=1

2

x+y
1

cos 2 = 2

cos x y = 1

cos x + y = 1 .
2
2

299
V d 44. Cng xt bt ng thc
3
cos A + cos B + cos C 6 , ABC,
2

. Ly
3


y
3y
y  3y
A=x , B = , C = x
+
= (x + y),
2
2
2
2

du bng xy ra khi v ch khi A = B = C =

ta c bi ton sau
Bi 57. Gii phng trnh
y
3y
3
cos x
+ cos
cos(x + y) = .
2
2
2


Gii
Phng trnh cho tng ng


x 2y
3
x+y
2 x+y
cos
2 cos
1 =
2 cos
2
2
2
2
x
+
y
x

2y
1
x
+
y
2 cos
cos
+ =0
2 cos2
2
2
2
2
x
+
y
x

2y
1
x
+
y
cos2
cos
cos
+ =0
2
2
2
4

2
x+y 1
x 2y
x 2y
1
cos
cos
=0
+ sin2
2
2
2
4
2

1
x

2y
x
+
y

cos x + y = 1 cos x 2y
cos
= cos
2
2
2
2
2
2

x 2y
x 2y

cos
sin
=0
= 1
2
2

cos x 2y = 1
cos x 2y = 1
2
2

1 hoc
1
x+y
x+y

cos
cos
=
= .
2
2
2
2
Cng vic tip theo tr nn n gin.

V d 45. Xt bt ng thc
sin

B
C
3
A
+ sin + sin 6 , ABC,
2
2
2
2

du bng xy ra khi v ch khi A = B = C =

. Ly A = 2x, B = 2y, C = (2x + 2y), ta


3

c bi ton sau
Bi 58. Gii phng trnh
3
sin x + sin y + cos(x + y) = .
2
Gii

300
Phng trnh cho tng ng
x+y
x+y
xy
3
cos
+ 1 2 sin2
=
2
2
2
2
x+y
xy 1
2 x+y
2 sin
cos
+ =0
2 sin
2
2
2
2
x+y
xy 1
2 x+y
sin
sin
cos
+ =0
2
2
2
4

2
x+y 1
xy
1 2 xy
sin
cos
+ sin
=0
2
2
2
4
2

sin x + y = 1 cos x y
sin x + y = 1 cos x y
2
2
2
2
2
2

sin x y = 0
cos x y = 1
2
2

y
x

y
cos
cos
=1
= 1
2
2
hoc

sin x + y = 1 .
sin x + y = 1
2
2
2
2
2 sin

V d 46. Xt bt ng thc
A
B
C
9
+ cos2 + cos2 6 , ABC,
2
2
2
4

du bng xy ra khi v ch khi A = B = C = . Ly A = 2x, B = 2y, C = (2x + 2y), ta


3
c bi ton sau
cos2

Bi 59. Gii phng trnh


9
cos2 x + cos2 y + sin2 (x + y) = .
4
Gii
Phng trnh cho tng ng
1+

cos 2x + cos 2y
9
+ sin2 (x + y) =
2
4

1 + cos(x + y) cos(x y) + 1 cos2 (x + y) =

9
4

1
cos2 (x + y) cos(x y) cos(x + y) + = 0
4

2
1
1
cos(x + y) cos(x y) + sin2 (x y) = 0
2
4

cos(x + y) 1 cos(x y) = 0

2
sin(x y) = 0

cos(x y) = 1

cos(x + y) = 1
cos(x + y) = 1 cos(x y) = 0

2
cos(x y) = 1
cos(x y) = 1

1
cos(x + y) = .
2
V d 47. Quan st cc bin i trn ta thy c th xy dng nn mt s phng trnh c li
gii tng t. Chng hn bi ton sau

301
Bi 60. Gii phng trnh
cos2 x + cos2 y + cos2 (x y) =

3
4

Gii
Phng trnh cho tng ng
1+

cos 2x + cos 2y
3
+ cos2 (x y) =
2
4

1 + cos(x + y) cos(x y) + cos2 (x y) =

3
4

1
cos2 (x + y) + cos(x y) cos(x + y) + = 0
4

2
1
1
cos(x + y) + cos(x y) + sin2 (x y) = 0
2
4

cos(x + y) + 1 cos(x y) = 0

2
sin(x y) = 0

cos(x y) = 1

cos(x + y) = 1
cos(x + y) = 1 cos(x y)

2
cos(x y) = 1
cos(x y) = 1

1
cos(x + y) = .
2
V d 48. Xt bt ng thc
A
B
C
1
sin sin 6 , ABC,
2
2
2
8

du bng xy ra khi v ch khi A = B = C = . Ly


3
A = 2x y, B = 3y, C = [(2x y) + 3y] = 2(x + y),
sin

ta c bi ton sau
Bi 61.
sin

y 2x
3y
1
sin
cos(x + y) = .
2
2
8
Gii

Phng trnh tng ng


4 [cos(x + y) cos(2y x)] cos(x + y) = 1
4 cos2 (x + y) 4 cos(x + y) cos(2y x) + 1 = 0
[2 cos(x + y) cos(2y x)]2 + sin2 (2y x) = 0
(
(
2 cos(x + y) = cos(2y x)
2 cos(x + y) = cos(2y x)

sin(2y x) = 0
cos(2y x) = 1

cos(2y x) = 1
cos(2y x) = 1

hoc
1
1
cos(x + y) =
cos(x + y) = .
2
2

302
V d 49. Xt bt ng thc
1
cos A cos B cos C 6 , ABC,
8

du bng xy ra khi v ch khi A = B = C = . Ly


3
A = x, B = y, C = (x + y),
ta c bi ton sau
Bi 62. Gii phng trnh
1
cos x. cos y. cos(x + y) = .
8
Gii
Phng trnh cho tng ng
4 cos(x + y) [cos(x + y) + cos(x y)] = 1
4 cos2 (x + y) + 4 cos(x + y) cos(x y) + 1 = 0
[2 cos(x + y) + cos(x y)]2 + sin2 (x y) = 0
(
(
2 cos(x + y) = cos(x y)
2 cos(x + y) = cos(x y)

sin(x y) = 0
cos(x y) = 1

cos(x y) = 1
cos(x y) = 1
hoc

1
1
cos(x + y) = .
cos(x + y) =
2
2
V d 50. Xt bt ng thc
3
cos A + cos B + cos C 6 , ABC,
2

du bng xy ra khi v ch khi A = B = C = . Ly


3
A = x, B = y, C = (x + y),
ta c phng trnh
3
cos x + cos y cos(x + y) = .
2

(1)

Ly
A = 2x y, B = 2y x, C = [(2x y) + (2y x)] = (x + y)
ta c phng trnh
3
cos(2x y) + cos(2y x) cos(x + y) = .
2
Cng (1) v (2) ta c bi ton sau
Bi 63. Gii phng trnh
cos x + cos y + cos(2x y) + cos(2y x) = 3 + 2 cos(x + y).

(2)

303
Gii
Phng trnh cho tng ng
[cos x + cos(2y x)] + [cos y + cos(2x y)] = 3 + 2 cos(x + y)
2 cos y cos(x y) + 2 cos x cos(x y) = 3 + 2 cos(x + y)
2 cos(x y) (cos x + cos y) = 3 + 2 cos(x + y)
x+y
xy
x+y
4 cos(x y) cos
cos
= 3 + 4 cos2
2
2
2
2
x+y
x+y
xy
4 cos2
4 cos(x y) cos
cos
+1=0
2
2
2

2
x+y
xy
xy
2 cos
cos(x y) cos
+ 1 cos2 (x y) cos2
= 0.
2
2
2
V 1 cos2 (x y) cos2

xy
> 0 nn
2

2 cos x + y cos(x y) cos x y = 0


2
2
(1)
1 cos2 (x y) cos2 x y = 0
2

x
+
y
xy

2 cos
cos(x y) cos
=0
2
2

1 + cos(x y)

1 cos2 (x y)
=0
2

2 cos x + y cos(x y) cos x y = 0


2
2

cos2 (x y) + cos3 (x y) = 2

cos(x y) = 1

xy
x+y
2 cos
= cos
2
2

(1)

(2)

xy
xy
1 = 1 cos2
= 1 nn
2
2

cos(x y) = 1
cos(x y) = 1
cos(x y) = 1

x+y
x+y
1
x+y
1
cos
cos2
2 cos
= 1
=
=
2
2
2
2
4

cos(x y) = 1
cos(x y) = 1
1
1
cos(x + y) = .
1 + cos(x + y) =
2
2

V cos(x y) = 1 2 cos2

(2)

S dng hm ngc sng tc mt s phng trnh, h phng


trnh.
y l mt phng php sng tc v gii phng trnh, h phng trnh kh hay. Gi s
hm s y = f (x) c hm s ngc l y = g(x). Nu v th ca hai hm s trn cng mt h
trc to -cc vung gc th hai th y i xng nhau qua ng phn gic th nht.
Do nu hm s y = f (x) c hm s ngc l y = g(x) th vic gii phng trnh f (x) = g(x)
quy v gii phng trnh f (x) = x (hoc g(x) = x). Tm li, trong mt s trng hp ta s

304
dng mnh sau.
Mnh . Cho y = f (x) l hm ng bin, c hm ngc y = f 1 (x) v tp xc nh ca
phng trnh f (x) = f 1 (x) bng Df Df 1 . Ta c php bin i sau
(

f (x) = x
x Df Df 1

(1)

f 1 (x) = x
x Df Df 1 .

(2)

f (x) = f 1 (x)

hoc
(
f (x) = f

(x)

Nh vy vic gii phng trnh f (x) = f 1 (x) ta thay th c bi cc h phng trnh tng
ng (1) hoc (2).
Chng minh.
Gi s f (x) = f 1 (x). Khi x = f (f (x)). Nu x > f (x) th do f ng bin nn f (x) >
f (f (x)) f (x) > x (v l). Nu x < f (x), tng
( t ta cng suy ra v l. Vy x = f (x).
x = f 1 (x)
Gi s x = f (x) v x Df Df 1 . Khi
f (x) = f 1 (x).
x = f (x)
Vy (1) c chng minh.
Ch 1. Cho hm s y = f (x) c hm ngc y = f 1 (x). Khi f ng bin f 1 ng
bin.
Ch 2. Nu hm s f lin tc v n iu nghim ngt trn khong (a; b) th tn ti hm
s ngc f 1 .
2
V d
r 51. Xt hm s y = f (x) = 2x + 4x 1 trn (1; +). Hm s ngc ca f (x) l
x+3
1 xc nh trn (3; +). Khi
y=
2

r
2

2x + 4x 1 =

x+3
1 2x2 + 4x =
2

x+3
.
2

Ta c bi ton sau.
Bi 64 ( ngh OLYMPIC 30/04/2007). Gii phng trnh
r
x+3
2x2 + 4x =
, vi x > 1.
2
Gii
iu kin x > 1 (gi thit).
r
x+3
Cch 1. Xt hm s y =
1, x [1; +) . Khi
2

(
r
y > 1
y > 1
x+3
y+1=

x+3
2
y + 2y + 1 =
2
x = 2y 2 + 4y 1.
2

(*)

305
r
2

Suy ra hm s y = 2x + 4x 1 l hm ngc ca hm y =

x+3
1 trn [1; +). Mt
2

khc hm s y = 2x2 + 4x 1 ng bin trn [1; +) nn

3 + 17
x+3
x=
4
1 2x2 + 4x 1 = x
2x2 + 4x 1 =
3 17
2
x=
.
4

3 + 17
l nghim duy nht ca (*). 2
Kt hp vi iu kin suy ra x =
4
r
x+3
~ Nhn xt: V sao li tm hm ngc ca hm y =
1, x [1; +)? l do ta
2
gi s
r
x+3
x+3
ay + b =
a2 y 2 + 2aby + b2 =
x = 2a2 y 2 + 4aby + 2b2 3.
2
2

So snh vi phng trnh cho thy rng cn chn a, b sao cho


(
(
(
2a2 = 2
a = 1
a=1

4ab = 4
b = 1
b = 1.
r

x+3
1, x [1; +). Ngoi ra ta cn c th gii thch
2
nh sau: T
rphng trnh cho thy rng cn xt hm s c tp gi tr l [1; +) v c

x+3
dng y = a
1 vi a > 0 . Khi
2
Vy nn ta mi xt hm s y =

x+3
2
4
2
y 2 + 2y + 1 = a2 .
x = 2 y2 + 2 y + 2 3
2
a r a
a


2 4
x+3

; 2 = (2; 4) a = 1 y =
1.
2
a a
2
r
x+3
Cch 2. t y + 1 =
, iu kin y > 0. Ta c h
2

(
(y + 1)2 = x + 3
2y 2 + 4y = x + 1 (1)

2
2x2 + 4x = y + 1
2x2 + 4x = y + 1. (2)
Ly (1) tr (2) theo v ta c
2(y 2 x2 ) + 4(y x) = x y 2(y x)(y + x) + 5(y x) = 0

"
y=x
yx=0

5 2x
2(y + x) + 5 = 0
y=
.
2
Khi y = x, thay vo (2) ta c

3 + 17
x=
4
2x2 + 4x = x + 1 2x2 + 3x 1 = 0
3 17
x=
(loi).
4

306
Khi y =

5 + 2x
, thay vo (2) ta c
2

5 13
(loi)
x=
4
4x2 + 10x + 3 = 0
5 + 13
x=
.
4

5 + 13
ta c
Vi x =
4

5 + 2x
5 + 13
y=
=
< 0 (khng tho mn y > 0)
2
4

3 + 17
Kt hp vi iu kin suy ra x =
l nghim duy nht ca (*). 2
4



3
; + . Hm s ngc ca f (x) l
2

V d 52. Xt hm s y = f (x) = x 3x trn



3 + 9 + 4x
9
1
y = f (x) =
xc nh trn ; + . Khi
2
4

3 + 9 + 4x
= x2 3x 9 + 4x = 2x2 6x 3.
2
Ta c bi ton sau.
Bi 65. Gii phng trnh

9 + 4x = 2x2 6x 3.

()

Gii

3
9
iu kin x > . t 2y 3 = 9 + 4x, iu kin y > . Ta c
4
2
(
(
(
(2y 3)2 = 9 + 4x
4y 2 12y = 4x
2y 2 6y = 2x (1)

2y 3 = 2x2 6x 3
2x2 6x = 2y
2x2 6x = 2y (2)
Ly (1) tr (2) theo v ta c
2(y 2 x2 ) 6y + 6x = 2x 2y 2(y 2 x2 ) + 4(x y) = 0
"
y=x
(y x)(y + x 2)
y = 2 x.
Khi y = x, thay vo (1) ta c
"
2

2x 6x = 2x x 4x = 0

x=0
x = 4.

Khi x = 0 th y = 0 (loi). Khi x = 4 th y = 4.


Khi y = 2 x, thay vo (2) ta c
"
x2 3x = 2 x x2 2x 2 = 0

x=1 3

x = 1 + 3.

307

Khi x = 1 + 3 th y = 1 3, khng tho mn iu kin.

Khi x = 1 3 th th vo phng trnh cho thy tho mn. Vy phng trnh cho c

hai nghim x = 4, x = 1 3. 2


9
~ Nhn xt: Phng trnh () c tp xc nh l ; + , trong khi ta c
4



3
Df Df 1 = ; + , v vy (*) nhn x = 1 3 lm nghim l iu d hiu, v gii phng
2


9
trnh (*) l tm x thuc tp xc nh ; + v tho mn (*).
4
V d 53. Hm s y =
bi ton sau.

x + 3 l hm s ngc ca hm s y = x3 3 trn R. Do ta c

Bi 66. Gii phng trnh

x + 3 = x3 3
Gii

Tp xc nh R.
(

y 3 = x + 3 (1)
t y = 3 x + 3. Khi ta c h
x3 = y + 3 (2)
Ly (2) tr (1) theo v ta c x3 y 3 = y x x3 + x = y 3 + y.
Xt f (t) = t3 + t. V f 0 (t) > 0, t R nn (3) f (x) = f (y) x = y. Do
x3 = x + 3 x3 x = 3

(3)

(4)

2
Gi s x = t, thay vo (4) ta c
3

9 3
4t 3t =
.
2
3

9 3
> 1. Xt
cho gn, k hiu m =
2



2
1
1
3
m=
+ 3 3 2m3 + 1 = 0 3 = m m2 1.
2

s
r
r

9
3
239
3
+
239
3 9
3
3
Do , nu t = m + m2 1 =
+
=
th
2
4
2

m=

1
2

3 +

1
3

1
3 239
3 9
, =
.

rng
1
2
Vy

1
+ 3

 
3
 

1
1
1
1
=4
+
3
+
.
2



1
1
1 3 9 3 + 239
3 239
3 9
t0 =
+
=
+
2

2
2
2

(5)

308
l mt nghim ca phng trnh (5). Ta s chng minh t0 l nghim duy nht ca phng
trnh. Ta c |t0 | > 1 v
4t3 3t = 4t30 3t0 (t t0 )(4t2 + 4t0 t + 4t20 3) = 0.
rng phng trnh 4t2 + 4t0 x + 4t20 3 = 0 v nghim do
0 = 4t20 4(4t20 3) = 12 12t20 = 12(1 t20 ) < 0 (v t20 > 1).

p

1 p
3
3
2
2
Vy t =
m + m 1 + m m 1 l nghim duy nht ca phng trnh (5). Do
2
phng trnh cho c nghim duy nht l
s

1
3 + 235
3 235
3 9
3 9
x=
+
.2
2
2
3
Bi 67. Cho trc s thc a. Gii phng trnh (n l x)
a2011 + x =

2011

a x.

Gii
Tp xc nh R. Xem a nh n s. Xt hm y = f (a) = a2011 + x. Ta c
y = a2011 + x a2011 = y x a =

y x.

2011

Vy hm s y = 2011 a x l hm ngc ca hm y = a2011 + x, a R. Mt khc v


0
(a2011 + x) = 2011a2010 > 0, a R nn y = a2011 + x l hm ng bin trn R, do
(

a2011 + x = a
2011
ax
x = a a2011 .
a2011 + x =
aR
Vy phng trnh cho c nghim duy nht x = a a2011 .

Sng tc h phng trnh


gii h phng trnh th phng php th v phng php cng l thng dng nht, l
phng php c bn nht. T bi hc "v lng" v h phng trnh c phng php ny.
Tuy nhin phng php ny vn thng xut hin nhng k thi ln, nhng k thi ch dnh
cho nhng hc sinh xut sc. c nhiu ti liu v h phng trnh cp n phng php
ny, do vy sau y ta ch trnh by mt s bi ton kh v i su hn vo vic phn tch k
thut gii cng nh k thut sng tc.

2
2
2
2 2

x (y + z) = (3x + x + 1) y z
Bi 68. Gii h phng trnh
y 2 (z + x)2 = (4y 2 + y + 1) z 2 x2

2
z (x + y)2 = (5z 2 + y + 1) x2 y 2 .
Gii

309
D thy rng (x; 0; 0) , (0; y; 0) , (0; 0; z) tho mn h cho. Tip theo xt xyz 6= 0. Khi
chia cc phng trnh ca h cho x2 y 2 z 2 ta c

2

1 1
1
1

1
1
1
2
1

+
+
=
3
+

+ 2 =3+ + 2
+
2

2
z
y
x
x

zy y
x x

2

z1
1
1
2
1
1
1
1 1
+
+
+
=
4
+
+
=4+ + 2

x2 zx z 2
y y2
y y

 x z 2

1
1
1
2
1

1
1
1 1

2+
+ 2 = 5 + + 2.

+
=
5
+
+
.

y
xy x
z z
y x
z z2
Cng li ta c
1 1 1
+ + = 3

y z
12 = 0 x
1 1 1
+ + = 4.
x y z

Vi

Vi

1 1 1
+ +
x y z

2

1 1 1
+ +
x y z

1 1 1
+ + = 3, ta c
x y z

2

1
1
1

5
3
+
+
=
3
+

= 6
x=
x
x x2


2
5
5
1
1
1
= 5
y=
=4+ + 2
3+

y
5
y
y
y

2


5
5

1
1
1
z= .

= 4

=5+ + 2
3+
4
z
z
z z
1 1 1
+ + = 4, ta c
x y z

2

1
1
1

4
=3+ + 2

=
13
x=
x
x x

x

9

2
1
1
1
= 12
y=
4
=4+ + 2

y
y
y
y

2


9 = 11
z=

1
1
1

4
=5+ + 2

z
z
z z

9
13
9
12
9
.
11

Cc nghim ca h l



9 3 9
(x; y; z) =
, (x; y; z) =
; ;
,
13 4 11

(a; 0; 0) , (0; b; 0) , (0; 0; c) vi a, b, c R, tu . 2
5
5
; 1;
6
4

x2 (y z) =
(1)

3
Bi 69. Gii h phng trnh
y 2 (z x) = 3
(2)

z 2 (x y) = 1 . (3)
3
Gii
Ta c hng ng thc sau :



x2 y 2 x2 z 2 + z 2 y 2 x2 y 2 + x2 z 2 y 2 z 2 = 0.

310
T nhn (1) vi y + z, nhn (2) vi z + x, nhn (3) vi x + y ta c
5
1
(y + z) + 3 (z + x) + (x + y) = 0
3
3
5y 5z + 9z + 9x + x + y = 0
5x + 2z
5x
4y = 10x 4z y =
yz =
.
(4)
2
2
5x
5
T (4), (1) c
= 2 , suy ra x, th vo (2), (3) c h hai n. Cng vic tip theo tr
2
3x
nn n gin.

(1)
xz + y = 7z
Bi 70. Gii h phng trnh
yz + x = 8z
(2)

x + y + z = 12. (3)
Gii
T (1) c y = 7z zx. Thay vo (2) v (3) c
(
(
(7z zx) z + x = 8z
x (1 z 2 ) = 8z 7z 2 (4)

x + (7z zx) + z = 12
x (1 z) = 12 8z. (5)
12 8z
. Thay vo (4) c
1z
"
z=2
(12 8z) (1 + z) = 8z 7z 2 z 2 + 4z 12 = 0
z = 6.

R rng z = 1 khng tho (5), do (5) x =

60
66
Vi z = 2 ta c x = 4, y = 6. Vi z = 6 ta c x = , y = .
7
7


60 66
Kt lun: h c hai nghim (4; 6; 2) ,
; ; 6 . 2
7 7
Bi 71 (HSG tnh Qung Bnh, nm hc 2010-2011). Gii h
(

x xy1=1
(1)

y 2 + x + 2y x y 2 x = 0. (2)
Gii
iu kin x > 0 v x y 1 > 0. Phng trnh (1) tng ng
p
p
p

x= xy1+1x=xy+2 xy1y =2 xy1

y 2 = 4 (x y 1) (y + 2)2 = 4x y + 2 = 2 x.
Phng trnh th hai ca h tng ng vi

2

y 2 + x + 2y x y 2 x = 0 y + x = xy 2 y + x = y x.
(
(

y+2=2 x
y+2=2 x
Ta thu c


y + x = y x
2y + (y + 2) = (y + 2) y.

1
9 17 17 5
T tm c nghim ca h l (x; y) = ( ; 1), (4; 2), (
;
). 2
4
8
2

311
Bi 72. Gii h phng trnh sau :
(
4x2 + y 4 4xy 3 = 1 (1)
4x2 + 2y 2 4xy = 2. (2)
Gii
Ly (1) tr (2), v theo v, ta c :
2

y 4 2y 2 4xy 3 + 4xy + 1 = 0 y 2 1 4xy y 2 1 = 0


y 2 1 y 2 1 4xy = 0 y = 1 hoc y = 1 hoc y 2 1 + 4xy = 0.
Nu y = 1, thay vo phng trnh (1) ta c :
4x2 + 1 4x = 1 x (x 1) = 0 x = 0 hoc x = 1.
Nu y = 1, thay vo phng trnh (1) ta c :
4x2 + 1 + 4x = 1 x (x + 1) = 0 x = 0 hoc x = 1.
Nu y 2 1 + 4xy = 0 x =

1 y2
(d thy trong trng hp ny y 6= 0), thay vo phng
4y

trnh u tin ta c:
2


1 y2
1 y2
4
4
+y 4
y3 = 1
4y
4y


2 2
6
4
1y
+ 4y 4y 1 y 2 = 4y 2
2


1 y 2 4y 4 1 y 2 = 4y 2 1 y 4



1 y2
1 y 2 4y 4 4y 2 1 + y 2 = 0


1 y 2 8y 4 + 5y 2 1 = 0


Cng vic n y tr nn n gin.


~ Nhn xt: y l mt dng h phng trnh a thc kh kh, r rng nu phng trnh th
hai, ngi ta chia hai v cho 2 th kh c th nhn bit gi tr ny m nhn vo ri tr tng v
nh trn. Vic pht hin ra gi tr 2 nhn vo c th dng cch t tham s ph ri la chn.
(
Bi 73. Gii h phng trnh

y 3 x x4 = 28

xy 2 + 2x2 y + x3 = 18 2
Gii

Ta bin i tng ng
(

x(y 3 x3 ) = 28
(1)

x(y + x)2 = 18 2. (2)

Suy ra (x; y) l nghim th y > x > 0. T (2), rt y theo x v th vo (1) c

348
x[( x)3 x3 ] = 28.
x

(3)

312
t t =

x (t > 0) v (3) thnh :

4
t9 (3 8 t3 )3 + 28t = 0.

(4)

Xt hm f (t) = t9 (3 8 t3 )3 + 28t, vi t > 0.


Ta c f 0 (t) > 0 vi mi t > 0.
Chng t hm f ng bin trn (0; +), nn (4) c nghim th nghim duy nht.
T h c nghim (x0 ; y0 ) th nghim l duy nht. D thy y = 2x th t (1), ta c: x4 = 4

hay x = 2. Suy ra y = 2 2.

Th li ( 2; 2 2) tha mn (2).

Tm li h cho c nghim duy nht l (x; y) = ( 2; 2 2). 2

KINH NGHIM GII MT S BI H PHNG TRNH


Khi gii h phng trnh, d c dng cch g bin i i chng na th mc ch cui cng ca
ta cng chuyn v phng trnh mt bin v gii phng trnh va thu c. cng l suy
ngh t nhin, vic lm gim bin l quy lut khng ch trong ton hc m c trong cuc sng
chng ta vn thng lm. Tm li, khi gii h phng trnh th chng ta phi tm cch lm
gim s n ca h thun li trong vic gii n. Sau y l mt s kinh nghim m nhm
bin son thu c trong qu trnh hc tp v ging dy, xin c chia s cng bn c.

Nu trong phng trnh ca h m c mt n xut hin di dng bc nht, th ta c th rt


n theo n cn li v th vo phng trnh th hai ca h v phng trnh thu c c bc
khng nh nhng tng gii l rt r rng.
(
Bi 1: Gii h phng trnh

2x3 + y(x + 1) = 4x2


5x4 4x6 = y 2
Gii

> Cch 1:
V phng trnh th nht ca h cha nhiu y nn ta ngh n vic rt y theo x v th vo
phng trnh th hai ca h.
2x2 (2 x)
Ta c y =
(do x = 1 khng l nghim ca h).
x+1
Thay vo phng trnh th hai ca h ta c :
"
2
4

x=0
4x
(2

x)
x4 5 4x2 =

(x + 1)2
(5 4x2 )(x2 + 2x + 1) = 4(4 4x + x2 )
"
"
x=0
x=0

4x4 + 8x3 + 3x2 26x + 11 = 0


(x 1)(2x 1)(2x2 + 7x + 11) = 0

x=0y=0

x = 1 y = 1

1
1
x= y=
2
2

313
1 1
Vy h cho c nghim: (x; y) = (0; 0), (1; 1), ( ; ) 2
2 2
~ Nhn xt: Cch gii ny c mt u im l khng cn phi mnh khe g c m ch cn bin
i ht sc bnh thng. Tuy nhin, n c mt nhc im l n ch gip chng ta gii quyt
bi ton thi, cn con ng sng tc ra bi ton th cch gii trn khng th lm r
c. hiu r c ngun gc ca bi ton v l cch m tc gi sng tc bi ton
trn, ta cng xem qua cch gii sau:
> Cch 2:
Ta vit li h nh sau:
(

2x3 + y(x + 1) = 4x2


y 2 + 4x6 = 5x4

Nhn thy x = 0 y = 0, hay (x; y) = (0; 0) l mt nghim ca h.


Vi x 6= 0 ta c h

2x + 2 (x + 1) = 4
x
 y 2

+ 4x2 = 5
x2
y
t a = 2x, b = 2 ta c c h:
x

a + b( a + 1) = 4
2
a2 + b 2 = 5
y l h i xng loi I. Vic gii h ny khng my kh khn.
~ Nhn xt: Qua li gii trn, ta thy con ng ch tc ra nhng h kiu ny l xut pht
t mt h bit thut gii, chng ta thay th hnh thc ca cc bin c mt trong h v bin
i rt gn ta thu c
( mt h c hnh thc hon ton xa l vi ci h ban u.
x + y + xy = 5
Chng hn: T h
(lu h ny c nghim (1; 2))
x2 + y 2 = 5
y
Ta thay th x bng 3 v y bng y 2 th ta c h:
2x

y
y3

3 + y2 + 3 = 5
y(y 2 + 2x3 y + 1) = 10x3
2x
2x

y2
y 2 (1 + 4x6 y 2 ) = 20x6

+
y
=
5
4x6
Vy ta c h phng trnh sau:
(

y(y 2 + 2x3 y + 1) = 10x3


y 2 (1 + 4x6 y 2 ) = 20x6

(
Bi 2: Gii h phng trnh

x2 2xy + x + y = 0 (1)
x4 4x2 y + 3x2 + y 2 = 0 (2)
Gii

314
Nhn thy phng trnh th nht ca h l phng trnh bc nht i vi x nn ta rt x theo
y v th vo phng trnh th hai c phng trnh mt n.
x2 + x
1
T (1) suy ra y =
( do x = khng l nghim ca h), thay vo (2) ta c
2x 1
2
"
 2
2
2
x=0
x
+
x
x
+
x
+ 3x2 +
=0
x4 4x2
2x 1
2x 1
f (x) = 0
Vi f (x) = x2 (2x 1)2 4(x2 + x)(2x 1) + 3(2x 1)2 + (x + 1)2 = 4x4 12x3 + 10x2 6x + 4
Ta c f (x) = 0 2x4 6x3 + 5x2 3x + 2 = 0 (x 1)(x 2)(2x2 + 1) = 0 x = 1, x = 2
Vy h cho c 3 nghim (x; y) = (0; 0), (1; 2), (2; 2) 2
~ Nhn xt: Cng nh v d 1, cch gii trn ch gii quyt c bi ton ch khng phi l
con ng sng tc bi ton . iu ny thi thc chng ta i tm mt li gii khc cho
bi ton trn. S xut hin x2 2xy v x4 4x2 y gi cho ta ngh n cc hng ng thc. Vy
ta th vit li h nh sau:
(
(x y)2 + x + y y 2 = 0
2

(x2 y) + 3x2 3y 2 = 0
Vic lm ny cng khng my kh quan, v khi nhn vo h chng ta cng cha pht hin c
mi lin h no. Theo cch lm v d 1 ta bin i:
Nu x = 0 y = 0 l nghim ca h.
Nu x 6= 0, ta c h

y
y

x + = 2y + 1
x 2y + 1 + = 0
x
x

2
y 2
y

(x + ) = 6y 3
x2 4y +
+
3
=
0
x
x2
Suy ra (2y + 1)2 = 6y 3. n y th bi ton tr nn n gin.
Vi cch gii trn, ta c th to ra c rt nhiu h phng trnh khc nhau. y chng ta
ch rng vic gii h cui cng quy v gii cc phng trnh bc hai nn chuyn cc h s
nhn nhng gi trno khng quan trng.
2y

x + x = 4x + 4
2y = 4x3 x2 + 4x


Chng hn t h
qua khai trin ta c h
2
2y

4x2 y + 4y 2 = 3x2

x+
= x2 3
x
hai bi trn chng ta gii theo phng php th. Du hiu nhn thy l vic xut hin ca
mt phng trnh l phng trnh bc nht i vi mt n. By gi chng ta chuyn qua xt
mt s h m chng ta thc hin rt th m phng trnh i vi mt n trong mt phng
trnh no khng phi l phng trnh bc nht.
(
Bi 3: Gii h phng trnh

x3 8x = y 3 + 2y (1)
x2 3 = 3(y 2 + 1) (2)
Gii

> Cch 1:

315
T (2) ta suy ra: x2 = 3(y 2 + 2) (3) , thay vo (1) ta c:

x=0
yx
x(3x2 xy 24) = 0
3x2 24
3
y=
x
Nu x = 0 thay vo (3) ta thy phng trnh v nghim.
3x2 24
thay vo (3) ta c
Nu y =
x
 2
2
3x 24
2
x =3
+ 6 13x4 213x2 + 864 = 0
x

2
x = 3 y = 1
x =9

96
96
78
2
x =
y=
x=
13
13
13
r
!
96
78
Vy h c 4 cp nghim l: (x; y) = (3; 1),
;
2
14
13
2

x3 8x = y(y 2 + 2) =

~ Nhn xt: Chng ta ngh n php th do phng trnh th nht ch cha y 3 v y; phng
trnh th hai ca h li cha y 2 nn nu ta thay y 2 vo phng trnh th nht th phng trnh
th nht ca h tr thnh phng trnh bc nht i vi n y v ta thc hin rt y nh trn.
Tuy nhin, c l y cng khng phi l con ng ch tc bi ton trn. T nhn xt trn, ta
thy phng trnh th nht hai bin x, y lch bc nhau 2 bc ( x3 v x; y 3 v y), ng thi
phng trnh th hai cng lch bc nhau 2 bc ( x2 , y 2 v hng s).
iu ny gi ta to ra s ng bc nh sau:
> Cch 2:
H cho tng ng
( 3
x y 3 = 8x + 2y
2

6 = x 3y

6(x3 y 3 ) = (8x + 2y)(x2 3y 2 )

y l phng trnh ng cp bc 3. Vic cn li gii quyt h khng cn kh khn na.


(
Bi 4: Gii h phng trnh

x3 + 3xy 2 = 49 (1)
x2 8xy + y 2 = 8y 17x (2)
Gii

Bi ton ny c hai cch gii bng h s bt nh v n ph tng - hiu. Sau y ta cng


xem qua hai cch gii khc:
> Cch 1:
Ta thy x = 0 khng phi l nghim ca h nn t (1) suy ra y 2 =

x3 + 49
()
3x

Th vo phng trnh (2) ta c


x3 + 49
= 8y 17 24y(x2 + x) = 2x3 + 51x2 49
3x

x = 1
24xy(x + 1) = (x + 1)(2x2 + 49x 49)
2x2 + 49x 49
y=
24x

x2 8xy

316
Nu x = 1 th vo (*) y = 4.
2x2 + 49x 49
Nu y =
th vo (*), ta c:
24x
 2
2
x3 + 49
2x + 49x 49
2

=
192x(x3 + 49) = (2x2 + 49x 49)
3x
24x
Bin i rt gn ta c:
4x4 + 4x3 + 45x2 + 94x + 49 = 0 (x + 1)2 (4x2 4x + 49) = 0 x = 1
Vy h c nghim: (x; y) = (1; 4) 2
> Cch 2:
V x = 0 khng l nghim ca h nn ta t y = tx.
Khi h tr thnh:
( 3
x (1 + 3t2 ) = 49
x2 (1 8t + t2 ) = x(8t 17)
Ta vit li h di dng

49
49
49
3

x = 1 + 3t2 = 49 + 3(t2 16) = 49 + 3a


vi a = t2 16; b = 8t 17
8t

17
b
8t

17

x = 2
= 2
=
t 8t + 1
(t 16) (8t 17)
ab
3
49
b
3
3

=
+ 3ab3 = 0
3 49 b + (a b)
49 + 3a
(a b)



2
a 49 b b(a b) + (a b)2 + 3b3 = 0
"
a=0


49 b2 b(a b) + (a b)2 + 3b3 = 0 (*)
Vi a = 0 t2 = 16, thay vo h ta c x = 1 y = 4.
Xt (*), khai trin v rt gn, ta c:
() 49t4 + 360t3 + 547t2 360t + 304 = 0 (t + 4)2 (49t2 32t + 19) = 0 t = 4
~ Nhn xt: Hai cch gii trn i hi tnh ton nhiu, v cha th ni ln tng ca ngi
ra . i vi bi ny cch gii hay nht l dng h s bt nh ( c cp trong chng
H phng trnh). Xut pht t nghim x = 1, ta s to ra cc phng trnh cha nhn t
(x + 1), t khai trin c(mt h mi. V d nh hai bi sau:
x3 + 2xy 2 = 5
Bi 4*: Gii h phng trnh :
2x2 + xy + y 2 = 4x + y
( 3
x + y 2 = (x y)(xy 1)
Bi 4**: Gii h phng trnh :
x3 x2 + y + 1 = xy(x y + 1)
(
1 + x3 y 3 = 19x3
Bi 5: Gii h phng trnh :
y + xy 2 = 6x2
Gii

317
V x = 0 khng l nghim ca h nn h tng ng

1
( 3

3 + y 3 = 19
a + y 3 = 19
1
x

(vi a = )
2
2
1
1

x
a y + y a = 6
y + y 2 = 6
2
x
x
t S = a + y, P = ay. Khi ta c
(

S(S 2 3P ) = 19
SP = 6

S=1
P = 6

a=3
y = 2

a = 2
y=3

1
1
Vy nghim ca h l: (x; y) = ( ; 2), ( ; 3) 2
3
2
~ Nhn xt: Ngoi cch gii trn, ta c th gii theo cch sau:
Do x = 0 khng l nghim ca h, ta vit li h di dng
(

6(1 + xy)(1 xy + x2 y 2 ) = 6.19x3


19xy(1 + xy) = 19.6x2

Cng hai phng trnh ca h li ta c: (1 + xy)(6x2 y 2 + 13xy + 25) = 0 n y, bi ton


tr nn n gin.
~ Nhn xt:Cng tng , (
ta c bi ton sau:
y + xy 2 = 6x2
Bi 5*: Gii h phng trnh
1 + x2 y 2 = 5x2
( 2
(x + 1)y 4 + 1 = 2xy 2 (y 3 1)
Bi 6: Gii h phng trnh
xy 2 (3xy 4 2) = xy 4 (x + 2y) + 1
Gii
H cho tng ng
(

x2 y 4 + 2xy 2 + 1 + y 4 2xy 5 = 0

3x2 y 6 2xy 2 x2 y 4 2xy 5 1 = 0

1
x

x + 2 2 + 4 2xy = 1
y
y

2x
1

3x2 y 2 2 x2 2xy 4 = 0 ( do y=0 khng l nghim ca h)


y
y

2

x + y 2 2xy = 1


2

2
2

=0
3x y 2xy x + 2
y
t a = x +

1
, b = xy, ta c h
y2
(

a2 2b = 1
a2 3b2 + 2b = 0

a2 = 2b 1
3b2 4b + 1 = 0

b=1
a = 1

318

1+ 5
51

1
1

x =
x +
x =
x =
=1
2 v
y2
y
2
Nu a = b = 1 :

y = 1 5
y = 1 + 5
y y1=0
xy = 1
2
2

x + 1 = 1
x = 1
y2
y
Nu a = 1; b = 1 :

(v nghim)

2
xy = 1
y +y+1=0
!

1 5 1 5
;
2
Vy nghim ca h cho l: (x; y) =
2
2

6x5 y
2

(x

1)
+
3
=

x2 + 2
s
Bi 7: Gii h phng trnh

4x 3x2 y 9xy 2

3y x =
x + 3y
Gii
(
T phng trnh th hai, ta c

3y > x
x + 3y 6= 0

H cho tng ng vi:

6
2
2
5

(x 2x + 4)(x + 2) = 6x y
x + 8 = 6x5 y
4x
4x 3x2 y 9x2 y 2 2
2

9y 6xy + x2 =
3xy
(3y x) =
x + 3y
x + 3y
( 6
( 6
x + 8 = 6x5 y
x + 8 = 6x5 y

(x2 3xy + 9y 2 )(x + 3y) = 4x


x3 + 27y 3 = 4x
V x = 0 khng l nghim ca h, nn ta c

6y
8

1 + 6 =
x
x
3
27y
4

1 +
= 2
3
x
x
t a =

2
3y
> 0, b = , ta thu c h
2
x
x
(
(
(
a=b
1 + a3 = 2b
1 + a3 = 2b

1 + b3 = 2a
(a b)(a2 + ab + b2 + 2) = 0
a3 2a + 1 = 0

a = b
a=b

5
1
+
2
a = 1, a =
(a 1)(a + a 1) = 0

Vi a = b = 1, ta c:

2 =1
x = 2
x = 2
x
v

2
2
3y

y =
y =
=1
3
3
x

319

1 + 5
Vi a = b =
ta c:
2

q
q

1
2

2 =
x =
x =
5+1
5+1
x
p
p

 v

2

5+1 51
5+1 51
5

1
3y

y =
y =
=
6
6
x
2
!
!
p

2
( 5 1)
5+1
,
2
Vy nghim ca h cho l (x; y) = 2;
5 + 1;
3
6
( 4

6x x3 x y 2 (y + 12) x2 = 6
Bi 8: Gii h phng trnh
2
5x4 x2 1 y 2 11x2 = 5
Gii
Nhn thy, x = 0 khng l nghim ca h nn chia hai v ca mi phng trnh cho x2 ta c:


 

2 

1
1

1
1
2
2
2

6 x x x x y y = 0
6 x + x2 x x y y 12 = 0


2
 
2 
2


1
1
1
1

2
2

11
=
0
5
x
+

x
y2 1 = 0
5 x

x2
x
x
x
1
t a = x , ta c h
x

6a2 ay 2 y = 0
5a2 a2 y 2 1 = 0

Chia hai v ca h cho a2 6= 0 ta c:


2
y
y

+
=6
a
a2

y2 + 1 = 5
a2
1
t u = , ta c h
a
(

y 2 u + u2 y = 6
2

uy(u + y) = 6
2

u+y =3

u +y =5
(u + y) 2uy = 5
uy = 2
(
(
u=1
u=2
T ta tm c:
hoc
y=2
y=1

1 5
1
2
Vi u = 1 a = 1 x = 1 x x 1 = 0 x =
x
2
1
1
1
1
17
Vi u = 2 a = x = 2x2 x 2 = 0 x =
2
x
2
4!
!

1 5
1 17
Vy nghim ca h cho l (x; y) =
;2 ,
;1 2
2
4

Bi tp t luyn

320
Gii
cc h phng
q
q trnh sau:

x+ y+ x y =2
q
1) q

y + x y x = 1
( 2
x xy + y 2 = 3(x y)
2)
x2 + xy + y 2 = 7(x y)2
(

x+y+ xy =8
3)

y xy =2
(
x3 + 3xy 2 = 6xy 3x 49
4)
x2 8xy + y 2 = 10y 25x 9
( 4
x + 2x3 y + x2 y 2 = 2x + 9
5)
x2 + 2xy = 6x + 6

x2 + y + x3 y + xy 2 + xy =
4
6)
5

x4 + y 2 + xy(1 + 2x) =
4
(
2
2
xy + x + y = x 2y
7)
p

x 2y y x 1 = 2x 2y
( 4
x x3 y + x2 y 2 = 1
8)
x3 y x2 + xy = 1
(
x+y >4
9)
(x4 + y 4 )(x7 + y 7 ) = x11 + y 11

2p 2
1 2

y
+
x

12y
+
1
=
(x + 17)

3
12
s
10)
2

2x
x
x3 x2 y

+
=
+

8y
3
3y
4
2
p
3 + 2x2 y x4 y 2 + x2 (1 2x2 ) = y 4
q
11)
1 + 1 + (x y)2 = x2 (x4 + 1 2x2 2xy 2 )
p
6
y + y 3 + 2x2 = xy x2 y 2
q
12)
4xy 3 + y 3 + 1 > 2x2 + 1 + (2x y)2
2
(
3
3
x(y x ) = 7
13)
x4 + x3 y + 9y = y 3 x + x2 y 2 + 9x

xy+1 = (y + 1)x
14)

2x2 9x + 6
4x2 + 18x 20 +
= y+1
2x2 9x + 8

Ph lc 1: GII TON BNG PHNG TRNH H PHNG TRNH


Bi 1 (IMO 1967): Trong 1 cuc thi u th thao, tng s huy chng l m, c pht
1
trong n ngy thi u. Trong ngy th nht, ngi ta pht 1 huy chng v s huy chng
7
1
cn li. Ngy th hai, ngi ta pht 2 huy chng v s huy chng cn li. Nhng ngy
7
cn li tng t. Ngy sau cng cn li n huy chng pht. Tnh m, n?
Gii
Gi s s huy chng cn li khi bt u ngy thi u th r l mr .
6(mk k)
Khi m1 = m, mn = n v k < n :
= mk+1
7
Ta bin i:
"
 2
 n1
 n
 n+1 #
7
7
7
7
7
7n (n 6)
m = 1+2. +3.
+...+n.
= 36 1 (n + 1).
+n
= 36+
6
6
6
6
6
6n1
.
Do (6; 7) = 1 n 6 .. 6n1 |n 6| > |6n1 |
D thy ch c n = 6 tho. Vy m = 36 2.
Bi 2 (APMOPS 2004): Mt ng c pht trin theo mt tc khng i:
200 con cu n ht c trong 100 ngy.
150 con cu n ht c trong 150 ngy.
Hi 100 con cu n ht c trong bao nhiu ngy?
Gii
Gi s mi con cu n ht 1 "ming" c mi ngy, vy trong 100 ngy 200 con cu s n ht
20000 "ming" c, tng t, 150 con cu s n ht 22500 "ming" c trong 150 ngy.
t x (n v: "ming" c) l lng c ng c ban u, a l tc mc c (n vi: "ming"
c/ngy), t l thi gian 100 con cu n ht ng c (n vi: ngy), t gi thit, ta c h:

x + 100a = 20000 (1)


x + 150a = 22500 (2)

x + at = 100t (3)

T (1) v (2) ta suy ra x = 15000, a = 50, vy t y ta d dng suy ra t = 300 2.


Bi 3: Mt ngi cha khi cht li di chc phn chia gia ti cho cc con l cc ng tin
1
vng nh sau : con c uc 100 ng v
s cn li , ngi con th hai c 200 ng
10
1
1
v
s cn li, ngi con th ba c 300 ng v
s cn li,......C nh vy cho n
10
10
khi gia ti c chia ht th s tin mi con bng nhau. Hi ngi cha c bao nhiu ng
tin vng, c bao nhiu con v mi ngi c chia bao nhiu ng tin vng?
321

322
Gii
Gi s tin ca cha l x (ng) (x > 0)
x 100
Con c c 100 +
10
x 100
x 300
10
Con th hai c 200+
10
x 100
x 300
x 100
10
Cho 100 +
= 200 +
10
10
Gii phng trnh trn ta tm c x = 8100 nn c 9 ngi con v mi ngi con c 900
ng. 2
Bi 4: Mt nhm gm 21 ngi i du lch n cc nc Anh, Php,, trong mi
ngi i t nht 1 nc v khng ai i c 3 nc, bit rng:
S ngi i c 2 nc v Anh gp i s ngi i c c 2 nc Php v .
S ngi i c Php v li gp i s ngi i Anh v Php.
S ngi ch i (khng i Php v Anh) hn s ngi ch i Anh (khng i Php v )
l mt ngi v bng s ngi i Php.
Hi:
a/ Hy tm s ngi i ng 1 nc.
b/ Hy tm s ngi i t nht 1 trong 2 nc Anh v Php.
Gii
Gi A, P, Y l tp hp s ngi i n cc nc Anh, Php, . Gi x, y, z l s ngi ch i
Anh, Php, . Gi t l s ngi i c 2 nc Anh v Php, th s ngi i Php v l 2t,
cn s ngi i 2 nc v Anh l 4t.
Theo bi ta c h phng trnh:

x + y + z + 7t = 21 (1)
z = x + 1 (2)

z = y + 3t (3)
T (1) 7t < 21 t < 3
.
T (1); (2); (3) 3z + 4t = 22 (t 1) .. 3
M 0 6 t < 2 t = 1
Khi t = 1 th:

x + y + z = 14
z =x+1

z = y + 3
Do (x; y; z) = (5; 3; 6)
Vy c 14 ngi ch i ng 1 nc, t nht 15 ngi i t nht 1 trong 2 nc Anh v Php. 2

323
Bi 5: Mt ngi i xe p in dc ng tu in v nhn thy rng: c 7 pht c 1
on tu chy cng chiu vi anh ta v c 5 pht c 1 on tu chy ngc li. Hi cch
nhau bao nhiu pht th ga u tin c 1 on tu khi hnh? Bit rng, cc on tu
khi hnh sau nhng khong thi gian nh nhau v chuyn ng vi vn tc khng i,
khng dng li bt k ni no trn ng v ngi i xe p in cng i vi vn tc khng
i.
Gii
t t l khong thi gian xut pht gia 2 tu in (tnh bng pht), x, y ln lt l vn tc
ca tu in v xe p (n v di/pht, x > y), ta c khong cch gia 2 tu in l xt.
Gi s khi gp mt tu in, th khi xe p s cch tu i ngay sau l xt, theo cng thc
khi 2 xe i ngc chiu, ta c
xt = 5(x + y) (1)
Tng t, ta cng c
xt = 7(x y) (2)
35
T (1) v (2) ta suy ra c x = 6y, th vo (1), ta li c 6yt = 7(6y y) t =
6
35
pht 2.
Vy 2 tu khi hnh cch nhau
6
Bi 6: Mt h nc c cung cp bi 3 vi nc. Bit rng nu tng vi cung cp nc
cho h th vi th nht lm y h nhanh hn vi th hai l 5 gi, vi th ba li lm y
h nhanh hn vi th nht l 4 gi. Cn nu vi th nht v th hai cng cung cp nc
cho h th thi gian chng lm y h bng thi gian ca vi th ba. Hi nu c 3 vi cng
cung cp nc cho h th chng s lm y h trong bao lu?
Gii
Gi thi gian vi th ba lm y h l t (gi), t > 0.
Thi gian vi th nht lm y h l t + 4 (gi).
Thi gian vi th hai lm y h l t + 9 (gi).
1
1
Trong 1 gi, vi th nht v th hai chy c
+
h nc v vi th ba chy c
t+4 t+9
1
h nc.
t
Theo bi,ta c phng trnh:
1
1
1
+
=
t+4 t+9
t
Tng ng:
t(t + 9) + t(t + 4) = (t + 4)(t + 9)
Ta tm c t = 6.
Vy trong 1 gi, c ba vi chy c:
1
1
1
1
+
+ =
t+4 t+9 t
3
Vy nu c ba vi cng cung cp nc th sau 3 gi h s y. 2

324
Bi 7: C 3 i xy dng cng lm chung mt cng vic. Lm chung c 4 ngy th i
III c iu ng lm vic khc, hai i cn li cng lm thm 12 ngy na th hon
thnh cng vic. Bit rng nng sut i I cao hn nng sut i II, nng sut i III l
trung bnh cng ca nng sut i I v II. Nu mi i lm mt mnh mt phn ba cng
vic th mt 37 ngy mi xong. Hi nu mi i lm mt mnh th bao nhiu ngy xong
cng vic trn?
Gii
Gi thi gian cc i I, II, III lm ring xong cng vic ln lt l x, y, z (n v: ngy).
Theo bi,ta c h phng trnh:




1 1
1 1 1

4
+ +
+ 12
+
= 1 (1)

x y z
x y

1
1

>
(2)
x
y


1
1 1 1

=
+
(3)

z
2 x y

x + y + z = 37 (4)
3


2 1
2
+
T (1) v (3) ta c: 4
+ 12. = 1 z = 36
z z
z
1 1
1
T (3) ta suy ra + = , kt hp vi (4) ta s c:
x y
18

x + y = 75
xy = 1350
Do x, y l nghim ca phng trnh: x2 75x + 1350 = 0 x1 = 30 v x2 = 45
Vy thi gian cn cc i I, II, III hon thnh cng vic ln lt l 30 ngy, 45 ngy, 36 ngy.2
Bi 8: Lp 9A c 28 hc sinh ng k d thi vo chuyn Ton, L, Ha ca trng Nng
Khiu. Trong khng c hc sinh no ch chn lp L hoc ch chn lp Ha, s hc sinh
chn vo lp Ton v L bng s hc sinh ch chn vo lp Ton, c 6 hc sinh chn vo
lp Ton v Ha, s hc sinh chn vo lp L v Ha gp 5 ln s hc sinh thi vo c 3
lp. Hy tnh s hc sinh chn vo tng lp.
Gii
Gi T, L, H, A, B, C, D ln lt l s hc sinh ch chn mn Ton, mn L, mn Ha, mn Ton
v L, mn Ton v Ha, mn L v Ha.T gi thit,ta c:

L=H=0

A+D =T

B + D = 6

C + D = 5D

D>3

T + A + B + C + D = 28

325
Do C = 4D nn A + D + A +
6 + 4D = 28 2A + 5D = 22 D chn v D 6 4

A=1

B = 2
Nh vy ta tm c D = 4

C = 16

T =5
Vy c 12 hc sinh thi vo lp Ton, 21 hc sinh thi vo lp L, 22 hc sinh thi vo lp Ha. 2
Bi 9: Trong k thi hc sinh gii ca trng, nu sp mi phng thi 22 hc sinh th tha
1 em, nu gim 1 phng thi th s hc sinh c chia u cho mi phng. Tnh s hc sinh
tham gia k thi bit rng mt phng thi khng cha qu 40 hc sinh.
Gii
Gi s phng thi l x (x N, n v: phng).
Theo bi, nu sp mi phng thi 22 hc sinh th tha 1 em nn tng s hc sinh l 22x + 1
(em).
S phng sau khi gim l x 1 (phng).
.
Do lc ny s hc sinh chia u cho mi phng nn 22x + 1 .. x 1
.
M 22x + 1 = 22(x 1) + 23 23 .. x 1 x = 2 hoc x = 24
Ti x = 2, s hc sinh c l 45 em, khi y s hc sinh mi phng sau khi gim 1 phng l 45
em (tri vi gi thit).
Ti x = 24, s hc sinh c l 529 em, iu ny ph hp vi bi.
Vy c tt c 529 hc sinh. 2

Ph lc 2: PHNG TRNH V CC NH TON


HC NI TING

Lch s pht trin ca phng trnh


C my cch gii phng trnh bc hai?
Lch s phng trnh bc 2 bt ngun t nn vn minh Babilon c i (khong 1800 nm trc
cng nguyn).Lc h bit cch gii tt c cc phng trnh bc 2 nhng khng din t
trong tp s thc.
Vo khong 1500 nm trc cng nguyn,trong mt tc phm ca ngi Ai Cp v cc bi
ton c th c nhng v d v gii phng trnh bc 2.
Trng phi Pythagores (th k VI trc cng nguyn) gii phng trnh bc 2 bng hnh
hc v v sau ngi ta gi l phng php Pythagores.
th k III trc cng nguyn,ngi Hy Lp c i bin vic gii phng trnh bc 2 thnh
c s cho ton b hnh hc ca h v c th lm vic trong tp hp s thc,h thay th
cc tnh ton ca ngi Babilon bng cc php dng hnh bng thc thng v compa.Tuy vy
h ch tnh ton tp hp s hu t dng,cho nn c nhiu phng trnh bc 2 h khng gii
c.Phi ch n th k XVI,khi xut hin s phc th mi gii c tt c cc phng trnh
bc 2.
Trung Hoa c i,cch gii phng trnh bc 2 cng c trnh by trong b sch "Sch ton
chn chng",trong cun "Trng Khu Kin ton kinh" v trong cun "S th cu chng".
Ngi Hindu tha nhn mt phng trnh bc 2 c li gii thc th c hai nghim hnh thc.H
thng nht php gii i s cc phng trnh bc 2 bng phng php b sung bnh phng
quen thuc,do vy ngy nay phng php ny c gi l phng php Hindu.
Nh thi c i,ngi Babilon,ngi Ai Cp,ngi Hy Lp,ngi trung Hoa,... bit cch gii
phng trnh bc 2 nhng cng thc nghim th mi n nm 825 nh ton hc Al-Khowarizmi
mi lp c.ng gii phng trnh bc 2 bng i s v hnh hc.ng vit: x2 + px + q = 0

p  2 p2
thnh x +
=
q
2
4
T ng tm c nghim:
r
p
p2
x=
q
2
4
326

327
p
v thay vo phng trnh u c
2

 2
p
2
q =0
z
4

V sau ng li gii theo cch khc,ng t z = x +

Vic gii phng trnh ny tr thnh n gin.


Trong chng trnh i s lp 9 c cng thc tnh nghim ca phng trnh bc 2 dng
chnh tc ax2 + bx + c = 0 bng cch t = b2 4ac Nu < 0 th phng trnh v nghim.
Nu = 0 th phng trnh c nghim kp:
x1 = x2 =

b
2a

Nu > 0 th phng trnh c hai nghim phn bit:

b
+

x 1 =
2a

x = b
2
2a
Francois Viete (1540-1603) ngi Php a ra h thc ca hai nghim ny:

x 1 + x 2 = b
a
x x = c
1 2
a
H thc ny v sau c mang tn ng (h thc Viete).

Francois Viete (1540-1603)


Ngoi ra cn c cc cch gii phng trnh bc hai bng hnh hc sau y: cch ca Sir John
Leslie (1766-1832). ngi Anh trong cun "Cc c s ca hnh hc", cch ca Thomas Carlyde
(1795-1881) ngi Anh, hc tr ca Leslie,...
Phng trnh v nh bc hai 2 n y 2 = ax2 + 1 trong a l s nguyn dng khng chnh

328
phng c Brahma Gupta v Bhaskara ngi n gii. Ngi ta gi l phng trnh Pell.
L thuyt y v phng trnh c Joshep-Louis Lagrange (1736-1813), ngi Php hon
tt vo 1766-1769.

Joshep-Louis Lagrange (1736-1813)

Cuc thch chn ng th gii ton hc


Nh chng ta bit, vic tm li gii cho phng trnh bc 3 l kh phc tp v cng thc
nghim l kh cng knh.
Trong mt bn Babilon tm thy c cc gi tr ca n3 + n2 vi n = 1 n n = 30 v nh vy
chng ta cng tm c nghim ca 30 phng trnh bc 3 c bit. Nh ton hc Neugebauer
tin rng ngi Babilon hon ton c th quy mt phng trnh bc 3 tng qut v dng "chun"
x3 + x2 = c.
Trung Hoa, b sch "Chc c ton kinh" ca Vng Hin Chng vit vo u thi ng
(th k VII) c nu cch gii phng trnh bc 3 tng qut bng i s. Trong b sch "S th
cu chng", Tn Cu Thiu cng a ra phng trnh bc 3. Nhng sau , ngi Trung Hoa
qu ch n phng php i s trn bn tnh, m t ch s dng cc k hiu i s
nh ang dng, do vy t th k VII nn ton hc Trung Hoa bt u lc hu so vi Phng Ty.
Omar Khayyam (1048-1122) Khorasan gii phng trnh bc 3 bng hnh hc mt cch
c o.
Sau Al-Khowarizmi c nhiu nh ton hc i tm cch gii phng trnh bc 3 mt cch
mit mi. Nhng tri qua sut 7 th k, ngoi vic tm c cch gii ca nhng phng trnh
khc th khng c bc tin no cho vic gii phng trnh bc 3. V vy c ngi t ra
chn nn, cho rng phng trnh bc 3 c th khng c thc. Nhng gio s Scippionel del
Ferro (1465-1526) ngi Italia, Trng i hc Tng hp Bologna th khng nh vy. ng vn

329
tip tc con ng ca mnh l tm mit mi nghin cu vn hc ba lc by gi l gii
phng trnh bc 3. Tri khng ph ng, vo mt ngy "khng th tin c", ng tm
ra bc t ph v n nm 1505, ng tuyn b tm ra cch gii c bit cho phng trnh:
x3 + mx = n (vi m, n > 0)
Vo thi , mi ngi u gi b mt cch gii ca mnh, v vy vic Ferro gi kn bo bi
cng l iu khng c g l. Nhng ng tic l, ng khng c dp no cng b thnh
tu ca mnh. Mi n khi sp qua i, ng mi li b mt ny cho ngi con r tin cn l
Anabel Nova. Nhng v sau, mt mn sinh ca Ferro l Antonio Fior ly cp bo bi ca ng.
Trong trng phi Italia th Fontana (1500-1557) cng i tin phong trong vic tm cch gii
phng trnh bc 3.
By gi li ni n mt nh ton hc khc, l Niccolo Tartaglia (1499-1557). Thi th u
ca ng tri qua tht nng n. ng sinh ra Brescia min bc Italia nn c gi l Niccolo
ca Brescia v thng c gi l Tartagli v lc nh ng b qun Php lm hi d man. Lc
ng mi 13 tui th qun Php trn vo Brescia. ng cng vi ngi cha (lc l ngi a
th) v dn chng chy trn vo ngi nh th n nhng qun Php rt theo v thm ha
xy ra: ngi cha b git cht cn cu b b chm vo hm v ming. Vt thng vm
ming lm ng ni nng rt kh khn nhng ng c t cht thng minh li ham hc. Sau khi
m ng qua i, ng phi t i tm ng sng cho mnh. ng hc vt l, ton hc v t r ti
nng rt sm, c nhiu ngi thi knh phc.
Vo nm 1530 mt nh ton hc a ra cho ng hai cu hi mang tnh thch thc, nhm
h uy tn ca ng:
1. Tm mt s m lp phng cng vi 3 ln bnh phng th bng 5.
2. Tm ba s m trong s th hai ln hn s th nht l 2, s th ba ln hn s th hai l
2 v tch ca chng l 1000.
y thc cht l tm nghim ca phng trnh bc 3, vi phng trnh cu hi 1 l:
x3 + 3x2 5 = 0 v cu hi 2 l: x3 + 6x2 + 8x 1000 = 0 ng tm c nghim ca
c hai phng trnh ny nn ng ngy cng ni ting.
Cc mn sinh ca Ferro cng tuyn b gii c phng trnh bc 3. Khng ai chu ai nn
cui cng cc nh ton hc Italia quyt nh m cuc thch u gia hai bn, mi bn a ra
30 bi ton lm trong 2 gi.
Sp n ngy thi, Tartaglia cm thy nao nng v ng l ngi t hc, s khng bo v c
cch gii ca mnh. ng suy ngh rt nhiu, a ra nhiu phng n khc nhau v trc ngy
thi 8 ngy, ng tim ra phng php mi gii phng trnh bc 3.
ng hc thuc cch gii mi v ngh ra 30 bi ton m ch c cch gii ny mi thc hin c.
Vo ngy 22/2/1535, cc nh ton hc ko v thnh ph Milan d cuc so ti. Ba chc bi
ton m mi bn a ra u l phng trnh bc 3. Tartaglia gii 30 bi ton m i phng
a ra trc gi quy nh, trong khi nhm mn sinh Ferro khng gii c bi no trong s

330
30 ton ca Tartaglia. Nh vy cuc thi kt thc v phn thng tuyt i thuc v Tartaglia.
Tin tc c truyn i lm chn ng c gii ton hc. thnh ph Milan c mt ngi ng
ngi khng yn, l Girolamo Cardano (1501-1576). ng khng ch l thy thuc ni ting
khp chu u, m cn l mt nh ton hc ti ba, dy ton v c nhiu cng trnh nghin cu
v ton hc. ng chuyn tm nghin cu phng trnh bc 3 nhng cha c kt qu. Cho nn
khi nghe tin Tartaglia gii c phng trnh bc 3, ng hy vng s c hng mt phn
thnh tu ca Tataglia.
Lc ny Tartaglia ni ting khp chu u nhng ng li khng mun cng b rng ri cng
trnh ca mnh. ng ch vit li trong tc phm "Nguyn tc hnh hc", cho nn trong t sch
ca nhiu ngi kh lng c c tc phm ca Tartaglia.
Vi thi chn tnh, hiu hc, sau nhiu ln Cardano ngh, nm 1539 Tartaglia ng
truyn li nhng b quyt cho Cardano. Nhng Cardano khng tn trng li ha, nm 1545
gii thiu cch gii phng trnh bc 3 vi li gii thch ca mnh trong cun "Ars magna".
Cardano vit:
"Khong 30 nm trc, Ferro tm c phng php gii ny, truyn li cho ngi khc
v tng tranh lun vi Tartaglia, Tartaglia cng pht hin c phng php ny. Ti
nhiu ln ngh thit tha v cui cng Tartaglia truyn t cho ti phng php gii
nhng li khng ch cho ti phng php chng minh, v vy buc ti phi tm ra nhiu cch
chng minh. V n rt kh, ti xin m t n nh sau:. . . "
Sau cuc tranh ci v bn quyn cch gii ny n ra gay gt nhng cui cng l phi
chin thng v ngi ta cng nhn li gii l ca Tartaglia. Tuy vy Cardano vn ni ting
nh cng b cch gii ny.
V sau Cardano cn a ra cch gii khc cho phng trnh x3 + mx = n vi m, n > 0 nh
sau: t x = u + v thay vo phng trnh c:
u3 + v 3 + (3uv + m)(u + v) n = 0
Vic gii phng trnh trn tng ng vic gii h phng trnh:

3uv + m = 0
u3 + v 3 n = 0
hoc
2 n

m3
=0
27

Trong mt cun sch ca Trung Quc li ni rng, sau 2 nm Cardano cng b cch gii
phng trnh bc 3 th trong bi "Nhng cu hi v pht minh", Tartaglia ph phn thi
thiu trung thc ca Cardano v yu cu thnh ph Milan t chc tranh lun cng khai vi
Cardano. n ngy gp nhau tranh lun th khng phi Cardano m li l mt hc tr ti
ba ca Cardano, l Lodovico Ferrari (1522-1565). Ferrari khng nhng nm c cch gii

331
phng trnh bc 3 m cn gii c phng trnh bc 4 nn Tartaglia chu tht bi. T
Tartaglia nh b vt thng lng v m hn n lc cht.
Trong cun "Lch s ton hc" ca Howard Eves ngi M xut bn nm 1969 li vit: ". . .
nhng li phn i mnh lit ca Tartaglia n tai Ferrari nn ngi ny lp lun cho thy
ca mnh rng, Cardano c c thng tin cn cho mnh t Ferro qua mt ngi th ba v ln
n Tartaglia l n cp t cng mt ngun . . . "
Cho d cc li n thi nh th no nhng cui cng li gii c lu truyn n ngy nay
vi tn gi chung l cng thc Cardano-Tartaglia:
s
x=

n
+
2

r 
n 2
2

s
+

 m 3
3

r 
n 2
2

 m 3
3

Cng cn ni thm rng, cng thc trn l cng thc nghim ca phng trnh bc 3 cha y
. Tuy nhin, t phng trnh bc 3 y (chnh tc hay hon chnh):
ax3 + bx2 + cx + d = 0
n phng trnh x3 + mx = n ch cn t:
y =x+

b
3a

Nm 1572, Bombelli cho cng b mt cun sch i s gp phn ng k vo vic gii phng
trnh
Trong cc sch gio khoa v l thuyt cc phng trnh cho bit rng, nu
 n 2 bc3.
m 3
+
l m th phng trnh x3 + mx = n c 3 nghim thc. Nhng trong trng
2
3
hp ny, cc nghim c biu th bng hiu ca hai cn bc 3 ca cc s phc lin hp.
iu tng chng bt thng ny gi l "trng hp bt kh quy ca phng trnh bc 3"
lm bn tm ng k cc nh i s hc thi xa. Bombelli ch ra tnh thc ca cc nghim
thot nhn l khng thc trong trng hp bt kh quy.
Trong cun "Canon mathematicus seu ad triangula" ca Viete xut bn nm 1579, tc gi c
gi mt cch gii bng lng gic cho trng hp bt kh quy ca cc phng trnh bc 3.
Trong lau65n vn ca Viete thy c cch gii rt p sau y cho phng trnh bc 3:
x3 + 3ax = 2b
Phng trnh trn l mt dng m phng trnh bc 3 no cng c th quy v c. Nu t:
x=

a
y
y

th phng trnh trn tr thnh:


y 6 + 2by 3 = a3
y l phng trnh bc 2 ca y 3 , ta c th tm ra y ri sau l x.
V phng trnh x3 + 3ax = 2b th Isaac Newton (1642-1727) cng c cch gii.

332

Nhng vinh quang sau khi qua i


chu u n th k XVI, khoa hc t nhin pht trin rt nhanh chng. Truyn thuyt
tn gio cho rng,Thng sinh ra th gii v Tri t c hnh vung. Nhng pht kin a
l ca Christophe Columbus (1451-1506), Fernand de Magellan (1480-1521) v nhng ngi
khc chng minh y rng Tri t c hnh cu, l iu khng th chi ci c.
Pht minh v a l ca Galileo em li cho nhn loi nhng nhn thc mi v v tr. Ton
hc c suy tn l "n hong" ca khoa hc t nhin. T th k XVI n th k XVIII xut
hin hng lot cc nh ton hc kit xut, h a nn ton hc ln mt nh cao mi. S
xut hin phng php ta , ng dng ca s phc, sng to ra vi-tch phn,... kt hp
nghin cu th gii khch quan trong trng thi tnh v ng.Vo thi gian , c chu u
gn nh b li ng sau s tr tr ca thi Trung C. Trong tin trnh pht trin ,ton
hc lun i tin phong.
Nhng nhnh phng trnh i s th tnh hnh li khng hon ton nh vy. Do cuc thch
chn ng c th gii ton hc vo u th k XVI nn ngi ta tm c cch gii
phng trnh bc ba, bc bn. T gia th k XVI tr i, ngi ta bt u i su nghin cu
phng trnh bc nm. Cc nh ton hc phn tch t m cch gii phng trnh t bc hai
n bc bn. Nu mt phng trnh c nghim vit c bng cng thc i s ca cc h s
th c gi l phng trnh gii c bng cn thc. Trc thi Bombelli v Viete ngi ta
xc nh c cng thc tng qut tnh nghim ca phng trnh t bc 1 n bc bn.
Khng bao lu, sau khi gii c phng trnh bc ba th phng trnh bc bn tng qut
cng c cch gii i s. Vic gii phng trnh bc bn tng qut quy v vic gii mt phng
trnh bc ba lin kt.
Nm 1540, Zuanne de Tonini da Coi ngi Italia ngh Cardano gii bi ton dn n
phng trnh bc bn nhng Cardano khng gii c, m hc tr l Ferrari li gii c v
sau Cardano cng b cch gii ny trong cun "Ars magna" ca ng.
Cch gii ca Ferrari vit gn theo cch k hiu hin nay nh sau: bin i n gin s quy
mt phng trnh bc bn chnh tc (y ) v dng:
x4 + px2 + qx + r = 0
T phng trnh trn, bin i c
x4 + 2px2 + p2 = px2 qx r + p2
hoc
(x2 + p)2 = px2 qx r + p2
Vi y bt k, t phng trnh trn ta c:
(x2 + p + y)2 = px2 qx r + p2 + 2y(x2 + p) + y 2 = (p + 2y)x2 qx + (p2 r + 2py + y 2 )
By gi ta chn y v phi ca phng trnh trn l mt bnh phng. y l trng hp
khi:
4(p + 2y)(p r 2py + y) q = 0

333
y l phng trnh bc ba ca y nn tm c cch gii. Mt gi tr ca y nh vy s quy
phng trnh bc bn lc u v vic ch phi ly cc cn bc hai.
Mt cch khc bng i s c Viete xut v mt cch na do Descartes a ra nm 1637
nhng trong nhiu sch gio khoa i hc c th cch gii ca Viete cng ging nh cch
gii ca Ferrari.
Nh ton hc Vanmec ngi Ty Ban Nha cng gii c phng trnh bc bn nhng tn
bo cha Tuocmacvada thiu cht ng v theo hn, ng lm tri Tri: phng trnh
bc bn khng hp vi kh nng ca ngi trn tc- l mun ca Tri.
V vic gii phng trnh bc bn tng qut c thc hin ty thuc vo vic gii phng
trnh bc ba lin kt, nn nm 1750 Euler c gng lm iu tng t l quy vic gii phng
trnh bc nm tng qut v php gii phng trnh bc bn lin kt nhng tht bi.
Rt nhiu nh ton hc cng vt c tm kim cch gii phng trnh bc nm nhng khng
i n kt qu. Trc tnh hnh , ngi ta bt u hoi nghi l liu c tn ti hay khng
mt cng thc tnh nghim ca phng trnh bc nm tng qut.
Nm 1778, nh ton hc Lagrange m c mt u mi quan trng. ng tp trung tm
kim cng thc chung gii cc phng trnh t bc hai n bc bn, v ng cho rng, nu tm
c cng thc chung th s suy lun tm c cch gii phng trnh bc nm. Nhng
cui cng ng pht hin thy nghim ca mt phng trnh bit c th c biu th bng
mt hm s i xng ca mt phng trnh h tr khc. Phng trnh h tr ny c ng gi
l cch gii d kin. Dng cch gii d kin ny, ng s dng gii phng trnh bc ba,
bc bn nhng khi n phng trnh bc nm th ng nh chu. Mt tia sng li le trong u
ng: cng thc nh vy l khng th tn ti c, nhng ng li khng th chng minh iu ny.
Phng php tng qut bin i mt phng trnh a thc bc n i vi x thnh mt phng
trnh bc n i vi y, trong cc h s ca y n1 , y n2 u bng 0 c Tschirnhausan (16511708) a ra. V sau php bin i phng trnh bc nm nh vy, trong cc h s ca
y 4 , y 3 , y 2 u bng 0 c Brings (1736-1798) a ra nm 1786. iu ny ng vai tr quan
trng trong vic gii phng trnh bc nm bng hm eliptic. Nm 1834, Jerrard (mt 1863)
li chng minh c h s ca y n3 cng bng 0.
Trong cc nm 1803, 1805, 1813, nh vt l Paolo Ruffini (1765-1822) ngi italia a ra
mt cch chng minh v mt iu m lc by gi c coi l mt s kin, l nghim ca
phng trnh tng qut bc nm hoc bc cao hn u khng th biu th c bng cc cn
thc theo cc h s ca phng trnh .
Loi ngi ng trc s thch thc v tr tu ht i ny n i khc, cho n khi xut
nhin nh ton hc tr tui Niels Henrik Abel (1802-1829) ngi Na Uy. ng l ngi rt can
m. Ngay t thu nh, ng bt u tm li gii cho phng trnh bc nm. Nh quyt tm
chin thng n cng nn vo nm 1824 (lc ng 22 tui), ng chng minh c mt cch
tng qut rng, phng trnh bc t 5 tr ln l khng th c cch gii kiu cn thc. Vi tr

334
tu ca tui tr, ng tuyn b vi th gii mt cch chn l: tr tu ca con ngi l bt
kh chin thng.
Nhng cn ng thnh cng ca Abel ht sc gp gnh. Tuy c nhng thnh cng trong thi
gian ngn ngi nhng nhng g l au kh cn nng n hn. Nhng vinh quang ca ng hu
ht n sau khi ng qua i.
ng sinh ra trong mt gia nh mc s nng thn. Cha m ca ng li ng con v ngho.
Trong s 7 anh ch em, ng l ngi anh th 2. Nm 13 tui, ng c a vo trong trng
dng. Ngay t u ng rt hng th vi ton hc. Nm 1817, trong trng xy ra mt
s kin c bit, trong mt m thay i c cuc i ng. Thy dy ton ca ng, do ngc
i hc tr nn b sa thi. Thay thy gio c l thy Homlboe (sinh nm 1795) mi 22 tui.
Thy Holmboe nhanh chng pht hin ti nng ton hc ca Abel. Khi Abel nu ra quyt
tm tn cng vo phng trnh bc nm, nhiu ngi ch diu, cho rng "ch nhi lm sao
li i n tht thin nga". Vy m Holmboe li rt ng h Abel, ng vin ng c gng vn ln.
Nm 1821, Abel thi u vo trng i hc. thc hin tng ca mnh, ng theo hc
thy Gauss. Ban u ng hc theo cch ca nhng ngi i trc, i tm cch gii p mt
cch chnh din. Sau nhiu ngy thng mit mi, nm 1824 ng chng minh l phng
trnh tng qut bc nm khng th c cng thc nghim bng cn thc. iu lm bn tm
buc loi ngi phi suy ngh sut 2 th k, cui cng c mt thanh nin khng c
ting tm gii quyt. Tuy nhin, iu Abel nu ra khng c cc tp ch ton hc ng
ti, buc ng phi t b tin ra in n. Nhng nhng au kh ca ng khng v th m gim bt.
Nm 1825, Abel n nhiu nc chu u v g ca nhiu ni nhng khng u coi ng ra
g c, k c nhng ni c gi l "vng quc ton hc". Cui cng ng n Berlin. Rt may
mn, y ng c k s Klaye hiu c tng ca ng. Tuy Klaye khng hiu ht ni
dung m Abel trnh by nhng Klaye li hiu c nng lc to ln ca Abel. Nm 1826 Klaye
gip Abel cho ra i tp ch "L lun v ton hc". Ba s u ca tp ch ng 22 bi
pht biu ca Abel, gii thiu cc cng trnh nghin cu ton hc ca Abel. Nhng thnh tu
xut sc ca Abel dn dn thu ht s ch ca gii ton hc chu u. Chnh v vy m tp
ch ny ni ting cho n tn ngy nay.
Thng 5/1827, vi tm lng thng nh qu hng T quc, Abel tr v th Otslo. Tuy
nhin, qu hng th ng li khng tm c cng vic g thch hp. Thng 9/1828 bn vin
s hn lm khoa hc Php yu cu vua Sal XIV gip vt cht, to iu kin Abel
nghin cu khoa hc. Nhng do vt v qu m bnh lao ca ng li ti pht, e da n
tnh mng ca ng. Ngy 6/4/1829, ngi sao sng rc trn bu tri tt ln.
Ngy 9/4/1829, ngi thn trong gia nh nhn c mt bc th t Berlin gi n vi ni
dung:
"ng Abel knh mn!
Trng chng ti quyt nh tn vinh ng l gio s ton hc ca trng.

335
Chc mng vinh d ca ng!"
Nhng bc th ny n chm, ng qua i trc 3 ngy.
Ngy 28/6/1830, Vin hn lm khoa hc Php trao gii thng ln cho Abel.
l nhng vinh quang sau khi ng qua i.
Li c cu chuyn v vic Abel gii phng trnh c bc rt ln nh sau: v i s H Lan c
ln ni dc vi vua Henry IV rng, nc Php khng c nh ton hc no c th gii c
bi ton do Adrianus Romanus (1561-1615) ngi H lan t ra nm 1593, v bi ton ny i
hi phi bit cch gii mt phng trnh bc bn5. Viete c triu n. Sau khi xem xt bi
ton, ng nhn ra mi lin h lng gic c bit v a ra 2 nghim, sau thm 21 nghim
na.
Trong cun "De numerosa", Viete trnh by mt qu trnh c h thng ni chung tm
xp x lin tip mt nghim ca mt phng trnh. Phng php ny s rt mt cng sc khi
gii cc phng trnh bc cao nhng c dng cho n ht th k XVI.

tiu s mt s nh ton hc ni ting


Mt cuc i trn bia m
Ngi ta khng bit g nhiu v cuc i ca nh ton hc c Hy Lp l i--phng (Diophante):
ng sng th k III trc Cng nguyn, sinh A-lch-xng-ri, tm nghin cu su v
phng trnh v tm tt cuc i mnh bng nhng hng ch trn bia m nh sau:
Hi du khch! Ni y yn ngh mt ngi tn l i--phng. V nhng con s nhim mu
c th s ni cho bn bit v nhng thng ngy di ca i ng. ng sng th ngy trong
mt phn su cuc i. Mt phn mi hai cuc i na, cm ng ln phn ru. Thm mt
phn by cuc i, ng mang nhn ci trn tay v 5 nm sau, c mt a con trai xinh
xn. Than i, d rt c thng yu, ngi con ny cht khi anh ta va bng na tui th
ca cha. Qu au kh, ngi cha bt hnh ch sng thm bn nm sau ci cht ca con.
Bn hy ni i: ng ta th bao nhiu tui v cuc i ng ra sao?.
Theo ngn ng phng trnh th khi t tui th ca i--phng l n s x, phin dch tng
cu trong li trn bia m, ta s c phng trnh sau:
x
x
x
x
+
+ +5+ +4=x
6 12 7
2
Gii phng trnh ny, bn d dng tm c x = 84. Vy i--phng sng 84 nm. Thi
nin thiu ca ng (84 : 6) l 14 nm, V 84 : 12 = 7 nn ng c ru lc 14 + 7 = 21 (tui).
Li do 84 : 7 = 12 nn ng ly v lc 21 + 12 =33 (tui). ng c con trai lc 38 (tui). V
ngi con ch sng bng na tui th ca cha nn anh ta cht lc 42 tui. Khi , ngi cha
38 + 42 = 80(tui). V ng nhm mt la i lc 80 + 4 = 84 (tui).
Tht l mt tm bia c o!

336

Ch v l sch qu hp!
Ngay t thi c ngi ta bit tam gic c ba cnh 3, 4, 5 (n v di) l tam gic vung v
32 + 42 = 52 ngoi ra, c v s s nguyn dng khc tha mn phng trnh x2 + y 2 = z 2 .
Chng hn cc b ba s 6; 8; 10 hay 5; 12; 13. . . Ta u bit y l cc s Pi-ta-go. T phng
trnh ny, ny ra ngh: liu c tm c cc b ba s nguyn x, y, z tha mn phng
trnh xn + y n = z n (n l s t nhin ln hn 2) hay khng. ngh ngy ln u tin c
i--phng xem xt k, v vy phng trnh c tn gi l phng trnh i--phng.
Ngi ta chng minh c l vi tt c cc s n t 3, 4. . . n 4000, chng c cc s nguyn
x, y, z no nghim ng phng trnh i--phng c. Ring Pi-e Phc-ma (Pierre Fermat, 16011665) mt nh ton hc Php, ngi tng cng vi Pa-xcan tm ra php tnh xc sut,
mnh dn nghi ng: Phng trnh i--phng s lun lun v nghim, d n l bt c s t
nhin no.
Nm 1637, Phc-ma vit bn l cung sch ca i--phng rng ng tm c cch chng
minh rt k diu iu nghi ng ni trn ca mnh l ng, nhng khng ghi ra c ch v. . .
l cun sch qu hp.
T n nay, hn ba trm nm tri qua, nhiu nh ton hc tn bao cng sc ri vn
chu b tay trong vic i tm chng minh ca Phc-ma.
iu nghi ng ni trn tr thnh mt bi ton lch s ni ting v phng trnh i--phng
v c tn gi bi ton Phc-ma. Nm 1993, nh ton hc ng-ry Oai (Andrew Wiles) ngi
Anh chng minh c Bi ton Phc-ma vi cc cng c ton hc hin i v vi hn . . .
200 trang giy.
Hai gng mt tr
Lch s ca i s trong nhng trang nghin cu v phng trnh u gi mi bng dng ca
hai chng trai tr thin ti: Ni-en hen-rch A-ben (Niels Henrik Abel, 1802-1829) nh ton hc
Na Uy v E-va-rt Ga-loa (Evariste Galois, 1811-1832) nh ton hc Php. Cng sng hi u
th k XIX, cuc i ngn ngi ca c hai ngi cng li nhng tc phm v gi cho ton
hc hin i.
C A-ben ln Ga-loa u bc l ti nng v ton ngay t lc cn ngi trn gh nh trng,
c hai cng quan tm n vic gii cc phng trnh i s c bc ln hn hai. Tuy khng h
bit n vic lm ca nhau, c hai cng gi cng trnh ca mnh n Vin hn lm khoa hoc
Php, v. . . cng gp ri ro: Cng trnh ca A-ben b ct vo t lu tr, mi lc anh cht
mi c ngi v tnh c v in ra, cn Ga-loa ba ln kin nhn gi ti mi n ln th ba mi
c nhn tr li cng trnh ca mnh vi li ph Khng th hiu c!. A-ben cht m thm
trong ngho tng, nhng li tn tui trong cc phng trnh A-ben v nhm A-ben ca
l thuyt nhm. Cn Ga-loa cht trong cuc quyt u v danh d vi bn khiu khch, li
60 trang th anh vit trong m cui cng, trong trnh by ngn gn mi kt qu quan
trng nht anh tm ra khi nghin cu phng trnh. L th di ny c vit bng nt ch
v cng hi h, xen gia nhng cng thc ton l nhng cu Ti vi qu. Ch cn my ting
ng h na thi, Ti khng kp. . .
Vy m 60 trang th m ra cho cc nh ton hoc th gii v s hng i, v li mt
l thuyt v phng trnh i s mt n s mang tn l thuyt Ga-loa.
Bn hy nhn li nm sinh v nm mt ca A-ben v Ga-loa: hai gng mt thin ti lun
ng cnh cnh ca m vo lu i ton hc hin i, mi mi tr trung.

337

Sng hay cht


Vo nm 1927, Erwin Schrodinger vit ra mt phng trnh cho nhng sng lng t. N
ph hp vi nhng th nghim mt cch tuyt vi ng thi v nn mt bc tranh ca mt th
gii rt khc l, trong nhng ht s cp nh electron khng phi l vt th r rng, m l
nhng m my xc sut. Spin ca electron ging nh mt ng tin c th na sp na nga
cho n khi n ri xung bn. Khng lu sau , cc nh l thuyt li lo lng khng yn trc
mi tnh cht l lng t, v d nhng con mo va sng va cht, v nhng v tr song song
trong Adolf Hitler l k chin thng cuc chin tranh th gii ln th hai.
C hc lng t khng b rng buc vi nhng b n trit l nh vy. Hu nh mi vt dng
hin i my vi tnh, in thoi di ng, my chi game, xe hi, t lnh, l vi sng u
cha nhng con chip nh gc transistor, dng c c s hot ng da trn c hc lng t
ca cht bn dn. Nhng cng dng mi cho c hc lng t xut hin gn nh hng tun.
Cc chm lng t - nhng ming nh xu ca mt cht bn dn c th pht ra nh sng
thuc mi mu sc v c s dng ghi nh sinh hc, trong chng thay th cho nhng
cht nhum truyn thng, thng l c hi. Cc k s v nh vt l ang c gng pht minh
ra my vi tnh lng t, mt dng c c th thc hin song song nhiu php tnh khc nhau,
ging ht nh con mo va sng va cht.
Laser l mt ng dng na ca c hc lng t. Chng ta s dng chng c thng tin t
nhng l nh li ti trn a CD, DVD v a Blu-ray. Cc nh thin vn s dng laser o
khong cch t Tri t n mt trng. Thm ch c th phng nhng tn la v tr ln t
Tri t vi sc y l mt chm laser mnh.
Chng cui trong cu chuyn ny c xut x t mt phng trnh gip chng ta hiu ngha
ca sng. N bt u vo nm 1807, khi Joseph Fourier ngh ra mt phng trnh cho dng
nhit. ng gi mt bi bo v ni v n n Vin hn lm Khoa hc Php nhng b t chi.
Vo nm 1812, vin hn lm Php a vn nhit thnh ti ca gii thng hng nm
ca vin. Fourier li gi mt bi bo di hn, c hiu chnh v git gii.
Ci hp dn nht ca bi bo ginh gii thng ca Fourier khng phi l phng trnh, m
l cch ng gii n. Mt bi ton in hnh l tm xem nhit dc theo mt thanh mng
thay i nh th no theo thi gian, cho bit trc c im nhit ban u. Fourier c th
gii phng trnh ny mt cch nh nhng nu nh nhit bin thin nh mt sng hnh sin
dc theo chiu di thanh. V th, ng biu din mt c trng phc tp hn l s kt hp ca
nhng ng hnh sin vi bc sng khc nhau, gii phng trnh cho mi ng cong hnh sin
thnh phn, v cng tt c nhng nghim ny li vi nhau. Fourier khng nh phng php
ny ng cho mi c trng nhit bt k, thm ch ng c vi trng hp trong nhit
c gi tr nhy cc. Tt c nhng g phi lm l cng gp mt s v hn nhng ng gp
t nhng ng cong hnh sin vi tn s ln dn.
Du vy, bi bo mi ca Fourier b ch trch l khng cht ch, v mt ln na vin
hn lm Php t chi ng ti. Vo nm 1822, Fourier pht l mi phn i v cho cng b
l thuyt ca ng di dng mt quyn sch. Hai nm sau , ng t b nhim mnh lm th
k ca vin hn lm, d mi ca ng vo nhng k ch trch ng, v cho ng bi bo gc ca
ng trong tp san ca vin. Tuy nhin, nhng ngi ch trch cha chu dng li. Cc nh ton
hc bt u nhn ra rng nhng chui v hn l nhng th nguy him; chng khng lun lun
hnh x ging nh nhng tng hu hn, p . Vic gii quyt nhng vn ny ha ra l

338
ht sc kh khn, nhng phn quyt cui cng l quan im ca Fourier c th c lm cho
cht ch bng cch ngoi suy ra nhng c trng rt khng u. Kt qu l php bin i
Fourier, mt phng trnh xem mt tn hiu bin thin theo thi gian l tng ca mt chui
nhng ng cong hnh sin thnh phn v tnh ra bin v tn s ca chng.
Ngy nay, php bin i Fourier nh hng n cuc sng ca chng ta theo v s kiu. Chng
hn, chng ta c th s dng n phn tch tn hiu dao ng to ra bi mt trn ng t
v tnh ra nhng tn s m nng lng truyn bi mt t chn ng l ln nht. Mt bc
tin ti xy dng nhng cng trnh chu c ng t l m bo rng tn s ring ca cng
trnh khc vi tn s ca ng t.
Nhng ng dng khc bao gm vic loi tp m ra khi nhng bn ghi m c, tm kim cu
trc ca ADN bng nh chp tia X, ci thin s thu nhn v tuyn v ngn cn nhng dao
ng khng mong mun xe hi. Thm mt ng dng na m a s mi ngi chng ta s
dng thng xuyn m khng , l chp nh k thut s.
Nu bn tnh xem cn bao nhiu thng tin biu din mu sc v sng ca mi pixel trong
mt bc nh k thut s, bn s pht hin ra rng mt chic camera k thut s nhi nht vo
th nh ca n lng d liu nhiu gp mi ln ci th nh c th cha. Camera lm cng
vic ny bng cch s dng s nn d liu JPEG gm nm bc nn khc nhau. Mt trong s
chng l mt phin bn k thut s ca php bin i Fourier, n hot ng vi mt tn hiu
khng thay i theo thi gian m thay i t u ny qua u kia bc nh. C s ton hc
hu nh l ging ht. Bn bc cn li tip tc lm gim d liu thm na, n khong bng
mt phn mi lng ban u.
y mi ch l by trong nhiu phng trnh m chng ta bt gp hng ngy, nhng khng
nhn ra chng ang hin din y. Nhng s tc ng ca nhng phng trnh ny i vi
lch s th su sc hn nhiu. Mt phng trnh tht s mang tnh cch mng c th c s tc
ng i vi s tn ti ca loi ngi ln hn c mi nh vua v hong hu c mu chon
y nhng quyn s hc ca chng ta.
C (hoc c th c) mt phng trnh, trn ht thy, m cc nh vt l v nh v tr hc t
ht nim tin yu vo y: mt l thuyt ca tt c thng nht c hc lng t v thuyt tng
i. Ni ting nht trong s nhiu ng c vin l l thuyt siu dy. Nhng nh mi ngi chng
ta u bit, cc phng trnh ca chng ta cho th gii vt cht c l ch l nhng phin bn
n gin ha khng bt gi c cu trc su sc ca thc ti. Ngay c nu t nhin c tun
theo nhng nh lut vn vt, th chng c th khng c biu din di dng nhng phng
trnh.
Mt s nh khoa hc ngh rng n lc chng ta t b nhng phng trnh truyn thng
theo ui nhng thut ton nhng cng thc khi qut hn tnh ton mi th, k c
vic ra quyt nh. Nhng cho n nhng ngy y, nu c, s hiu bit su sc nht ca chng
ta v cc nh lut ca t nhin s tip tc c dng thc nhng phng trnh, v chng ta s
hc cch tm hiu chng v thch ng vi chng. Cc phng trnh c thnh tu ca chng,
Chng tht s lm bin chuyn th gii v chng s li tip tc lm th gii bin chuyn.

TI LIU THAM KHO

1. Nguyn Vn Mu ; Nguyn Vn Tin


Mt s chuyn i s bi dng hc sinh gii. NXB Gio dc 2010.
2. Thanh Sn
Mt s chuyn hnh hc phng bi dng hc sinh gii. NXB Gio dc 2010.
3. V Dng Thy ; Nguyn Vn Nho
40 nm Olympic Ton hc quc t. NXB Gio dc 2006.
4. on Qunh ; Don Minh Cng ; Trn Nam Dng ; ng Hng Thng
Ti liu chuyn ton i s 10. NXB Gio dc 2010.
5. Nguyn Vn Mu
Phng php gii phng trnh v bt phng trnh. NXB Gio dc 2010.
6. Thanh Sn
Mt s chuyn hnh hc khng gian bi dng hc sinh gii. NXB Gio dc 2010.
7. V Hu Bnh
Nng cao v pht trin ton 9. NXB Gio dc 2006.
8. Tp ch Ton hc v tui tr.
9. Tuyn tp 10 nm thi Olympic 30/4. NXB Gio dc 2006.
10. Cc bi thi Olympic Ton THPT Vit Nam. NXB Gio dc 2007.
11. Din n http://mathscope.org.
12. Din n http://onluyentoan.vn.
13. Din n http://www.artofproblemsolving.com/Forum.
14. Bch khoa ton th m Wikipedia.

339

You might also like